Вы находитесь на странице: 1из 108

CITY OF BATANGAS VS.

PHILIPPINE SHELL

FACTS:

Batangas City is a local government unit created by virtue of its charter, Republic Act No. 5495 (RA 5495). Under RA 5495, Batangas
City constitutes a political body corporate, and is endowed with powers which pertain to a municipal corporation.[9] The Sangguniang
Panlungsod is the legislative body of Batangas City.

PSPC owns and operates a refinery situated in Tabangao, Batangas City (Tabangao Refinery).[11]

In furtherance of the mandate of Presidential Decree No. 87 (PD 87) to promote the discovery and production of indigenous petroleum,
the Department of Energy (DOE) executed Service Contract No. 38 (SC 38) with SPEX under which SPEX was tasked to explore and
develop possible petroleum sources in North Western Palawan.

SPEX's exploration led to the discovery of an abundant source of natural gas in the Malampaya field off the shores of Palawan, which
thereafter gave rise to the Malampaya Project.

The Malampaya Project required the construction of a 504-kilometer offshore pipeline for the transport of natural gas from Malampaya
field to Batangas, for treatment in PSPC's Tabangao Refinery.

On May 28, 2001, the Sangguniang Panlungsod enacted the Assailed Ordinance which requires heavy industries operating along the
portions of Batangas Bay within the territorial jurisdiction of Batangas City to construct desalination plants to facilitate the use of seawater
as coolant for their industrial facilities

The Assailed Ordinance was approved by the city mayor on June 7, 2001

Heavy industries subject of the Assailed Ordinance had until May 28, 2006 to comply with its provisions. Among the facilities affected by
the Assailed Ordinance is PSPC's Tabangao Refinery.

On May 23, 2006, PSPC filed against Batangas City and the Sangguniang Panlungsod a Petition for Declaration of Nullity (PSPC Petition)
before the RTC praying that the Assailed Ordinance be declared null and void. The PSPC Petition was raffled to Branch 84, and docketed
as SP Civil Case No. 7924

For its part, PSPC averred that the Assailed Ordinance constitutes an invalid exercise of police power as it failed to meet the substantive
requirements for validity.

Particularly, PSPC argued that the Assailed Ordinance contravenes the Water Code of the Philippines (Water Code), and encroaches
upon the power of the National Water Resources Board (NWRB) to regulate and control the Philippines' water resources.

In addition, Batangas City and the Sangguniang Panlungsod failed to sufficiently show the factual or technical basis for its enactment.

In this connection, PSPC alleged that the Assailed Ordinance unduly singles out heavy industries, and holds them solely accountable for
the loss of water and destruction of aquifers without basis, resulting in the deprivation of their property rights without due process of law

PSPC contended that the Assailed Ordinance was not posted or published in a newspaper of general circulation in the province, nor were
public hearings or consultations involving concerned parties conducted thereon

Further, there are no records showing that the Assailed Ordinance, as approved by the Sangguniang Panlungsod, was forwarded to the
Sangguniang Panlalawigan of the Province of Batangas after it was approved by the city mayor

SPEX essentially adopted the allegations of PSPC and prayed for the same relief, asserting that it possesses material and direct interest
in the subject matter of the PSPC Petition.[28]

In response, Batangas City and the Sangguniang Panlungsod maintained that they have the power to enact the Assailed Ordinance
pursuant to the general welfare clause under the LGC.[29] According to them, the rationale of the Assailed Ordinance is to stop PSPC
and other industries similarly situated from relying "too much" on gro... round water as coolants for their machineries, and alternatively
promote the use of seawater for such purpose, considering that fresh ground water is a "perishable commodity."... that the "regulation or
prohibition" on the use of ground water is merely incidental to the main purpose of the Assailed Ordinance, which is to compel heavy
industries such as PSPC to construct desalination plants. Hence, provisions having regulatory and prohibitive effect may be taken out of
the Assailed Ordinance without entirely impairing its validity

Further, Batangas City and the Sangguniang Panlungsod took exception to PSPC's allegations and asserted that the Assailed Ordinance
had been published in Dyaryo Veritas, a newspaper of general circulation in the area. Moreover, Batangas City and the Sangguniang
Panlungsod claimed that a joint public hearing on the Assailed Ordinance had in fact been conducted by the Sangguniang Panlungsod
and Sangguniang Panlalawigan, where PSPC was duly represented

Finally, Batangas City and the Sangguniang Panlungsod averred that since PSPC and SPEX, along with other concerned heavy
industries, essentially question the former's authority to regulate and prohibit the use of fresh ground water, they should have first referred
their grievances to NWRB by filing a complaint for adjudication on the threatened revocation of their existing water permits.[34]
RTC rendered a Decision

The dispositive portion of said Decision reads:

It is evident that from foregoing factual milieu and parameters, the questioned ordinance is INVALID, as it is hereby declared INVALID,
in its entirety for want of necessity and for not conducting prior public hearing, and for violating the due process clause of the Constitution
with respect to its (sic) Sec. 8, City Ordinance No.3,... The RTC gave credence to the testimony of PSPC's witness Engineer Joeffrey
Caranto (Engineer Caranto) who conducted a hydrogeology study on the Tabangao-Malitam watershed from which PSPC sources fresh
ground water.[38] The RTC summarized the findings of said study in this wise:A water balance x x x calculation of the Tabangao-Malitam
groundwater system shows that the natural recharge (replenishment) rate far exceeds the current demand for water in the area. Hence,
there is no threat of depletion of the groundwater resource[s] in the Tabangao-Malitam [w]atershed that purportedly may result from
PSPC's deep well pumping.

Water levels in the PSPC wells have not lowered significantly over the last three (3) decades, indicating that there is no substantial
diminution of the supply of groundwater.

The chloride levels in the other nearby PSPC wells are all within drinking water standards and have not increased in the last four (4)
decades of usage. This indicates that salt water intrusion is not occurring in the PSPC wells.

The RTC also noted that the Sangguniang Panlungsod failed to consult the NWRB before enacting the Assailed Ordinance, thereby
encroaching upon its authority

Anent Section 8, the RTC concluded that the power granted to the city mayor to cause the issuance of cease and desist orders against
the use of ground water without prior notice and hearing constitutes a violation of the due process clause.

The appeals against JG Summit and First Gas were raffled to the Fourth Division (CA Fourth Division) and were docketed as CA-G.R.
CV Nos. 90324 (JG Summit Appeal) and 90365 (First Gas Appeal), respectively. Meanwhile, the appeal filed against PSPC and SPEX
was raffled to the Tenth Division (CA Tenth Division), and docketed as CA-G.R. CV No. 90373 (PSPC Appeal).

These testimonies, according to Batangas City and the Sangguniang Panlungsod, serve as sufficient factual bases for the enactment of
the Assailed Ordinance, as "there could be no higher degree of evidence than the actual experience of the inhabitants in the area.

On the basis of the submissions of the parties, the CA Tenth Division issued the Assailed Decision dismissing the appeal filed against
PSPC and SPEX for lack of merit. The relevant portions of the Assailed Decision read:City Ordinance No. 3, S. 2001 contravenes
Presidential Decree No. 1067, better known as "The Water Code of the Philippines" as it is an encroachment into the authority of the
[NWRB]. The use of water resources is under the regulatory power of the national government. This is explicit from the provisions of the
Water Code which states that -"The utilization, explo[i]tation, development, conservation and protection of water resources shall be subject
to the control and regulation of the government through the [NWRB]".

Although respondents-appellants insist that the city ordinance is not an absolute prohibition but merely a regulation on the use of fresh
groundwater for cooling systems and industrial purposes the argument cannot justify the attempt to usurp the NWRB's power to regulate
and control water resources. Moreover, not only does the city ordinance prohibit or regulate the use of fresh groundwater in disregard of
previously granted water permits from the NWRB but also directs the installation of desalination plants for purposes of utilizing sea water,
without the requisite water permit from the NWRB.

The police power of the Sangguniang Panglungsod is subordinate to the constitutional limitations that its exercise must be reasonable
and for the public good. Without the concurrence of these two requisites, the ordinance will not muster the test of a valid police measure
and should be struck down

There must be a reasonable relation between the purposes of the police measure and the means employed for its accomplishment.
Arbitrary invasion of personal rights and those pertaining to private property will not be allowed even under the guise of protecting public
interest.

WHEREFORE, the appeal is DISMISSED. The Decision dated 29 June 2007 rendered by the Regional Trial Court of Batangas City,
Branch 84, in SP Civil Case No. 7924, declaring invalid City Ordinance No. 3, S. 2001 is hereby AFFIRMED.

Batangas City and the Sangguniang Panlungsod filed a Motion for Reconsideration

The CA Tenth Division found that the MR merely reiterated the arguments relied upon in the appeal, which were already passed upon in
the Assailed Decision

On January 25, 2011, Batangas City filed the present Petition.[55] Notably, the Petition does not name the Sangguniang Panlungsod as
party

ISSUES:

The sole issue for this Court's determination is whether the CA erred in affirming the RTC Decision which declared the Assailed Ordinance
invalid.
In this Petition, the Court is called upon to determine whether the control and regulation of the use of water may be made subject of a city
ordinance under the regime of the Water Code - a national statute governing the same subject matter.

RULING:

The Assailed Ordinance is void for being ultra vires, for being contrary to existing law, and for lack of evidence showing the existence of
factual basis for its enactment.

The requisites for a valid ordinance are well established. Time and again, the Court has ruled that in order for an ordinance to be valid, it
must not only be within the corporate powers of the concerned LGU to enact, but must also be passed in accordance with the procedure
prescribed by law. Moreover, substantively, the ordinance (i) must not contravene the Constitution or any statute; (ii) must not be unfair
or oppressive; (iii) must not be partial or discriminatory; (iv) must not prohibit, but may regulate trade; (v) must be general and consistent
with public policy; and (vi) must not be unreasonable

Batangas City claims that the enactment of the Assailed Ordinance constitutes a valid exercise of its police power. This claim is erroneous

Since LGUs exercise delegated police power as agents of the State, it is incumbent upon them to act in conformity to the will of their
principal, the State.[75] Necessarily, therefore, ordinances enacted pursuant to the general welfare clause may not subvert the State's
will by contradicting national statutes

The Water Code governs the ownership, appropriation, utilization, exploitation, development, conservation and protection of water
resources.[... water resources are placed under the control and regulation of the government through the National Water Resources
Council, now the NWRB.[79] In turn, the privilege to appropriate and use water is one which is exclusively granted and regulated by the
State through water permits issued by the NWRB.[80] Once granted, these water permits continue to be valid save only for reasons
spelled out under the Water Code itself.[81]

Conversely, the power to modify, suspend, cancel or revoke water permits already issued also rests with NWRB.

There is no doubt, therefore, that the Assailed Ordinance effectively contravenes the provisions of the Water Code as it arrogates unto
Batangas City the power to control and regulate the use of ground water which, by virtue of the provisions of the Water Code, pertains
solely to the NWRB.

By enacting the Assailed Ordinance, Batangas City acted in excess of the powers granted to it as an LGU, rendering the Assailed
Ordinance ultra vires.

In any case, it bears emphasizing that the measure of the substantive validity of an ordinance is the underlying factual basis for which it
was enacted. Hence, without factual basis, an ordinance will necessarily fail the substantive test for validity.

While the Assailed Ordinance has been struck down as invalid, the pronouncements hereunder should not be misconstrued by heavy
industries to be carte blanche to abuse their respective water rights at the expense of the health and safety of the inhabitants of Batangas
City, the environment within which these inhabitants live, and the resources upon which these inhabitants rely. The Court recognizes
fresh ground water as an invaluable natural resource, and deems it necessary to emphasize that Batangas City is not precluded from
exercising its right to protect its inhabitants from injurious effects which may result from the misuse of natural water resources within its
territorial jurisdiction, should these effects later arise, provided that such exercise is done within the framework of applicable national law,
particularly, the Water Code.

Principles:

The policy of ensuring the autonomy of local governments was not intended to create an imperium in imperio and install intra-sovereign
political subdivisions independent of the sovereign state.[2] As agents of the state, local governments should bear in mind that the police
power devolved to them by law must be, at all times, exercised in a manner consistent with the will of their principal.

Police power is the power to prescribe regulations to promote the health, morals, peace, education, good order, safety, and general
welfare of the people.[71] As an inherent attribute of sovereignty, police power primarily rests with the State. In furtherance of the State's
policy to foster genuine and meaningful local autonomy, the national legislature delegated the exercise of police power to local government
units (LGUs) as agents of the State

Such delegation can be found in Section 16[73] of the LGC, which embodies the general welfare clause.[74]

"that ordinances passed by virtue of the implied power found in the general welfare clause must be reasonable, consonant with the
general powers and purposes of the corporation, and not inconsistent with the laws or policy of the State."

In this regard, it is appropriate to stress that where the state legislature has made provision for the regulation of conduct, it has manifested
its intention that the subject matter shall be fully covered by the statute, and that a municipality, under its general powers, cannot regulate
the same conduct.
Where the subject is o(statewide concern, and the legislature has appropriated the field and declared the rule, its declarati on is binding
throughout the State." A reason advanced for this view is that such ordinances are in excess of the powers granted to the municipal
corporation.

It is a fundamental principle that municipal ordinances are inferior in status and subordinate to the laws of the state. An ordinance in
conflict with a state law of general character and statewide application is universally held to be invalid. The principle is frequently
expressed in the declaration that municipal authorities, under a general grant of power, cannot adopt ordinances which infringe the spirit
of a state law or repugnant to the general policy of the state.

To prohibit an act or to compel something to be done, there must be a shown reason for the same. The purpose must also be cogent to
the means adopted by the law to attain it. In this case, as seen in the "whereas clause," the purpose of the ordinance is to protect the
environment and prevent ecological imbalance, especially the drying up of the aquifers of Batangas City

It would have been acceptable had there been a specific study or findings that the local government conducted (sic) and not j ust its
reliance on the complaints of some constituents who merely made its conclusion that the drying up of wells or its salination was due to
the "heavy industries" use of groundwater.

ASSESSOR OF AGUSAN DEL SURE VS. FPOIPI

FACTS:

The exemption from real property taxes given to cooperatives applies regardless of whether or not the land owned is leased. This
exemption benefits the cooperative's lessee. The characterization of machinery as real property is governed by the Local Government
Code and not the Civil Code.

This Petition1 for review assails the Decision2 dated September 26, 2007 and the Resolution3 dated May 26, 2008 of the Court of Appeals
in CA-G.R. SP No. 74060. The Court of Appeals affirmed the Decision of the Central Board of Assessment Appeals (CBAA) exempting
Filipinas Palm Oil Plantation Inc. from payment of real property taxes.4chanrobleslaw

Filipinas Palm Oil Plantation Inc. (Filipinas) is a private organization engaged in palm oil plantation5 with a total land area of more than
7,000 hectares of National Development Company (NDC) lands in Agusan del Sur.6 Harvested fruits from oil palm trees are converted
into oil through Filipinas' milling plant in the middle of the plantation area.7 Within the plantation, there are also three (3) plantation roads
and a number of residential homes constructed by Filipinas for its employees.8chanrobleslaw

After the Comprehensive Agrarian Reform Law9 was passed, NDC lands were transferred to Comprehensive Agrarian Reform Law
beneficiaries who formed themselves as the merged NDC-Guthrie Plantations, Inc. - NDC-Guthrie Estates, Inc. (NGPI-NGEI)
Cooperatives.10 Filipinas entered into a lease contract agreement with NGPI-NGEI.11chanrobleslaw

The Provincial-Assessor of Agusan del Sur (Provincial Assessor) is a government agency in charge with the assessment of lands under
the public domain.12 It assessed Filipinas' properties found within the plantation area,13 which Filipinas assailed before the Local Board
of Assessment Appeals (LBAA) on the following grounds:

chanRoblesvirtualLawlibrary

(1.) The [petitioner] Provincial Assessors of Agusan del Sur ERRED in finding that the Market Value of a single fruit bearing oil palm tree
is P207.00 when it should only be P42.00 pesos per tree;

(2.) The [petitioner] ERRED in finding that the total number of standing and fruit bearing oil palm tree is PI 10 [sic] trees per hectare when
it should be only 92 trees;

(3.) The [petitioner] ERRED in finding that the Market Value[s] of the plantation roads are:ChanRoblesVirtualawlibrary
A.) P270,000.00 per kilometer for primary roads
B.) P135,000.00 for secondary roads
C.) P67,567.00 for tertiary roads constructed by the company.
It should only be:ChanRoblesVirtualawlibrary
A.) P105,000.00 for primary roads
B.) P52,300.00 for secondary roads
C.) P26,250.00 for tertiary roads
Likewise, bridges, culverts, canals and pipes should not be assessed separately from plantation roads, the same being components of
the roads thereof;

(4.) The [petitioner] ERRED in imposing real property taxes against the petitioner for roads, bridges, culverts, pipes and canals as these
belonged to the cooperatives;

([5].) The [petitioner] ERRED in finding that the Market Value of NDC service area is P11,000.00 per hectare when it should only be
P6,000.00 per hectare;

([6].) The [petitioner] ERRED in imposing realty taxes on Residential areas built by [respondent] except for three of them;
([7].) The [petitioner] ERRED when it included haulers and other equipments [sic] which are unmovable as taxable real properties.14

In its Decision15 dated June 8, 1999, the LBAA found that the P207.00 market value declared in the assessment by the Provincial
Assessor was unreasonable.16 It found that the market value should not have been more than P85.00 per oil palm tree.17 The sudden
increase of realty tax assessment level from P42.00 for each oil palm tree in 1993 to P207.00 was confiscatory.18chanrobleslaw

The LBAA adopted Filipinas' claim that the basis for assessment should only be 98 trees.19 Although one (1) hectare of land can
accommodate 124 oil palm trees, the mountainous terrain of the plantation should be considered.20 Because of the terrain, not every
meter of land can be fully planted with trees.21 The LBAA found that roads of any kind, as well as all their improvements, should not be
taxed since these roads were intermittently used by the public.22 It resolved that the market valuation should be based on the laws of the
Department of Agrarian Reform since the area is owned by the NDC, a quasi-governmental body of the Philippines.23chanrobleslaw

The LBAA exempted the low-cost housing units from taxation except those with a market value of more than P150,000.00 under the
Local Government Code.24 Finally, the LBAA considered the road equipment and mini haulers as movables that are vital to Filipinas'
business.

Filipinas appealed before the CBAA on July 16, 1999.26 On November 21, 2001, the CBAA rendered a decision, the dispositive portion
of which reads:

WHEREFORE, this Board has decided to set aside, as it does hereby set aside, the decision rendered by the Local Board of Assessment
Appeals of the Province of Agusan del Sur on June 8, 1999 in an unnumbered case entitled "[F]ilipinas Palm Oil Co., Inc. Petitioner,
versus the Provincial Assessors Office of Agusan del Sur, Respondent" and hereby orders as follows:

A. The market value for each oil palm tree should be FIFTY- SEVEN & 55/100 PESOS (57.55), effective January 1, 1991. The assessment
for each municipality shall be based on the corresponding number of trees as listed in Petitioner-Appellee's "Hectarage Statement"
discussed hereinabove;

B. Petitioner-Appellee should not be made to pay for the real property taxes due on the roads starting from January 1, 1991;

C. Petitioner-Appellee is not liable to the Government for real property taxes on the lands owned by the Multi-purpose Cooperative;

D. The housing units with a market value of PI75,000.00 or less each shall be subjected to 0% assessment level, starting 1994;

E. Road Equipment and haulers are not real properties and, accordingly, Petitioner-Appellee is not liable for real property tax thereon;

F. Any real property taxes already paid by Petitioner-Appellee which, by virtue "of this decision, were not due, shall be applied to future
taxes rightfully due from Petitioner-Appellee.

SO ORDERED.

The CBAA denied the Motion for Reconsideration filed by the Provincial Assessor.28 The Provincial Assessor filed a Petition for Review
before the Court of Appeals, which, in turn, sustained the CBAA's Decision.

The Court of Appeals held that the land owned by NGPI-NGEI, which Filipinas has been leasing, cannot be subjected to real property tax
since these are owned by cooperatives that are tax-exempt.30 Section 133(n) of the Local Government Code provides:

SECTION 133. Common Limitations on the Taxing Powers of Local Government Units. — Unless otherwise provided herein, the exercise
of the taxing powers of provinces, cities, municipalities, and barangays shall not extend to the levy of the following:
....

(n) Taxes, fees, or charges, on Countryside and Barangay Business Enterprises and cooperatives duly registered under R.A. No. 6810
and Republic Act Numbered Sixty-nine hundred thirty-eight (R.A. No. 6938) otherwise known as the "Cooperative Code of the Philippines."
(Emphasis supplied)

Section 234(d) of the Local Government Code exempts duly registered cooperatives, like NGPI-NGEI, from payment of real property
taxes:

SECTION 234. Exemptions from Real Property Tax. — The following are exempted from payment of the real property tax:
....

(d) All real property owned by duly registered cooperatives as provided for under R.A. No. 6938[.] (Emphasis supplied)

The Court of Appeals held that the pertinent provisions "neither distinguishes nor specifies" that the exemption only applies to real
properties used by the cooperatives.31 It ruled that "[t]he clear absence of any restriction or limitation in the provision could only mean
that the exemption applies to wherever the properties are situated and to whoever uses them."32 Therefore, the exemption privilege
extends to Filipinas as the cooperatives' lessee.33chanrobleslaw
On the roads constructed by Filipinas, the Court of Appeals held that although it is undisputed that the roads were built pri marily for
Filipinas' benefit, the roads should be tax-exempt since these roads were also being used by the cooperatives and the public.34 It applied,
by analogy, Bislig Bay Lumber Company, Inc. v. Provincial Government of Surigao:

We are inclined to uphold the theory of appellee. In the first place, it cannot be disputed that the ownership of the road that was constructed
by appellee belongs to the government by right accession not only because it is inherently incorporated or attached to the timber land
leased to appellee but also because upon the expiration of the concession, said road would ultimately pass to the national government.
In the second place, while the road was constructed by appellee primarily for its use and benefit, the privilege is not exclusive, for, under
the lease contract entered into by the appellee and the government and by public in by the general. Thus, under said lease contract,
appellee cannot prevent the use of portions, of the concession for homesteading purposes. It is also in duty bound to allow the free use
of forest products within the concession for the personal use of individuals residing in or within the vicinity of the land. . . . In other words,
the government has practically reserved the rights to use the road to promote its varied activities. Since, as above shown, the road in
question cannot be considered as an improvement which belongs to appellee, although in part is for its benefit, it is clear that the same
cannot be the subject of assessment within the meaning of section 2 of Commonwealth Act No. 470.36 (Citations omitted)

Furthermore, the Court of Appeals agreed with the CBAA that the roads constructed by Filipinas had become permanent improvements
on the land owned by NGPI-NGEI.37 Articles 440 and 445 of the Civil Code provide that these improvements redound to the benefit of
the land owner under the right of accession:

Article 440. The ownership of property gives the right by accession to everything which is produced thereby, or which is incorporated or
attached thereto, either naturally or artificially.
....

Article 445. Whatever is built, planted or sown on the land of another and the improvements or repairs made thereon, belong to the owner
of the land, subject to the provisions of the following articles.

On the road equipment and mini haulers as real properties subject to tax, the Court of Appeals affirmed the CBAA's Decision that these
are only movables.39 Section 199(o) of the Local Government Code provides a definition of machinery subject to real property taxation:

SECTION 199. Definition of Terms. — When used in this Title, the term:
....

(o) "Machinery" embraces machines, equipment, mechanical contrivances, instruments, appliances or apparatus which may or may not
be attached, permanently or temporarily, to the real property. It includes the physical facilities for production, the installations and
appurtenant service facilities, those which are mobile, self-powered or self-propelled, and those not permanently attached to the real
property which are actually, directly, and exclusively used to meet the needs of the particular industry, business or activity and which by
their very nature and purpose are designed for, or necessary to its manufacturing, mining.

The Court of Appeals held that Section 19^(o) of the Local Government Code should be construed to include machineries covered by the
meaning of real properties provided for under Article 415(5) of the Civil Code:40chanrobleslaw

Article 415. The following are immovable property:


....
(5) Machinery, receptacles, instruments or implements intended by the owner of the tenement for an industry or works which may be
carried on in a building or on a piece of land, and which tend directly to meet the needs of the said industry or works[.]

The Court of Appeals cited Davao Sawmill Company v. Castillo,41 where it has been held that machinery that is movable by nature
becomes immobilized only when placed by the owner of the tenement, but not so when placed by a tenant or any other person having a
temporary right unless this person acts as an agent of the owner.42 Thus, the mini haulers and other road equipment retain their nature
as movables.43chanrobleslaw

The Provincial Assessor filed before this Court a Petition for Review raising the following issues:

First, whether the exemption privilege of NGPI-NGEI from payment of real property tax extends to respondent Filipinas Palm Oil Plantation
Inc. as lessee of the parcel of land owned by cooperatives; and

Second, whether respondent's road equipment and mini haulers are movable properties and have not been immobilized by destination
for real property taxation.

Petitioner argues that based on Mactan Cebu International Airport Authority v. Ferdinand J. Marcos,44 cooperatives cannot extend its
exemption from real property tax to taxable persons.45 It argues that Sections 198, 199, 205, and 217 of the Local Government Code
provide that real property taxes are assessed based on actual use.46 Moreover, the exemption of cooperatives applies only when it is
the cooperative that actually, directly, and exclusively uses and possesses the properties.47 Sections 198, 199, 205, and 217 of the Local
Government Code provide:

SECTION 198. Fundamental Principles. — The appraisal, assessment, levy and collection of real property tax shall be guided by the
following fundamental principles:
....
(b) Real property shall be classified for assessment purposes on the basis of its actual use[.]
....
SECTION 199. Definition of Terms. — When used in this Title, the term:
....
(b) "Actual Use" refers to the purpose for which the property is principally or predominantly utilized by the person in possession thereof[.]
....
SECTION 205. Listing of Real Property in the Assessment Rolls. —
....
(d) Real property owned by the Republic of the Philippines, its instrumentalities and political subdivisions, the beneficial use of which has
been granted, for consideration or otherwise, to a taxable person, shall be listed, valued and assessed in the name of the possessor,
grantee or of the public entity if such property has been acquired or held for resale or lease.
....

SECTION 217. Actual Use of Real Property as Basis for Assessment. — Real property shall be classified, valued and assessed on the
basis of its actual use regardless of where located, whoever owns it, and whoever uses it. (Emphasis supplied)

Petitioner claims that Section 199(o) of the Local Government Code specifically covers respondent's road equipment and mini haulers
since these are directly and exclusively used to meet the needs of respondent's industry, business, or activity.48 Article 415(5) of the Civil
Code, which defines real property, should not be made to control the Local Government Code,49 a subsequent legislation that specifically
defines "machinery" for taxation purposes.

In the Resolution51 dated October 13, 2008, this Court denied the Petition for Review due to procedural missteps, which included the
failure to attach legible duplicate original or certified true copies of the assailed decision and failure to pay proper fees. On November 25,
2008, petitioner moved for reconsideration,52 praying for the reversal of the Petition's denial due to mere technicalities.

On January 26, 2009, this Court granted Petitioner's Motion for Reconsideration.53 It directed the reinstatement of the Petition and
required respondent to comment.

On November 20, 2009, respondent filed its Comment.

Respondent reiterates the rulings of the CBAA and the Court of Appeals that the exemption of cooperatives from real property taxes
extends to it as the lessee.56 It asserts that under its lease agreement with NGPI-NGEI, it pays an Annual Fixed Rental, which includes
the payment of taxes.57 It claims that in case NGPI-NGEI is liable to the local government for real property tax on the land, the tax should
be taken from the Annual Fixed Rental.58 To make respondent pay real property taxes on the leased land would be equivalent to
assessing it twice for the same property.59chanrobleslaw

On the road equipment and mini haulers being subjected to real property taxation, respondent maintains that it should be spared from
real property tax since the equipment and mini haulers are movables.60chanrobleslaw

The Petition is granted to modify the Court of Appeals Decision, but only with respect to the nature of respondent's road equipment and
mini haulers.

Under Section 133(n) of the Local Government Code, the taxing power of local government units shall not extend to the levy of taxes,
fees, or charges on duly registered cooperatives under the Cooperative Code.61 Section 234(d) of the Local Government Code
specifically provides for real property tax exemption to cooperatives:

SECTION 234. Exemptions from Real Property Tax. — The following are exempted from payment of the real property tax:
....

(d) All real property owned by duly registered cooperatives as provided for under [Republic Act] No. 6938[.] (Emphasis supplied)

NGPI-NGEI, as the owner of the land being leased by respondent, falls within the purview of the law. Section 234 of the Local Government
Code exempts all real property owned by cooperatives without distinction. Nothing in the law suggests that the real property tax exemption
only applies when the property is used by the cooperative itself. Similarly, the instance that the real property is leased to either an
individual or corporation is not a ground for withdrawal of tax exemption.

In arguing the first issue, petitioner hinges its claim on a misplaced reliance in Mactan, which refers to the revocation of tax exemption
due to the effectivity of the Local Government Code. However, Mactan does not refer to the tax exemption extended to cooperatives. The
portion that petitioner cited specifically mentions that the exemption granted to cooperatives has not been withdrawn by the effectivity of
the Local Government Code:

[S]ection 232 must be deemed to qualify Section 133.

Thus, reading together Sections 133, 232, and 234 of the L[ocal] G[overnment] C[ode], we conclude that as a general rule, as laid down
in Section 133, the taxing powers of local government units cannot extend to the levy of, inter alia, "taxes, fees and charges of any kind
on the National Government, its agencies and instrumentalities, and local government units"; however, pursuant to Section 232,
provinces, cities, and municipalities in the Metropolitan Manila Area may impose the real property tax except on, inter alia, "real property
owned by the Republic of the Philippines or any of its political subdivisions except when the beneficial use thereof has been granted, for
consideration or otherwise, to a taxable person," as provided in item (a) of the first paragraph of Section 234.
As to tax exemptions or incentives granted to or presently enjoyed by natural or juridical persons, including government -owned and
controlled corporations, Section 193 of the L[ocal] G[overnment] C[ode] prescribes the general rule, viz., they are withdrawn upon the
effectivity of the L[ocal] G[overnment] C[ode], except those granted to local water districts, cooperatives duly registered under R.A. No.
6938, non-stock and non-profit hospitals and educational institutions, and unless otherwise provided in the L[ocal] Gfovernment] C[ode].
The latter proviso could refer to Section 234 which enumerates the properties exempt from real property tax. But the last paragraph of
Section 234 further qualifies the retention of the exemption insofar as real property taxes are concerned by limiting the retention only to
those enumerated therein; all others not included in the enumeration lost the privilege upon the effectivity of the L[ocal] G[overnment]
C[ode]. Moreover, even as to real property owned by the Republic of the Philippines or any of its political subdivisions covered by item
(a) of the first paragraph of Section 234, the exemption is withdrawn if the beneficial use of such property has been granted to a taxable
person for consideration or otherwise.

Since the last paragraph of Section 234 unequivocally withdrew, upon the effectivity of the L[ocal] G[overnment] C[ode], exemptions from
payment of real property taxes granted to natural or juridical persons, including government-owned or controlled corporations, except as
provided in the said section, and the petitioner is, undoubtedly, a government-owned corporation, it necessarily follows that its exemption
from such tax granted it in Section 14 of its Charter, R.A. No. 6958, has been withdrawn. Any claim to the contrary can only be justified if
the petitioner can seek refuge under any of the exceptions provided in Section 234, but not under Section 133, as it now asserts, since,
as shown above, the said section is qualified by Sections 232 and 234.

In short, the petitioner can no longer invoke the general rule in Section 133 that the taxing powers of the local government units cannot
extend to the levy of:

(o) taxes, fees or charges of any kind on the National Government, its agencies or instrumentalities, and local government units.

It must show that the parcels of land in question, which are real property, are any one of those enumerated in Section 234, either by virtue
of ownership, character, or use of the property.63 (Emphasis supplied)

The roads that respondent constructed within the leased area should not be assessed with real property taxes. Bislig Bay finds application
here. Bislig Bay Lumber Company, Inc. (Bislig Bay) was a timber concessionaire of a portion of public forest in the provinces of Agusan
and Surigao.64 To aid in developing its concession, Bislig Bay built a road at its expense from a barrio leading towards its area.65 The
Provincial Assessor of Surigao assessed Bislig Bay with real property tax on the constructed road, which was paid by the company under
protest.66 It claimed that even if the road was constructed on public land, it should be subjected to real property tax because it was built
by the company for its own benefit.67 On the other hand, Bislig Bay asserted that the road should be exempted from real property tax
because it belonged to national government by right of accession.68 Moreover, the road constructed already became an inseparable part
of the land.69 The records also showed that the road was not only built for the benefit of Bislig Bay, but also of the public.70 This Court
ruled for Bislig Bay, thus:

We are inclined to uphold the theory of appellee. In the first place, it cannot be disputed that the ownership of the road that was constructed
by appellee belongs to the government by right accession not only because it is inherently incorporated or attached to the timber land
leased to appellee but also because upon the expiration of the concession, said road would ultimately pass to the national government.
... In the second place, while the road was constructed by appellee primarily for its use and benefit, the privilege is not exclusive, for,
under the lease contract entered into by the appellee and the government and by public in by the general. Thus, under said lease contract,
appellee cannot prevent the use of portions, of the concession for homesteading purposes. ... It is also in duty bound to allow the free
use of forest products within the concession for the personal use of individuals residing in or within the vicinity of the land. ... In other
words, the government has practically reserved the rights to use the road to promote its varied activities. Since, as above shown, the
road in question cannot be considered as an improvement which belongs to appellee, although in part is for its benefit, it is clear that the
same cannot be the subject of assessment within the meaning of section 2 of Commonwealth Act No. 470.71

This was reiterated in Board of Assessment Appeals ofZamboanga del Sur v. Samar Mining Company, Inc.72 Samar Mining Company,
Inc. (Samar Mining) was a domestic corporation engaged in the mining industry.73 Since Samar Mining's mining site and mill were in an
inland location entailing long distance from its area to the loading point, Samar Mining was constrained to construct a road for its
convenience.74 Initially, Samar Mining filed miscellaneous lease applications for a road right of way covering lands under the jurisdiction
of the Bureau of Lands and the Bureau of Forestry where the proposed road would pass through.75 Samar Mining was given a "temporary
permit to occupy and use the lands applied for by it";76 hence, it was able to build what was eventually known as the Samico Road.
Samar Mining was assessed by the Provincial Assessor of Zamboanga del Sur with real property taxes on the road, which prompted it to
appeal before the Board of Assessment Appeals.77 Invoking Bislig Bay, Samar Mining claimed that it should not be assessed with real
property tax since the road was constructed on public land. This Court ruled for Samar Mining, thus:

There is no question that the road constructed by respondent Saimar on the public lands leased to it by the government is an improvement.
But as to whether the same is taxable under the aforequoted provision of the Assessment Law, this question has already been answered
in the negaitive by this Court. In the case of Bislig Bay Lumber Co., Inc. vs. Provincial Government of Surigao, where a similar issue was
raised.
....

. . . What is emphasized in the Bislig case is that the improvement is exempt from taxation because it is an integral part of the public land
on which it is constructed and the improvement is the property of the government by right of accession. Under Section 3(a) of the
Assessment Law, all properties owned by the government, without any distinction, are exempt from taxation.79 (Emphasis supplied,
citations omitted)
The roads that respondent constructed became permanent improvements on the land owned by the NGPI-NGEI by right of accession
under the Civil Code, thus:

Article 440. The ownership of property gives the right by accession to everything which is produced thereby, or which is incorporated or
attached thereto, either naturally or artificially.
....
Article 445. Whatever is built, planted or sown on the land of another and the improvements or repairs made thereon, belong to the owner
of the land[.]

Despite the land being leased by respondent when the roads were constructed, the ownership of the improvement still belongs to NGPI-
NGEI. As provided under Article 440 and 445 of the Civil Code, the land is owned by the cooperatives at the time respondent built the
roads. Hence, whatever is incorporated in the land, either naturally or artificially, belongs to the NGPI-NGEI as the landowner.

Although the roads were primarily built for respondent's benefit, the roads were also being used by the members of NGPI and the public.80
Furthermore, the roads inured to the benefit of NGPI-NGEI as owners of the land not only by right of accession but through the express
provision in the lease agreement:

chanRoblesvirtualLawlibrary
On March 7, 1990 NGPI Multi-Purpose Cooperative, Inc., as Lessor, and NDC-Guthrie Plantations, Inc., as Lessee, entered into a "Lease
Agreement" . . . covering the agricultural lands transferred by NDC to the DAR, which lands the DAR ultimately distributed undivided to
qualified workers-beneficiaries. . . .
....

Clause No. 6.3 of the same lease agreement provides that "All taxes due on the improvements on the Leased Property except those
improvements on the Area that the LESSOR shall have utilized under Clause 1.2 hereof, shall be for the account of the LESSEE."

Clause No. 9.4 of the same lease agreement provides that ". . . All fixed and permanent improvements, such as roads and palm trees
introduced on the Leased Property, shall automatically accrue to the LESSOR upon termination of this Lease Agreement without need of
reimbursement."

All the above-cited stipulations in the lease agreement between NGPI Multi-Purpose Cooperative and NDC-Guthrie Plantations, Inc. were
reconfirmed and reaffirmed in the Addendum to Lease Agreement entered into by and between NGPI Multi-Purpose Cooperative and
Filipinas Palmoil Plantations, Inc. on January 30, 1998. . . . The main subject of the said Addendum was the extension of the term of the
lease agreement up to December 31, 2032, along with economic benefits to the lessor other than rentals.

There is no dispute that the roads are on the land owned by NGPI Multi-Purpose Cooperative which leased the same to Petitioner-
Appellee. These roads belong to the Multi-Purpose Cooperative, not only by right of accession but also by express provisions of the
Contract of Lease[.]81

Respondent claims that under its lease agreement with NGPI-NGEI, it pays an Annual Fixed Rental, which includes the payment of
taxes.82 If NGPI-NGEI were liable to the local government for real property tax on the land, the tax should be taken from the Annual
Fixed Rental:

"2.1. In consideration of this Lease Agreement, the LESSEE shall pay the LESSOR the following annual

"1) An annual fixed rental, in the following amount — "SIX HUNDRED THIRTY FIVE PESOS" (P635.00) PER HECTARE PER ANNUM
which would cover the following:

"(1) All Taxes on the Land


"(2) Administration Charges
"(3) Amortization charges

"It is understood that, if the annual fixed rental of "SIX HUNDRED THIRTY FIVE PESOS" (p 635.00) is insufficient to pay any increase
on the land taxes, the Lessee shall pay the difference, provided such increase does not exceed ten percent (10%) of the immediately
preceding tax imposed on the land; provided further, that any increase beyond these percentage shall be borne equally by the LESSOR
and LESSEE.

"The foregoing notwithstanding, it is understood and agreed that at all times, liability for realty taxes on the Leased Property Primarily and
principally lies with the LESSOR and any reference herein to payment by LESSEE of said taxes is only for purposes of earmarking the
proceeds of the rentals herein agreed upon."
Clause No. 6.3 of the same lease agreement provides that "All taxes due on the improvements on the Leased Property except those
improvements on the Area that the LESSOR shall have utilized under Clause 1.2 hereof, shall be for the account of the LESSEE."83
(Emphasis supplied)

Therefore, NGPI-NGEI, as owner of the roads that permanently became part of the land being leased by respondent, shall be liable for
real property taxes, if any. However, by express provision of the Local Government Code, NGPI-NGEI is exempted from payment of real
property tax.84chanrobleslaw

II
The road equipment and mini haulers shall be considered as real property, subject to real property tax.

Section 199(o) of the Local Government Code defines "machinery" as real property subject to real property tax,85 thus:

SECTION 199. Definition of Terms. — When used in this Title, the term:
....

(o) "Machinery" embraces machines, equipment, mechanical contrivances, instruments, appliances or apparatus which may or may not
be attached, permanently or temporarily, to the real property. It includes the physical facilities for production, the installations and
appurtenant service facilities, those which are mobile, self-powered or self-propelled, and those not permanently attached to the real
property which are actually, directly, and exclusively used to meet the needs of the particular industry, business or activity and which by
their very nature and purpose are designed for, or necessary to its manufacturing, mining, logging, commercial, industrial or agricultural
purposes[.]

Article 415(5) of the New Civil Code defines "machinery" as that which constitutes an immovable property:

Article 415. The following are immovable property:


....
(5) Machinery, receptacles, instruments or implements intended by the owner of the tenement for an industry or works which may be
carried on in a building or on a piece of land, and which tend directly to meet the needs of the said industry or works[.]

Petitioner contends that the second sentence of Section 199(o) includes the road equipment and mini haulers since these are directly
and exclusively used by respondent to meet the needs of its operations.86 It further claims that Article 415(5) of the New Civil Code
should not control the Local Government Code, a subsequent legislation.

On the other hand, respondent claims that the road equipment and mini haulers are movables by nature. It asserts that although there
may be a difference between the meaning of "machinery" under the Local Government Code arid that of immovable property under Article
415(5) of the Civil Code, "the controlling interpretation of Section 199(o) of [the Local Government Code] is the interpretation of Article
415(5) of the Civil Code."

In Manila Electric Company v. City Assessor,89 a similar issue of which definition of "machinery" prevails to warrant the assessment of
real property tax on it was raised.

Manila Electric Company (MERALCO) insisted on harmonizing the provisions of the Civil Code and the Local Government Code and
asserted that "machinery" contemplated under Section 199(o) of the Local Government must still be within the contemplation of
immovable property under Article 415 of the Civil Code.90 However, this Court ruled that harmonizing such laws "would necessarily mean
imposing additional requirements for classifying machinery as real property for real property tax purposes not provided for, or even in
direct conflict with, the provisions of the Local Government Code."91 Thus:

chanRoblesvirtualLawlibrary
While the Local Government Code still does not provide for a specific definition of "real property," Sections 199(o) and 232 of the said
Code, respectively, gives an extensive definition of what constitutes "machinery" and unequivocally subjects such machinery to real
property tax. The Court reiterates that the machinery subject to real property tax under the Local Government Code "may or may not be
attached, permanently or temporarily to the real property"; and the physical facilities for production, installations, and appurtenant service
facilities, those which are mobile, self-powered or self-propelled, or are not permanently attached must (a) be actually, directly, and
exclusively used to meet the needs of the particular industry, business, or activity; and (b) by their very nature and purpose, be designed
for, or necessary for manufacturing, mining, logging, commercial, industrial, or agricultural purposes.
....

Article 415, paragraph (5) of the Civil Code considers as immovables or real properties "[m]achinery, receptacles, instruments or
implements intended by the owner of the tenement for an industry or works which may be carried on in a building or on a piece of land,
and which tend directly to meet the needs of the said industry or works." The Civil Code, however, does not define "machinery."

The properties under Article 415, paragraph (5) of the Civil Code are immovables by destination, or "those which are essentially movables,
but by the purpose for which they have been placed in an immovable, partake of the nature of the latter because of the added utility
derived therefrom." These properties, including machinery, become immobilized if the following requisites concur: (a) they are placed in
the tenement by the owner of such tenement; (b) they are destined for use in the industry or work in the tenement; and (c) they tend to
directly meet the needs of said industry or works. The first two requisites are not found anywhere in the Local Government Code.92
(Emphasis supplied, citations omitted)

Section 199(o) of the Local Government prevails over Article 415(5) of the Civil Code. In Manila Electric Company:

As between the Civil Code, a general law governing property and property relations, and the Local Government Code, a special law
granting local government units the power to impose real property tax, then the latter shall prevail. As the Court pronounced in
Disomangcop v. The Secretary of the Department of Public Works and Highways Simeon A. Datumanong:

It is a finely-imbedded principle in statutory construction that a special provision or law prevails over a general one. Lex specialis derogant
generali. As this Court expressed in the case of Leveriza v. Intermediate Appellate Court, "another basic principle of statutory construction
mandates that general legislation must give way to special legislation on the same subject, and generally be so interpreted as to embrace
only cases in which the special provisions are not applicable, that specific statute prevails over a general statute and that where two
statutes are of equal theoretical application to a particular case, the one designed therefor specially should prevail."

The Court also very clearly explicated in Vinzons-Chato v. Fortune Tobacco Corporation that:

A general law and a special law on the same subject are statutes in pari materia and should, accordingly, be read together and
harmonized, if possible, with a view to giving effect to both. The rule is that where there are two acts, one of which is special and particular
and the other general which, if standing alone, would include the same matter and thus conflict with the special act, the special law must
prevail since it evinces the legislative intent more clearly than that of a general statute and must not be taken as intended to affect the
more particular and specific provisions of the earlier act, unless it is absolutely necessary so to construe it in order to give its words any
meaning at all.

The circumstance that the special law is passed before or after the general act does not change the principle. Where the special law is
later, it will be regarded as an exception to, or a qualification of, the prior general act; and where the general act is later, the special statute
will be construed as remaining an exception to its terms, unless repealed expressly or by necessary implication.
Furthermore, in Caltex (Philippines), Inc. v. Central Board of Assessment Appeals, the Court acknowledged that "[i]t is a familiar
phenomenon to see things classed as real property for purposes of taxation which on general principle might be considered personal
property[.]"

Therefore, for determining whether machinery is real property subject to real property tax, the definition and requirements under the Local
Government Code are controlling.

Respondent is engaged in palm oil plantation.94 Thus, it harvests fruits from palm trees for oil conversion through its milling plant.95 By
the nature of respondent's business, transportation is indispensable for its operations.

Under the definition provided in Section 199(o) of the Local Government Code, the road equipment and the mini haulers are classified as
machinery, thus:

SECTION 199. Definition of Terms. — When used in this Title, the terra:
....

(o) "Machinery" . . . includes the physical facilities for production, the installations and appurtenant service facilities, those which are
mobile, self-powered or self-propelled, and those not permanently attached to the real property which are actually, directly, and exclusively
used to meet the needs of the particular industry, business or activity and which by their very nature and purpose are designed for, or
necessary to its manufacturing, mining, logging, commercial, industrial or agricultural purposes [.]

Petitioner is correct in claiming that the phrase pertaining to physical facilities for production is comprehensive enough to include the road
equipment and mini haulers as actually, directly, and exclusively used by respondent to meet the needs of its operations in palm oil
production.96 Moreover, "mini-haulers are farm tractors pulling attached trailers used in the hauling of seedlings during planting season
and in transferring fresh palm fruits from the farm [or] field to the processing plant within the plantation area."97 The indispensability of
the road equipment and mini haulers in transportation makes it actually, directly, and exclusively used in the operation of respondent's
business.

In its Comment, respondent claims that the equipment is no longer vital to its operation because it is currently employing equipment
outside the company to do the task.98 However, respondent never raised this contention before the lower courts. Hence, this is a factual
issue of which this Court cannot take cognizance. This Court is not a trier of facts. Only questions of law are entertained in a petition for
review assailing a Court of Appeals decision.

WHEREFORE, the Petition is PARTLY GRANTED.

MERALCO VS. CITY ASSESSOR

FACTS:

Before the Court is a Petition for Review on Certiorari seeking the reversal of the Decision1 dated May 13, 2004 and Resolution dated
November 18, 2004 of the Court of Appeals in CA-G.R. SP No. 67027. The appellate court affirmed the Decision3 dated May 3, 2001 of
the Central Board of Assessment Appeals (CBAA) in CBAA Case No. L-20-98, which, in turn, affirmed with modification the Decision4
dated June 17, 19985 of the Local Board of Assessment Appeals (LBAA) of Lucena City, Quezon Province, as regards Tax Declaration
Nos. 019-6500 and 019-7394, ruling that MERALCO is liable for real property tax on its transformers, electric posts (or poles), transmission
lines, insulators, and electric meters, beginning 1992.

MERALCO failed to persuade the Court of Appeals that the transformers, transmission lines, insulators, and electric meters mounted on
the electric posts of MERALCO were not real properties. The appellate court invoked the definition of "machinery" under Section 199(o)
of the Local Government Code and then wrote that:

We firmly believe and so hold that the wires, insulators, transformers and electric meters mounted on the poles of [MERALCO] may
nevertheless be considered as improvements on the land, enhancing its utility and rendering it useful in distributing electricity. The said
properties are actually, directly and exclusively used to meet the needs of [MERALCO] in the distribution of electricity.
In addition, "improvements on land are commonly taxed as realty even though for some purposes they might be considered person alty.
It is a familiar personalty phenomenon to see things classed as real property for purposes of taxation which on general principle might be
considered personal property."

Issue: Whether or not the transformers, electric posts (or poles), transmission lines, insulators, and electric meters are real properties.
Held: While the Local Government Code still does not provide for a specific definition of "real property," Sections 199(o) and 232 of the
said Code, respectively, gives an extensive definition of what constitutes "machinery" and unequivocally subjects such machinery to real
property tax. The Court reiterates that the machinery subject to real property tax under the Local Government Code "may or may not be
attached, permanently or temporarily to the real property;" and the physical facilities for production, installations, and appurtenant service
facilities, those which are mobile, self-powered or self-propelled, or are not permanently attached must (a) be actually, directly, and
exclusively used to meet the needs of the particular industry, business, or activity; and (2) by their very nature and purpose, be designed
for, or necessary for manufacturing, mining, logging, commercial, industrial, or agricultural purposes.

Article 415, paragraph (1) of the Civil Code declares as immovables or real properties "[l]and, buildings, roads and constructions of all
kinds adhered to the soil." The land, buildings, and roads are immovables by nature "which cannot be moved from place to place,"
whereas the constructions adhered to the soil are immovables by incorporation "which are essentially movables, but are attached to an
immovable in such manner as to be an integral part thereof."57 Article 415, paragraph (3) of the Civil Code, referring to "[ejverything
attached to an immovable in a fixed manner, in such a way that it cannot be separated therefrom without breaking the material or
deterioration of the object," are likewise immovables by incorporation. In contrast, the Local Government Code considers as real property
machinery which "may or may not be attached, permanently or temporarily to the real property," and even those which are "mobile."

Article 415, paragraph (5) of the Civil Code considers as immovables or real properties "[machinery, receptacles, instruments or
implements intended by the owner of the tenement for an industry or works which may be carried on in a building or on a piece of land,
and which tend directly to meet the needs of the said industry or works." The Civil Code, however, does not define "machinery."

The properties under Article 415, paragraph (5) of the Civil Code are immovables by destination, or "those which are essentially movables,
but by the purpose for which they have been placed in an immovable, partake of the nature of the latter because of the added utility
derived therefrom."58 These properties, including machinery, become immobilized if the following requisites concur: (a) they are placed
in the tenement by the owner of such tenement; (b) they are destined for use in the industry or work in the tenement; and (c) they tend to
directly meet the needs of said industry or works.59 The first two requisites are not found anywhere in the Local Government Code.

Furthermore, in Caltex (Philippines), Inc. v. Central Board of Assessment Appeals,62 the Court acknowledged that "[i]t is a familiar
phenomenon to see things classed as real property for purposes of taxation which on general principle might be considered personal
property[.]" Therefore, for determining whether machinery is real property subject to real property tax, the definition and requirements
under the Local Government Code are controlling.

REPUBLIC VS. RURAL BANK OF KABACAN

FACTS:

NIA is a government-owned-and-controlled corporation specifically authorized under P.D. 552 to exercise the power of eminent domain.

NIA needed some parcels of land for the purpose of constructing the Malitubog-Marigadao Irrigation Project. On 08 September 1994, it
filed with the RTC of Kabacan, Cotabato a Complaint for the expropriation of a portion of three (3) parcels of land covering a total of
14,497.91... square meters.

On 11 July 1995, NIA filed an Amended Complaint to include Leosa Nanette A. Agdeppa and Marcelino Viernes

NIA filed a Second Amended Complaint to allege properly the area sought to be expropriated, the exact address of the expropriated
properties and the owners thereof. NIA further prayed that it be authorized to take immediate possession of the properties... after
depositing with the Philippine National Bank the amount of ?19,246.58 representing the provisional value thereof.

respondents filed their Answer with Affirmative and Special Defenses and Counterclaim.[6] They alleged, inter alia, that NIA had no
authority to expropriate portions of their land, because it was not a sovereign political entity;... that it was not necessary to expropriate
their properties, because there was an abandoned government property adjacent to theirs, where the project could pass through; that Lot
No. 3080 was no longer owned by the Rural Bank of Kabacan; that NIA's valuation of their expropriated... properties was inaccurate
because of the improvements on the land that should have placed its value at ?5 million; and that NIA never negotiated with the
landowners before taking their properties for the project, causing permanent and irreparable damages to their properties... valued at
?250,000

On 11 September 1996, the RTC issued an Order forming a committee tasked to determine the fair market value of the expropriated...
properties to establish the just compensation to be paid to the owners. The committee was composed of the Clerk of Court of RTC Branch
22 as chairperson and two (2) members of the parties to the case.

the lower court issued an Order stating it would issue a writ of possession in favor of NIA upon the determination of the fair market value
of the properties, subject of the expropriation proceedings.[10] The lower court later... amended its ruling and, on 21 October 1996, issued
a Writ of Possession in favor of NIA.
On 15 October 1996, the committee submitted a Commissioners' Report[12] to the RTC... stated that the committee members could not
agree on the market value of the subject properties and recommended the appointment of new independent commissioners to replace
the ones coming from the parties only.

the RTC... appointed Renato Sambrano, Assistant Provincial Assessor of the Province of Cotabato; and Jack Tumacmol, Division Chief...
of the Land Bank of the Philippines-Kidapawan Branch.

The committee had agreed that the fair market value of the land to be expropriated should be ?65 per square meter based on the zonal
valuation of the Bureau of Internal Revenue (BIR).

On 03 December 1997, the committee submitted to the RTC another report

[18] Petitioner objected to the inclusion of the value of the excavated soil in the computation of the value of the land... the RTC promulgated
its "Judgment,... the court finds and so holds that the commissioners have arrived at and were able to determine the fair market value of
the properties. The court adopts their findings, and orders

NIA assailed the trial court's adoption of the Commissioners' Report, which had determined the just compensation to be awarded to... the
owners of the lands expropriated. NIA also impugned as error the RTC's inclusion for compensation of the excavated soil from the
expropriated properties. Finally, it disputed the trial court's Order to deliver the payment intended for the Rural Bank of Kabacan to...
defendants-intervenors, who allegedly acquired ownership of the land still titled in the name of the said rural bank.

the CA... affirming with modification the RTC Decision.

deleted the inclusion of the value of the soil excavated from the properties in the just compensation. It ruled that the property owner was
entitled to compensation only for the value of the property at the time of the taking.[

Thus, it concluded that NIA, as the new owner of the affected properties, had the right to... enjoy and make use of the property, including
the excavated soil, pursuant to the latter's objectives... the CA affirmed the trial court's ruling that recognized defendants-intervenors
Margarita Tabaoda and Portia Charisma Ruth Ortiz as the new owners of Lot No. 3080 and held that they were thus entitled to just
compensation. The appellate court based its conclusion... on the non-participation by the Rural Bank of Kabacan in the expropriation
proceedings and the latter's Manifestation that it no longer owned Lot No. 3080

ISSUES:

THE COURT OF APPEALS SERIOUSLY ERRED IN AFFIRMING THE TRIAL COURT'S FINDING OF JUST COMPENSATION OF THE
LAND AND THE IMPROVEMENTS THEREON BASED ON THE REPORT OF THE COMMISSIONERS.

THE COURT OF APPEALS ERRED IN RULING THAT THE PAYMENT OF JUST COMPENSATION FOR LOT NO. 3080 SHOULD BE
MADE TO RESPONDENTS MARGARITA TABOADA AND PORTIA CHARISMA RUTH ORTIZ.[31]

RULING:

On the first issue, the Petition is not meritorious.

The constitutional limitation of "just compensation" is considered to be a sum equivalent to the... market value of the property, broadly
defined as the price fixed by the seller in open market in the usual and ordinary course of legal action and competition; or the fair value
of the property; as between one who receives and one who desires to sell it, fixed at the time of the... actual taking by the government

In the instant case, we affirm the appellate court's ruling that the commissioners properly determined the just compensation to be awarded
to the landowners whose properties were expropriated by petitioner.

The records show that the trial court dutifully followed the procedure under Rule 67 of the 1997 Rules of Civil Procedure when it formed
a committee that was tasked to determine the just compensation for the expropriated properties. The first set of committee members
made an... ocular inspection of the properties, subject of the expropriation. They also determined the exact areas affected, as well as the
kinds and the number of improvements on the properties.[34] When the members were unable to agree on the valuation of the lan d...
and the improvements thereon, the trial court selected another batch of disinterested members to carry out the task of determining the
value of the land and the improvements.

The new committee members even made a second ocular inspection of the expropriated areas. They also obtained data from the BIR to
determine the zonal valuation of the expropriated properties, interviewed the adjacent property owners, and considered other factors such
as distance... from the highway and the nearby town center.[35] Further, the committee members also considered Provincial Ordinance
No. 173, which was promulgated by the Province of Cotabato on 15 June 1999, and which provide for the value of the properties and
the... improvements for taxation purposes.

In National Power Corporation v. Diato-Bernal,[37] this Court emphasized that the "just"-ness of the compensation could only be attained
by using reliable and actual data as bases for fixing the value of the condemned property.

In the instant case, the committee members based their recommendations on reliable data and, as aptly noted by the appellate court,
considered various factors that affected the value of the land and the improvements.
We note that petitioner had ample opportunity to rebut the testimonial, as well as documentary evidence presented by respondents when
the case was still on trial. It failed to do so, however.

Moreover, factual findings of the CA are generally binding on this Court. The rule admits of exceptions, though, such as when the factual
findings of the appellate court and the trial court are contradictory, or when the findings are not supported by the evidence on... record.[42]
These exceptions, however, are not present in the instant case.

Thus, in the absence of contrary evidence, we affirm the findings of the CA, which sustained the trial court's Decision adopting the
committee's recommendations on the just compensation to be awarded to herein respondents.

We also uphold the CA ruling, which deleted the inclusion of the value of the excavated soil in the payment for just compensation. There
is no legal basis to separate the value of the excavated soil from that of the expropriated properties, contrary to what the trial court did.

In the context of expropriation proceedings, the soil has no value separate from that of the expropriated land. Just compensation ordinarily
refers to the value of the land to compensate for what the owner actually loses. Such value could only be that which prevailed at... the
time of the taking.

In National Power Corporation v. Ibrahim, et al.,[43] we held that rights over lands are indivisible

On the second issue, the Petition is meritorious.

The CA affirmed the ruling of the trial court, which had awarded the payment of just compensation - intended for Lot No. 3080 registered
in the name of the Rural Bank of Kabacan - to the defendants-intervenors on the basis of the non-participation of the rural bank in the...
proceedings and the latter's subsequent Manifestation that it was no longer the owner of that lot. The appellate court erred on this matte...
me of the Rural Bank of Kabacan - to the defendants-intervenors on the basis of the non-participation of the rural bank in the...
proceedings and the latter's subsequent Manifestation that it was no longer the owner of that lot. The appellate court erred on this matter.

It should be noted that eminent domain cases involve the expenditure of public funds.[45] In this kind of proceeding, we require trial courts
to be more circumspect in their evaluation of the just compensation to be awarded to the owner of the expropriated... property.[46] Thus,
it was imprudent for the appellate court to rely on the Rural Bank of Kabacan's mere declaration of non-ownership and non-participation
in the expropriation proceeding to validate defendants-intervenors' claim of entitlement to that... payment.

The law imposes certain legal requirements in order for a conveyance of real property to be valid. It should be noted that Lot No. 3080 is
a registered parcel of land covered by TCT No. T-61963. In order for the reconveyance of real property to be valid, the conveyance...
must be embodied in a public document[47] and registered in the office of the Register of Deeds where the property is situated.

We have scrupulously examined the records of the case and found no proof of conveyance or evidence of transfer of ownership of Lot
No. 3080 from its registered owner, the Rural Bank of Kabacan, to defendants-intervenors.

The trial court should have been guided by Rule 67, Section 9 of the 1997 Rules of Court, which provides thus:

SEC. 9. Uncertain ownership; conflicting claims. -- If the ownership of the property taken is uncertain, or there are conflicting claims to
any part thereof, the court may order any sum or sums awarded as compensation for the property to be paid to the court for the... benefit
of the person adjudged in the same proceeding to be entitled thereto. But the judgment shall require the payment of the sum or sums
awarded to either the defendant or the court before the plaintiff can enter upon the property, or retain it for the public use or purpose... if
entry has already been made.

Hence, the appellate court erred in affirming the trial court's Order to award payment of just compensation to the defendants-intervenors.
There is doubt as to the real owner of Lot No. 3080. Despite the fact that the lot was covered by TCT No. T-61963 and was registered...
under its name, the Rural Bank of Kabacan manifested that the owner of the lot was no longer the bank, but the defendants-intervenors;
however, it presented no proof as to the conveyance thereof. In this regard, we deem it proper to remand this case to the trial court for
the... reception of evidence to establish the present owner of Lot No. 3080 who will be entitled to receive the payment of just compensation.

LUIS MARCOS P. LAUREL v. ZEUS C. ABROGAR

FACTS:

Petitioner is one of the accused in Criminal Case... unlawfully and feloniously take, steal and use the international long distance calls
belonging to PLDT by conducting International Simple Resale (ISR)

Petitioner filed a "Motion to Quash... on the ground that the factual allegations in the Amended Information do not constitute the felony of
theft.

The trial court denied the Motion to Quash

Petitioner's special civil action for certiorari was dismissed by the Court of Appeals.
this Court held that the Amended Information does not contain material allegations charging petitioner with theft of personal property
since international long distance calls and the business of providing telecommunication or telephone services are... not personal
properties under Article 308 of the Revised Penal Code.

Respondent

PLDT... filed a Motion for Reconsideration with Motion to Refer the Case to the Supreme Court En Banc.

PLDT further insists that the Revised Penal Code should be interpreted in the context of the Civil Code's definition of real and personal
property.

The enumeration of real properties in Article 415 of the Civil Code is exclusive such that all those not included therein are... personal
properties. Since Article 308 of the Revised Penal Code used the words "personal property" without qualification, it follows that all
"personal properties" as understood in the context of the Civil Code, may be the subject of theft under Article 308 of the Revised Penal

Code.

PLDT alleges that the international calls and business of providing telecommunication or telephone service are personal properties
capable of appropriation and can be objects of theft.

According to respondent, the "international phone calls"... are personal properties which may be subject of theft.

Article 416(3)... of the Civil Code deems "forces of nature" (which includes electricity) which are brought under the control by science, are
personal property.

petitioner Laurel claims that a telephone call is a conversation on the phone or a communication carried out using the telephone.

It is not synonymous to electric current or impulses. Hence, it may not be considered as... personal property susceptible of appropriation.

Petitioner claims that the analogy between generated electricity and telephone calls is misplaced. PLDT does not produce or generate
telephone calls. It only provides the facilities or services for the transmission and switching of... the calls.

He also insists that "business" is not personal property.

Since the services of PLDT cannot be considered as "property," the same may not... be subject of theft.

The Office of the Solicitor General (OSG) agrees with respondent PLDT

ISSUES:

whether or not "the unauthorized use or appropriation of PLDT international telephone calls, service and facilities... constitutes... theft.

RULING:

We resolve to grant the Motion for Reconsideration but remand the case to the trial court for proper clarification of the Amended
Information.

Article 308 of the Revised Penal Code provides:

Art. 308. Who are liable for theft. - Theft is committed by any person who, with intent to gain but without violence against, or intimidation
of persons nor force upon things, shall take personal property of another without the latter's... consent.

The elements of theft under Article 308 of the Revised Penal Code are as follows: (1) that there be taking of personal property; (2) that
said property belongs to another; (3) that the taking be done with intent to gain; (4) that the taking be done without the consent of the...
owner; and (5) that the taking be accomplished without the use of violence against or intimidation of persons or force upon things.

Prior to the passage of the Revised Penal Code on December 8, 1930, the definition of the term "personal property" in the penal code
provision on theft had been established in Philippine jurisprudence.

This Court consistently ruled that any personal property, tangible or intangible, corporeal or incorporeal, capable of appropriation can be
the object of theft.

the term "personal property" in the Revised Penal Code should be interpreted in the context of the Civil Code provisions in accordance
with the rule on statutory construction

In fact, this Court used the Civil Code definition of "personal property" in interpreting the theft provision of the penal code in United States
v. Carlos.
any property which is not included in the... enumeration of real properties under the Civil Code and capable of appropriation can be the
subject of theft under the Revised Penal Code.

The only requirement for a personal property to be the object of theft under the penal code is that it be capable of appropriation. It need
not be capable of "asportation," which is defined as "carrying away."

To appropriate means to deprive the lawful owner of the thing.

appropriation of forces of nature which are brought under control by science such as electrical energy can be achieved by tampering with
any apparatus used for generating or measuring such forces of nature... the Court declared in Genato that ownership over electricity...
as well as telephone service, is protected by the provisions on theft of the Penal Code.

The business of providing telecommunication or telephone service is likewise personal property which can be the object of theft under
Article 308 of the Revised Penal Code.

Business may be appropriated under Section 2 of Act No. 3952 (Bulk Sales Law), hence, could be object of... theft... petitioner's acts
constitute theft of respondent PLDT's business and service, committed by means of the unlawful use of the latter's facilities.

intangible property such as electrical energy is capable of appropriation because it may be taken and carried away. Electricity is personal
property under Article 416 (3) of the Civil Code, which enumerates "forces of nature which are brought under control... by science."

PLDT not being the owner of said telephone calls, then it could not validly claim... that such telephone calls were taken without its consent.
It is the use of these communications facilities without the consent of PLDT that constitutes the crime of theft, which is the unlawful taking
of the telephone services and business.

Therefore, the business of providing telecommunication and the telephone service are personal property under Article 308 of the Revised
Penal Code, and the act of engaging in ISR is an act of "subtraction" penalized under said article.

ACCORDINGLY, the motion for reconsideration is GRANTED. The assailed Decision dated February 27, 2006 is RECONSIDERED and
SET ASIDE

Principles:

any personal property, tangible or intangible, corporeal or incorporeal, capable of appropriation can be the object of theft.

any property which is not included in the... enumeration of real properties under the Civil Code and capable of appropriation can be the
subject of theft under the Revised Penal Code.

The only requirement for a personal property to be the object of theft under the penal code is that it be capable of appropriation. It need
not be capable of "asportation," which is defined as "carrying away."

Business may be appropriated under Section 2 of Act No. 3952 (Bulk Sales Law), hence, could be object of... theft... that intangible
property such as electrical energy is capable of appropriation because it may be taken and carried away. Electricity is personal property
under Article 416 (3) of the Civil Code, which enumerates "forces of nature which are brought under control... by science."

SERG’S PRODUCTS VS. PCI LEASING

FACTS:

"On February 13, 1998, respondent PCI Leasing and Finance, Inc. ("PCI Leasing" for short) filed with the RTC-QC a complaint for [a]
sum of money (Annex 'E'), with an application for a writ of replevin docketed as Civil Case No. Q-98-33500.

"On March 6, 1998, upon an ex-parte application of PCI Leasing, respondent judge issued a writ of replevin (Annex 'B') directing its sheriff
to seize and deliver the machineries and equipment to PCI Leasing after 5 days and upon the payment of the necessary expenses.

"On March 24, 1998, in implementation of said writ, the sheriff proceeded to petitioner's factory, seized one machinery with [the] word
that he [would] return for the other machineries.

"On March 25, 1998, petitioners filed a motion for special protective order (Annex 'C'), invoking the power of the court to control the
conduct of its officers and amend and control its processes, praying for a directive for the sheriff to defer enforcement of the writ of...
replevin.

"This motion was opposed by PCI Leasing (Annex 'F'), on the ground that the properties [were] still personal and therefore still subject to
seizure and a writ of replevin.

"In their Reply, petitioners asserted that the properties sought to be seized [were] immovable as defined in Article 415 of the Civil Code,
the parties' agreement to the contrary notwithstanding. They argued that to give effect to the agreement would be prejudicial to... innocent
third parties. They further stated that PCI Leasing [was] estopped from treating these machineries as personal because the contracts in
which the alleged agreement [were] embodied [were] totally sham and farcical.
Ruling of the Court of Appeals

Citing the Agreement of the parties, the appellate court held that the subject machines were personal property, and that they had only
been leased, not owned, by petitioners. It also ruled that the "words of the contract are clear and leave no doubt upon the true intention
of... the contracting parties." Observing that Petitioner Goquiolay was an experienced businessman who was "not unfamiliar with the
ways of the trade," it ruled that he "should have realized the import of the document he signed." The CA further held:

"Furthermore, to accord merit to this petition would be to preempt the trial court in ruling upon the case below, since the merits of the
whole matter are laid down before us via a petition whose sole purpose is to inquire upon the existence of a grave abuse of d iscretion
on... the part of the [RTC] in issuing the assailed Order and Resolution. The issues raised herein are proper subjects of a full-blown trial,
necessitating presentation of evidence by both parties. The contract is being enforced by one, and [its] validity is attacked by the other
a... matter x x x which respondent court is in the best position to determine."

Hence, this Petition.[11]

ISSUES:

whether the said machines are personal, not immovable, property which may be a proper subject of a writ of replevin.

RULING:

The Petition is not meritorious.

Main Issue: Nature of the Subject Machinery

On the other hand, Article 415 of the Civil Code enumerates immovable or real property as follows:

"ART. 415. The following are immovable property:... x x x....................................x x x....................................x x x

(5) Machinery, receptacles, instruments or implements intended by the owner of the tenement for an industry or works which may be
carried on in a building or on a piece of land, and which tend directly to meet the needs of the said industry or works;

I... n the present case, the machines that were the subjects of the Writ of Seizure were placed by petitioners in the factory built on their
own land. Indisputably, they were essential and principal elements of their chocolate-making industry. Hence, although each of them
was... movable or personal property on its own, all of them have become "immobilized by destination because they are essential and
principal elements in the industry."[16] In that sense, petitioners are correct in arguing that the said machines are real, not... personal,
property pursuant to Article 415 (5) of the Civil Code.[17]

Be that as it may, we disagree with the submission of the petitioners that the said machines are not proper subjects of the Writ of Seizure.

The Court has held that contracting parties may validly stipulate that a real property be considered as personal.[18] After agreeing to such
stipulation, they are consequently estopped from claiming otherwise

NOTE that the reason why the machine was deemed as a personal property was because of the lease agreement which binded both
parties. HERE petitioner was ESTOPPED from declaring that the machines subject in the writ was agreed to be a personal property in
the lease contract.

In the present case, the Lease Agreement clearly provides that the machines in question are to be considered as personal property.
Specifically, Section 12.1 of the Agreement reads as follows:[21]

"12.1 The PROPERTY is, and shall at all times be and remain, personal property notwithstanding that the PROPERTY or any part thereof
may now be, or hereafter become, in any manner affixed or attached to or embedded in, or permanently resting upon, real property or
any... building thereon, or attached in any manner to what is permanent."

Clearly then, petitioners are estopped from denying the characterization of the subject machines as personal property. Under the
circumstances, they are proper subjects of the Writ of Seizure.

It should be stressed, however, that our holding -- that the machines should be deemed personal property pursuant to the Lease
Agreement is good only insofar as the contracting parties are concerned.[22] Hence, while the parties are bound by the

Agreement, third persons acting in good faith are not affected by its stipulation characterizing the subject machinery as personal.[23] In
any event, there is no showing that any specific third party would be adversely affected.

The validity and the nature of the contract are the lis mota of the civil action pending before the RTC. A resolution of these questions,
therefore, is effectively a resolution of the merits of the case. Hence, they should be threshed out in... the trial, not in the proceedings
involving the issuance of the Writ of Seizure.
It should be pointed out that the Court in this case may rely on the Lease Agreement, for nothing on record shows that it has been nullified
or annulled. In fact, petitioners assailed it first only in the RTC proceedings, which had ironically been instituted by respondent.

Accordingly, it must be presumed valid and binding as the law between the parties.

Moreover, even granting that the charge is true, such fact alone does not render a contract void ab initio, but can only be a ground for
rendering said contract voidable, or annullable pursuant to Article 1390 of the new Civil Code, by a proper action in court. There... is
nothing on record to show that the mortgage has been annulled. Neither is it disclosed that steps were taken to nullify the same.

Principles:

After agreeing to a contract stipulating that a real or immovable property be considered as personal or movable, a party is estopped from
subsequently claiming otherwise. Hence, such property is a proper subject of a writ of replevin obtained by the other... contracting party.

Under the principle of estoppel, a party to... a contract is ordinarily precluded from denying the truth of any material fact found therein.

PRUDENTIAL VS. PANIS

FACTS:

On November 19, 1971, plaintiffs-spouses Fernando A. Magcale and Teodula Baluyut Magcale secured a loan in the sum of P70,000.00
from the defendant Prudential Bank. To secure payment of this loan, plaintiffs executed in favor of defendant on the aforesaid date a
deed of Real Estate Mortgage over the following described properties:

l. A 2-STOREY, SEMI-CONCRETE, residential building with warehouse spaces containing a total floor area of 263 sq. meters, more or
less, generally constructed of mixed hard wood and concrete materials, under a roofing of cor. g. i. sheets; declared and assessed in the
name of FERNANDO MAGCALE under Tax Declaration No. 21109, issued by the Assessor of Olongapo City with an assessed value of
P35,290.00. This building is the only improvement of the lot.

2. THE PROPERTY hereby conveyed by way of MORTGAGE includes the right of occupancy on the lot where the above property is
erected, and more particularly described and bounded, as follows:

A first class residential land Identffied as Lot No. 720, (Ts-308, Olongapo Townsite Subdivision) Ardoin Street, East Bajac-Bajac,
Olongapo City, containing an area of 465 sq. m. more or less, declared and assessed in the name of FERNANDO MAGCALE under Tax
Duration No. 19595 issued by the Assessor of Olongapo City with an assessed value of P1,860.00; bounded on the

NORTH: By No. 6, Ardoin Street

SOUTH: By No. 2, Ardoin Street

EAST: By 37 Canda Street, and

WEST: By Ardoin Street.

All corners of the lot marked by conc. cylindrical monuments of the Bureau of Lands as visible limits. ( Exhibit "A, " also Exhibit "1" for
defendant).

Apart from the stipulations in the printed portion of the aforestated deed of mortgage, there appears a rider typed at the bottom of the
reverse side of the document under the lists of the properties mortgaged which reads, as follows:

AND IT IS FURTHER AGREED that in the event the Sales Patent on the lot applied for by the Mortgagors as herein stated is released
or issued by the Bureau of Lands, the Mortgagors hereby authorize the Register of Deeds to hold the Registration of same until this
Mortgage is cancelled, or to annotate this encumbrance on the Title upon authority from the Secretary of Agriculture and Natural
Resources, which title with annotation, shall be released in favor of the herein Mortgage.

From the aforequoted stipulation, it is obvious that the mortgagee (defendant Prudential Bank) was at the outset aware of the fact that
the mortgagors (plaintiffs) have already filed a Miscellaneous Sales Application over the lot, possessory rights over which, were
mortgaged to it.

Exhibit "A" (Real Estate Mortgage) was registered under the Provisions of Act 3344 with the Registry of Deeds of Zambales on November
23, 1971.

On May 2, 1973, plaintiffs secured an additional loan from defendant Prudential Bank in the sum of P20,000.00. To secure payment of
this additional loan, plaintiffs executed in favor of the said defendant another deed of Real Estate Mortgage over the same properties
previously mortgaged in Exhibit "A." (Exhibit "B;" also Exhibit "2" for defendant). This second deed of Real Estate Mortgage was likewise
registered with the Registry of Deeds, this time in Olongapo City, on May 2,1973.

On April 24, 1973, the Secretary of Agriculture issued Miscellaneous Sales Patent No. 4776 over the parcel of land, possessory rights
over which were mortgaged to defendant Prudential Bank, in favor of plaintiffs. On the basis of the aforesaid Patent, and upon its
transcription in the Registration Book of the Province of Zambales, Original Certificate of Title No. P-2554 was issued in the name of
Plaintiff Fernando Magcale, by the Ex-Oficio Register of Deeds of Zambales, on May 15, 1972.

For failure of plaintiffs to pay their obligation to defendant Bank after it became due, and upon application of said defendant, the deeds of
Real Estate Mortgage (Exhibits "A" and "B") were extrajudicially foreclosed. Consequent to the foreclosure was the sale of the properties
therein mortgaged to defendant as the highest bidder in a public auction sale conducted by the defendant City Sheriff on April 12, 1978
(Exhibit "E"). The auction sale aforesaid was held despite written request from plaintiffs through counsel dated March 29, 1978, for the
defendant City Sheriff to desist from going with the scheduled public auction sale (Exhibit "D")." (Decision, Civil Case No. 2443-0, Rollo,
pp. 29-31).

Respondent Court, in a Decision dated November 3, 1978 declared the deeds of Real Estate Mortgage as null and void (Ibid., p. 35).

On December 14, 1978, petitioner filed a Motion for Reconsideration (Ibid., pp. 41-53), opposed by private respondents on January 5,
1979 (Ibid., pp. 54-62), and in an Order dated January 10, 1979 (Ibid., p. 63), the Motion for Reconsideration was denied for lack of merit.
Hence, the instant petition (Ibid., pp. 5-28).

The first Division of this Court, in a Resolution dated March 9, 1979, resolved to require the respondents to comment (Ibid., p. 65), which
order was complied with the Resolution dated May 18,1979, (Ibid., p. 100), petitioner filed its Reply on June 2,1979 (Ibid., pp. 101-112).

Thereafter, in the Resolution dated June 13, 1979, the petition was given due course and the parties were required to submit
simultaneously their respective memoranda. (Ibid., p. 114).

On July 18, 1979, petitioner filed its Memorandum (Ibid., pp. 116-144), while private respondents filed their Memorandum on August 1,
1979 (Ibid., pp. 146-155).

In a Resolution dated August 10, 1979, this case was considered submitted for decision (Ibid., P. 158).

In its Memorandum, petitioner raised the following issues:

1. WHETHER OR NOT THE DEEDS OF REAL ESTATE MORTGAGE ARE VALID; AND

2. WHETHER OR NOT THE SUPERVENING ISSUANCE IN FAVOR OF PRIVATE RESPONDENTS OF MISCELLANEOUS SALES
PATENT NO. 4776 ON APRIL 24, 1972 UNDER ACT NO. 730 AND THE COVERING ORIGINAL CERTIFICATE OF TITLE NO. P-2554
ON MAY 15,1972 HAVE THE EFFECT OF INVALIDATING THE DEEDS OF REAL ESTATE MORTGAGE. (Memorandum for Petitioner,
Rollo, p. 122).

This petition is impressed with merit.

The pivotal issue in this case is whether or not a valid real estate mortgage can be constituted on the building erected on the land
belonging to another.

The answer is in the affirmative.

In the enumeration of properties under Article 415 of the Civil Code of the Philippines, this Court ruled that, "it is obvious that the inclusion
of "building" separate and distinct from the land, in said provision of law can only mean that a building is by itself an immovable property."
(Lopez vs. Orosa, Jr., et al., L-10817-18, Feb. 28, 1958; Associated Inc. and Surety Co., Inc. vs. Iya, et al., L-10837-38, May 30,1958).

Thus, while it is true that a mortgage of land necessarily includes, in the absence of stipulation of the improvements thereon, buildings,
still a building by itself may be mortgaged apart from the land on which it has been built. Such a mortgage would be still a real estate
mortgage for the building would still be considered immovable property even if dealt with separately and apart from the land (Leung Yee
vs. Strong Machinery Co., 37 Phil. 644). In the same manner, this Court has also established that possessory rights over said properties
before title is vested on the grantee, may be validly transferred or conveyed as in a deed of mortgage (Vda. de Bautista vs. Marcos, 3
SCRA 438 [1961]).

Coming back to the case at bar, the records show, as aforestated that the original mortgage deed on the 2-storey semi-concrete residential
building with warehouse and on the right of occupancy on the lot where the building was erected, was executed on November 19, 1971
and registered under the provisions of Act 3344 with the Register of Deeds of Zambales on November 23, 1971. Miscellaneous Sales
Patent No. 4776 on the land was issued on April 24, 1972, on the basis of which OCT No. 2554 was issued in the name of private
respondent Fernando Magcale on May 15, 1972. It is therefore without question that the original mortgage was executed before the
issuance of the final patent and before the government was divested of its title to the land, an event which takes effect only on the issuance
of the sales patent and its subsequent registration in the Office of the Register of Deeds (Visayan Realty Inc. vs. Meer, 96 Phil. 515;
Director of Lands vs. De Leon, 110 Phil. 28; Director of Lands vs. Jurado, L-14702, May 23, 1961; Pena "Law on Natural Resources", p.
49). Under the foregoing considerations, it is evident that the mortgage executed by private respondent on his own building which was
erected on the land belonging to the government is to all intents and purposes a valid mortgage.

As to restrictions expressly mentioned on the face of respondents' OCT No. P-2554, it will be noted that Sections 121, 122 and 124 of
the Public Land Act, refer to land already acquired under the Public Land Act, or any improvement thereon and therefore have no
application to the assailed mortgage in the case at bar which was executed before such eventuality. Likewise, Section 2 of Republic Act
No. 730, also a restriction appearing on the face of private respondent's title has likewise no application in the instant case, despite its
reference to encumbrance or alienation before the patent is issued because it refers specifically to encumbrance or alienation on the land
itself and does not mention anything regarding the improvements existing thereon.

But it is a different matter, as regards the second mortgage executed over the same properties on May 2, 1973 for an additional loan of
P20,000.00 which was registered with the Registry of Deeds of Olongapo City on the same date. Relative thereto, it is evident that such
mortgage executed after the issuance of the sales patent and of the Original Certificate of Title, falls squarely under the prohibitions stated
in Sections 121, 122 and 124 of the Public Land Act and Section 2 of Republic Act 730, and is therefore null and void.

Petitioner points out that private respondents, after physically possessing the title for five years, voluntarily surrendered the same to the
bank in 1977 in order that the mortgaged may be annotated, without requiring the bank to get the prior approval of the Ministry of Natural
Resources beforehand, thereby implicitly authorizing Prudential Bank to cause the annotation of said mortgage on their title.

However, the Court, in recently ruling on violations of Section 124 which refers to Sections 118, 120, 122 and 123 of Commonwealth Act
141, has held:

... Nonetheless, we apply our earlier rulings because we believe that as in pari delicto may not be invoked to defeat the policy of the State
neither may the doctrine of estoppel give a validating effect to a void contract. Indeed, it is generally considered that as between parties
to a contract, validity cannot be given to it by estoppel if it is prohibited by law or is against public policy (19 Am. Jur. 802). It is not within
the competence of any citizen to barter away what public policy by law was to preserve (Gonzalo Puyat & Sons, Inc. vs. De los Amas
and Alino supra). ... (Arsenal vs. IAC, 143 SCRA 54 [1986]).

This pronouncement covers only the previous transaction already alluded to and does not pass upon any new contract between the
parties (Ibid), as in the case at bar. It should not preclude new contracts that may be entered into between petitioner bank and private
respondents that are in accordance with the requirements of the law. After all, private respondents themselves declare that they are not
denying the legitimacy of their debts and appear to be open to new negotiations under the law (Comment; Rollo, pp. 95-96). Any new
transaction, however, would be subject to whatever steps the Government may take for the reversion of the land in its favor.

PREMISES CONSIDERED, the decision of the Court of First Instance of Zambales & Olongapo City is hereby MODIFIED, declaring that
the Deed of Real Estate Mortgage for P70,000.00 is valid but ruling that the Deed of Real Estate Mortgage for an additional loan of
P20,000.00 is null and void, without prejudice to any appropriate action the Government may take against private respondents.

REPUBLIC VS. NORTHERN CEMENT

FACTS:

On June 16, 2000, Northern Cement5 filed with the RTC an application for the registration of title over the Subject Lot - a Fifty Eight
Thousand Six Hundred Seventeen point Ninety Six (58,617.96) square meters lot in Barangay Labayug, Sison, Pangasinan6 - pursuant
to Presidential Decree No. 1529 (PD 1529)7 and to have the title thereto registered and confirmed under its name (Application).8

In its Application, Northern Cement alleged, inter alia, that: (1) it is the owner in fee simple of the Subject Lot which it acquired by way of
a Deed of Absolute Sale (Deed of Sale) from the former owner, Rodolfo Chichioco (Chichioco);9 (2) the Subject Lot was last assessed
at P17,630.00 per Tax Declaration No. 023-01677;10 and (3) Northern Cement is occupying said lot.11

To support its Application, Northern Cement offered, inter alia, the following documents: (1) Deed of Sale dated December 28, 196812
executed by Chichioco in favor of Northern Cement; (2) Affidavits13 of alleged adjoining landowners Eugenia Batnag and Placido Saro
attesting that Northern Cement is the owner and possessor of the Subject Lot; (3) seven (7) Tax Declarations14 for various years from
1971 to 2003 in the name of Northern Cement and a Tax Declaration15 for year 1970 in the name of Chichioco; (4) Tax Clearance
Certificate16 dated May 21, 2007; (5) Technical Description17 of the Subject Lot; (6) Approved Plan18 certified by the Department of
Environment and Natural Resources (DENR) stating that the Subject Lot is "x x x inside alienable and disposable area as per project No.
63, L.C. Map No. 698, certified on November 21, 1927 x x x."19

Likewise, Northern Cement submitted a Report20 dated March 16, 2003 from Alfredo Reyes, Special Investigator I, Community
Environment and Natural Resources Office (CENRO), DENR, Urdaneta City, stating, among others, that: (1) the land is agricultural;21
(2) it has not been earmarked for public purposes;22 (3) the entire area is within the alienable and disposable zone as class ified on
November 21, 192723 and (4) Northern Cement is the actual occupant of the Subject Lot with the improvement: "Cogon."24

Northern Cement likewise adduced in evidence the testimonies of the following witnesses: (1) Angelita Cabana, Northern Cement's duly
authorized representative, who testified that Northern Cement acquired ownership over the Subject Lot from Chichioco by virtue of a
Deed of Absolute Sale dated December 28, 1968, that Northern Cement has been paying the realty taxes due thereon, and that there is
no other person who claims interest over the same;25 and (2) Lilia Macanlalay and Macaria Lopez, Jr., Records Officer and Special
Investigator, respectively, of the CENRO Regional Office of Urdaneta City, who both testified that an investigation was conducted over
the Subject Lot and that all the records relative thereto are complete.26

The Office of the Solicitor General (OSG) filed its Notice of Appearance27 for the Republic, deputizing the City Prosecutor of Urdaneta
City to appear in the case.

Ruling of the RTC

In its Decision28 dated July 6, 2009, the RTC granted the Application for registration of Northern Cement in this wise:
WHEREFORE, premises considered, the Court, after confirming the Order of General Default, hereby adjudicates Lot 3250, Ap-01-
004756, Pls 796 Sison Public Land Subd., which is the subject land of this registration proceedings in favor of applicant NORTHERN
CEMENT CORPORATION, as its real property and hereby likewise orders the registration of title thereto in accordance with
PRESIDENTIAL DECREE No. 1529 in the name of the applicant and on the basis of the approved Technical Description (Exh. "J").

Upon finality of the Decision, let a corresponding Order for the issuance of Decree of Registration be issued.

SO ORDERED.29
The RTC ruled that from the evidence presented, Northern Cement was able to prove, by preponderance of evidence, its claim of
ownership over the Subject Lot.

The Republic appealed to the CA, alleging that the RTC erred in granting the application for registration despite the failure of Northern
Cement to observe the requirements for original registration of title under PD 1529. The Republic pointed out, among others, that the
CENRO Report and the Approved Plan submitted in evidence by Northern Cement hardly suffice to prove that the Subject Lot is an
alienable portion of the public domain.

Ruling of the CA

In the assailed Decision30 dated August 15, 2011, the CA denied the Republic's appeal and affirmed in toto the Decision of the RTC,
disposing of the case as follows:
WHEREFORE, in the light of the foregoing, the present appeal is hereby DENIED and the assailed decision dated 06 July 2009 is
AFFIRMED in toto.

SO ORDERED.31
The CA ruled that the evidence sufficed to comply with the requirements of PD 1529.

The Republic filed a Motion for Reconsideration32 but the same was denied in the assailed CA Resolution33 for raising no additional
arguments to warrant reconsideration of the assailed Decision.

Hence, this Petition.

ISSUE:

Whether the CA erred in affirming the RTC's Decision granting the application for registration of title in favor of Northern Cement despite
non-compliance with the requirements under PD 1529.

RULING:

YES! Northern Cement is not qualified to have the Subject Lot registered in its name under Section 14 of PD 1529, whether under (1) or
(2), which states,

SECTION 14. Who may apply. - The following persons may file in the proper Court of First Instance an application for registration of title
to land, whether personally or through their duly authorized representatives:

(1) Those who by themselves or through their predecessors-in--interest have been in open, continuous, exclusive and notorious
possession and occupation of alienable and disposable lands of the public domain under a bona fide claim of ownership since June 12,
1945, or earlier.

(2) Those who have acquired ownership over private lands by prescription under the provisions of existing laws.
The Republic is correct.

At the outset, the Court notes that while the Republic makes a fairly lengthy disquisition on compliance by Northern Cement with the
requirements of Section 14(1) of PD 1529 and while the RTC quoted34in passing this provision of the law, nowhere else in the records
does it appear that Northern Cement's case is specifically hinged thereon. The Application itself does not enlighten as to whether it was
filed under Section 14(1) or Section 14(2) of PD 1529. Northern Cement made no allegation nor presented evidence that it had been in
possession of the subject property since June 12, 1945 or earlier. At any rate, the evidence presented, the allegations in the pleadings
as well as the discussion of the CA and the RTC in their respective decisions and resolutions, reveal that the present controversy was
filed and tried based on Section 14(2) of PD 1529. Thus, the Petition shall be resolved on Northern Cement's proof of its acquisition of
the Subject Lot by prescription.

Unlike Section 14(1) which requires an open, continuous, exclusive, and notorious manner of possession and occupation since June 12,
1945 or earlier, Section 14(2) is silent as to the nature and period of such possession and occupation necessary. This necessitates a
reference to the relevant provisions of the Civil Code on prescription - in this case, Articles 113735 and 1118 thereof, to wit:
Article 1137. Ownership and other real rights over immovables also prescribe through uninterrupted adverse possession thereof for thirty
years, without need of title or of good faith.

Article 1118. Possession has to be in the concept of an owner, public, peaceful and uninterrupted. (Emphasis and underscoring supplied)
The Court, in the case of Heirs of Crisologo v. Rañon, stated:
Prescription is another mode of acquiring ownership and other real rights over immovable property. It is concerned with lapse of time in
the manner and under conditions laid down by law, namely, that the possession should be in the concept of an owner, public, peaceful,
uninterrupted and adverse. Possession is open when it is patent, visible, apparent, notorious and not clandestine.It is continuous when
uninterrupted, unbroken and not intermittent or occasional; exclusive when the adverse possessor can show exclusive dominion over the
land and an appropriation of it to his own use and benefit; and notorious when it is so conspicuous that it is generally known and talked
of by the public or the people in the neighborhood x x x. (Emphasis and underscoring supplied; citations omitted)
The phrase "adverse, continuous, open, public, and in concept of owner," is a conclusion of law.37 The burden of proof is on the person
seeking original registration of land to prove by clear, positive and convincing evidence that his possession and that of his predecessors-
in--interest was of the nature and duration required by law.

Applying the foregoing to the present case, the Court is unconvinced by the pieces of evidence submitted by Northern Cement to prove
compliance with the requirement of possession under Section 14(2) of PD 1529 in relation to Articles 1137 and 1118 of the Civil Code for
original registration of land. The RTC erred in haphazardly concluding otherwise and the CA, in turn, erred in affirming the RTC.

First, the seven (7) tax declarations (1971, 1974, 1980, 1985, 1995, 2001 and 2003) in the name of Northern Cement and one (1) tax
declaration (1970) in the name of its predecessor-in-interest for a claimed possession of at least thirty-two (32) years (1968 - 2000) do
not qualify as competent evidence to prove the required possession. It has been held that this type of intermittent and sporadic assertion
of alleged ownership does not prove open, continuous, exclusive and notorious possession and occupation.38 The Court has, in a catena
of cases, found as lacking, episodic and random payments of realty taxes including five (5) Tax Declarations for a claimed possession of
forty-five (45) years,39 twenty-three (23) Tax Declarations on two (2) areas for a claimed possession of forty-six (46) years40 and twenty
(20) Tax Declarations on three (3) areas for a claimed possession of sixty-five (65) years.41 The Court finds no reason to decide this
case differently being that it shares the same factual milieu.

Moreover, Tax Declarations are not conclusive evidence of ownership but only a basis for inferring possession.42 It is only when these
tax declarations are coupled with proof of actual possession of the property that they may become the basis of a claim of ownership.43

Second, even if it be assumed that Northern Cement had been in possession of the subject property since 1968, it still failed to sufficiently
demonstrate that its supposed possession was of the nature and character contemplated by law.

The testimonies of the adjoining owners presented by Northern Cement do not deserve serious consideration and they do not augment
the inadequacy of the Tax Declarations. The two witnesses, claiming to be heirs of the owners of the lands adjoining the subject property,
did not testify as to the specific acts of possession and ownership exercised by Northern Cement and/or its predecessors-in-interest.
They merely made a uniform and sweeping claim that the subject property "is owned and possessed by [Northern Cement],"44 which is
a mere conclusion of law. This evidence is tenuous, at best.

Third, Northern Cement miserably failed to prove possession of the Subject Lot in the concept of an owner, with the records bare as to
any acts of occupation, development, cultivation or maintenance by it over the property. Indeed, from the evidence presented, the only
"improvements" on the Subject Lot were "cogon"45 and "unirrigated rice."46

Cogon grass is hardly the "improvement" contemplated by law to prove satisfaction of the requirements of registering lands. It is a matter
of common knowledge that cogon grass grows casually on lands in this country, without need of cultivation, and hardly has utility. More
than anything, it is usually indicia that the land on which it grows is idle.47

As for the unirrigated rice which appeared latest in the 1995 Tax Declaration, plain common sense dictates that the fact of it being
unirrigated and uncultivated further cements the character of the land as idle.

The importance of exercising acts of dominion on a land sought to be registered cannot be downplayed. In a plethora of cases, the Court
has disallowed registration of lands where, although plants and fruit-bearing trees existed on the contested lands, it was not proven that
they were cultivated by the registrant, or that they were actively and regularly cultivated and maintained and not merely casually or
occasionally tended to by the registrant,48 or that they were planted by him or his predecessors-in-interest.49

Evidently, this case where cogon and unirrigated rice appear to be the only things standing on the Subject Lot and with no allegations or
testimony that the same had been planted or cultivated by Northern Cement, pales in comparison with the aforementioned cases.

On a final note, this Court is well-aware that the Republic has raised issues bearing on the registrable nature of the subject property,
pursuant to the landmark and oft-quoted case of Malabanan v. Republic50 in relation to the relevant Civil Code provisions, i.e., whether
it was validly and sufficiently declared alienable and disposable and, even so, if it was further declared as no longer intended for public
use or service or for the development of national wealth and whether the latter declaration is necessary for the subject land to be
registrable. The Court deems it no longer necessary to address these matters as this case can be amply decided on the basis of the
evident failure of Northern Cement to satisfy the required possession under PD 1529, Section 14(2) in relation to Articles 1137 and 1118
of the Civil Code. Perhaps, that issue is fated to be scrupulously discussed in a more opportune case.

REPUBLIC VS. SAROMO

The Facts and Antecedent Proceedings

As culled from the CA Decision, the facts are as follows:


On September 25, 1980, Geodetic Engineer Francisco C. Guevarra surveyed the land subject of this case for x x x Filemon Saromo.
Engineer Guevarra then prepared Survey Plan No. PSU-4-A-004479 (Exhibit "A"). At the bottom left hand portion of the plan is a NOTE
that states: "This survey is formerly a portion of China Sea. This survey is inside unclassified public forest land and is apparently inside
the area covered by Proclamation No. 1801 dated November 10, 1978. This survey is within 100.00 meters strip along the shore line.
This survey was endorsed by the District Land Officer D.L.O. No. (IV-A-1), Batangas City dated December 11, 1980." The survey plan of
the subject lot includes the salvage zone.

On September 30, 1980, Survey Plan No. PSU-4-A-004479 was submitted to Region IV-A for approval.

On December 11, 1980, the survey plan was endorsed by the District Land Officer, Batangas City and on the following day, December
12, 1980, the plan was approved by Flor U. Pelayo, Officer-in-Charge.

On December 24, 1980, Saromo, then fifty [50] years old, executed an Application for Free Patent (Exh. "N"), covering the subject
property, which he filed with the Bureau of Lands, District Land Office No. IV-A-1 in Batangas City. The application stated among others
that the land is an agricultural public land covered by Survey No. PSU-4-A-004479, containing an area of forty five thousand eight hundred
eight (45,808) square meters and that Saromo first occupied and cultivated the land by himself in 1944 (Exh. "N-2" and. "N-3").

xxxx

On the same date, Saromo executed an affidavit (Exh. "4"), stating that he is the holder of Free Patent Application No. (IV-A-1) 15603
and that he holds himself responsible for any liability, whether civil and/or criminal that may arise if the land has already been adjudicated
as private property and/or the corresponding certificate of title had in fact been issued and for any statement he had made therein that
may be found untrue or false.

On January 24, 1981, Saromo executed an affidavit (Exh. ["]3["]) in support of a Notice of Application for Free Patent stating that said
Notice of Application for Free Patent (which was not signed by the Director of Lands) was posted on the bulletin board.of the barrio where
the land is situated and at the door of the municipal building on December 24, 1980 until the 24th day of January 1981.

On March 4, 1981, Alberto A. Aguilar executed an investigation report (Exh. "P") stating that on January 14, 1981, he went to and
examined the land applied for by Saromo; that the land applied for is inside agricultural area under proposed Project No. 31 LC Map 225.
While the certified true copy of said investigation report submitted by the Republic mentions "LC Map 225", the xerox copy of the same
investigation report offered in evidence by Saromo as "Exhibit 26", contains an insertion of the number ["]#235" above the words LC Map
225.

On May 18, 1981, Jaime Juanillo, District Land Officer, issued an Order (Exh. "O") approving the application for free patent of Saromo
and ordering the issuance of Patent No. 17522 in his favor. The Order stated that the land applied for has been classified as alienable
and disposable; the investigation conducted by Land Investigation/Inspector Alberto A. Aguilar revealed that the land applied for has been
occupied and cultivated by the applicant himself and/or his predecessors[-]in[-]interest since July 4, 1926 or prior thereto.

On May 26, 1981, Original Certificate of Title No. P-331 (Exh. "C") was issued in the name of Filemon Saromo by Deputy Register of
Deeds for the Province of Batangas, Gregorio C. Sembrano.

On October 16, 1981, a certain Luis Mendoza filed with the Bureau of Lands a protest against the Free Patent awarded to Saromo. The
investigation was not terminated because of the resignation of the investigator from the Bureau and his departure for the United States.
(Exh. "B"; p. 21, TSN, Aprill5, 2002, Atty. Rogelio Mandar)

On September 6, 1999, the Director of Lands issued Special Order No. 99-99 creating an investigation team headed by Atty. Rogelio C.
Mandar to verify and determine the legality of the issuance of Free Patent No. 17522, now OCT No. P-331, in the name of Saromo
covering the subject parcel of land identified as Lot No. 3, Plan PSU-4-A-004479, containing an area of forty five thousand eight hundred
eight (45,808) square meters (Exhs. "B"; pp. 6-7, TSN, April 15, 2002, Atty. Mandar). The investigation team found from the documents
gathered that:
a) the subject lot covered by Free Patent No. 17522 in the name of Saromo, identified and described under Plan PSU-4-A-004479, was
not alienable and disposable at the time of the issuance thereof, as it was found upon investigation to be "inside unclassified public forest
and covered by Proclamation No. 1801 declaring the whole of Batangas Coastline as tourist zone (Exh. "B", p. 2)

b) the issuance of Free Patent No. 17522 in the name of Saromo was highly improper and irregular, and Free Patent No. 17522 and the
corresponding OCT N[o]. P-331 issued to Saromo is null and void ab initio and the land covered must be reverted to the State. x x x
x x x (O]n September 19, 2001, the Republic filed this case for Reversion/Cancellation of Title before the [RTC].

[The Republic], in its Complaint, alleged that the subject lot covered by OCT No. P-331 is inside the unclassified forest [land] and also
inside the area covered by Proclamation No. 1801 dated November 10, 1978 declaring the land as Tourist Zones and Marine Preserve
under the administration and control of the Philippine Tourism Industry. It further alleged that upon ocular inspection, it was ascertained
that the land is situated along the coastline of Brgy. Balibago and that since it is part of the shore, it concluded that the subject lot is part
of the public dominion and therefore, cannot be titled in the name of private person.

On the other hand, (Saromo), in his Answer, denied the allegations of [the Republic] and countered that the subject land is disposable
and alienable the same being an agricultural land suited for cultivation and plantation of fruit bearing trees at the time the free patent was
issued to him. He claimed that he is the owner of the subject lot in fee simple by virtue of OCT No. [P-]331 and Free Patent No. 17522,
which was lawfully issued to him by the Lands Management Bureau (formerly, Bureau of Lands).[6]
Ruling of the RTC
The RTC rendered a Decision[7] dated October 24, 2005 in favor of Saromo, the dispositive portion of which states:
WHEREFORE, premises considered, the instant complaint is hereby DISMISSED for lack of merit.

No pronouncement as to the costs.

SO ORDERED.[8]
The RTC relied heavily on the testimony of Engr. Francisco Guevara[9] (Engr. Guevara), who testified that the note appearing on the
survey plan indicated "past and present annotations" placed by the office of the Bureau of Lands and that the "land is no longer a forest
land and it belongs to what was alienated and disposed by the [then] Bureau of Lands and therefore, it is suited for plantation,
cultivation[.]"[10]

The RTC also stated that the then Bureau of Lands verified the truthfulness of the information given by Saromo before it approved the
free patent application; and the fact that the free patent was issued to Saromo only confirmed his statement in his application that the
subject land was alienable and disposable, being agricultural land.[11] The RTC concluded that the findings of the field investigator of the
then Bureau of Lands as to the nature of the subject land after conducting his ocular inspection at the time of the application for free
patent should be given more weight since that is the foremost issue to be considered by the concerned agency before granting the
application for free patent.[12] The RTC found that the Republic failed to overturn the presumption of regularity in the performance of the
official function of the employee of the then Bureau of Lands who approved the free patent.[13]

Regarding the issue that the subject land is covered by Proclamation No. 1801,[14] the RTC stated that it "was so explicit in enumerating
the areas covered by the said law and it shows that the subject property was not one of those listed therein."[15] According to the RTC,
there is, likewise, nothing in the law which provides that those covered thereby is inalienable and non--disposable because the law
declares certain islands, coves and peninsulas in the Philippines as Tourist Zones and Marine Reserve under the administration and
control of the Philippine Tourism Authority (PTA).[16]

The RTC concluded that the subject land is well within the purview of a public land which is alienable and disposable, and the patent title
issued to Saromo is not tainted with any irregularity as claimed by the Republic.[17]

The Republic filed a motion for reconsideration, which was opposed by Saromo. The RTC denied the motion in its Resolution dated April
24, 2006.[18]

The Republic appealed the RTC Decision to the CA.

Ruling of the CA

The CA in its Decision[19] dated June 30, 2009 denied the appeal of the Republic. The dispositive portion thereof states:
IN THE LIGHT OF ALL THE FOREGOING, the appeal is hereby DENIED. The decision dated 24 October 2005 of the Regional Trial
Court of Balayan, Batangas, Branch 9, in Civil Case No. 3929 is hereby AFFIRMED.

No costs.

SO ORDERED.[20]
The CA also relied on the testimony of Engr. Guevara, who was the person who prepared the survey plan referred to above, to the effect
that the subject land is an agricultural land and, therefore, alienable and disposable.[21] The CA noted the explanation of Engr. Guevara
on the meaning of "unclassified public forest land" annotated on the survey plan to the effect that since the subject land is "capable of
being cultivated and planted with trees, vegetables and other plantation done by any occupants," it follows that the same is already
alienable and disposable.[22] Thus, the CA ruled that the Republic failed to prove its cause of action by preponderance of evidence.[23]

The CA further noted that Saromo complied with all the necessary requirements for the issuance of a free patent and he relied on the
knowledge and expertise of the District Land Office, which is tasked to manage and issue patents pursuant to existing laws.[24] The CA
determined that the Republic failed to prove the fraud and misrepresentation that Saromo allegedly committed.[25]

The Republic filed a motion for reconsideration, which was opposed by Saromo and denied by the CA in its Resolution dated October
12, 2009.[26]

Hence, the instant Petition. Saromo filed his Comment[27] dated March 9, 2010.

The Issues

The Petition raises the following issues:


1. Whether the CA erred on a question of law in upholding that the subject land is alienable and disposable at the time of issuance of free
patent title to Saromo.

2. Whether the CA erred in not applying Section 91 of the Public Land Act on fraud and misrepresentation and in disregarding the
attendant fraud and misrepresentation of Saromo in his free patent application.

3. Whether the CA erred in applying the presumption of regularity in the performance of official duties of the officer who issued Saromo's
free patent.
4. Whether the principle of Regalian doctrine applies in the present case.[28]
The Court's Ruling

The Petition is impressed with merit.

While the Republic seeks the reversal of the finding of both the CA and the RTC that the subject land is alienable and disposable via a
question of law issue, it actually seeks a review by the Court of their factual findings. The Court cannot make the legal conclusion that
the Republic seeks without a review of the facts upon which the CA and the RTC based their ruling that the subject land is alienable and
disposable.

As a rule, the factual findings of the CA affirming those of the trial court are final and conclusive, and they cannot be reviewed by the
Court which has jurisdiction to rule only on questions of law in petitions to review decisions of the CA filed before the Court, save only in
the following circumstances: (1) when the factual conclusion is a finding grounded entirely on speculations, surmises and conjectures;
(2) when the inference is manifestly mistaken, absurd or impossible; (3) when there is grave abuse of discretion; (4) when the judgment
is based on a misapprehension of facts; (5) when the findings of fact are conflicting; (6) when the CA went beyond the issues of the case
in making its findings, which are further contrary to the admissions of both the appellant and the appellee; (7) when the CA's findings are
contrary to those of the trial court; (8) when the conclusions do not cite the specific evidence on which they are based; (9) when the facts
set forth in the petition as well as in the petitioner's main and reply briefs are not disputed by the respondents; (10) when the CA's findings
of fact, supposedly premised on the absence of evidence, are contradicted by the evidence on record;[29] or (11) when the CA manifestly
overlooked certain relevant facts not disputed by the parties, which, if properly considered, would justify a different conclusion.[30] Thus,
for the Court to review the factual findings of the courts below, any of these exceptions must be present in this case.

The subject land is unclassified public forest land.

From the outset, the Republic argues that Proclamation No. 1801 expressly declared the Batangas Coastline as a tourist zone; hence, it
is a reserved area incapable of alienation and disposition by private individuals.[31]

The Court is not persuaded by this argument of the Republic.

Indeed, Proclamation No. 1801 includes the "Whole of Batangas Coastline"[32] as a tourist zone and marine reserve under the
administration and control of the PTA, and the law requires that: "No development projects or construction for any purposes shall be
introduced within the zones without prior approval of the President of the Philippines upon recommendation of the Philippine Tourism
Authority."[33] However, as correctly observed by the RTC, there is nothing in the law which provides that the areas covered thereby are
necessarily inalienable and non--disposable.[34]

Section 4 of Presidential Decree No. 564[35] provides that the PTA has the purpose of promoting "the development into integrated resort
complexes of selected and well defined geographic areas with potential tourism value, known otherwise as 'tourist zones'."

On the other hand, the Tourism Act of 2009 or Republic Act No. (RA) 9593[36] defines "Tourism Enterprise Zones" or TEZs in the following
manner:
SEC. 59. Tourism Enterprise Zones. - Any geographic area with the following characteristics may be designated as a Tourism Enterprise
Zone:

(a) The area is capable of being defined into one contiguous territory;

(b) It has historical and cultural significance, environmental beauty, or existing or potential integrated leisure facilities within its bounds or
within reasonable distances from it;

(c) It has, or it may have, strategic access through transportation infrastructure, and reasonable connection with utilities infrastructure
systems;

(d) It is sufficient in size, such that it may be further utilized for bringing in new investments in tourism establishments and services; and

(e) It is in a strategic location such as to catalyze the socioeconomic development of their neighboring communities.
Under RA 9593, it is the newly created TIEZA (Tourism Infrastructure and Enterprise Zone Authority) that shall designate TEZs, upon
recommendation of any local government unit (LGU) or private entity, or through joint ventures between the public and the private
sectors.[37]

From the above descriptions of "tourist zones" and TEZs, they appear to be the same. But, there is nothing from their descriptions from
which it can be deduced that as tourist zones or TEZs, they are therefore inalienable and non-disposable.

Proclamation No. 1801 also declares the "Whole of Batangas Coastline" a marine reserve. As defined: "A Marine Reserve is an MPA
where strict sanctuary conditions are not mandated for the entire area, but there is still a desire to control access and activities, such as
boating, mooring and various fishing techniques. It may consist of multiple zones including a sanctuary area,"[38] while "[a] Marine
Protected Area (MPA) is any specific marine area that has been reserved by law or other effective means and is governed by special
rules or guidelines to manage activities and protect the entire, or part of, the enclosed coastal and marine environment."[39]

Based on the above definitions, there may be indications that the concerned area may be subject to special rules or guidelines for its
management and protection; but, it does not follow that as a marine reserve, the area is automatically inalienable and non-disposable.
Given the foregoing, the presidential declaration that the whole of the Batangas coastline is a tourist zone and marine reserve is not
sufficient to prove that the subject land is inalienable and non-disposable.

Unfortunately, the very survey plan that Saromo submitted to the then Bureau of Lands as basis for his application for free patent and its
approval contains a notation that the subject land is "inside unclassified public forest land."[40] To recall, the NOTE appearing at the
bottom left hand portion of the Survey Plan No. PSU-4A-004479 (Exhibit "A"[41]) prepared by Engr. Guevara states: "This survey is
formerly a portion of China Sea. This survey is inside unclassified public forest land and is apparently inside the area cover[ed] by
Proclamation No. 1801 dated Nov[ember] 10, 1978. This survey is within 100.00 meters strip along the shore line. This survey was
indorsed by the District Land Officer D.L.O. No. (IV-A-1), Batangas City dated December 11, 1980."[42]

As is, the NOTE qualifies as an admission of Saromo under Section 26, Rule 130 of the Rules of Court, which provides: "[t]he act,
declaration or omission of a party as to a relevant fact may be given in evidence against him." The NOTE is an admission by Saromo that
the subject land is "inside unclassified public forest land." Thus, unless Saromo is able to rebut in a clear and convincing manner such
admission or declaration, it will remain as an admission against his interest and binding upon him.

Saromo presented the testimonies of Engr. Guevara, Alberto Aguilar (Aguilar) and Engineer Carlita Cabrera (Engr. Cabrera) to rebut the
land classification expressly indicated in the NOTE.

Both the RTC and the CA were convinced of the testimonial evidence that Saromo adduced, and they relied heavily on the testimony of
Engr. Guevara in arriving at the factual conclusion that the subject land is agricultural land and, thus, alienable and disposable. The CA
even quoted Engr. Guevara's testimony on cross-examination,[43] to wit:
[Atty. Benjamin C. Asido: (to the witness)]

Q
May we ask you again, what you mean by the note, "This survey is inside unclassified public forest land," what is the meaning of that?
A
It meant that the place was already alienable and disposable as classified by the Bureau of Forestry and if there are any improvements
such as grasses, they really reflect as unclassified forest. But then, this is capable o[f] being cultivated and planted with trees, vegetables
and other plantation done by any occupants, sir.
Q
In other words, what you are saying is, is that the meaning of inside unclassified public forest is that it is already alienable and disposable,
is that what you mean?
A
Yes, sir.[44]
Aside from the foregoing explanation, Engr. Guevara commented on the significance of the said NOTE during his direct examination, to
wit:
[Atty. Paciano B. Balita (to witness)]
Q
In your plan, there is a note, what is the significance of that note, if any?
A
In the note it is placed here that all corners not otherwise described PLS are cyl. concrete monuments 15x60 cm, and the others were
planted PS cyl. concrete monuments 15x60 and these comers are formerly a portion of China sea and this survey is inside unclassified
public forest land and is apparently inside the area covered by Proclamation No. 1801 dated November 10, 1978 and all the survey is
within 100 meters strip along the shoreline and this survey was indorsed by the district land officer D.L.O. Bo. (IV-A1), Batangas City
dated December 11, 1980. These are the notes placed by the office of the Bureau of Lands, indicating that all these are past and present
annotations in the place, sir.[45]
The CA also stated: "And his testimony on the meaning of 'unclassified public forest land' was not rebutted by the [Republic]."[46]

The CA further mentioned the testimony of Aguilar, who was the investigator of the District Land Office of the then Bureau of Lands in
Batangas City and conducted an ocular inspection of the subject land during the processing of Saromo's free patent application. Aside
from identifying his investigation report[47] and the order of approval of Saromo's application,[48] Aguilar merely made a conclusion when
asked as to the "physical feature" of the land, to wit:
[Atty. Paciano B. Balita (to the witness)]
Q
You made a report. Now, during your inspection, would you tell the Court what actually was the physical feature of the land?
A
The land being applied for free-patent is agricultural in nature, sir.[49]
As reflected in his investigation report, the improvements in the land consisted of "coconuts" and that "the land applied for is inside
Agricultural area under proposed project No. 31 L.C. map 225."[50]

Engr. Cabrera, a geodetic engineer, who was assigned as a final verifier of the Chief Survey Division of the then Bureau of Lands and
conducted a verification survey, testified as well on the "physical feature or condition" of the subject land in this manner:
[Atty. Paciano B. Balita (to the witness)]
Q
Would you be able to tell the Honorable Court, actually the physical feature or condition of this property subject of this suit?
A
That is agricultural in nature because there was an improvement thereon; planted with coconut trees, beach houses, sir.
Q
It is not a forest land or timber land?
A
No, sir.[51]
Both the RTC and the CA erred in unduly relying on the testimony of Engr. Guevara because his observation as to the physical features
of the subject land is not conclusive to remove the subject land from its "unclassified forest land" classification and overturn the NOTE
that the area he surveyed was "inside unclassified public forest land." Similarly, the testimonies of Engr. Guevara, Aguilar and Engr.
Cabrera on their observations as to the physical features of the subject land during their ocular inspection are not clear and convincing
proof that the subject land is alienable and disposable.

As the Court held in The Secretary of the Department of Environment and Natural Resources v. Yap,[52] forest land of the public domain
in the context of both the Public Land Act and the Constitution is a classification descriptive of its legal nature or status and does not have
to be descriptive of what the land looks like, viz.:
Forests, in the context of both the Public Land Act and the Constitution[53] classifying lands of the public domain into "agricultural, forest
or timber, mineral lands and national parks," do not necessarily refer to large tracts of wooded land or expanses covered by dense growths
of trees and underbrushes.[54] The discussion in Heirs of Amunategui v. Director of Forestry[55] is particularly instructive:
A forested area classified as forest land of the public domain does not lose such classification simply because loggers or settlers have
stripped it of its forest cover. Parcels of land classified as forest land may actually be covered with grass or planted to crops by kaingin
cultivators or other farmers. "Forest lands" do not have to be on mountains or in out of the way places. Swampy areas covered by
mangrove trees, nipa palms, and other trees growing in brackish or sea water may also be classified as forest land. The classification is
descriptive of its legal nature or status and does not have to be descriptive of what the land actually looks like. Unless and until the land
classified as "forest" is released in an official proclamation to that effect so that it may form part of the disposable agricultural lands of the
public domain, the rules on confirmation of imperfect title do not apply.[56] (Emphasis supplied)
There is a big difference between "forest" as defined in a dictionary and "forest or timber land" as a classification of lands of the public
domain as appearing in our statutes. One is descriptive of what appears on the land while the other is a legal status, a classification for
legal purposes.[57] At any rate, the Court is tasked to determine the legal status ofBoracay Island, and not look into its physical layout.
Hence, even if its forest cover has been replaced by beach resorts, restaurants and other commercial establishments, it has not been
automatically converted from public forest to alienable agricultural land.[58]

From the foregoing, testimonial evidence on the physical layout or condition of the subject land-that it was planted with coconut trees and
beach houses had been constructed thereon - are not conclusive on the classification of the subject land as alienable agricultural land.
Rather, it is the official proclamation releasing the land classified as public forest land to form part of disposable agricultural lands of the
public domain that is definitive. Such official proclamation, if there is any, is conspicuously missing in the instant case.

The term "unclassified land" is likewise a legal classification and a positive act is required to declassify inalienable public land into
disposable agricultural land. The Court in Heirs of the late Sps. Palanca v. Republic[59] observed that:
While it is true that the land classification map does not categorically state that the islands are public forests, the fact that they were
unclassified lands leads to the same result. In the absence of the classification as mineral or timber land, the land remains unclassified
land until released and rendered open to disposition.[60] When the property is still unclassified, whatever possession applicants may
have had, and however long, still cannot ripen into private ownership.[61] This is because, pursuant to Constitutional precepts, all lands
of the public domain belong to the State, and the State is the source of any asserted right to ownership in such lands and is charged with
the conservation of such patrimony.[62] Thus, the Court has emphasized the need to show in registration proceedings that the
government, through a positive act, has declassified inalienable public land into disposable land for agricultural or other purposes.[63]
Given the foregoing, the misapprehension of the "facts" as adduced by Saromo through the foregoing testimonial evidence warrants the
review by the Court of the findings of fact of both the CA and the RTC. Without the official declaration that the subject land is alienable
and disposable or proof of its declassification into disposable agricultural land, the "unclassified public forest land's" legal classification of
the subject land remains.

Engr. Guevara even admitted that the NOTE in his survey plan indicated "past and present annotations" placed by the "office of the
Bureau of Lands." This is confirmation of the land classification status of the subject land as "unclassified public forest land" and such
remained even at the time when he executed the survey plan. Otherwise, the NOTE should have contained a further annotation that said
classification had been changed. Also, Engr. Guevara did not present and testify on the applicable land classification map that would
corroborate his finding that the subject land was already disposable agricultural land.

In addition to the exception that the judgments of the courts below are based on misapprehension of facts, the other exception that is
applicable in this case is when the findings of fact are contradicted by the evidence on record.

The Republic has adduced compelling evidence, which were not contradicted by Saromo, that the subject land was inalienable and non--
disposable at the time of his application.

The Republic presented as witness Leonito D. Calubayan (Engr. Calubayan), a geodetic engineer and Community Environment
Resources Officer of Calaca, Batangas of the Department of Environment and Natural Resources, who testified as follows:
[Atty. Benjamin C. Asido (to witness)]
Q
Sometime in July 2002, did you receive a letter request [from] one Atty. Benjamin Asido in relation to this complaint in this particular case?
A
Yes, sir.
xxxx
Q
Do you have [the] letter request of Atty. Asido?
xxxx
A
Yes, it is on file, sir.
ASIDO:
Q
May I have that record?
INTERPRETER:
Witness showing a letter request addressed to CENRO Officer dated July 10, 2002.
ASIDO:
May I make [of] record, your Honor, that the letter request be marked as Exhibit "1". A letter request dated July 10, 2002 requesting the
CENRO Officer to certify whether or not the land subject of this case is alienable or disposable.
Q
What action, if any, did you take on the letter request?
A
It is a standard operating procedure that whenever communication of this nature has been received by our office, I used to forward this
to our Chief of Forestry, the Chief of Forest Management Service, sir.
Q
Q What action, if any, did your Chief of Management Service take?
A
Well, as requested in the request, the office through the Chief of Forest Management Service prepared a certification, sir.
Q
May I have that Certification?
A
(Witness showing a Certification dated October 9, 2002)
ASIDO:
May I request, your Honor, that this Certification prepared by Pedro Caringal, Jr. be marked as Exhibit "J".
xxxx
ASIDO:
Q
In this Certification marked as Exhibit "J", you stated under paragraph 1 and I quote: "Plan PSU-4A-004479 surveyed in the name of
Filemon Saromo covered by Original Certificate of Title No. P-331 with an area of 4.5 hectares more or less in the Municipality of
Calatagan, Batangas," do you have that plan with you now? Plan PSU-4A-004479?
A
I have the copy of that plan, sir. This is the copy of the plan on record, sir.
(Witness showing a plan of the land surveyed for Filemon Saromo)
xxxx
Q
Under paragraph 1 of this Certification, Exhibit "J", you stated that the area covered by OCT No. P-331 is within the foreshore area of the
Municipality of Calatagan?
A
Yes, sir, because the approved plan of PSU-4A-004479 was projected and verified against [sheet] 5 of 9 sheets land classification map
number, in short, under LC Map 3276 verified on June 29, 1987, sir.
Q
Do you have that LC map with you?
A
Yes, sir.
Q
May I have that LC Map?
A
This is the LC Map that Iam referring to (Witness showing LC Map 3276)
Q
Will you please indicate in your report the land subject of this case in the LC Map 3276?
A
This is the area where the subject PSU Plan falls when verified and plotted in the LC Map. It falls on Project No. 38-A, Block C, which
states that it is forest land (permanent forest) with an area of 38.8 hectares the overall area of the project where that PSU falls, sir.
ASIDO:
May we request that the LC Map be marked as Exhibit "L" for the plaintiff and area indicated by the witness subject of this case be marked
as Exhibit "L-1", your Honor.
xxxx
ASIDO:
We [request] that the investigation report relative to the application for Free Patent [of Saromo] be marked as Exhibit "P", your Honor.
xxxx
Q
Now in this Investigation Report under paragraph 7 it states that the land is not inside agricultural area LC Map No. 225, do you have this
LC Map 225?
A
I have with me the record of LC Map 225 (Witness showing LC Map 225)
Q
Is this LC Map for the Province ofBatangas?
A
It says here, it is Sibulan, sir.
Q
Where is Sibulan?
A
May I see the map, sir. According to this LC [Map] 225, it appears that it covers the Municipality of Sibulan, Negros Oriental.
ASIDO:
May we request, your Honor, that the LC Map No. 225 be marked as an evidence as Exhibit "Q" and the Municipality of Sibulan, Negros
Oriental be marked as Exhibit "Q-1",your Honor.
May we request that paragraph 7 of the investigation report be marked as Exhibit "P-1", your Honor.
Q
Also this LC Map, it made mention [of] Project No. 31. Do you have that map?
A
This LC 718, there is written project No. 31 but this subdivision, the Municipality ofTaal, sir.
Q
Where is that?
A
This is also in Batangas, sir.
Q
How far is Barrio Balibago from Taal? A It is so far away, Taal and Balibago, sir.
ASIDO:
May we request, your Honor, that Project No. 31 be marked as Exhibit "R", your Honor.[64]
On cross-examination, Engr. Calubayan explained that based on the projection of the survey plan for Saromo, it is within the Municipality
of Calatagan despite the indication in OCT No. P-331 issued to Saromo that it is in Balibago, Lian, Batangas, to wit:
[Atty. Paciano B. Balita (to witness)]
Q
Did you see before that the property, subject of this suit, is located at Calatagan, Batangas?
A
According to our findings, when the property in question was projected, the foreshore area is within the Municipality of Calatagan, sir.
Q
What is the basis of your findings?
A
Based on our projection with the land classification map, it appears that it falls within the foreshore area of the Municipality of Calatagan.
There is a technical data. The land classification map has a latitude and longtitude. The land in question is also provided with that
geographic coordinate so we computed that, so by means of that coordinate, we can project on the land classification map where the
property could be located or could fall, sir.
Q
So, your basis was a technical data?
A
Yes, sir.
xxxx
Q
The torrens title of OCT No. P-331, from the description, would you still insist that the property could be traced as indicated in the title?
A
The title states that this is located in Lian, however, when we issued a certification that was based on the land classification map, that
was issued sometime in 1987, so the survey appears to be executed earlier than what the land classification map was issued, sir.[65]
From the foregoing, it is clear that when Plan Psu-4A-004479 surveyed in the name of Saromo was verified and plotted by the Forest
Management Service in the corresponding land classification map, it falls on Project No. 38-A, Block C, of the Land Classification (LC)
Map No. 3276 (Exh. "L") certified on June 29, 1987, which is forest land (permanent forest) within the foreshore area of Calatagan,
Batangas.[66]

In addition, LC Map No. 3342 (Exh. "M") was presented to prove that as of October 10, 1984, the whole of Calatagan, Batangas was
unclassified public forest and that there was no land classification certified or declared prior to 1984 covering the subject land.[67] Engr.
Calubayan explained the reference to the LC Map of Calatagan, Batangas as warranted by the technical data found in the survey plan
prepared by Engr. Guevara for Saromo such that when the said data are projected, they fall within the LC Map of Calatagan, Batangas.

In fine, the Republic presented credible evidence to show that the subject land remains within unclassified forest land, which conforms
with the NOTE in the survey plan for Saromo. The subject land, is therefore, inalienable and non-disposable and could not have been the
valid subject of a free patent application because only agricultural public lands subject to disposition can be the subject of free patents.[68]

There are attenuating circumstances that put in doubt the applicability of the presumption of regularity in the performance of official duties.

The presumption of regularity in the performance of official duties in the processing and approval of Saromo's free patent has been
controverted by the evidence presented by the Republic. Also, the evidence presented by Saromo put in serious doubt the regularity in
the processing and approval of his free patent.

The survey plan in question includes a NOTE that the subject land is within "unclassified public forest land." The investigator and verifier
of the then Bureau of Lands, who processed Saromo's application, did not present any land classification map that would negate such
NOTE.
Also, as testified to by Engr. Calubayan, the investigation report of Aguilar mentioned that the land applied for is inside agricultural land
under proposed project No. 31, LC Map 225 (Exh. "26" as corrected[69] but LC Map 225 is for Sibulan, Negros Oriental. LC Map 718
mentioned in the Survey Authority (Exh. "25" as corrected[70]) refers to Taal, Batangas.

Even Saromo himself contradicted the investigation report of Aguilar which indicated that "[t]he occupation and cultivation of the applicant
[Saromo], as far as [Aguilar has] been able to ascertain date from 1944" and the subject land was "first occupied and cultivated by Filemon
Saromo in 1944."[71] His very Application for Free Patent (Exh."2"[72]), which is under oath, contained untrue information, as confirmed
by him, although he attributed the incorrectness to clerical error. Since the year "1944" appears in both his Application for Free Patent
and in the investigation report of Aguilar, the error can no longer be categorized as clerical. Rather, an intention to mislead or make a
false representation is evident.

Saromo testified as follows:


[Atty. Paciano Balita (to witness)]
Q
Since when have you been occupying this property, subject of this suit?
A
When I purchased the adjacent land, it was [in] 1967 and some of it was in the year 1969, sir.
xxxx
Q
By the way, in your affidavit or application it is stated here that when you submitted an application you were only 11 years old, what can
you say to that?
A
No, sir. I was already 44 or 46 years old.
Q
Why it was indicated here that you were 11 years old, who prepared this?
A
It was the surveyor and it was a pro forma of the Bureau of Lands. I believed that is a clerical error. It is impossible that I was only 11
years old because I'm not in a position to purchase a lot yet, sir.
Q
That was in 1980?
A
Yes, sir.
Q
And now, 2004 that is 24 years ago?
A
Yes, sir.
Q
How old are you now?
A
69, sir. This coming March I'll be 70 years old.[73]
Saromo could not have first occupied the subject land in 1944 as indicated in his sworn Application for Free Patent and in the investigation
report, because he bought the subject land in 1967 at the earliest or 1969 at the latest, and he was then 44 or 46 years old.

Given the foregoing discrepancies in the documents relative to Saromo's free patent application, the processing and approval of his free
patent were far from regular. Thus, the validity of his free patent cannot be affirmed based on the mere presumption of regularity in the
performance of official duties.

Reversion of the subject land is warranted under Section 91 of Commonwealth Act No. (CA) 141.

Section 91 of CA 141, otherwise known as The Public Land Act, provides:


SEC. 91. The statements made in the application shall be considered as essential conditions and parts of any concession, title, or permit
issued on the basis of such application, and any false statement therein or omission of facts altering, changing, or modifying the
consideration of the facts set forth in such statements, and any subsequent modification, alteration, or change of the material facts set
forth in the application shall ipso facto produce the cancellation of the concession, title, or permit granted. It shall be the duty of the
Director of Lands, from time to time and whenever he may deem it advisable, to make the necessary investigations for the purpose of
ascertaining whether the material facts set out in the application are true, or whether they continue to exist and are maintained and
preserved in good faith, and for the purposes of such investigation, the Director of Lands is hereby empowered to issue subpoenas or
subpoena duces tecum and, if necessary, to obtain compulsory process from the courts. In every investigation made in accordance with
this section, the existence of bad faith, fraud, concealment, or fraudulent and illegal modification of essential facts shall be presumed if
the grantee or possessor of the land shall refuse or fail to obey a subpoena or subpoena duces tecum lawfully issued by the Director of
Lands or his authorized delegates or agents, or shall refuse or fail to give direct and specific answers to pertinent questions, and on the
basis of such presumption, an order of cancellation may issue without further proceedings.
As mentioned above, there are several discrepancies in the documents relative to Saromo's free patent application, which indicate
incorrect and misleading facts and statements. Taken together, they can be considered as "false statements" on the essential conditions
for the grant of the free patent in favor of Saromo, and as such, they ipso facto justify the cancellation of the free patent and the
corresponding Torrens certificate of title issued to him.

Even if Section 91 of CA 141 is ruled out, reversion is warranted based on mistake or error on the part of government officials or agents.
In Republic v. Hachero,[74] the Court observed:
Reversion is an action where the ultimate relief sought is to revert the land back to the government under the Regalian doctrine.
Considering that the land subject of the action originated from a grant by the government, its cancellation therefore is a matter between
the grantor and the grantee.[75] In Republic v. Guerrero,[76] the Court gave a more general statement that "this remedy of reversion can
only be availed of in cases of fraudulent or unlawful inclusion of the land in patents or certificates of title."[77] Nonetheless, the Court
recognized in Republic v. Mangotara,[78] that there were instances when it granted reversion for reasons other than fraud:
x x x. In Estate of the Late Jesus S. Yujuico v. Republic (Yujuico case), reversion was defined as an action which seeks to restore public
land fraudulently awarded and disposed of to private individuals or corporations to the mass of public domain. It bears to point out, though,
that the Court also allowed the resort by the Government to actions for reversion to cancel titles that were void for reasons other than
fraud, i.e., violation by the grantee of a patent of the conditions imposed by law; and lack of jurisdiction of the Director of Lands to grant
a patent covering inalienable forest land or portion of a river, even when such grant was made through mere oversight.[79] [Emphasis
Supplied]
In the case at bench, although the Republic's action for cancellation of patent and title and for reversion was not based on fraud or
misrepresentation on the part of Hachero, his title could still be cancelled and the subject land reverted back to the State because the
grant was made through mistake or oversight. x x x[80]
The Court further observed in Hachero:
At any rate, it is a time-honored principle that the statute of limitations or the lapse of time does not run against the State. Jurisprudence
also recognizes the State's immunity from estoppel as a result of the mistakes or errors of its officials and agents. These well- established
principles apply in the case at bench. The Court in Republic v. Roxas elucidated:
xxxx

Be that as it may, the mistake or error of the officials or agents of the [Bureau of Lands] in this regard cannot be invoked against the
government with regard to property of the public domain. It has been said that the State cannot be estopped by the omission, mistake or
error of its officials or agents.

It is well-recognized that if a person obtains a title under the Public Land Act which includes, by oversight, lands which cannot be registered
under the Torrens system, or when the Director of Lands did not have jurisdiction over the same because it is a public domain, the grantee
does not, by virtue of the said certificate of title alone, become the owner of the land or property illegally included. Otherwise stated,
property of the public domain is incapable of registration and its inclusion in a title nullifies that title.[81]
Since, at the very least, the government officials concerned in the processing and approval of Saromo's free patent application erred or
were mistaken in granting a free patent over unclassified public forest land, which could not be registered under the Torrens system and
over which the Director of Lands had no jurisdiction, the free patent issued to Saromo ought to be cancelled. In the same vein, the Torrens
title issued pursuant to the invalid free patent should likewise be cancelled.

Since the reversion of the subject land to the State is in order, needless to say that the Regalian doctrine has been accordingly applied
in the resolution of this case.

WHEREFORE, the Petition is hereby GRANTED

GATCHALIAN VS. FLORES

FACTS:

Petitioner is one of the co-owners of a parcel of land (Road Lot 23) covered by Transfer Certificate of Title No. 79180 located at Brgy.
Vitalez, Parañaque City. Road Lot 23 is registered under the name of petitioner's parents, spouses Sixto Gatchalian and Liceria
Gatchalian. On June 2, 2011, petitioner filed a Complaint for Ejectment with Damages against respondents Cesar Flores, Jose Paolo
Araneta (sic), Corazon Quing and Cynthia Flores (respondents) with the Metropolitan Trial Court (MeTC) of Parafiaque City, Branch 77
and docketed as Civil Case No. 2011-49.

The survey conducted on the property established that the lot of Segundo Mendoza encroached a portion of Road Lot 23 which the
Gatchalian's had tolerated. But after several years, the lot of Segundo Mendoza was sold and 'subdivided among the new owners
including herein respondents. When the latter demonstrated acts of gross ingratitude to the Gatchalian family, petitioner and his family
were constrained to withdraw their tolerated possession, use and occupation of the portion of Road Lot 23. Verbal and written demands
to vacate were then served upon them but remained unheeded. Their dispute reached the Lupong Tagapamayapa but all in vain. Hence,
the filing of the ejectment case against the respondents.

For their part, respondents denied that they usurped the property of petitioner. In fact, it was the Gatchalians who have encroached on
Road Lot 23 when they put up a fence in their (respondents) property. They insisted that Road Lot 23 is a public road and is now known
as "Don Juan Street Cat-Mendoza". In the subdivision plan of the GAT Mendoza Housing area, Road Lot 23 is constituted as a right of
way. Respondents believed that petitioner has no cause of action against them and has no authority to file the instant case because it is
the City Government of Parañaque which has the right to do so.5

On December 9, 2011, the MeTC rendered a Decision6 ordering respondents to vacate Road Lot 23, thus:

WHEREFORE, premises considered, judgment is hereby rendered as follows ordering the defendants CESAR FLORES, JOSE PAOLO
ARANETA, CORAZON QUING AND CYNTHIA FLORES and all persons claiming rights under them, to wit:
1) to vacate the 140.50 square meter portion of the Road (Lot 23) encroached by them which is covered by TCT No. 79180 and located
at Don Juan St., Barangay Vitalez, Paranaque City;

2) to pay reasonable amount of rental in the amount of ₱20,000.00 a month plus legal rate of interest reckoned from June 2, 2011 until
the defendants shall have fully vacated the encroached portion of the Road (Lot 23);

3) ₱20,000.00 as and (sic) for Attorney's fees;

4) Cost of suit.

SO ORDERED.7

Respondents appealed the same to the RTC, which reversed the ruling of the MeTC in its Decision8 dated June 8, 2012, to wit:

WHEREFORE, premises considered, the appealed Decision dated December 9, 2011 by Branch 78 of the Metropolitan Trial Court of
Parañaque docketed under Civil Case No. 2011-49 is REVERSED and the Complaint dated June 2, 2011 is herewith DISMISSED for
lack of merit. SO ORDERED.9

Aggrieved, petitioner appealed to the CA. The latter in its Decision10 dated March 13, 2015, reversed the RTC and reinstated the ruling
of the MeTC. However, upon reconsideration, the CA reversed itself and affirmed the RTC, thus:

WHEREFORE, respondent's Motion for Reconsideration is hereby GRANTED. Accordingly, we REVERSE and SET ASIDE our findings
in our Decision dated March 13, 2015. The instant petition fore review is hereby DISMISSED and the Decision dated June 8, 2012 of the
Regional Trial Court of Parafiaque City, Branch 196 in Civil Case No. 12- 0050 is UPHELD.

SO ORDERED.11

Hence, this petition.

Petitioner claimed that the CA committed grave error in ruling that the private character of Road Lot 23 has been stripped by Municipal
Ordinance No. 88-04, series of 1988 constituting the said road lot as a public right-of-way. Petitioner also claimed that the CA erred in
stating that by virtue of laches, the road lot has been converted to public property of the municipality.

Petitioner further alleged that the road lot is still private property it being covered by TCT No. 79180 under the name of Spouses Sixto
Gatchalian and Liceria Gatchalian. The mere usage by the public of the road lot does not make it public property. To convert the same to
public property, it must be expropriated by the government or the registered owner must donate or sell the same to the government.

The petition is granted.

At the outset, petitioner filed before the MeTC an action for ejectment against the respondents. It is settled that in ejectment proceedings,
the only issue for the Court's resolution is, who between the parties is entitled to the physical or material possession of the subject
property. Issues as to ownership are not involved, except only for the purpose of determining the issue of possession.12

In the instant case, petitioner asserts that he is entitled to the possession of the road lot being one of the co-owners of the same since it
is registered under the name of petitioner's parents. While respondents do not claim ownership of the subject lot, they argued that the
road lot is now public property because of Ordinance No. 88-04, series of 1988 constituting it as "Don Juan St. Gat-Mendoza". As such,
petitioner cannot evict respondents.

It is undisputed that the road lot is registered under the name of petitioner's parents. Even the respondents did not dispute this fact. It is
also undisputed that the municipal government has not undertaken any expropriation proceedings to acquire the subject property neither
did the petitioner donate or sell the same to the municipal government.1âwphi1 Therefore, absent any expropriation proceedings and
without any evidence that the petitioner donated or sold the subject property to the municipal government, the same is still private property.

In the case of Woodridge School, Inc. v. ARB Construction Co., Inc. 13, this Court held that:

In the case of Abellana, Sr. v. Court of Appeals, the Court held that "the road lots in a private subdivision are private property, hence, the
local government should first acquire them by donation, purchase or expropriation, if they are to be utilized as a public road." Otherwise,
they remain to be private properties of the owner-developer.

Contrary to the position of petitioners, the use of the subdivision roads by the general public does not strip it of its private character. The
road is not converted into public property by mere tolerance of the subdivision owner of the public's passage through it. To repeat, "the
local government should first acquire them by donation, purchase or expropriation, if they are to be utilized as a public road."14

As reiterated in the recent case of Republic of the Philippines, represented by the Department of Public Works and Highways (DPWH) v.
Sps. Llamas15, this Court held that:

As there is no such thing as an automatic cessation to [the] government of subdivision road lots, an actual transfer must first be effected
by the subdivision owner: "subdivision streets belonged to the owner until donated to the government or until expropriated upon payment
of just compensation."16
Since the local government of Parañaque has not purchased nor undertaken any expropriation proceedings, neither did the petitioner
and his siblings donate the subject property, the latter is still a private property and Ordinance No. 88-04 did not convert the same to
public property.

As to the CA's finding that by virtue of laches the subject property has been converted into public property, We do not agree.

It is well-settled that an "owner of [a] registered land does not lose his rights over a property on the ground of laches as long as the
opposing claimant's possession was merely tolerated by the owner."17

A torrens title is irrevocable and its validity can only be challenged in a direct proceeding.18 A torrens title is an indefeasible and
impresciptible title to a property in favor of the person in whose name the title appears. The owner is entitled to all the attributes of
ownership of the property, including possession. The person who has a torrens title over a land is entitled to possession thereof. As such,
petitioner can file an ejectment case against herein respondents who encroached upon a portion of petitioner's property.

WHEREFORE, premises considered, the petition is GRANTED.

REPUBLIC VS. ROVENCY REALTY CORP.

THE FACTS

On 22 March 2001, RRDC filed before the RTC an Amended Application for Registration4 covering a parcel of land identified as Lot No.
3009 (subject land) situated in Barangay Balulang, Cagayan de Oro City, described as follows:

A parcel of land (Lot No. 3009, Cad-237, Cagayan Cadastre) situated in the Barrio of Carmen, City of Cagayan de Oro, Island of Mindanao.
Bounded on the S., along line 1-2 by Lot 6648; on the NW., along line 2-3 by Lot 30011; along line 3-4 by Lot 301 O; along line 4-5 by
Lot 3047; along line 5-6 by Lot 3020; on the N., along line 6-7 by Lot 3007; on the SE., along line 8-9 by Lot 6645; along line 9-1 by Lot
3008; all of Cad-237, Cagayan Cadastre.

Beginning at the point marked "1" on the plan being N. 51 deg. 24'W., 1091.05 m. from PBM No. 24, Cad-237, Thence;

1-2 S. 79 deg. 15'W. 260.92 m.


2-3 N. 19 deg. 02'E. 231.49 m.
3-4 N. 13 deg. 32'E. 489.77 m.
4-5 N. 61 deg. 39'E. 302.54 m.
5-6 N. 40 deg. 09'E. 146.06 m.
6-7 S. 82 deg. 14'E. 140.06 m.
7-8 S. 24 deg. 28'E. 152.88 m.
8-9 S. 34 deg. 00'W. 448.33 m.
9-1 S. 33 deg. 26'W. 445.73 m.
beginning; containing an area of THREE HUNDRED EIGHTEEN THOUSAND THREE HUNDRED FORTY FIVE (318,345) square meters
more or less. All points referred to are indicated on the plan and marked on the ground by Old BL., cyl. cone. mons. 15 x 60 cm. Bearing
true, date of Original Survey August 9 & 13, 1929, and that of the preparation June 29, 2000, executed by Crisanto M. Bagares, Geodetic
Engineer and approved on August 1, 2000.5

RRDC alleged, among others, that it is a domestic corporation duly organized and existing under and by virtue of the laws of the Republic
of the Philippines; that it is the absolute owner in fee simple of the subject land having acquired the same from its previous owner, P.N.
Roa Enterprises, Inc., by virtue of a notarized deed of absolute sale executed on 05 March 1997; that the subject land was assessed at
₱2,228,000.00 as shown in the Tax Declaration (TD) No. 141011; that it has registered the subject land for taxation purposes and paid
the realty taxes due therein from its acquisition, to the filing of the application; that immediately after acquiring the subject land, it took
actual physical possession of the same and has been continuously occupying the subject land; and that it and its predecessors-in- interest
have been in open, continuous, adverse, and peaceful possession in concept of owner of the subject land since time immemorial, or for
more than thirty (30) years.

Attached to the application are: original copy of the technical description of the subject land6; the Tracing Cloth Plan of the survey plan7;
Certification in Lieu of Surveyor's/Geodetic Engineer's Certificate8 issued by the Chief of the Land Surveys Assistance Section,
Department of Environment and Natural Resources, Region X; T.D. No. 141011 in the name of RRDC9 ; and the Deed of Absolute Sale
between RRDC and P.N. Roa Enterprises, Inc., dated 5 March 1997.10

On 16 July 2001, an opposition to the application was filed by the Heirs of Paulino Avancena. They alleged, that the subject land was
already claimed and owned by the late Atty. Paulino Avancena (Paulino), their father and predecessor-in-interest, as early as 1926; that
Paulino had been in open, continuous, notorious, adverse, and exclusive possession and occupation of the subject land; that Paulino
registered the subject land for taxation purposes and has paid the taxes due thereon in 1948; that their parents, Paulino and Rizalina Neri
(Rizalina) merely allowed and tolerated Pedro N. Roa's (Pedro) possession of the subject land after the latter approached them and
requested that he be allowed to use the subject land for his businesses; that Pedro is one of RRDC's predecessors-in-interest; that
sometime in 1994, Rizalina demanded the return of the subject land from the heirs of Pedro, but to no avail; that in 1996, Rizalina died
leaving the private oppositors as the rightful heirs of the subject land; that their parents never sold the subject land to Pedro nor to RRDC,
and as such, no right or title over the subject land was passed on to RRDC. Thus, they prayed that RRDC's application be dismissed,
and that their opposition be treated as their own application for registration.11
On 3 August 2001, the petitioner Republic of the Philippines (Republic), through the Office of the Solicitor General (OSG), filed its
opposition to the application on the following grounds: that neither RRDC nor its predecessors-in-interest have been in open, continuous,
exclusive, and notorious possession and occupation of the land in question since 12 June 1945 or prior thereto; that the subject land
exceeds the twelve (12)-hectare limit for confirmation of imperfect title set by Section 47 of Commonwealth Act (CA.) No. 141, as amended
by Republic Act (R.A.) No. 6940; and that the subject land forms part of the public domain belonging to the Republic and, thus, not subject
to private appropriation.12

During trial, RRDC presented the following documents in support of its application: (i) Deed of Absolute Sale notarized by notary public
Paulino Avancena showing that the subject land was sold by Catalino Ebalo to Nicolas Beja and Maximo Amper on 21 June 193713 ; (ii)
Deed of Absolute Sale notarized by notary public Paulino A vancefia showing that a portion of the subject land consisting of 159, 178.5
square meters (first portion) was sold by Maximo Amper to Perfecto Virtudazo on 07 October 194014 ; (iii) Deed of Absolute Sale notarized
by notary public Troadio C. Ubay-ubay showing that the first portion consisting of 15 hectares, 91 ares and 72 centares (159,172 square
meters) was sold by Trinidad Virtudazo, Israel Virtudazo, and Adelina Virtudazo to Victor D. Beja on 22 April 196115 ; (iv) Deed of
Absolute Sale showing that the first portion of the subject land consisting of 159,172 square meters was sold by Victor D. Beja to Pedro
N. Roa on 01 February 19616 ; (v) Deed of Absolute Sale notarized by notary public Troadio C. Ubay-ubay showing that the other portion
(second portion) of the subject land was sold by Nicolas Beja to Victor Beja on 22 April 196117 ; (vi) Deed of Sale showing that the
second portion was sold by Victor Beja to Pedro N. Roa on 01 February 196718 ; (vii) Deed of Exchange notarized by notary public Jose
L. Sabio, Jr. showing that the two portions of the subject land were conveyed by Pedro N. Roa in favor of P.N. Roa Enterprises, Inc. on
23 September 1987;19 and (viii) Deed of Sale notarized by Rene C. Barbaso showing that the two (2) portions of the subject land were
sold by P.N. Roa Enterprises, Inc. to RRDC on 25 July 1996.20

RRDC also presented a certification21 from the Community Environment and Natural Resources Office (CENRO), Cagayan de Oro City,
certifying that the subject land is alienable and disposable and not covered by any public land application patent and hence, no patent
has been issued thereon. Lastly, RRDC presented several tax declarations in the name of its predecessors-in-interest, the earliest of
which is T.D. No. 91264, which showed that realty taxes on the subject land have been paid in 1947.22

On the other hand, to support their claim that a patent over the subject land had been issued in the name of their father, the private
oppositors presented a certification23 issued by the Records Management Division of the Lands Management Bureau of the Department
of Environment and Natural Resources which merely states that " ...according to the verification made by the Geodetic Surveys Division,
survey plan no. Psu-45882 with an accession no. 284578 is located at Cagayan, Misamis, as per their EDP listing. It is unfortunate
however that as of this moment, this office (Records Management Division) cannot locate said records despite diligent search made
thereon."

The RTC Ruling

In its decision, dated 7 November 2003, the RTC granted RRDC's application for registration of the subject land. It opined that the CENRO
certification, stating that the subject land is alienable and disposable and not covered by any public land application, is sufficient to show
the character of the land. It further ruled, that RRDC and its predecessors-in-interest had been in open and continuous possession under
a bona fide claim of ownership over the subject land based on the documentary and testimonial evidence offered by RRDC, without
discussing how these pieces of evidence established the required possession.

The trial court further brushed aside the opposition interposed by the heirs of Paulino Avanceña. It was not convinced that the evidence
they presented were sufficient to grant the application in their favor. It noted that the oppositors' claim that they were the rightful owners
of the subject land does not hold water considering that the deeds of sale presented by RRDC in support of their claim were notarized by
Paulino himself.

The dispositive portion of the RTC decision reads:

WHEREFORE, this Court considering the evidence of the applicant, the reports of the Land Registration Authority, Director of Lands and
the Certification of the CENRO, DENR, Cagayan de Oro City, hereby declares that the applicant, Rovency Realty & Development
Corporation, have sufficient title proper for registration over the parcel of land subject of this application. The opposition of the Heirs of
Paulino Avanceña, is hereby ordered dismissed, being lack of merit.

Accordingly, in accordance with the prayer of the applicant herein, the Commissioner, or anyone acting on his behalf is hereby directed
to ISSUE A DECREE OF REGISTRATION and the CORRESPONDING CERTIFICATE OF TITLE FOR THE PARCEL OF LAND
described in the instant application in favor of RO VEN CY REAL TY and DEVELOPMENT CORPORATION. SO ORDERED.24

Unconvinced, the Republic, through the OSG, and private oppositors heirs of Paulino Avancena, elevated their respective appeals to the
CA.25

The Republic contended that the trial court erred in granting the application for registration, considering that the land applied for is in
excess of what is allowed by the Constitution; and that the Corporation Code further prohibits RRDC to acquire the subject land unless
the acquisition thereof is reasonably necessary for its business. On the other hand, the Avancena heirs insisted that they are the rightful
owners of the subject land, by virtue of the homestead patent granted to their predecessor-in-interest.

The CA Ruling
In its assailed decision, dated 10 March 2009, the CA affirmed the 7 November 2003 RTC decision. The appellate court concurred with
the trial court's findings that the subject land is alienable and disposable, and that RRDC has sufficiently established the required period
and character of possession. Likewise, the appellate court was not persuaded by the claims of the heirs. It noted that the private oppositors
anchored their claim on the alleged homestead grant to Paulino, their predecessor-in-interest, which claim was unsupported by sufficient
documentary evidence.

The appellate court also ruled that the 12-hectare limit under the Constitution was not violated. It explained that Section 3 of Article XII of
the 1987 Constitution, the constitutional provision which provided for the 12-hectare limit in the acquisition of land, covers only agricultural
lands of the public domain. It ratiocinated that when the subject land was acquired through acquisitive prescription by RRDC's
predecessors-in-interest, it was converted into a private property and, as such, it ceased to be part of the public domain. Thus, when
RRDC acquired the subject land by purchase, it was no longer within the ambit of the constitutional limitation.

As to the contention that the Corporation Code bars RRDC to acquire the subject land, the appellate court simply stated that while the
said code imposes certain limitations on the acquisition of real property, there is no such prohibition. It stressed that RRDC is an artificial
being imbued with the power to purchase, hold, and convey real and personal property for such purposes that are within the objects of
its creation. Considering that RRDC is a corporation engaged in realty business, it has the power to purchase real properties. The
dispositive portion of said decision states:

WHEREFORE, the appeal is DENIED. The assailed November 7, 2003 Decision of the Regional Trial Court (RTC) of Misamis Oriental,
Branch 41, Cagayan de Oro City is hereby AFFIRMED. SO ORDERED.26

The Republic moved for reconsideration; while the Heirs of Paulino Avanceña adopted the Republic's motion for reconsideration as their
own. In its resolution, dated 3 December 2009, the CA denied the motion for reconsideration.

Hence, this petition.

THE ISSUES

I.

THE TRIAL COURT ERRED IN GRANTING THE AMENDED APPLICATION FOR REGISTRATION AND ORDERING THE ISSUANCE
OF A DECREE OF REGISTRATION AND THE CORRESPONDING CERTIFICATE OF TITLE FOR A PARCEL OF LAND CONTAINING
AN AREA OF THREE HUNDRED EIGHTEEN THOUSAND THREE HUNDRED FORTY FIVE (318,345) SQUARE METERS IN FAVOR
OF ROVENCY REALTY AND DEVELOPMENT CORPORATION, DESPITE THE FACTS THAT-

(i) THE LAND APPLIED FOR REGISTRATION OF TITLE IS IN EXCESS OF WHAT IS ALLOWED BY LAW; AND,

(ii) RESPONDENT'S RIGHT TO ACQUIRE THE SUBJECT PARCEL OF LAND IS FURTHER LIMITED BY THE CORPORATION CODE.

II.

RESPONDENT'S EVIDENCE IS INSUFFICIENT TO PROVE THAT IT OR ITS PREDECESSORS-IN-INTEREST HAVE BEEN IN OPEN,
CONTINUOUS, EXCLUSIVE AND NOTORIOUS POSSESSION UNDER A BONA FIDE CLAIM OF OWNERSHIP SINCE JUNE 12, 1945
OR EARLIER AND THE SUBJECT PROPERTY IS NO LONGER INTENDED FOR PUBLIC USE OR FOR THE DEVELOPMENT OF
THE NATIONAL WEALTH.27

THE COURT'S RULING

The petition is meritorious.

12-hectare limit under Section 3, Article XII of the 1987 Constitution

The Republic argues that the trial and appellate courts erred in granting RRDC's application for the registration of the subject land, as the
same has a total land area of 31.8 hectares, which is way beyond the 12-hectare limit under Section 3, Article XII of the 1987 Constitution,
which provides:

SECTION 3. Lands of the public domain are classified into agricultural, forest or timber, mineral lands, and national parks. Agricultural
lands of the public domain may be further classified by law according to the uses which they may be devoted. Alienable lands of the
public domain shall be limited to agricultural lands. Private corporations or associations may not hold such alienable lands of the public
domain except by lease, for a period not exceeding twenty-five years, renewable for not more than twenty-five years, and not to exceed
one thousand hectares in area. Citizens of the Philippines may lease not more than five hundred hectares, or acquire not more than
twelve hectares thereof by purchase, homestead, or grant. [emphasis supplied]

As can be clearly gleaned from its language, Section 3, Article XII applies only to lands of the public domain. Private lands are, therefore,
outside of the prohibitions and limitations stated therein. Thus, the appellate court correctly declared that the 12-hectare limitation on the
acquisition of lands under Section 3, Article XII of the 1987 Constitution has no application to private lands.

A case in point is the absolute prohibition on private corporations from acquiring any kind of alienable land of the public domain. This
prohibition could be traced to the 1973 Constitution which limited the alienation of lands of the public domain to individuals who were
citizens of the Philippines. This constitutional prohibition, however, does not necessarily mean that corporations may not apply for original
registration of title to lands. In fact, the Court, in several instances, affirmed the grant of applications for original registration filed by
corporations,28 for as long as the lands were already converted to private ownership by operation of law as a result of satisfying the
requisite possession required by the Public Land Act.29

In Director of Lands v. Intermediate Appellate Court30 (Director of Lands), the Court granted the application for original registration of
parcels of land filed by a corporation which acquired the lands by purchase from members of the Dumagat tribe. The Court ratiocinated
that the lands applied for registration were already private lands even before the corporation acquired them. The Court observed that the
sellers, being members of the national cultural minorities, had by themselves and through their predecessors, possessed and occupied
the lands since time immemorial. As a consequence of their open, exclusive, and undisputed possession over the said lands for the
period required by law for the acquisition of alienable lands of the public domain, said lands ceased to become part of the public land and
were converted, by operation of law, into private ownership. As such, the sellers, if not for their conveyance of the lands in question to
the corporation, were entitled to exercise the right granted to them by the Public Land Act to have their title judicially confirmed.
Considering further that the lands in question were already private in character at the time the corporation acquired them, the constitutional
prohibition does not apply to the corporation.

In Republic v. TA.N. Properties 31 (TA.N. Properties), the Court stressed that what is determinative for the application of the doctrine in
Director of Lands is for the corporate applicant for land registration to establish that when it acquired the land, the same was already
private land by operation of law because the statutory acquisitive prescriptive period of 30 years had already lapsed.

The pronouncements in Director of Lands and TA.N. Properties apply with equal force to the 12-hectare limitation, considering that both
the limitation and the prohibition on corporations to acquire lands, do not cover ownership of private lands. Stated differently, whether
RRDC can acquire the subject land and to what extent, depends on whether the pieces of evidence it presented before the trial court
sufficiently established that the subject land is alienable and disposable land of the public domain; and that the nature and duration of the
possession of its individual predecessors-in-interest converted the subject land to private land by operation of law.

Requirements for original registration of title to land

In Republic of the Philippines vs. Cortez,32 the Court explained that applicants for original registration of title to land must first establish
compliance with the provisions of either Section 14(1) or Section 14(2) of P.D. No. 1529, which state:

Sec. 14. Who may apply. The following persons may file in the proper Court of First Instance an application for registration of title to land,
whether personally or through their duly authorized representatives:

(1) Those who by themselves or through their predecessors-in interest have been in open, continuous, exclusive and notorious
possession and occupation of alienable and disposable lands of the public domain under a bona fide claim of ownership since June 12,
1945, or earlier.

(2) Those who have acquired ownership of private lands by prescription under the provision of existing laws.

It must be emphasized that the requirements and bases for registration under these two provisions of law differ from one another. Section
14 (1) mandates registration on the basis of possession, while Section 14 (2) entitles registration on the basis of prescription.33 Thus, it
is important to ascertain under what provision of Section 14 the registration is sought.

A reading of the application, however, is unavailing. In its application, RRDC alleged that it and its predecessors-in-interest "had been in
open, continuous, adverse, and peaceful possession in concept of owner of the subject property since time immemorial or for more than
thirty years." This allegation made it unclear whether registration is sought under Section 14(1) - possession since 12 June 1945 or earlier;
or under Section 14(2) - possession for more than thirty years.

An examination of the 7 November 2003 RTC decision also proved futile considering that, and as previously pointed out, aside from
enumerating the exhibits offered by the applicant, the trial court did not discuss how these pieces of evidence established the requisites
for registration. Thus, for the proper resolution of the issues and arguments raised herein, it becomes necessary for the present application
to be scrutinized based on the requirements of the provisions of Sections 14 (1) and (2) of P.D. No. 1529.

Registration under Section 14(1) of P.D. No. 1529

Under Section 14(1), applicants for registration of title must sufficiently establish the following requisites: first, that the subject land forms
part of the disposable and alienable lands of the public domain; second, that the applicant and his predecessors-in-interest have been in
open, continuous, exclusive, and notorious possession and occupation of the same; and third, that the possession is under a bona fide
claim of ownership since 12 June 1945, or earlier.34

The first requisite of Section 14(1) entails only that the property sought to be registered be alienable and disposable at the time of the
filing of the application for registration.35 To prove that the land sought to be registered is alienable and disposable, the present rule is
that the application for original registration must be accompanied by (1) a CENRO or PENRO Certification; and (2) a copy of the original
classification approved by the DENR Secretary, and certified as true copy by the legal custodian of the official records.36This strict
requirement for the registration of lands enunciated in TA.N Properties had been consistently applied and affirmed by the Court in a
plethora of cases.37
In the present case, to prove that the subject land is alienable and disposable, RRDC presented a CENRO certification stating that the
subject land is "alienable and disposable and not covered by any public land application." RRDC, however, failed to present a certified
true copy of the original classification approved by the DENR Secretary declaring the subject land alienable and disposable. Clearly, the
evidence presented by RRDC falls short of the requirements in TA.N. Properties. Thus, the trial and appellate courts erred when they
ruled that the subject land is alienable and disposable part of the public domain and susceptible to original registration.

Furthermore, RRDC also failed to prove that it and its individual predecessors-in-interest sufficiently complied with the required period
and nature of possession.

An applicant for land registration must exhibit that it and its predecessors-in-interest had been in open, continuous, exclusive, and
notorious possession and occupation of the land under a bona fide claim of ownership since 12 June 1945 or earlier. It has been held
that possession is open when it is patent, visible, apparent, notorious, and not clandestine; it is continuous when uninterrupted, unbroken,
and not intermittent or occasional; it is exclusive when the adverse possessor can show exclusive dominion over the land and an
appropriation of it to his own use and benefit; and notorious when it is so conspicuous, that it is generally known and talked of by the
public or the people in the neighborhood.38

In Republic vs. Remman Enterprises, Inc., 39 the Court held that for purposes of land registration under Section 14(1) of P.D. No. 1529,
proof of specific acts of ownership must be presented to substantiate the claim of open, continuous, exclusive, and notorious possession
and occupation of the land subject of the application. Applicants for land registration cannot just offer general statements which are mere
conclusions of law rather than factual evidence of possession. Actual possession is in the manifestation of acts of dominion over it of
such nature as a party would actually exercise over his own property.

In Republic v. Gielczyk, the Court explained that "possession" and "occupation" are not synonymous to each other. Possession is broader
than occupation because it includes constructive possession; whereas occupation delimits the all-encompassing effect of constructive
possession. Thus, taken together with the words open, continuous, exclusive, and notorious, the word occupation means that for one's
title to land to be judicially recognized, his possession of the land must not be mere fiction.40

In this case, aside from the deeds of absolute sale covering the subject land which were executed prior to 12 June 1945, RRDC did not
present any evidence which would show that its predecessors-in-interest actually exercised acts of dominion over the subject land even
before the cut-off period. As such, RRDC failed to prove that its possession of the land, or at the very least, its individual predecessors-
in-interest's possession over the same was not mere fiction.

Neither would the tax declarations presented by RRDC suffice to prove the required possession. To recall, the earliest of these tax
declarations dates back only to 1948. Clearly, the required possession and occupation since 12 June 1945 or earlier, was not
demonstrated.

From the foregoing, it is clear that RRDC failed to prove that its individual predecessors-in-interest had been in open, continuous, exclusive
and notorious possession and occupation of the subject land under a bona fide claim of ownership since 12 June 1945 or earlier; and
that said possession and occupation converted the subject land into a private property by operation of law. Consequently, the subject
land cannot be registered in the name of RRDC under Section 14(1) of P.D. No. 1529.

Requirements under Section 14(2) of P.D. No. 1529

RRDC also failed to establish compliance with the requirements for registration under Section 14(2).

In Heirs of Mario Malabanan vs. Republic (Malabanan),41 the Court explained that when Section 14(2) of P.D. No. 1529 provides that
persons "who have acquired ownership over private lands by prescription under the provisions of existing laws," it unmistakably refers to
the Civil Code as a valid basis for the registration of lands. The Civil Code is the only existing law that specifically allows the acquisition
by prescription of private lands, including patrimonial property belonging to the State.

The Civil Code makes it clear that patrimonial property of the State may be acquired by private persons through prescription.1âwphi1
This is brought about by Article 1113, which states that all things which are within the commerce of man are susceptible to prescription,
and that property of the State or any of its subdivisions not patrimonial in character shall not be the object of prescription.42

Nonetheless, this does not necessarily mean that when a piece of land is declared alienable and disposable part of the public domain, it
can already be acquired by prescription. In Malabanan, this Court ruled that declaration of alienability and disposability is not enough -
there must be an express declaration that the public dominion property is no longer intended for public service or the development of the
national wealth or that the property has been converted into patrimonial, thus:

"(2) In complying with Section 14(2) of the Property Registration Decree, consider that under the Civil Code, prescription is recognized
as a mode of acquiring ownership of patrimonial property. However, public domain lands become only patrimonial property not only with
a declaration that these are alienable or disposable. There must also be an express government manifestation that the property is already
patrimonial or no longer retained for public service or the development of national wealth, under Article 422 of the Civil Code. And only
when the property has become patrimonial can the prescriptive period for the acquisition of property of the public dominion begin to run.
"43 [emphasis supplied]

The classification of the land as alienable and disposable land of the public domain does not change its status as property of the public
dominion under Article 420(2) of the Civil Code. As such, said land, although classified as alienable and disposable, is insusceptible to
acquisition by prescription.44
In this case, RRDC did not present any evidence which would show that the subject land was expressly declared as no longer intended
for public service or the development of the national wealth, or that the property has been converted into patrimonial. Hence, it failed to
prove that acquisitive prescription has begun to run against the State, and that it has acquired title to the subject land by virtue thereof.

In fine, RRDC failed to satisfy all the requisites for registration of title to land under either Sections 14(1) or (2) of P.D. No. 1529. RRDC
also failed to establish that when it or P.N. Roa Enterprises, Inc., also a corporation and its direct predecessor-in-interest, acquired the
subject land, it had already been converted to private property, thus, the prohibition on the corporation's acquisition of agricultural lands
of the public domain under Section 3, Article XII of the 1987 Constitution applies. RRDC's application for original registration of imperfect
title over Lot No. 3009 must perforce be denied.

WHEREFORE, the instant petition is GRANTED.

LEONIDAS VS. VARGAS

Factual Antecedents

On February 2, 2002, Tomas R. Leonidas (herein petitioner) filed an application for land registration5 (Application) covering Lot 566 and
Lot 1677 which are both situated in Concepcion, Iloilo (collectively, subject lots).

Petitioner alleged that he inherited the subject lots from his parents, Ponciano Leonidas, Jr. (Ponciano) and Asuncion Roxas de Leonidas
(Asuncion); that as evidenced by the 1-fay 17, 193 7 Certificate of Sale issued by the Provincial Treasurer of Iloilo, the subject lots, then
covered by Tax Declaration (TD) No. 722, were purchased by Asuncion when auctioned due to delinquency in the payment of real
property taxes by the original owners, the heirs of Inis Luching; that Asuncion immediately took possession of the subject lots and
exercised dominical rights thereover notoriously, continuously, and exclusively; that upon Asuncion’s death in 1986, Ponciano succeeded
to the ownership and possession of the subject lots; that after Ponciano’s death in l 991, the subject lots became his (petitioner’s) own
exclusive property; that he permitted and tolerated the occupation of some portions of the subject lots by Juanito Tisolan, Pancing
Guevarra, Carmencita Guevarra, Delia .Aspera-Ecleo, Victorino Mosqueda, Nora Binas, Crisanto Amangas (Amangas),6 Rosana
Vasquez, Henry Asturias, Ronnie Astorias, Antonio Asturias, and Jacob Narciso; that as far as known to him (petitioner), the following
are the owners of all adjoining properties, i.e. the owners of Lot 564, Lot 565 Lot 1578, and Lot 1677, Mansueto Sicad, Francisco Aspero,
Brigido Celestial, and Eugenio Bondoc, Jr. who are all from Poblacion, Concepcion, Iloilo, and Carmen Paoli of unknown address; that
Lot 566 is bounded on the west by the provincial road and the (petitioner) does not claim any portion thereof; that the latest assessed
value of the subject lots is ₱52, 660.00 as certified by the Provincial Treasurer of Iloilo; that to the best of his knowledge and belief, there
is no mortgage or encumbrance of any kind whatsoever affecting the subject lots except for taxes due thereon; that a certain Tomas
Vargas (Tomas), however, had declared a portion of the subject lots in his name for taxation purposes; but that Tomas died shortly after
the end of the Second World War, and the whereabouts of his heirs, if any, are unknown, despite his diligent search to locate them in
Concepcion, Iloilo, and elsewhere.

Petitioner also alleged that he was 77 years old, Filipino, a resident of No. 55 Chestnut St., West Fairview, Quezon City, and married to
Ofelia Gustilo Leonidas (Ofelia); that attached to his Application were original Survey Plans with photographic copies each, the Tracing
Cloth Plan land Management Service, Department of Environment and Natural Resources (DENR), Region VI, Iloilo City, in lieu of the
surveyor’s Certificate, Technical Description with three photographic copies, the Certificate in quadruplicate of the Provincial Treasurer
showing the latest assessed value of the subject lots, and a copy of the muniment of title to prove ownership of the subject lots, with the
original to be presented at the trial.

Petitioner thus prayed that the subject lots be brought under the operation of the Property Registration Decree7 (PD 1529) and that the
titles thereto be registered and confirmed in his name.

The Republic of the Philippines (Republic), represented by the Office of the Solicitor General (OSG), opposed the said Application. The
Republic claimed that neither the petitioner nor his predecessors-in-interest had been in continuous, exclusive, and notorious possession
and occupation of the subject lots since June 12, 1945, or prior thereto, as required by Section 48 of Commonwealth Act (CA) No. 141,
as amended by PD 1073; that the petitioner’s muniment/s of title, tax declarations, and tax payment receipts did not constitute competent
and sufficient evidence of either a bona fide acquisition of the subject lots, and neither did the petitioner's bare claim of open, continuous,
exclusive, and notorious possession and occupation thereof in the concept of owner since June 12, 1945, or prior thereto, amount to
convincing proof of his claim of possession and ownership over the subject lots; that, although the petitioner's muniments of title might
appear genuine, the tax declarations and/or tax payments showing the pretended possession were, in fact, of recent vintage; that the
claim of ownership in fee simple on the basis of a Spanish title or grant could no longer be availed of by petitioner who had failed to file
an appropriate application therefor within the period of six months from Feb1uary 16, 1976, as required by PD 892; and that the subject
lots are portions of the public domain belonging to the Republic which are not subject to private appropriation. Thus, the Republic prayed
that the petitioner's Application be denied and that the subject lots be declared part of the public domain.

On March 11, 2003, Tancredo Vargas (Tancredo) also filed an Opposition8 to the Application. Tancredo averred that he is Tomas'
legitimate son and compulsory heir; that during Tomas's lifetime, the latter was the absolute and exclusive owner of a certain parcel of
land located at Loong, Concepcion, Iloilo, which parcel of land is bounded on the north by the seashore, on the south by Severino Asturias
(Asturias),9 on the east by the seashore, and on the west by Asturias and Braulio Celestial; that this parcel of land had an area of 36,237
square meters and was covered by TD No. 3549 in Tomas's name; that the petitioner does not exclusively own Lot 1677 since it had
been split into two, viz. Lot 1677-A and Lot 1677-B; that he (Tancredo) is the owner of Lot 1677-A; that Lot 566 was also not exclusively
owned by the petitioner, as this Lot 566 had also been divided into two lots, viz. Lot 566-A and Lot 566-B; that he (Tancredo) is the owner
of Lot 566-A as shown in the RPTA Tax Mapping project in the Municipal of Concepcion, Iloilo; that the petitioner's allegation that the
owners of the property covered by TD 772 became delinquent in the payment of the tax due thereon, for which reason the Provincial
Treasurer of Iloilo allegedly sold the same to Asuncion, was not at all true; that the property covered by TD 772 was not sold at public
auction because the forfeiture was lifted prior to the public auction sale; and that the fact that the Office of the Provincial Treasurer of
Iloilo did not have a copy of the Certificate of Sale dated May 17, 1937 bolstered the argument that petitioner’s allegation is questionable.
Tancredo thus prayed that the petitioner's Application be denied insofar as the portions covered by the TDs in the name of Tomas
(disputed portions) are concerned.

On March 21, 2003, another Opposition10 to the Application was filed by Moncerat A. Sicad-De Julian, Gil A. Sicad, represented by his
wife, Elizabeth Sicad, Teresita A. Sicad-Bayuran, Villaluz Sicad-Zarriz, Eden A. Sicad, and Melchor Sicad, represented by his wife, Elena
D. Sicad, (Elena; collectively, the Sicads) all represented by their attorney-in-fact, Elena.11 These oppositors claimed that they are the
heirs of the late Mansueto Sicad (Mansueto) who was the owner of a portion of the subject lots (Sicads’s contested portion); that the
Sicads's contested portion was bought by Mansueto from Asturias as evidenced by the Deed of Definite Sale of a Parcel of Land described
as Doc. No. 75, Page No. 35, Book No. 1, Series of 1950 of the notarial register of notary public Crespo Celestial; that the Sicads's
contested portion had been in the possession of Mansueto during the latter's lifetime; that they had been in possession of the Sicads’s
contested portion since Mansueto's death; that part of the Sicads’s contested portion had already been registered under Original
Certificate of Title (OCT) No. F-36795; and that the petitioner had never been in possession of the lots subject of his Application. The
Sicads thus prayed that the petitioner's Application be dismissed, insofar as it concerned the Sicads’s contested portion as set forth in
the aforesaid Deed of Definite Sale; and that the Sicads’s contested portion be registered instead in their names.

At the trial, the petitioner presented himself and Geronimo C. Pefiaflorida (Peñaflorida), Land Management Inspector, DENR, Community
Environment and Natural Resources Office (CENRO), at Sara, lloilo as witnesses.12 On the other hand, Catalino Guinez, Emeliana
Isturias Matulac, and Elena testified for the Sicads.13 For his part, Tancredo presented himself and a forn1er overseer or tenant of the
Vargas family,14 Jose Etchona (Etchona).15 Then on August 8, 2003, the petitioner filed his Formal Offer

of Evidence16 wherein he submitted the Certificate of Sale dated May 17, 1937, TD 014134 for the year 1976 in Asuncion's name and
covering Cadastral Lot Nos. 1, 2, and 3 PSU-216090, TD 0037 for the year 1994 in the names of Asuncion and Ponciano and covering
Cadastral Lot No. 1677, TD 0036 for the year 1994 in the names of Asuncion and Ponciano and covering Cadastral Lot No. 566, TD
0114_ for the year 2003 in the names of Asuncion and Ponciano and covering CadastraJ Lot No. 1677-A, TD 0118 for the year 2003 in
the names of Asuncion and Ponciano and covering Cadastral Lot No. 1677-B, TD 0116 for the year 2003 in the names of Asuncion and
Ponciano and covering Cadastral Lot No. 566-A; and TD 0117 for the year 2003 in the names of Asuncion and Ponciano and covering
Cadastral Lot No. 566-B,17 tax receipts for the years 1986, 1987, 1988, 1989, 1990, 1991, 1994, 2002 and 2003, statement of the
assessed value issued by the Provincial Assessor of Iloilo on March 26, 1996, Lot No. 566's Blue Print Survey Plan with technical
description, Lot l 677’s Blue Print Survey Plan with technical description, Certificate of Unavailability of Surveyor's Certificate of Survey
for Lots 566 and 1677, and Survey Inspection Report dated August 28, 1997 for Lot Nos. 566 and 1677 issued by Peñaflorida,18 i.e.
CENRO Report dated August 28, 1997, to the effect that the subject lots are free from liens and encumbrances, and are moreover within
the alienable and disposable area. Pursuant to the RTC's directive, petitioner also offered as additional evidence the originally-approved
subdivision plan covering Lot No. 1677, Csd-06-008798 to prove the identity and location of the easement for public use;19 and a
certification by Joel B. Diaz, CENRO at Sara, Iloilo, to the effect that Lot No. 1677, Pls 1099, situated in Brgy; Loong, Concepcion, Iloilo,
with an area of 8,062 square meters was issued Patent No. 063015-92-846 dated May 28, 1992 in the name ofFlordeluz Sedigo, but that
Lot No. 1677 has doubled with the lot situated at Poblacion, Concepcion, Iloilo in the name of the Heirs of Ponciano and that this latter
lot is not covered by any public land application filed with the CENRO in Sara, Iloilo, which explained why no patent has been issued
therefor, hence indicating that this other Lot No. 1677, Pls 1099, which is situated in Brgy. Aglusong, Concepcion, Iloilo is entirely different
from Lot No. 1677, which is situated in Sitio Loong, Poblacion, Concepcion, Iloilo.20

The petitioner likewise submitted in evidence an Ocular Inspection Report covering an ocular inspection earlier ordered by the RTC.21

Ruling of the Regional Trial Court

In its Decision dated March 19, 2007, the RTC disposed of this case in this wise:

WHEREFORE, general default having been declared and the [A]pplication supported by evidence, the adjudication and registration of
portion of Lot No. 566 with an area of 3.1161 hectares and portion of Lot 1677 with an area of 3. 7255 hectares, all of Concepcion
Cadastre, together with all the improvements thereon are hereby ordered in favor of applicant [petitioner], of legal age, married to [Ofelia],
Filipino, and resident of Fairview, Quezon City, Philippines. Portions of Lot [No.] 1677 with an area of 2.3642 hectares and portion of Lot
[No.] 566 with an area of 1.1782 hectares are hereby adjudicated in favor of [Tancredo ], of legal age, single, Filipino, and resident of
Lawa-an Village, Balantang, Jaro, Iloilo City, Philippines which portions shall be segregated in a proper subdivision survey and to follow
the description of the plan of Municipal Assessor of Concepcion, Iloilo commensurate to Lot 1677-A under [T.D.] No. 054822 and 566-A
under [T.D.] No. 0550.

The easement of right of way of the lots, highways, streets, alleys, shorelines and other portion[s] of land not specified as lots located
within the borders of the land covered by this case are declared to be the properties of the [Republic].

The Clerk of Court is directed to forward copies of this decision to all government agencies concerned.

And finally, the Administrator, Land Registration Authority, is hereby directed, after this decision shall have become final for which he
shall be duly advised by specific order of this Court, to issue [a] decree of registration and title in accordance with the amended plan on
file in the record.

SO ORDERED.23
The RTC held that petitioner had sufficiently established that his predecessors-in-interest had possessed and owned a parcel of land in
Barangay Loong, Concepcion, Iloilo to the extent not covered by Tancredo’s Opposition; that while petitioner and his predecessors-in-
interest might not have been in actual possession of the subject lots at all time, they nonetheless had been consistently visiting the same;
and that petitioner’s claim of possession and ownership is supported by documents consisting of the Certificate of Sale issued by the
Provincial Treasurer of Iloilo on May 17, 1937, the tax declarations in Asuncion's name for the years 1976, 1994, and 2003, the official
receipts showing payments of real estate taxes thereon, and the statement of the assessed value issued by the Provincial Assessor of
Iloilo on May 26, 1996. The RTC stressed that the period of possession by petitioner and his predecessors-in-interest sufficed to confer
a registrable title upon petitioner.

The RTC likewise ruled that Tancredo was also able to establish a superior claim with respect to his disputed portions; that all of the tax
declarations in Asuncion's name continuously bore the annotation acknowledging Tomas's adverse claim relative to Tancredo's disputed
portions; that Tomas's open and continuous possession for more than the required number of years was sufficiently shown by a tax
declaration issued as early as the year 1945; that the overseers and other persons authorized to manage Tancredo's disputed portions
were never driven out by petitioner; and that Tancredo had visited the disputed portions more frequently than petitioner who, as the
evidence shows, has his permanent residence in Quezon City, Metro Manila.

With regard to the claim of the Sicads, the RTC held that Mansueto and his successors-in-interest had no more interest in the Sicads'
contested portion because what was shown to have been sold by Asturias to Mansueto pertained to a lot measuring only two hectares,
52 acres, and 92 ares, a parcel of land at par with the land covered by the aforementioned free patent issued to Mansueto.

The RTC emphasized that it is well-entrenched in jurisprudence that alienable public land openly, continuously, and exclusively possessed
by a person personally or through his predecessors-in-interest for at least 30 years becomes ipsojure private property by mere lapse of
time, or by completion of said period pursuant to Section 48(b) of CA 141, as amended by RA 1942 and RA 3872.

Ruling of the Court of Appeals

Only the petitioner and the Republic filed their respective Notices of Appeal24 which were given due course by the RTC in its Order of
May 25, 2007.25 These notices of appeal were consolidated and docketed as CA-G.R. CV No. 02296. In a Decision dated August 13,
2009, the CA disposed as follows:

WHEREFORE, the Decision dated March 19, 2007 is modified, as follows: 1.) the portion pertaining to the award of [Lot No.] 566 with an
area of3.1161 hectares and [Lot No.] 1677 with an area of 3.7255 hectares to [petitioner], is REVERSED and SET ASIDE; and 2.) the
portion pertaining to the award of [Lot No.] 1677 with an area of 2.3642 hectares and [Lot No.] 566 with an area of 1.1782 hectares in
favor of [Tancredo] is AFFIRMED.

SO ORDERED.26

The CA held that, contrary to the Republic's stance, the records showed that there had been compliance with the jurisdictional
requirements of publication, posting, and notice; that petitioner had properly identified the subject lots; that the subject lots had already
been classified as alienable and disposable at the time that petitioner filed the Application in 2002, pursuant to the CENRO Report dated
August 28, 1997 issued by Peñaflorida; that it has been held that "[a] certification by the CENRO of the DENR stating that the subject lots
are found to be within the alienable and disposable site per land classification project map is sufficient evidence to show the real character
of the land subject of the application;"27 that these notwithstanding, petitioner failed to prove with the requisite evidence the kind of
possession and the length of time required by law for the registration of the subject lots in his name, because his lone testimony did not
suffice to establish his and his predecessors-in-interest’s alleged open, continuous, exclusive, and notorious possession over the subject
lots since June 12, 1945, or earlier; that petitioner's alleged acts of swimming in, and planting trees on the subject lots, his having finished
high school at the Victorino Salcedo High School in the neighboring town of Sara, Iloilo, and his having left the subject lots when he
attended college - all these neither added up nor supported his assertion of dominion or ownership· over the subject lots; that his allegation
that his childhood memories regarding the subject lots all came back to him after the death of his father Ponciano was indicative of the
fact that he was really unaware of the existence of the subject lots; that his Application was even opposed by Tancredo and by the Sicads
who claimed exclusive possession over certain portions of the subject lots; that petitioner's failure to explain why he or his predecessors-
in-interest declared the subject lots for taxation purposes only in 1976, was inconsistent with his claim of possession thereover since
1937; and that it is an axiom of the law that the burden of proof in a land registration case rests upon the applicant who mu st present
clear, positive, and convincing evidence establishing the alleged possession and occupation in good faith, and for the period required by
law.

On the other hand, the CA ruled that Tancredo had sufficiently proven his open, continuous, exclusive, and notorious possession and
occupation for the period required by law, over the portions of the subject lots he was claiming in the concept of an owner; that Tomas's
adverse claims were annotated on the TDs issued in Asuncion’s name covering the disputed portions, i.e. TD 014134, 0114, and 0117
;28 that Tomas declared the disputed portions for taxation purposes in his name as early as 1945; that Tancredo himself testified that
Tomas first used the disputed portions as rice land and converted the same into coconut land in the 1960s; that Tancredo’s witness,
Etchona, likewise testified that Tomas employed him and Domingo Celestial not only to cultivate, but also to guard the disputed portions,
and that Tomas himself appropriated the harvest from the disputed portions and introduced improvements thereon; and that even
petitioner himself admitted in his Application that Tomas had declared the disputed portions in his (Tomas’) name for taxation purposes.

Petitioner moved for reconsideration29 but was denied by the CA in its Resolution of February 22, 2012.30

Issue
Before this Court, petitioner now raises the following issue:

[Whether] the [CA] gravely abused its discretion in denying the registration of [his] already vested title [over] Lot [Nos.] 566 and 1677 of
the Concepcion, Iloilo Cadastre as his private property, and in awarding some portions thereof in favor of [Tancredo] in this land
registration proceeding.31

Petitioner’s arguments

Petitioner insists in his Petition,32 Consolidated Reply,33 and Memorandum34 that the CA erred in finding that he failed to prove that he
and his predecessors-in-interest had been in open, continuous, exclusive, and notorious possession and occupation of the subject lots
since June 12, 1945, or earlier, and that there is indubitable evidence that the subject lots were in fact sold in a tax sale on May 1 7, 1937
by the gove1nment through the Provincial Treasurer of Iloilo; that he filed the present Application so that an OCT can be issued in his
name as evidence of his vested title over the subject lots; that assuming that the subject lots are still part of the public domain, he is
nevertheless still entitled to have the subject lots registered in his name by reason of his and his predecessors-in-interest's exclusive
possession and occupation thereof for more than 30 years, as compared to Tancredo's possession which supposedly began only in 1945;
that under the Land Registration Act, as amended, the possessor is deemed to have acquired by operation of law the right to a government
grant upon compliance with the conditions therefor, which was just what he did in this case; that the confirmation proceeding is a mere
formality and the registration thereunder does not confer title but merely recognizes a title that is already vested; that rejection of his
vested title to the questioned lots will occasion loss of confidence in the government's sales of forfeited property by reason of tax
delinquency; that the CA erred in finding that the TDs in Asuncion's name carried Tomas's adverse claim, as the attached copies thereof
did not bear any such annotations; that the CA also erred in stating that petitioner did not present any TDs to support his claim of ownership
over the subject lots for the reason that the CA Decision itself mentioned that he submitted a TD for the year 1976; that contrary to the
CA's findings, he did testify that he had visited the subject lots every so often to plant trees after he and his parents left Concepcion in
1945, and that such improvements were reflected in his exhibits; that the CA likewise erred in holding that he only came to know about
the subject lots after the death of his father, Ponciano, for the fact is that he did testify that he and his cousins used to swim in the sea
near the subject lots, as early as when he was 12 years old; that the CA moreover erred in concluding that Tancredo had successfully
established his claims over the disputed portions of the subject lots because the TDs in Asuncion's name are all annotated with Tomas’s
adverse claim, and that Tomas had declared said disputed portions in his name as early as 1945; that the tax declarations supposedly in
Tomas's name were neither presented nor offered in evidence; that Tancredo admitted during his cross-examination that Tomas’s 1945
tax declaration was procured notwithstanding the fact that the subject lots had already been declared in Asuncion’s name; that Tancredo
did not comply with the pertinent provisions of the Land Registration Act, as amended, because he did not present evidence to prove the
specific date in 1945 when Tomas acquired the disputed portions, or how Tomas in fact acquired the same; that besides these, Tancredo
could not identify the disputed portions that he was claiming; that if Tancredo wanted to vindicate his claims of ownership over the disputed
portions, then Tancredo should institute the proper action before a court of general jurisdiction, and not in the land registration court, as
the subject lots were no longer part of the public domain; that the issue of whether the sale by the government to Asuncion on May 17,
1937 changed the classification of the subject lots from public to private is of first impression and should be resolved by the Supreme
Court EnBanc; and that the circumstances obtaining in this case are exceptions to the rule that only questions of law are allowed in a
petition filed pursuant to Rule 45 of the Revised Rules of Court; and that to deny his Application, or to render judgment ordering the
reversion to public ownership of the subject lots would amount to grave abuse on the part of the judiciary.

The Republic’s Arguments

In its Comment35 and Memorandum,36 the Republic counters that the instant Petition merely raises questions of fact which are proscribed
under Rule 45 of the Revised Rules of Court; that this Court is not a trier of facts; that petitioner's case does not fall under any of the
exceptions to the rule that factual findings of the CA are invariably binding upon the Supreme Court; and that the assailed CA Decision
should not be disturbed because the CA had amply justified the reversal of the RTC Decision which was erected upon the petitioner's
failure to substantiate his claim of ownership over the subject lots.

Tancredo’s Arguments

In his Comment37 and Memorandum,38 Tancredo maintains that the disputed portions had been in the absolute possession and
dominion of Tomas; that the findings of the RTC and the CA regarding petitioner's ineligibility to obtain title to the disputed portions due
to non-compliance with the requirements of the law, and for insufficiency of evidence, should not be disturbed; that the CA’s finding that
petitioner's TDs bore the annotated claims of Tomas on the subject lots is a factual finding and should not be disturbed; that petitioner's
possession is not the possession required by law for purposes of land registration because petitioner failed to present evidence that would
prove actual, notorious, continuous, and exclusive possession and occupation of the subject lots; that the evidence adduced by petitioner
is self-serving, hence undeserving of any weight; that the origin of the disputed portions as pointed out by the RTC is Assessor's Lot No.
337, which is individually identified after the Cadastral Survey as Lot Nos. 1676- A, 1677-A, and 566-A, all of the Concepcion (Iloilo)
Cadastre; that petitioner is barred or estopped from questioning the identity of the disputed portions that had been adjudicated to him
(Tancredo ), as the lack of sufficient identification pertained to the subject lots that petitioner himself was trying to register; and that the
issues raised by petitioner were factual in nature, and the same is proscribed under Rule 45 of the Revised Rules of Court.

The fundamental issues to be resolved in this case are: (1) Whether the petitioner is entitled to obtain a title over the subject lots; and (2)
Wether Tancredo has established, by his own evidence, that he was qualified to acquire title over the disputed portions claimed by him.

The Court’s Ruling

The Petition is denied.


Requisites for the confirmation and
registration of an imperfect and
incomplete title under CA 141 and
PD 1529

"The Regalian doctrine, embodied in Section 2, Article XII of the 1987 Constitution, provides that all lands of the public domain belong to
the State, which is the source of any asserted right to ownership of land."39 "[Commonwealth Act No. 141, in turn,] goven1s the
classification and disposition of lands of the public domain. Section 11 [thereof] provides, as one of the modes of disposing public lands
that are suitable for agriculture, the 'confirmation of imperfect or incomplete titles.’ Section 48 [thereof], on the other hand, enumerates
those who are considered to have acquired an imperfect or incomplete title over public lands and, therefore, entitled to confirmation and
registration under the Land Registration Act [now PD 1529]."40 The latter law then "specifies who are qualified to apply for registration of
land."41 Taken together, all the foregoing provide for the requisites for the confirmation and registration of an imperfect and incomplete
title, thus -

x x x In particular, Section 14 (l) [of PD 1529] in relation to Section 48 (b) of[CA] 141, as amended by Section4 of P.D. No. 1073, states:

SEC. 14. Who may apply. -The following persons may file in the proper Court of First Instance [now Regional Trial Court] an application
for registration of title to land, whether personally or through their duly authorized representatives:

(1) Those who by themselves or through their predecessors-in-interest have been in open, continuous, exclusive and notorious
possession and occupation of alienable and disposable lands of the public domain under a bonafide claim of ownership since June 12,
1945, or earlier.

xxx xxx xxx

Section 48. The following described citizens of the Philippines, occupying lands of the public domain or claiming to own any such lands
or an interest therein, but whose titles have not been perfected or completed, may apply to the Court of First Instance [now Regional Trial
Court] of the province where the land is located for confirmation of their claims and the issuance of a certificate of title therefor, under [PD
1529], to wit:

xxx xxx xxx

(b) Those who by themselves or through their predecessors-in-interest have been in open, continuous, exclusive and notorious
possession and occupation of [alienable and disposable lands] of the public domain, under a bonafide claim of acquisition of ownership,
since June 12, 1945, or earlier, immediately preceding the filing of the application for confirmation of title except when prevented by war
or force majeure. These shall be conclusively presumed to have performed all the conditions essential to a Government grant and shall
be entitled to a certificate of title under the provisions of this chapter.

Based on these legal parameters, applicants for registration of title under Section 14 (1) must sufficiently establish: (1) that the subject
land forms part of the disposable and alienable lands of the public domain; (2) that the applicant and his predecessors-in-interest have
been in open, continuous, exclusive and notorious possession and occupation of the same; and (3) that his possession has been under
a bonafide claim of ownership since June 12, 1945, or earlier.

These triple requirements of alienability and possession and occupation since June 12, 1945 or earlier under Section 14 (1) are
indispensable prerequisites to a favorable registration of title to the property. Each element must necessarily be proven by no less than
clear, positive and convincing evidence; otherwise, the application for registration should be denied.42

Petitioner did not cite the specific provision of CA 141 upon which he based his Application. Neve1theless, the allegations therein seem
to establish the fact that his claim is one of imperfect title under the above- quoted Section 48(b) of CA 141 in relation to Section 14(1) of
PD 1529

The subject lots are considered


alienable and disposable lands of the
public domain

The first· requirement is complied with in the case at bench. Notwithstanding that only a CENRO certification covering the subject lots
was presented in the instant case, the subject lots are considered alienable and disposable lands of the public domain because of this
Court’s ruling that an application for land registration may be granted despite the absence of the DENR Secretary's certification, provided
that the same was pending at the time Republic v. Vega43 was promulgated on January 17, 2011. In Republic v. Alora,44 this Court
expressly clarified this matter in this wise:

x x x [I]n Republic v. T.A.N Properties, Inc., which was promulgated on 26 June 2008 x x x we held that applicants for land registration
must present a copy of the original classification approved by the DENR Secretary and certified as true copy by the legal custodian of the
official records. x x x

x x x In Republic v. Serrano [(decided on 24 February 2010)], we allowed the approval of a land registration application even without the
submission of the certification from the DENR Secretary. As this ruling presented an apparent contradiction with our earlier
pronouncement in Republic v. T.A.N Properties, Inc., we sought to harmonize our previous rulings in Republic v. vega [(decided on 17
January 201 l)]. We then said that the applications for land registration may be granted even without the DENR Secretary's certification
provided that the application was currently pending at the time Republic v. vega was promulgated. x x x45

It is worth stressing, however, that the foregoing ruling is the exception, not the rule. As explicitly elucidated in Republic v. Vega:46

It must be emphasized that the present ruling on substantial compliance applies pro hac vice. It does not in any way detract from our
rulings in Republic v T.A.N Properties, Inc., and similar cases which impose a strict requirement to prove that the public land is alienable
and disposable, especially in this case when the Decisions of the lower court and the [CA] were rendered prior to these rulings. To
establish that the land subject of the application is alienable and disposable public land, the general rule remains: all applications for
original registration under [PD 1529] must include both (1) a CENRO or PENRO certification and (2) a certified true of the original
classification made by the DENR Secretary.

As an exception, however, the courts - in their sound discretion and based solely on the evidence presented on record - may approve the
application, pro hac vice, on the ground of substantial compliance showing that there has been a positive act of government to show the
nature and character of the land and an absence of effective opposition from the government. This exception shall only apply to
applications for registration currently pending before the trial court prior to this Decision and shall be inapplicable to all future applications.
(Underscoring and emphases in the original)47

That said, we hold that both the petitioner and Tancredo failed to establish clearly and convincingly their respective rights to registration
of imperfect titles under CA 141 and PD 1529, as will be discussed below.

Petitioner failed to prove possession


of the subject lots in the manner and
for the period required by law

First off, petitioner failed to establish bonafide possession and ownership over the subject lots since June 12, 1945 or earlier. His
contention that his predecessors-in-interest became the owners of the subject lots pursuant to the May 17, 1937 Certificate of Sale48 of
the Forfeited Real Property issued by the Provincial Treasurer of Iloilo appears to be consistent with the fact that TD 3549 in Tomas’s
name which was found by the CA as issued in 1945 bears an annotation stating that such is "[ c ]ontested by [Asuncion]".49 Even then,
the Certificate of Public Sale indicated that the balance of the purchase price in the amount of ₱29.44, was yet to be paid on or before
December 31, 1937.50

No incontrovertible proof was, however, presented to establish the fact that this balance of the purchase price in the said amount of ₱29
.44 had indeed been paid on or before December 31, 193 7. In addition, the CA also correctly pointed out that even as petitioner was
able to submit TDs and evidence of tax payments only for a few years, he nevertheless failed to explain why he or his predece ssors-in-
interest declared the subject lots for taxation purposes only in 1976, this despite his claim that his predecessors-in-interest had been in
possession and occupation of the subject lots since 1937, as allegedly shown in the Provincial Treasurer’s Certificate of Sale. It is settled
that intermittent and irregular tax payments run counter to a claim of ownership or possession.51

Second, even assuming for argument's sake that petitioner’s predecessors-in-interest had paid the balance of the delinquent tax payment,
petitioner nonetheless failed to prove his and his predecessors-in-interests actual, notorious, exclusive and continuous possession of the
subject lots for the length of time required by law.

To be sure, petitioner's failure to explain what happened after his family supposedly left the subject lots in 1941, when the war broke out,
vis-a-vis his failure to prove that he had indeed introduced valuable improvements in the subject lots during the time that he and his
parents had been allegedly in actual possession and occupation thereof, cast doubts upon his claim of actual possession and occupation
thereof. Withal, petitioner's testimony of having swum near the subject lots, of having planted trees thereon, and his having finished high
school at the Victorino Salcedo High School in the neighboring town of Sara can hardly be considered as acts of dominion or ownership
over the subject lots. Besides, petitioner did not present clear and convincing evidence that the subject lots had indeed been cultivated
by him or by his predecessors-in-interest for the period of time required by law. Needless to say, all these failings weaken his claim that
he has been a bonafide possessor and occupant of the subject lots in the manner and for the period prescribed by law, to wit:

The possession contemplated by Section 48 (b) of [CA] 141 is actual, not fictional or constructive. In Carlos v. Republic of the Philippines,
the Court explained the character of the required possession, as follows:

The law speaks of possession and occupation. Since these words are separated by the conjunction and, the clear intention of the law is
not to make one synonymous with the other. Possession is broader than occupation because it includes constructive possession. When,
therefore, the law adds the word occupation, it seeks to delimit the all-encompassing effect of constructive possession, Taken together
with the words open, continuous, exclusive and notorious, the word occupation serves to highlight the fact ti1at for an applicant to qualify,
his possession must not be a mere fiction. Actual possession of a land consists in the manifestation of acts of dominion over it of such a
nature as a party would naturally exercise over his own property.52 (Emphases in the original)

Oddly enough, while in its Decision the RTC appeared to have granted petitioner’s Application, said Decision seemed to have indulged
in a bit of non-sequitur when it said that "[petitioner] and his predecessors were not in actual possession of the [subject Jots] all the time"
x x x.53 Simply said, the -A effectively ruled that since petitioner failed to prove that he or his predecessors-in-interest had indeed
performed the required acts of possession and occupation, or specific acts of dominion over the subject lots, it stands to reason that
registration thereof in his name cannot be allowed.

Tancredo also failed to establish


possession and occupation over the
disputed portions in the manner and
for the period required by law

At this juncture, we shall revisit the uniform finding by both the RTC and the CA, which in effect upheld Tancredo's right to register the
disputed portions in his name (as an exception to the settled rule that questions of fact are proscribed in a Rule 45 petition since a correct
evaluation of the facts will yield a different conclusion).54

First off, Tancredo failed to show that his or his predecessor-ininterest’s possession and occupation over the disputed portions had been
under a bonafide claim of ownership since June 12, 1945, or earlier. We are inclined to agree with petitioner's posture that Tancredo
failed to adduce clear and convincing evidence which established the origin or antecedents of Tomas's straightforward possession and
occupation, or claim of ownership, over the disputed portions. Consider the following exchange/s between/among Tancredo, the
petitioner, and the Court -

[Petitioner]: (to the witness[, Tancredo])

Q: When did your father acquire this property?

A: In 1945.

Q: From whom?

A: I have no idea.

xxxx

Q: Did you not ask your father from whom he acquired this property?

A: No, I did not.

Q: As a matter of fact[,] until the death of your father[,] you have not ask[ed] him from whom did he acquire the property?

A: No, Sir.

xxxx

COURT: (to the witness[, Tancredo])

Q: Your father died in 1995 [,] why did you not [cause] the transfer of tax declaration in your name or to the heirs?

A: Because the plan of the heirs is, if the property [is registered] in my father[']s name [then] the title should be transferred in my name.

xxxx

Q: Your tax receipts correspond only [to] the year 2003, how about other tax receipts?

A: I [will just [try] to find out if the Provincial Treasurer’s Office still has the copy.

Q: Even just a certification stating that you [continued] in paying realty tax from 1946 up to 2003?

A: Yes, I can ask the provincial treasurer for that matter.

Q: When you secure[d] the tax declaration[,] you [knew] that the lot was also declared in the name of [Asuncion], is it not?

A: Yes, Your Honor.

Q: That was in the office of the Municipal Assessor?

A: Yes, Your Honor.

Q: Did you verify if they were paying taxes also?

A: No, Your Honor.

Q: You did not?

A: I [did] not[,] Your Honor.


Q: If that is the case[,] why did you [say] a while ago that you [knew]. only [about] the case of [petitioner] when this case was filed because
the tax declaration itself [stated] that the lot was also declared in the name of [Asuncion]?

A: Although I have already seen the notation on the tax declaration that they also [secured a] tax declaration [over] the [disputed Portions].
I did not mind it Your Honor because they did not openly claim ownership over the [disputed portions]. And in the same manner[.] Your
Honor[,] in their tax declaration it is also indicated that the [disputed portions] is also declare[d] in the name of [Tomas].55

More than this, Tancredo did not present clear, convincing evidence to support his claim that the disputed portions were in fact transferred
to him by his father, Tomas. Tancredo merely testified that the disputed portions were given to him solely by Tomas, an act that was
allegedly consented to by his siblings. Thus-

[Petitioner]: (to the witness, Tancredo)

Q: You have siblings, meaning brothers and sisters?

A: Yes, Sir.

Q: You said a while ago that you succeeded to the ownership of the [subject lots] when your father died in 1985, how about your siblings[?]
[Did they] not succeed to the [ownership of the subject lots?]

A: They sign[ed] a deed of adjudication in favor of me[.] I have a copy and it was notarized.

xxxx

Q: In your [O]pposition you said that you were authorized?

A: Yes, Sir.

Q: By whom?

A: By my brothers and sisters.

Q: Where is your authority?

A: I can produce it. I can pass [sic] it anytime.

Q: You did not [s]tate in your [O]pposition that you have your siblings with you?

A: Because the property was given to me by my father. 56

Nonetheless, there is nothing in the records to support or confirm Tancredo’s claim that the property was in fact deeded over to him by
his father, Tomas.

In Buenaventura v. Pascual,57 this Court affirmed the lower courts' dismissal of the claims for registration of imperfect titles because,
among others, both the applicant and oppositors failed to adduce evidence as to how they acquired the subject property from their
respective predecessors-in-interest, i.e., whether by succession or by donation or by some other mode. Furthermore, we stressed therein
that the applicant failed to prove the manner by which her predecessors-in-interest possessed the subject property.

Then, again, Tancredo also failed to establish that he and his predecessors-in-interest had/have been in open, continuous, exclusive and
notorious possession and occupation of the disputed portions since June 12, 1945, or prior thereto.

If anything, the records showed that Tancredo merely submitted photocopies of four tax declarations which were attached as annexes to
his Opposition.1âwphi1 These included the 1945 TD 3549 as adverted to by the CA in the records58 pertaining to a 3.6237-hectare lot
in an unstated cadastral lot, TD 0548 covering an 813-hectare lot in Cadastral Lot No. 1676-A,59 TD 0549 for a 2.3642-hectare lot in
Cadastral Lot No. 1677-A,60 and TD 0550 concen1ing a 1.1782-hectare lot in Cadastral Lot No. 566-A.61 All four TDs are in Tomas's
name, without copies of the dorsal portions thereof, and bearing annotations stating either "[c]ontested by [Asuncion]" or "[a]lso declared
in the name of [Asuncion] or [Ponciano]".

It would thus appear that Tancredo had erected his opposition/claim to the lots in question upon the said photocopies of four tax
declarations whose authenticity or genuineness is open to the most serious doubts. And, even on the assumption that the said tax
declarations are in fact authentic and genuine, still it is settled that tax declarations are not conclusive proof of ownership. If anything, tax
declarations are merely corroborative of a person's claim of possession. More than that, as elsewhere indicated, intermittent and irregular
tax payments, as in this case, do not really provide strong support for a claim of ownership or possession.62

It is axiomatic of course that "[i]t is the policy of the State to encourage and promote the distribution of alienable public lands as a spur to
economic growth and in line with the social justice ideal enshrined in the Constitution. At the same time, the law imposes stringent
safeguards upon the grant of such resources lest they fall into the wrong hands to the prejudice of the national patrimony."63 This ruling
controls the present case.
As a final note: All of the foregoing discussion showed that the issues raised in this case have all been previously resolved and determined
by settled jurisprudence; hence, there is no reason to grant petitioner's prayer for this case to be referred to or heard by the Court EnBanc,
as this is not a case of first impression at all.

WHEREFORE, the Petition is hereby DENIED.

REPUBLIC VS. MENDIOLA

FACTS:

On July 27, 2007, respondents filed a verified application for registration of title to land under Presidential Decree (P.D.) No. 1529,5 as
amended, otherwise known as the Property Registration Decree over the subject property before the RTC of Pasig City.6 They claimed
that they inherited the subject property from their late parents and have been in physical and continuous possession thereof in the concept
of an owner even before June 17, 1945.7

During the initial hearing, considering that no opposition to the application was registered, the R TC issued an order of general default
except against herein petitioner.

Upon presentation of evidence, the respondents submitted the following: (i) Conversion Plan and Geodetic Engineer's Certificate of the
subject property; (ii) Tax Declarations; and (iii) the Certification from the Department of Environment and Natural Resources (DENR)-
National Capital Region (NCR) verifying the subject property as alienable and disposable.8

The Ruling of the RTC

On February 22, 2010, the RTC rendered its Order9 wherein it ruled that the respondents herein have sufficient title deemed proper for
registration under P.D. No. 1529. The dispositive portion thereof reads:

WHEREFORE, judgment is hereby rendered thus: the title of Margarita C. Mendiola, widow; Lualhati M. Talavera, married to Celso
Talavera; Zenaida M. Estacio, widow; Francisco C. Mendiola, Jr., married to Corazon Marindo; Estrellita M. Espiritu, married to Danilo
Espiritu; Mario C. Mendiola, married to Leticia Mendiola; Wilfredo C. Mendiola, married to Teresita E. Mendiola; Liwaywa y C. Mendiola,
single; Orlando C. Mendiola, married to Melinda Mendiola; Sherry Comeling, married to Antonio Comeling; Mamencia M. Lacsa, married;
Racquel Lacsa, married to Ferdinand San Juan; Paraluman M. Casinsinan, married to Leonardo Casinsinan, to that parcel of land (as
described on conversion survey plan, Swo-007607-000730-D, being a portion of Lot 2750, Mcadm-590-D, Taguig Cadastral Mapping),
situated in Brgy. lbayo, Tipas, Taguig, Metro Manila consisting of more or less 1,256 square meters with the aforequoted technical
descriptions, is hereby CONFIRMED.

Upon the finality of the judgment, let the proper Decree of Registration and Certificate of Title be issued to the applicants pursuant to
Section 39 of P.D. [No.] 1529.

Let two (2) copies of this Order be furnished [to] the Land Registration Authority Administrator Benedicto B. Julep, thru Salvador L. Oriel,
the Chief of the Docket Division of said Office, East A venue, Quezon City.

SO ORDERED.10

The RTC held that the subject property was determined to be alienable and disposable as per Certification issued by the DENR-NCR
dated January 3, 1968.1âwphi1 Also, it held that the respondents had acquired title to the subject property after finding that they have
been in continued possession thereof for more than 30 years.11

Thus, petitioner, represented by the Office of the Solicitor General (OSG), filed an appeal under Rule 41 before the CA.

The Ruling of the CA

On September 20, 2013, the CA issued its Decision12 wherein it denied the appeal of the OSG and accordingly affirmed the Order dated
February 22, 2010 of the RTC. The dispositive portion thereof states:

WHEREFORE, premises considered, the instant appeal is hereby DENIED for lack of merit. Accordingly, the assailed Order of Branch
266 of the [RTC] of Pasig City dated February 22, 2010 is AFFIRMED.

SO ORDERED.13

In denying the appeal, the CA asseverated that respondents sufficiently established their entitlement over the property by presenting
evidence relevant to their possession and occupation of the property. Moreover, the CA based its ruling on the declaration in Rep. of the
Phils. v. Serrano, et al. 14 which allowed the registration application even without the submission of the certification from the DENR
Secretary classifying the land as alienable and disposable:

While Cayetano failed to submit any certification which would formally attest to the alienable and disposable character of the land applied
for, the Certification by DENR Regional Technical Director Celso V. Loriega, Jr., as annotated on the subdivision plan submitted in
evidence by Paulita, constitutes substantial compliance with the legal requirement. It clearly indicates that lot 249 had been verified as
belonging to the alienable and disposable area as early as July 18, 1925.
The DENR certification enjoys the presumption of regularity absent any evidence to the contrary. It bears noting that no opposition was
filed or registered by the Land Registration Authority or the DENR to contest respondents' applications on the ground that their respective
shares of the lot are inalienable. There being no substantive rights which stand to be prejudiced, the benefit of the Certification may thus
be equitably extended in favor ofrespondents.15 [Emphasis and underscoring deleted)

Consequently, petitioner filed a petition for review on certiorari under Rule 45.

The Issue

Essentiaily, the main issue ir1 the present case is whether or not the CA erred in affirming the findings of the trial court that herein
respondents are entitled to their application for registration of title over the subject property.

The Ruling of the Court

The petition is meritorious.

The conversion plan, technical descriptions of the property, and the Certification issued by the DENR-NCR are insufficient proof of the
alienable and disposable character of the subject property. Clearly, respondents failed to prove their entitlement thereto under Chapter
III, Section 14 of P.D. No. 1529, which states:

Sec. 14. Who may apply. The following persons may file in the proper Court of First Instance an application for registration of title to land,
whether personally or through their duly authorized representatives:

(1) Those who by themselves or through their predecessors-in-interest have been in open, continuous, exclusive and notorious
possession and occupation of alienable and disposable lands of the public domain under a bona fide claim of ownership since June 12,
1945, or earlier.

xxxx

Thus, it is imperative for an applicant for registration of title over a parcel of land to establish the following: (i) possession of the parcel of
land under a bona fide claim of ownership, by himself and/or through his predecessors-in-interest since June 12, 1945, or earlier; and (ii)
that the property sought to be registered is already declared alienable and disposable at the time of the application.

In the present case, respondents submitted DENR-NCR's Certification dated May 22, 2009, wherein it stated that the subject property
was alienable and disposable.16

The Court, however, finds respondents' reliance on the Certification issued by the DENR-NCR misplaced.

In Rep. of the Phils. v. Lualhati, 17 the Court ruled that the applicant for land registration must prove that the DENR Secretary had
approved the subject property as alienable and disposable, to wit:

Accordingly, in a number of subsequent rulings, this Court consistently deemed it appropriate to reiterate the pronouncements in T.A.N.
Properties in denying applications for registration on the ground of failure to prove the alienable and disposable nature of the land subject
therein. In said cases, it has been repeatedly ruled that certifications issued by the CENRO, or specialists of the DENR, as well as Survey
Plans prepared by the DENR containing annotations that the subject lots are alienable, do not constitute incontrovertible evidence to
overcome the presumption that the property sought to be registered belongs to the inalienable public domain. Rather, this Court stressed
the importance of proving alienability by presenting a copy of the original classification of the land approved by the DENR Secretary and
certified as true copy by the legal custodian of the official records.18 (Citations omitted and emphasis ours)

WHEREFORE, the petition is GRANTED.

TAAR VS. LAWAN


FACTS:

The present case involves two (2) free patent applications5 over a 71,014-square-meter parcel of land (the Property) located in Barangay
Parsolingan, Genova, Tarlac.6

Narcisa Taar (Narcisa), Alipio Duenas (Alipio), Fortunata Duenas (Fortunata), and Pantaleon Taar (Pantaleon) inherited two (2) vast
tracts of land situated in Tarlac. One (1) parcel of land was adjudicated exclusively in favor of Pantaleon while the other parcel of land
was given to Pantaleon, Narcisa, Alipio, and Fortunata.7 Narcisa sold her share to Spouses Primitive T. Adaoag and Pilar Tandoc (the
Adaoag Spouses) and to Spouses Ignacio Gragasin and Genoveva Adaoag (the Gragasin Spouses).8

Later, Pantaleon, Alipio, Fortunata, the Adaoag Spouses, and the Gragasin Spouses executed an agreement to partition the second
parcel of land. This agreement was approved by the Court of First Instance of Tarlac in its February 18, 1948 Decision.9

Pantaleon, Alipio, and Fortunata were the predecessors-in-interest of Francisca,10 Joaquina, Lucia, and Oscar L. Galo.11
Based on the February 18, 1948 Decision, petitioners prepared a subdivision plan12 over the Property in 2000.13 The subdivision plan,
denominated as Subdivision Plan No. Ccs-03-000964-D, was approved on February 6, 2001.14 Petitioners then applied for free patents
over the Property.15

On March 16, 2001, Claudio Lawan (Claudio), Marcelino M. Galo (Marcelino), A.rtemio Abarquez (Artemio), Augusto B. Lawan (Augusto),
and Adolfo L. Galo (herein private respondents) filed a verified protest16 alleging that their predecessors-in-interest had been in "actual,
physical, exclusive[,] and notorious possession and occupation of the land ... since 1948,"17 Petitioners countered that private
respondents occupied the property as tenants.18

The Regional Office of the Department of Environment and Natural Resources in Region III conducted an ocular inspection of the Property
and required the parties to submit their respective documentary evidence.19

In its May 29, 2002 Order,20 Department of Environment and Natural Resoqrces Regional Executive Director for Region III Leonardo R.
Sibbaluca (Director Sibbaluca) found that private respondents were the actual occupants of the Property. There were no improvements
or other traces of possession by petitioners. aased on his findings, Director Sibbaluca cancelled Subdivision Plan No. Ccs-03-000964-D
and denied petitioners' free patent applications,21

Neither of the parties interposed an appeal or moved for reconsideration.22 Hence, Director Sibbaluca's May 29, 2002 Order attained
finality.23

Later that year, private respondents filed their free patent applications before the Tarlac Community Environment and Natural Resources
Office.24 Their applications covered the Property, which was also claimed by petitioners.25

On January 23, 2004, private respondents' applications were approved.26 The corresponding free patents27 and certificates of title
denominated as "Katibayan ng Orihinal na Titulo" were then issued in their favor.28

On July 29, 2004,29 petitioners filed before the Secretary of the Department o:f Environment and Natural Resources30 a Verified
Petition31 to annul Director Sibbaluca's May 29, 2002 Order on the ground of extrinsic fraud and to cancel private respondents' free
patents and certificates of title.32 Petitioners alleged that they were deprived of due process.33

The Department of Environment and Natural Resources Undersecretary for Legal Affairs formed an investigating team34 to ascertain the
actual occupants of the Property.35 During the ocular inspection, the investigating team found "concrete residential houses of [petitioners]
with fences, fn1it trees[,] and coconut trees" and "other houses ... owned [by] relatives and friends of the parties."36 The investigating
team gathered documentary 1evidence, discovered several leasehold contracts between the parties,37 ; and saw rice fields cultivated by
Augusto, Marcelino, Claudio, Artemio Tabuyo, Artemio Abarquez, and Romy Tabuyo.38 Based on their findings, the team concluded that
petitioners were entitled to the Property.39

In his Decision40 dated January 18, 2007, then Secretary of Department of Environment and Natural Resources Angelo T. Reyes
(Secretary Reyes) adopted the findings of the investigating team and ordered the cancellation of the free patents and the certificates of
title issued in favor of private respondents.

Private respondents moved for reconsideration but their Motion was denied in the June 14, 2007 Order.41 Hence, they appealed
Secretary Reyes' January 18, 2007 Decision before the Office of the President.42

In its October 20, 2008 Decision,43 the Office of the President, through then Executive Secretary Eduardo R. Ermita (Executive Secretary
Ermita), reversed Secretary Reyes' January 18, 2007 Decision and reinstated Director Sibbaluca's May 29, 2002 Order. The Office of the
President held that Secretary Reyes erred in reversing Director Sibbaluca's May 29, 2002 Order as it had already attained finality,44

Petitioners moved for reconsideration but their Motion was denied in the Resolution45 dated March 26, 2009.

Petitioners filed a petition for certiorari46 against private respondents and Executive Secretary Ermita before the Court of Appeals.47
They alleged that the Office of the President committed grave abuse of discretion in reinstating Director Sibbaluca's May 29, 2002 Order
considering that their predecessors-in-interest had been declared ipso Jure owners of the Property as early as 1948 by the Court of First
Instance of Tarlac.48

In its July 20, 2009 Resolution,49 the Court of Appeals dismissed the petition for certiorari outright for being an inappropriate remedy.
The Court of Appeals noted that an appeal could have been taken from the Decision and the Resolution of the Office .of the President.50
Instead of filing an original action for certiorari, they should have filed a petition for review under Rule 43 of the Rules of Court.51

Petitioners moved for reconsideration but their Motion was denied for lack of merit in the Resolution52 dated January 15, 2010.

On March 4, 2010, petitioners53 filed a Petition for Review on Certiorari54 before this Court assailing the Resolutions dated July 20, 2009
and January 15, 2010 of the Court of Appeals. In its April 5, 2010 Resolution,55 this Court required private and public respondents to
comment on the petition for review.

Private respondents filed their Comment56 on June 1 7, 2010 while public respondent Executive Secretary Ermita, through the Office of
the Solicitor General, filed his Comment57 on August 15, 2011. Petitioners then filed their Consolidated Reply on December 5, 2011.58
In its January 23, 2013 Resolution,59 this Court gave due course to the Petition and required the parties to submit their res pective
memoranda.

Petitioners filed their Memorandum60 on April 12, 2013. On the other hand, the Office of the Solicitor General manifested that it would
no longer file a memorandum considering that it had exhaustively discussed its arguments in the Comment.61 Private respondents filed
their Memorandum on July 19, 2013.62

Petitioners claim that the Court of Appeals erred in dismissing their petition for certiorari and that the Office of the President acted with
grave abuse of discretion in reinstating Director Sibbaluca's May 29, 2002 Order. Petitioners insist that their predecessors.-in-interest
were declared ipso Jure owners of the Property by the Court of First Instance of Tarlac in its February 18, 1948 Decision.63 According to
petitioners, the Court of First Instance recognized that their predecessors-in-interest "possessed, occupied[,] and cultivated the ... lots for
more than thirty (30) years since 1915."64 Therefore, the principle of res judicata bars private respondents from asserting title to the
Property.65

Petitioners add that private respondents procured their free patents through fraud and misrepresentation.66 They pray for the cancellation
of private respondents' free patents and certificates of title.67

On the other hand, private respondents assert that the Court of Appeals correctly dismissed the petition for certiorari. They claim that
petitioners filed their petition ''after the lapse of more than two (2) months from the date they received the adverse decision of the Office
of the President."68 Moreover, they allege that petitioners raised errors of judgment, not efforts of jurisdiction.69

Private respondents contend that they are not bound by the February 18, 1948 Decision of the Court of First Instance. They assert that
the principle of res judicata does not apply because there is no identity of parties and subject matter.70 The Office of the Solicitor General
shares this view and points out that the February 18, 1948 Decision of the Court of First Instance simnply adopted an agreement of
partition, which arose out of a dispute "between and among petitioners' ... predecessors-in-[interest]."71 Private respondents insist that
petitioners are bound by Director Sibbaluca's May 29, 2002 Order, which had already attained finality.72

The present case presents the following issues for this Court's resolution:

First, whether or not the Court of Appeals erred in dismissing the petition for certiorari filed by Francisca Taar, Joaquina Taar, Lucia Taar,
and the Heirs of Oscar L. Galo;73

Second, whether or not the February 18, 1948 Decision of the Court of First Instance bars Claudio Lawan, Marcelino L. Galo, Artemio
Abarquez, Augusto B. Lawan, and Adolfo L. Galo from applying for free patents over the Property;74

Lastly, whether or not the free patents and certificates of title issued in favor of Claudio Lawan, Marcelino L. Galo, Artemio Abarquez,
Augusto B. Lawan, and Adolfo L. Galo are valid and were secured through fraud and misrepresentation.75

The Petition is denied.

A petition for certiorari under Rule 65 of the Rules of Court is an extraordinary remedy.76 Its scope of review is narrow, limited only to
errors of jurisdiction. Errors of judgment can only be reviewed through an appeal. In Fernando v. Vasquez,77 this Court made a general
distinction between errors of jurisdiction and en-ors of judgment, thus:

An error of judgment is one which the court may commit in the exercise of its jurisdiction. An error of jurisdiction renders an order or
judgment void or voidable. Errors of jurisdiction an: reviewable on certiorari; errors of judgment only by appeal. Let us not lose sight of
the true function of the writ of certiorari - "to keep an inferior court within the bounds of its jurisdiction or to prevent it from committing such
a grave abuse of discretion amounting to excess of jurisdiction." And, abuse of discretion must be so grave and patent to justify the
issuance of the writ.78 (Citation omitted)

Errors of judgment may involve a court's appreciation of the facts and conclusions of law drawn from such facts.79 If a court acts within
its jurisdiction, then "any alleged errors committed in the exercise of its discretion will amount to nothing more than mere errors of
judgment[.]"80 In Madrigal Transport, Inc. v. Lapanday Holdings Corporation:81

The supervisory jurisdiction of a court over the issuance of a writ of certiorari cannot be exercised for the purpose of reviewing the intrinsic
correctness of a judgment of the lower court - on the basis either of the law or the facts of the case, or of the wisdom or legal soundness
of the decision. Even if the findings of the court are incorrect, as long as it has jurisdiction over the case, such correction is normally
beyond the province of certiorari. Where the error is not one of jurisdiction, but of an error of law or fact - a mistake of judgment - appeal
is the remedy.82 (Emphasis supplied, citations omitted)

On the other hand, errors of jurisdiction are those where the act or acts complained of were done without jurisdiction, in excess of
jurisdiction, or with grave abuse of discretion amounting to lack or excess of jurisdiction.83 Milwaukee Industries Corporation v. Court of
Tax Appeals84 discussed these concepts; thus:

Without jurisdiction denotes that the tribunal, board, or officer acted with absolute lack of authority. There is excess of jurisdiction when
the public respondent exceeds its power or acts without any statutory authority. Grave abuse of discretion connotes such capricious and
whimsical exercise of judgment as to be equivalent to lack or excess of jurisdiction; otherwise stated, power is exercised in an arbitrary
or despotic manner by reason of passion, prejudice, or personal hostility; and such exercise is so patent or so gross as to amount to an
evasion of a positive duty or to a virtual refusal either to perform the duty enjoined or to act at all in contemplation of law.85 (Emphasis in
the original)

The mere allegation of grave abuse of discretion in a petition for certiorari does not mean that the petition will automatically be given due
course, The general invocation of grave abuse of discretion is insufficient. Parties must satisfy other exacting requirements under the
Rules of Court.

A petition for certiorari brought under Rule 65, Section 1 of the Rules of Court is specifically required to have "no appeal, or any plain,
speedy, and adequate remedy in the ordinary course of law." Ordinarily, if an appeal can be taken from a judgment or order, then the
remedy of certiorari will not lie. The mere possibility of delay arising from an appeal does not warrant direct recourse to a petition for
certiorari.86

However, there are exceptions to this rule. For instance, the availability of an appeal does not necessarily proscribe the institution of a
petition for certiorari if it is shown that an appeal is "inadequate, slow, insufficient and will not promptly relieve a party from the injurious
effects of the order complained of."87 In Silvestre v. Torres: 88

[T]he availability of the ordinary recourse of appeal does not constitute sufficient ground to prevent a party from making use of the
extraordinary remedy of certiorari; but it is necessary, besides, that the ordinary appeal be an adequate remedy, that is, "a remedy which
is equally beneficial, speedy and sufficient, not merely a remedy which at some time in the future will bring about {J revival of the judgment
of the lower court complained of in the certiorari proceeding, but a remedy which will promptly relieve the petitioner from the injurious
effects of that judgment and the acts of the inferior court or tribunal[.]"89 (Emphasis supplied)

This was reiterated later in Jaca v. Davao Lumber Company,90 where this Court underscored the standard in determining the propriety
of a petition for certiorari, thus:

The availability of the ordinary course of appeal course not constitute sufficient ground to prevent a party from making use of the
extraordinary remedy of certiorari where the appeal is not an adequate remedy or equally beneficial, speedy and sufficient. It is the
inadequacy - not the mere absence; - of all other legal remedies and the danger of failure of justice without the writ, that must usually
determine the propriety of certiorari.91 (Citation omitted, emphasis supplied)

In the present case, petitioners' allegation that the Office of the President, through then Executive Secretary Ermita, gravely abused its
discretion in failing to appreciate the merits of the February 18, 1948 Decision of the Court of First Instance92 involves an error of
judgment, not of jurisdiction. Assuming that the issue raised by petitioners pertains to an error of jurisdiction, there is no showing that the
Office of the President exercised its power in an "arbitrary or despotic manner by reason of passion, prejudice, or personal hostility."93

Petitioners could have taken an appeal from the October 20, 2008 Decision and March 26, 2009 Resolution of the Office of the President
by filing a petition for review under Rule 43 of the Rules of Court, which governs appeals from judgments rendered by quasi-judicial
agencies in the exercise of quasi-judicial powers.94

While it is true that courts may take cognizance of a petition for certiorari despite the availability of appeal,95 petitioners failed to allege
and prove that appeal would be inadeq11ate to promptly relieve them of the effects of the assailed Decision and Resolution of the Office
of the President. Well-settled is the rule that a petition for certiorari cannot be used as a substitute for a lost appeal "especially if one's
own negligence or error in one's choice of remedy occasioned such loss or lapse."96

In this regard, the Court of Appeals did not err in dismissing the petition for certiorari outright.

II

The core of the controversy lies in whether or not private respondents are barred by the principle of res judicata from instituting free patent
applications over the Property claimed by petitioners.

The rule on res judicata states that a '"final judgment or decree rendered on the merits . . . by a court of competent jurisdiction . . . is
conclusive of the rights of the parties or their privies, in all other [subsequent] actions or suits" and on all points and matters determined
in the first suit.97

Res judicata has two (2) aspects. The first aspect refers to bar by prior judgment while the second refers to conclusiveness of judgment.98

In bar by prior judgment, the first judgment "precludes the prosecution of a second action upon the same claim, demand or cause of
action."99 On the other hand, conclusiveness of judgment states that "issues actually and directly resolved in a former suit cannot again
be raised in any future case between the same parties involving a different cause of action.''100 Thus, the concept of conclusiveness of
judgment is also known as preclusion of issues.101 All that is required is identity of issues.102

Parties invoking the application of res judicata must establish the following elements:

(1) the judgment sought to bar the new action must be final;

(2) the decision must have been rendered by a court having jurisdiction over the subject matter and the parties;
(3) the disposition of the case must be a judgment on the merits; and

(4) there must be as between the first and second action identity of parties, subject matter, and causes of action.103

In this case, only the first three (3) elements of res judicata are present.

The February 18, 1948 Decision of the Court of First Instance is a final judgment on the merits rendered by a court of competent
jurisdiction. However, it does not bar private respondents from instituting their free patent applications over the Property. There is no
identity or substantial identity of parties and identity of subject matter between the February 18, 1948 Decision of the Court of First
Instance and private respondents' free patent applications.

The principle of res judicata does not require absolute identity of parties. It requires, at the very least, substantial identity of parties. There
is substantial identity of parties when there exists a "community of interest between a party in the first case and a party in the second
case even if the latter was not impleaded in the first case."104 For instance, there is substantial identity of parties when one intervenes
as a party-defendant and creates a common cause with the original defendant.105

The February 18, 1948 Decision of the Court of First Instance involved an agreement between petitioners' predecessors-in-interest,
namely: Alipio Duenas, Fortunata Duenas, Spouses Primitivo T. Adaoag and Pilar Tandoc, Spouses Ignacio Gragasin and Genoveva
Adaoag, Pantaleon Taar, Lucia Taar, Joaquina Taar, Feliciano Taar, Paulino Taar, and Oscar Galo.106 Clearly, private respondents
were not parties to the agreement. Moreover, there is no clear showing that private respondents or their predecessors-in-interest shared
a common interest with any of the parties to the agreement.

However, assuming that there is identity or substantial identity of parties, there is no identity of subject matter between the February 18,
1948 Decision of the Court of First Instance and private respondents' free patent applications. Although both relate to the same Property,
the February 18, 1948 Decision of the Court of First Instance was simply an agreement partitioning the bigger parcel of land, which
embraced the smaller portion claimed by petitioners and private respondents. On the other hand, private respondents' free patent
applications involved the establishment of their rights as the purported occupants and cultivators of the Property. Evidently, there is no
identity of subject matter. The principle of res judicata does not apply.

In addition, the Court of First Instance did not recognize, expressly or impliedly, that private petitioners' predecessors-in-interest occupied
and cultivated the Property for more than 30 years since 1915. It also did not declare petitioners' predecessors-in-interest as the ipso
Jure owners of the same.

Therefore, the February 18, 1948 Decision of the Court of First Instance cannot bar the filing of a subsequent free patent application over
the Property. Likewise, petitioners cannot rely solely on this Decision to obtain free patents. Entitlement to agricultural lands of the public
domain requires compliance with the provisions of Commonwealth Act No. 141, otherwise known as the Public Land Act, There are four
(4) modes of disposition of agricultural lands under Section 11 of the Public Land Act, namely:"(1) for homestead settlement; (2) by sale;
(3) by lease; or (4) by confirmation of imperfect or incomplete titles[.]"107

The applicant of a homestead must be a "citizen of the Philippines over the age of eighteen years, or the head of a family[.]"108 The
applicant must prove compliance with the residency and cultivation requirements under Chapter IV of Public Land Act. Under the
Constitution, only 12 hectares of agricultural land of the public domain may be acquired through homestead.109

Sales patents are governed by Chapter V of the Public Land Act. The applicant must be a citizen of the Philippines who is of legal age or
a head of the family.110 The land must first be appraised before it can be sold through public bidding.111 As an additional requirement,
the purchaser must "have not less than one-fifth of the land broken and cultivated within five years after the date of the award."112 The
purchaser must also show "actual occupancy, cultivation, and improvement of at least one-fifth of the land applied for until the date on
which final payment is made" before the issuance of a sales patent.113 Only 12 hectares of agricultural land of the public domain may
be acquired through a sales patent.114 The Public Land Act authorized domestic corporations to apply for sales patents over . . .
agricultural lands. However, under the present Constitution, private corporations and associations can only lease agricultural lands.115

The third mode of disposition of agricultural lands of the public domain is through a lease. The government can only award the right to
lease through an auction, the procedure of which shall be the same as that prescribed for sales patents.116 An inherent condition of the
lease is that the lessee should have cultivated 1/3 of the land "within five years after the date of the approval of the lease."117 Under the
Constitution, citizens may lease not more than 500 hectares of agricultural lands of the public domain. For private corporations and
associations, they may lease a maximum of 1,000 hectares of agricultural lands for a period of 25 years, renewable for another 25
years.118

The last mode of disposition is by confirmation of imperfect or incomplete titles either through judicial legalization or through administrative
legalization. The second sub-category refers to the grant of free patents.119

Judicial legalization or judicial confirmation of imperfect or incomplete titles is governed by Section 48 of the Public Land Act, as amended
by Republic Act No. 3872 and Presidential Decree No. 1073, which states:

Section 48. The following described citizens of the Philippines, occupying lands of the public domain or claiming to own any such lands
or an interest therein, but whose titles have not been perfected or completed, may apply to the Court of First Instance of the province
where the land is located for confirmation of their claims and the issuance of a certificate of title therefor, under the Land Registration Act,
to wit:
...

(b) Those who by themselves or through their predecessors in interest have been in the open, continuous, exclusive, and notorious
possession and occupation of agricultural lands of the public domain, under a bona fide claim of acquisition or ownership, except as
against the government, since [June 12, 1945], immediately preceding the filing of the: applications for confirmation of title, except when
prevented by war or force majeure. Those shall be conclusively presumed to have performed all the conditions essential to a government
grant and shall be entitled to a certificate of title under the provisions of this chapter.

In Heirs of Malabanan v. Republic of the Philippines, 120 this Court made an important qualification regarding the registration of lands
through judicial confirmation of imperfect title, thus:

If the mode is judicial confirmation of imperfect title under Section 48 (b) of the Public Land Act, the agricultural land subject of the
application needs only to be classified as alienable and disposable as of the time of the application, provided the applicant's possession
and occupation of the land dated back to June 12, 1945, or earlier. Thereby, a conclusive presumption that the applicant has performed
all the conditions essential to a government grant arises, and the applicant becomes the owner of the land by virtue of an imperfect or
incomplete title. By legal fiction, the land has already ceased to be part of the public domain and has become private property.121
(Citations omitted)

On the other hand, the grant of free patents is governed by Section 44, paragraph 1 of the Public Land Act, as amended by Republic Act
No. 6940, which states:

Section 44. Any natural-born citizen of the Philippines who is not the owner of more than twelve (12) hectares and who, for at least thirty
(30) years prior to the effectivity of this amendatory Act, has continuously occupied and cultivated, either by himself or through his
predecessors-in-interest a tract or tracts of agricultural public lands subject to disposition, who shall have paid the real estate tax thereon
while the same has not been occupied by any person shall be entitled, under the provisions of this Chapter, to have a free patent issued
to him for such tract or tracts of such land not to exceed twelve (12) hectares.

The applicant for a free patent should comply with the following requisites: (1) the applicant mu.st be a natural-born citizen of the
Philippines; (2) the applicant must not own more than 12 hectares of land; (3) the applicant or his or her predecessors-in-interest must
have continuously occupied and cultivated the land; (4) the continuous occupation and cultivation must be for a period of at least 30 years
before April 15, 1990, which is the date of effectivity of Republic Act No. 6940;122 and (5) payment of real estate taxes on the land while
-it has not been occupied by other persons.

Applicants are free to avail any of the two (2) modes. Both judicial legalization and administrative legalization involve agricultural lands of
the public domain and require "continuous occupation and cultivation either by the applicant himself or through his predecessors-in-
interest for a certain length of time."123

In judicial legalization or judicial confirmation, the applicant "already holds an imperfect title to an agricultural land of the public domain
after having occupied it from June 12, 1945 or earlier."124 On the other hand, the applicant of a free patent does not claim that the land
is his or her private property but acknowledges that the land is still part of the public domain.125 This distinction was reiterated in De
Leon v. De Leon-Reyes,126 thus:

Under Section 11 of the Public Land Act (PLA), there are two modes of disposing public lands through confirmation of imperfect or
incomplete titles: (l) by judicial confirmation; and (2) by administrative legalization, otherwise known as the grant of free patents.

...

Section 48 of the PLA particularly specifies who are entitled to judicial confirmation or completion of imperfect titles:

(b) Those who by themselves or through their predecessors-in-interest have been in open, continuous, exclusive, and notorious
possession and, occupation of agricultural lands of the public domain, under a bona fide claim of acquisition or ownership, since June
12, 1945, immediately preceding the filing of the application for confirmation of title, except when prevented by war or force majeure.
Those shall be conclusively presumed to have performed all the conditions essential to a government grant and shall be entitled to a
certificate of title under the provisions of this chapter.

Upon compliance with the conditions of Sec. 48 (b) of the PLA, the possessor is deemed to have acquired, by operation of law, right to a
grant over the land. For all legal intents and purposes, the land is segregated from the public domain, because the beneficiary is
conclusively presumed to have performed all the conditions essential to a Government grant. The land becomes private in character and
is now beyond the authority of the director of lands to dispose of.

At that point, original registration of the title, via judicial proceedings, takes place as a matter of course; the registration court does not
grant the applicant title over the property but merely recognizes the applicant's existing title which had already vested upon the applicant's
compliance with the requirement of open, continuous, exclusive, and notorious possession and occupation of the land since June 12.
1945.

On the other hand, Chapter VII (Sections 44-46) of the PLA substantively governs administrative legalization through the grant of free
patents. Section 44 particularly identifies who are entitled to a grant of a free patent[.]
Sec. 44. Any natural-born citizen of the Philippines who is not the owner of more than twelve (12) hectares and who, for at least thirty
(30) years prior to the effectivity of this amendatory Act, has continuously occupied and cultivated, either by himself or through his
predecessors-in-interest a tract or tracts of agricultural public lands subject to disposition, who shall have paid the real estate tax thereon
while the same has not been occupied by any person shall be entitled, under the provisions of this Chapter, to have a free patent issued
to him for such tract or tracts of such land not to exceed twelve (12) hectares ...

Unlike an applicant in judicial confirmation of title who claims ownership over the land, the applicant for a free patent recognizes that the
land applied for belongs to the government. A patent, by its very definition, is a governmental grant of a right, a privilege, or authority. A
free patent [...] is an instrument by which the government conveys a grant of public land to a private person.

Pursuant to the Administrative Code and the PLA, the DENR has exclusive jurisdiction over the management and disposition of public
lands. In the exercise of this jurisdiction, the DENR has the power to resolve conflicting claims over public lands and determine an
applicant's entitlement to the grant of a free patent.127 (Emphasis supplied, citations omitted)

Petitioners, in choosing to apply for free patents, acknowledged that the land covered by their application still belongs to the government
and is still part of the: public domain.128 Under Section 44 of the Public Land Act as amended by Republic Act No. 6940, they are required
to prove continuous occupation and cultivation for 30 years prior to April 15, 1990 and payment of real estate taxes while the land has
not been occupied by other persons. Petitioners insist that the February 18, 1948 Decision of the Court of First Instance automatically
vests them with ownership over the property. This Decision cannot be used as proof of compliance with the requirements of the Public
Land Act. Again, the Court of First Instance simply approved an agreement of partition. If at all, the February 18, 1948 Decision could
only be used as the basis of a subdivision plan.

III

Section 91 of the Public Land Act provides the automatic cancellation of the applications filed on the ground of fraud and
misrepresentation, thus:

Section 91 . The statements made in the application shall be considered as essential conditions and parts of any concession, title, or
permit issued on the basis of such application, and any false statements therein or omission of facts altering, changing, or modifying the
consideration of the facts set forth in such statements, and any subsequent modification, alteration, or change of the material facts set
forth in the application shall ipso facto produce the cancellation of the concession, title, or permit granted. It shall be the duty of the
Director of Lands, from time to time and whenever he may deem it advisable, to make the necessary investigations for the purpose of
ascertaining whether the material facts set out in the application are true, or whether they continue to exist and are maintained and
preserved in good faith, and for the purposes of such investigation, the Director of Lands is hereby empowered to issue subpoenas and
subpoenas duces tecum and, if necessary, to obtain compulsory process from the courts. In every investigation made in accordance with
this section, the existence of bad faith, fraud, concealment, or fraudulent and illegal modification of essential facts shall be presumed if
the grantee or possessor of the land shall refuse or fail to obey a subpoena or subpoena duces tecum lawfully issued by the Director of
Lands or his authorized delegates or agents, or shall refuse or fail to give direct and specific answers to pertinent questions, and on the
basis of such presumption, an order of cancellation may issue without further proceedings.

Only extrinsic fraud may be raised as a ground to "review or reopen a decree of registration."129 Extrinsic fraud has a specific meaning
under the law. It refers to that type of fraud that "is employed to deprive parties of their day in court and thus prevent them from asserting
their right to the property registered in the name of the applicant."130

Petitioners invoke Section 91 of the Public Land Act impliedly by insisting that private respondents procured their free patents and
certificates of title through extrinsic fraud and misrepresentation. However, petitioners failed to substantiate their claims. Petitioners allege
that private respondents committed extrinsic fraud and misrepresentation but failed to establish the circumstances constituting them.
They could have pointed to irregularities131 during the proceedings to prove that the issuance of the free patents was not made in
accordance with the Public Land Act.132

The determination on the existence or nonexistence of fraud is a factual matter that is beyond the scope of a petition for review on
certiorari.133 Although there are exceptions to this rule,134 petitioners failed to allege and prove that this case falls under the exceptions.
Assuming that private respondents procured their free patents and certificates of title through extrinsic fraud and misrepresentation, the
petition must still be denied.

While it is true that "a title emanating from a free patent which was secured through fraud does not become indefeasible ... because the
patent from whence the title sprung is itself void[,]"135 petitioners are not the proper parties to bring an action for the cancellation of free
patents and certificates of title. The validity or invalidity of free patents granted by the government and the corresponding certificates of
title is a matter between the grantee and the government. In explaining this rule, this Court in Sumail v. Court of First Instance of
Cotabato13 underscored the nature of a free patent application, thus:

Consequently, Sumail may not bring such action or any action which would have the effect of cancelling a free patent and the
corresponding certificate of title issued on the basis thereof, with the result that the land covered thereby will again form part of the public
domain. Furthermore, there is another reason for withholding legal personality from Sumail. He does not claim the land to be his private
property. In fact, by his application for a free patent, he had formally acknowledged and recognized the land to be a part of the public
domain; this, aside from the declaration made by the cadastral court that lot 3633 was public land. Consequently, even if the parcel were
declared reverted to the public domain, Sumail does not automatically become owner thereof He is a mere public land applicant like
others who might apply for the same.137 (Emphasis supplied)
This principle was reiterated later in Caw is v. Cerilles, 138 a case involving the validity of a sales patent. Thus:

[W]e must point out that petitioners' complaint questioning the validity of the sales patent and the original certificate of title over Lot No.
47 is, in reality, a reversion suit. The objective of an action for reversion of public land is the: cancellation of the certificate of title and the
resulting reversion of the land covered by the title to the State. This is why an action for reversion is oftentimes designated as an annulment
suit or a cancellation suit.

Coming now to the first issue, Section 101 of the Public Land Act clearly states:

SEC. 101. All actions for the reversion to the Government of lands of the public domain or improvements thereon shall be instituted by
the Solicitor General or the officer acting in his stead, in the proper courts, in the name of the Republic of the Philippines.

Even assuming that private respondent indeed acquired title to Lot No. 47 in bad faith, only the State can institute reversion proceedings,
pursuant to Section 101 of the Public Land Act and our ruling in Alvarico v. Sola Private persons may not bring an action for reversion or
any action which would have the effect of canceling a land patent and the corresponding certificate of title issued on the basis of the
patent, suchthat the land covered thereby ·will again form part of the public domain. Only the Office] [of the] S[olicitor] G[eneral] or the
officer acting in his stead may do so. Since the title originated from a grant by the government, its cancellation is a matter between the
grantor and the grantee.

Similarly, in Urquiaga v. CA, this Court held that there is no need to pass upon any allegation of actual fraud in the acquisition of a title
based on a sales patent. Private persons have no right or interest over land considered public at the time the sales application was filed.
They have no personality to question the validity of the title. We further stated that granting, for the sake of argument, that fraud was
committed in obtaining the title, it is the State, in a reversion case, which is the proper party to file the necessary action. 139 (Emphasis
supplied, citations omitted)

Lorzano v. Tabayag, 140 citing Kayaban v. Republic, 141 explained the purpose of the rule:

In Kayaban, et al. v. Republic, et al., this Court explained the reason for the rule that only the government, through the OSG, upon the
recommendation of the Director of Lands, may bring an action assailing a certificate of title issued pursuant to a fraudulently acquired
free patent:

Since it was the Director of Lands who processed and approved the applications of the appellants and who ordered the issuance of the
corresponding free patents in their favor in his capacity as administrator of the disposable lands of the public domain, the action for
annulment should have been initiated by him, or at least with his prior authority and consent.142

WHEREFORE, the Petition for Review on Certiorari is DENIED.

ARJONILLO VS. PAGULAYAN

FACTS

Avelardo Cue (Cue) died intestate on 8 December 1987 in Tuguegarao, Cagayan. Cue died single with no surviving descendants or
ascendants but was survived by the following: 1) his brother, Felix Cue; 2) Alfonsa Sim and Rodolfo Sia, his niece and nephew by his
deceased sister Marta Cue; 3) the herein petitioner Florencia Arjonillo (Arjonillo), his niece by his deceased sister Angelita Cue; and 4)
Antonio, Isidra, Jacinto, Juanio, Nenita and Teodora, all surnamed Cue, his nieces and nephews by his deceased brother Francisco Cue.
On 21 June 1989, they executed an extra judicial settlement of the estate of Cue.

According to the heirs of Cue, the decedent acquired the following properties during his lifetime:

a) Lot 999-B-3-B, Psd-57204, being a portion of Lot 999-B-3, Psd- 52698, located at Poblacion, Tuguegarao, Cagayan, with an area of
two hundred ten (210) square meters, more or less; bounded on the N. along line 1-2 by Calle Commercio; on the N and E, along lines
2-3-4 by Lot 999-B-3-A, of the subdivision plan, and on the S, along line 4-1 by Lot 999-A, Psd-46471 (Pedro Abraham and Josefina
Abraham); reasonably assessed at ₱105,000.00;

b) A 2-storey commercial building erected on lot 999-B-3-B, Psd-57204, made of strong materials; assessed at ₱73,320.00.4

Lot 999-B-3-B, however, is registered in the name of Demetria Pagulayan (Pagulayan) per Transfer Certificate of Title (TCT) No. T-35506,
issued by the Register of Deeds for the Province of Cagayan.

Some of the heirs of Cue, including Arjonillo, instituted Civil Case No. 4778 with the RTC for "Reivindicacion, with Partition and Application
for Temporary Restraining Order and Preliminary Mandatory Injunction."5 They alleged that although the property was registered in the
name of Pagulayan, it was Cue who purchased it using his own funds; that being his paramour., Pagulayan exercised undue influence
on him in order to register the property exclusively in her own name; and that the registration of the property in the name of Pagulayan is
void as it is against public policy.

On the other hand, Pagulayan alleged that she acquired the property from Spouses Chua Bun Gui6 and Esmeralda Valdepanas Chua
(Spouses Chua) for and in consideration of ₱20,000.00 which was acknowledged to have been received in full by the vendors as
evidenced by the deed of absolute sale executed on 25 August 1976.7 She prayed in her answer that the complaint be dismissed since
the plaintiffs have no legal personality or cause of action against her.
The Ruling of the RTC

On 31 August 2006, the RTC rendered a decision declaring that Pagulayan is not the rightful owner of the subject property and,
consequently, ordered the partition of the subject lot and building among the heirs of Cue. According to the RTC, "[Demetria] failed to
substantiate her financial .capability to acquire the properties subject of the suit, more so to erect and put up a building thereon jointly
with Avelardo Cue."8 Its findings were based, among others, on the testimony of Dr. Benito Valdepanas (Dr. Valdepanas), who is a
nephew of Spouses Chua:

After making a thorough evaluation on the merits of the case, as it has been well substantiated by the testimonies of witnesses presented
during the court proceedings, Demetria Pagulayan failed to prove her claim that she bought the lot in question and put up a building
thereon. Noted as well in the records of the case is the Deposition of a witness who testified among others that he knows the lot described
in TCT No. T- 35506; that said witness has personal knowledge of the sale of the lot in question by his uncle to the late A velardo Cue;
and that Defendant Demetria Pagulayan is a mere salesgirl of the late A velardo Cue.

The allegations of the Plaintiffs as above-discussed have been, in the mind of the Court, preponderantly proven as evidenced by the
testimonies and documents presented during the trial of the case."9

The Ruling of the CA

Upon review, the CA, in its Decision dated 7 January 2011, reversed and set aside the RTC decision and dismissed the case. A motion
for reconsideration was filed which was denied in the CA Resolution dated 16 March 2011.

In dismissing the case, the CA found that petitioners failed to discharge the burden of proving their allegation that the properties in dispute
form part of the estate of Cue. It was also found that the testimonies of their witnesses could be considered as mere hearsay because
they did not have personal knowledge of the circumstances attending the execution of the deed of sale in favor of Pagulayan and the
consequent issuance of TCT No. T- 35506 in her name.10

ISSUES

Arjonillo is now before the Court assailing the decision of the CA on the following grounds:

I. THE HONORABLE COURT OF APPEALS GRAVELY ERRED WHEN IT REVERSED OR SET ASIDE THE TRIAL COURT'S 31
AUGUST 2006 DECISION AND DISMISSING THE COMPLAINT IN CIVIL CASE NO. 4778 ABANDONING THE FACTUAL FINDINGS
OF THE COURT A QUO.

II. THE HONORABLE COURT OF APPEALS GRAVELY ERRED WHEN IT RULED ON THE INDEFEASIBILITY OF RESPONDENT
DEMETRIA PAGULAYAN'S TITLE AND CATEGORICALLY DECLARED THAT THE OWNERSHIP OF THE DISPUTED PROPERTIES
BELONG TO HER.

III. THE HONORABLE COURT OF APPEALS COMMITTED GRAVE ERROR WHEN IT CONSIDERED WITNESS DR. BENITO V
ALDEPANAS' TESTIMONY AS HEARSAY.11

THE COURT'S RULING

The petition is without merit.

When a case is appealed to the CA, it is thrown wide open for review by that court which thereby has the authority to affirm, reverse, or
modify the assailed decision of the lower court. The appellate court can render an entirely new decision in the exercise of its power of
review in order to correct patent errors committed by the lower courts.12

Arjonillo and her co-heirs claim that the subject properties were owned by their predecessor, Cue. They sought to recover its full
possession from Pagulayan by filing an accion reivindicatoria before the RTC. It is then incumbent upon them to convince the court by
competent evidence that the subject properties form part of Cue's estate because in order to successfully maintain actions for recovery
of ownership of a real property, the complainants must prove the identity of the land and their title thereto as provided under Article 434
of the Civil Code.13 They have the burden of proof to establish the averments in the complaint by preponderance of evidence,14 relying
on the strength of their own evidence and not upon the weakness of their opponent's evidence.15

Rather than dispensing with their burden of proof as required under the law, Arjonillo and her co-heirs concentrated on attacking
Pagulayan's claim of ownership over the subject properties on the ground of the latter's alleged lack of financial capability to purchase
the land and erect a building thereon. It was consistently emphasized that Pagulayan was a mere salesgirl who only had an annual salary
of ₱1,950.00 in 1976.16 On this basis, Arjonillo and her co-heirs maintained that Pagulayan could not have acquired the property on 25
August 1976 as reflected in the Deed of Absolute Sale executed by Spouses Chua.17

They also tried to prove that contrary to what appears in the deed of sale, the actual transaction transpired between Chua Bun Gui and
Cue. But Chua Bun Gui did not testify during the trial. Neither his wife nor any witness to the sale was presented. Instead, Arjonillo and
her co-heirs presented the testimony of Dr. Valdepanas who, as earlier noted, is the nephew of Spouses Chua and has a clinic adjacent
to the property under scrutiny. The subject of his testimony, however, is not of matters he himself knows; thus, it should be disregarded
for being hearsay.
Dr. Valdepanas testified as follows:

Q: Now, you said a while ago that Chua Bun [Gui] was the former owner of the lot in question, what did Chua Bun [Gui] do with the lot in
question?

A: Two or three days after the fire that was August 22 1977 my uncle Chua Bun [Gui] went home to had a cup of coffee he told me that
he sold the lot in question to A velardo Cue when in fact I was also interested to buy it.

Q: Are we made to understand that the transaction regarding the sale of the lot in question to A velardo Cue was made in your house?

A: No, sir. Avelardo Cue told me that the lot in question was sold in installment basis when infact I offered to purchase the lot in question
in cash basis, sir.

xxxx

Q: Were you present whenever the late Avelardo Cue made payments to your uncle Chua Bun Gin?

A: [A]side from knowing it personally, the late Avelardo Cue told me that he paid fifty percent of the fiurchased price and the remaining
balance on installment basis, sir.18

Despite claiming knowledge of the terms and conditions of the sale, perusal of the deed of absolute sale revealed that Dr. Valdepanas
was neither a party nor a witness to the transaction. It is noticeable that Dr. Valdepan_as merely repeated statements he heard from Cue
and Chua Bun Gui. When asked if he was present whenever Cue paid Chua Bun Gui, he did not give a categorical answer but simply
claimed that he knew about it personally. More importantly, proponent offered the testimony to prove "that the lot in question was
purchased by the late A velardo Cue and not by the defendant, Demetria Pagulayan, although the Deed of Sale was in the name of the
said defendant Demetria Pagulayan."19 It was offered as evidence of the truth of the fact being asserted. Clearly, the above-quoted
testimony is hearsay and thus inadmissible in evidence. A witness can only testify on facts within his personal knowledge.20 This is a
substantive prerequisite for accepting testimonial evidence that establishes the truth of a disputed fact.21 Unless the testimony falls under
any of the recognized exceptions, hearsay evidence whether objected to or not cannot be given credence for it has no probative value.22

On the other hand, to shed light on how she could afford to purchase the land, Pagulayan testified that she worked with free board and
lodging from 1954 to 1976 and deposited her earnings in an account with the Philippine National Bank.23 She further testified that she
withdrew some of the money and used it in re-sellingpalay and pigs.24

The following documents were offered and admitted in evidence25 to support Pagulayan's claim that it is indeed she who owns the land
in question: 1) a notarized deed of absolute sale26 executed by Spouses Chua on 25 August 1976 conveying the property to Pagulayan;
2) TCT No. T- 3550627 registered in the name of Pagulayan; and 3) Real Property Tax Receipts for 199328 and 199429 which were
offered to prove that the land's tax declaration was in the name of Pagulayan.

We agree with the finding of the CA that "[t]he documentary and testimonial evidence on record clearly support [Pagulayan's] ownership
of the disputed property as reflected in TCT No. T-35506, which was issued in her name pursuant to the aforesaid Deed of Sale."30 It is
fundamental that a certificate of title serves as evidence of an indefeasible and incontrovertible title to the property in favor of the person
whose name appears therein. The titleholder is entitled to all the attributes of ownership, including possession of the property.31

Though it has been held that placing a parcel of land under the mantle of the Torrens system does not mean that ownership thereof can
no longer be disputed,32 this Court cannot ignore the fact that Arjonillo, together with her co-heirs, failed to discharge the burden of
proving their claim by a preponderance of evidence as required under the law. Based on the foregoing, we find no persuasive argument
in the instant petition that will convince us to overturn the assailed judgment of the appellate court.

WHEREFORE, the Petition for Review on Certiorari is DENIED.

REPUBLIC VS. NICOLAS

FACTUAL ANTECEDENTS

On 22 March 1996, respondent filed a Petition before the RTC of San Mateo, Rizal,8 seeking to register her title over Lot 2 of Survey
Plan Psu-213331, a parcel of land located in Brgy. San Isidro, Rodriguez, Rizal, with an area of 118,448 square meters.9 She asserted
that she was entitled to confirmation and registration of title, as she had been in "natural, open, public, adverse, continuous, uninterrupted"
possession of the land in the concept of an owner since October 1964.10

Petitioner Republic of the Philippines filed an Opposition11 to the Petition. It contended that (a) neither respondent nor her predecessors-
ininterest had been in open, continuous, exclusive and notorious possession of the land since 12 June 1945;12 (b) the Tax Declarations
attached to the Petition did not constitute sufficient evidence of the acquisition or possession of the property;13 (c) respondent failed to
apply for registration of title within six months from 16 February 1976 as required by Presidential Decree No. (P.D.) 892;14 and (d) the
land in question was part of the public domain and not subject to private appropriation.15

After the conduct of proceedings to confirm compliance with jurisdictional requisites,16 the RTC directed respondent to submit documents
to establish that (a) the property that was the subject of the application for registration of title was not covered by the Comprehensive
Agrarian Reform Program of the Government; (b) there were no tenants on the property; and (c) the land was not subject to any
homestead, free patent, or grant of title from the Land Registration Authority (LRA), the Bureau of Lands, or the Department of Agrarian
Reform.17 Respondent was also directed to begin the presentation of her evidence.18

In line with this directive, the Community Environment and Natural Resources Office (CENRO) submitted a Report19 on the results of its
verification of the existing records on the subject property. The Report stated that the land "appears to be [n]ot covered by any public land
application nor embraced by any administrative title."20 However, the entry with respect to whether the land was within the alienable and
disposable zone was left blank with a notation that the area was "not projected due to [u]navailability of coordinates re[:] Tala Estate Tie-
Line."21

The LRA likewise submitted a Report22 stating that it "was not in a position to verify whether or not the parcel of land subject of registration
is already covered by land patent and is within the area classified as alienable and disposable land of the public domain."23 Hence, the
LRA recommended that the CENRO of Antipolo, Rizal, be ordered to submit a report on the status of the land.24 This proposal was
adopted by the RTC in an Order25 dated 28 December 1998.

During trial, respondent presented three witnesses to prove her right to register the property: Leonila Alfaro, her daughter and attorney-
in-fact, who testified that respondent had occupied the land since 1940 and had paid the real estate taxes therefor since 1969;26 Santiago
Eulin, who was allegedly hired by respondent to plant vegetables and fruit trees on the land and who acted as its caretaker since 1942;27
and Roberto M. Valdez of the LRA, who identified the original tracing cloth plan for the property.28

The following documents were likewise submitted to the trial court: Survey Plan PSU-213331,29 a Surveyor's Certificate30 and technical
descriptions of the property,31 which purportedly proved that the land had been duly surveyed by the Land Management Sector; various
Tax Declarations and receipts;32 and a Certification issued by the CENRO that the land applied for was not covered by any public land
application.33

Petitioner, on the other hand, decided to have the case submitted for resolution without any further submission.34

THE RULING OF THE RTC

In a Decision dated 31 July 2002, the RTC granted the Petition and ordered the issuance of a Decree of Registration in favor of
respondent.35 It declared that she had acquired ownership of the land by way of open, continuous, public, adverse, actual and bona fide
possession in the concept of an owner since 1940.36

Petitioner appealed the RTC Decision to the CA. In the Appellant's Brief,37 the Republic argued that respondent had failed to clearly and
convincingly establish that she had actual, continuous, exclusive and notorious possession of the property since 12 June 1945 or earlier
as required by Section 14(1) of P.D. 1529 or the Property Registration Decree.38 Petitioner further asserted that there was no basis for
the finding of the RTC that she had occupied the land since 1940.39

Respondent failed to file an appellee's brief.40 Consequently, the CA considered the case submitted for resolution.41

THE RULING OF THE CA

On 23 August 2007, the CA dismissed petitioner's appeal.42 According to the appellate court, the evidence presented proved that
respondent had occupied the land since 1940. Even assuming that her possession of the property started only when she had it privately
surveyed in 1964, she had been its occupant for more than 30 years.43 As such, she was still entitled to registration of title under Section
14(2) of P.D. 1529.

The CA further characterized the land as private property:

The fact that the subject land is covered by a private survey (PSU) (EXH. "J") way back in 1964, which survey was approved on April
1965 by Director Nicanor Jorge of the then Bureau of Lands, is a clear indication that it is already private in nature. Moreover, applicant's
evidence consisting of the DENR-CENRO Certifications (Exhs. "O" and "P") that Lot 2 of PSY 213331 is not covered by any public land
application and that its equivalent is Lot No, 10549 of the Montalban Cadastre have substantial probative value which established (sic)
that the land is alienable and disposable and not covered by any land grant from the government.

Petitioner moved for reconsideration of the Decision.44 The CA, however, denied the motion in a Resolution45 dated 22 January 2008,
prompting petitioner to elevate the case to this Court.

PROCEEDINGS BEFORE THIS COURT

In its Petition for Review, the Republic argues that (a) the decision of the CA and the RTC to confirm the title of respondent to the land
based on her possession and occupation thereof was not supported by evidence; and (b) the testimonial and documentary evidence she
presented did not establish possession of the property in the manner and period required by law, that is, her possession of the property
since 12 June 1945 or earlier. Petitioner also emphasizes that the lower courts gave undue importance to the Tax Declarations and
receipts presented,46 as well as to the testimonies of respondent's witnesses, notwithstanding the inconsistencies in their statements.

On 26 September 2008, respondent filed a Manifestation and Comment47 in which she pointed out that the grounds relied upon by
petitioner all pertain to allegedly erroneous findings of fact. She argued that these grounds could not be raised in a Rule 45 proceeding;
hence, the dismissal of the petition was warranted.48
Petitioner reiterate its arguments in its RepIy49 and Memorandum50 filed on 17 March 2009 and 19 February 2010, respectively.

ISSUES

Based on the submissions of the parties and the Decisions of the CA and the RTC, two issues are presented for resolution by this Court:

(1) Whether the CA erroneously allowed the judicial confirmation of respondent's title to the property under Section 14(1) of P.D. 1529;
and

(2) Whether the CA erred in declaring that respondent is likewise entitled to registration of title based on ownership by acquisitive
prescription under Section 14(2) of P.D. 1529.

OUR RULING

We GRANT the Petition.

Applications for registration of title to land, both public and private, are governed by Section 14 of P.D. 1529:

SECTION 14. Who May Apply. -The following persons may file in the proper Court of First Instance an application for registration of title
to land, whether personally or through their duly authorized representatives:

(1) Those who by themselves or through their predecessors-in-interest have been in open, continuous, exclusive and notorious
possession and occupation of alienable and disposable lands of the public domain under a bona fide claim of ownership since June 12,
1945, or earlier.

(2) Those who have acquired ownership of private lands by prescription under the provisions of existing laws.

(3) Those who have acquired ownership of private lands or abandoned river beds by right of accession or accretion under the existing
laws.

(4) Those who have acquired ownership of land in any other manner provided for by law.

Where the land is owned in common, all the co-owners shall file the application jointly.

Where the land has been sold under pacto de retro, the vendor a retro may file an application for the original registration of the land,
provided, however, that should the period for redemption expire during the pendency of the registration proceedings and ownership to
the property consolidated in the vendee a retro, the latter shall be substituted for the applicant and may continue the proceedings.

A trustee on behalf of his principal may apply for original registration of any land held in trust by him, unless prohibited by the instrument
creating the trust.

Each paragraph of Section 14 refers to a distinct type of application depending on the applicable legal ground. Since each type is governed
by its own set of legal principles, the framework for analysis to be used in resolving an application would vary depending on the paragraph
invoked.51 Hence, it is important for the Court to first determine the exact legal ground used by an applicant for registration.52

In this case, we note that the application filed by respondent before the RTC did not state the exact legal basis of her request. At best,
the pleading implied that her claim was one for registration and confirmation of title based on her possession and occupation of the
property:

COMES NOW Petitioner Rosario L. Nicolas, of legal age, widow, Pilipino [sic] with address at Brgy. San Isidro, Rodriguez (formerly
Montalban), Rizal Province, Philippines, by her undersigned counsel and to this Honorable Court respectfully petitions to have the land
hereinafter described below brought under the operation of the Land Registration Act and to have said land titled, registered and confirmed
in her name and further declares that:

xxxx

6. Petitioner acquired the subject parcel of land by way of occupation and has been in natural, open, public, adverse, contin[u]ous,
uninterrupted and in the concept of an owner/possessor thereof since October 1964 up to the present.53 (Emphases supplied)

From the foregoing allegations, it appears that the claim of respondent is anchored on either of the first two paragraphs of Section 14.
However, it is unclear whether she sought judicial confirmation and registration of her title pursuant to Section 14(1) of P.D. 1529, or of
the registration of her title on the ground of acquisitive prescription under Section 14(2) of the same law.

Similarly, no specific provision in P.D. 1529 was identified by the RTC when it granted the Petition.54 Its mention of the Civil Code,
however, seems to indicate an application of the principle of acquisitive prescription. The CA, for its part, delineated the differences
between the first two paragraphs of Section 14, but decided to apply both clauses. In its Decision, it ruled that respondent is entitled to
register her title under either paragraph:
From the evidence adduced, applicant-appellee has convincingly established her registrable title to the subject land, which is entitled to
confirmation and registration by the trial court. As testified by the daughter of applicant, her mother commenced occupying the subject
land since 1940 and up to the present which (sic) has been planted with fruit-bearing trees and vegetables by their caretaker. Her
testimony was corroborated by Santiago Eulin, their caretaker since 1942 who took over after his father, the original caretaker. These
witnesses declared that they even stayed on the land in question where the applicant has a hut. It was also established that the applicant
had the property surveyed in 1964 resulting in the approval of Plan PS U 213 31 by the Bureau of Lands. This qualifies applic ant under
Section 14, par. 1 of the Property Registration Decree.

Even assuming that applicant's occupation and possession of the subject land did not start on July 12, 1945 or earlier but only in 1964
when she had it surveyed, still she can apply for registration of title under Sec. 14, par. 2 of the Property Registration Decree as she has
been occupying the land continuously for more than thirty (30) years from the time the application was filed in 1996.55 (Emphases
supplied)

Given these findings, the Court has examined the application for registration in this case under the legal framework of both Section 14(1)
and (2) of P.D. 1529. We find that respondent has failed to sufficiently establish the requisites of both paragraphs; in particular, with
respect to the classification and the character of the land in question. Hence, we are constrained to reverse the CA and the RTC Decisions
allowing the registration of her title to the property.

Respondent has failed to prove that the property is alienable and disposable agricultural land that may be registered under Section 14(1)
of P.D. 1529.

Section 14(1) of P.D. 1529 governs applications for registration of alienable and disposable lands of the public domain. This paragraph
operationalizes Section 48(b) of Commonwealth Act No. 141 as amended.56 This provision grants occupants of public land the right to
judicial confirmation of their title. Based on these two provisions and other related sections of C.A. 141, registration is allowed provided
the following requisites have been complied with:

1. The applicant is a Filipino citizen.57

2. The applicant, by himself or through his predecessors-in-interest, has been in open, continuous, exclusive and notorious possession
and occupation of the property since 12 June 1945.58

3. The prope1ty has been declared alienable and disposable as of the filing of the application.59

4. If the area applied for does not exceed 12 hectares, the application should be filed by 31 December 2020.60

As earlier stated, respondent failed to establish the third requisite, i.e., that the property subject of the application is alienable and
disposable agricultural land.

The Court has emphasized in a long line of cases61 that an applicant for registration under Section 14(1) must prove that the subject
property has been classified as alienable and disposable agricultural land by virtue of a positive act of the Executive Department. In Heirs
of Malabanan v. Republic, 62 we declared:

Alienable and disposable lands of the State fall into two categories, to wit: (a) patrimonial lands of the State, or those classified as lands
of private ownership under Article 425 of the Civil Code, without limitation; and (b) lands of the public domain, or the public lands as
provided by the Constitution, but with the limitation that the lands must only be agricultural. Consequently, lands classified as forest or
timber, mineral, or national parks are not susceptible of alienation or disposition unless they are reclassified as agricultural. A positive act
of the Government is necessary to enable such reclassification, and the exclusive prerogative to classify public lands under existing laws
is vested in the Executive Department, not in the courts. xxx Thus, until the Executive Department exercises its prerogative to classify or
reclassify lands, or until Congress or the President declares that the State no longer intends the land to be used for public service or for
the development of national wealth, the Regalian Doctrine is applicable.

In this case, we note that both the RTC and the CA glossed over this requirement. The RTC, for instance, only made a general conclusion
as to the classification and alienability of the property, but without any discussion of the evidence presented:

From the evidence adduced, applicant-appellee has convincingly established her registrable title to the subject land which is entitled to
confirmation and registration by the trial court. x x x It was also established that the applicant had the property surveyed in 1964 resulting
in the approval of Plan PSU-213331 by the Bureau of Lands. This qualifies applicant under Sec. 14, par. 1 of the Property Registration
Decree.63

The CA, on the other hand, simply relied on the fact that the property had been the subject of a private survey in 1964:

From the evidence adduced, the following facts have been duly proved:

xxxx

That the land applied for is neither subject to any water, oil/nor (sic) mineral rights, not within any government reservation, naval or
military, or mineral rights, within the forest zone, and neither is it part of the inalienable or undisposable land of the public domain nor
covered by the Code on Comprehensive Agrarian Reform or subject to any subsisting Public Patent application;
xxxx

That the said parcel of land applied for is duly surveyed for registration (Exh. "J"), classified as agricultural; that they planted mangoes,
buko, sometimes corn in the area through their caretaker x x x.64

While a petition for review on certiorari under Rule 45 is generally limited to a review of errors of law, the Court may conduct its own
review of the evidence if the findings of the lower courts are bereft of legal and factual bases.65 In this case, the conclusions of the RTC
and the CA are not only contradicted by the evidence on record; they are likewise contrary to law and jurisprudence. As a result, the Court
is constrained to set aside these pronouncements.

To prove that the property subject of an application for original registration is part of the alienable and disposable lands of the public
domain, applicants must "identify a positive act of the government, such as an official proclamation, declassifying inalienable public land
into disposable land for agricultural or other purposes."66 To sufficiently establish this positive act, they must submit (1) a certification
from the CENRO or the Provincial Environment and Natural Resources Office (PENRO); and (2) a copy of the original classification
approved by the DENR Secretary and certified as a true copy by the legal custodian of the official records.67

Here, respondent presented the following pieces of evidence to establish her claim that the land had been classified as agricultural and
considered alienable and disposable:

(1) A CENRO Report68 stating that the land was not covered by any public land application or embraced by any administrative title, but
with a notation that that the alienability of the land was "[n]ot projected due to [u]navailability of coordinates re: Tala Estate Tieline'';

(2) A CENRO Certification69 that the lot "is not covered by any kind of public land application";

(3) A Report70 from the Land Registration Authority (LRA) declaring that it was "not in a position to verify whether or not the parcel of
land subject of registration is already covered by land patent and is within the area classified as alienable and disposable land of the
public domain"; and

(4) The testimonies of Leonila Alfaro,71 her daughter, and Santiago Eulin72 (the caretaker of the land) confirming that the property is
agricultural in nature.

It is evident from the foregoing enumeration that respondent not only neglected to submit the required CENRO/PENRO certification and
DENR classification, but also presented evidence that completely failed to prove her assertion.

First, the testimonies of Leonila and Santiago on the classification of the land have very little evidentiary value. That they consider the
property agricultural in nature is irrelevant, as their statements are mere opinions bereft of any legal significance.

Second, none of the documents submitted by respondent to the trial court indicated that the subject property was agricultural or part of
the alienable and disposable lands of the public domain. At most, the CENRO Report and Certification stated that the land was not
covered by any kind of public land application. This was far from an adequate proof of the classification of the land. In fact, in Republic v
Lualhati, 73 the Court rejected an attempt to prove the alienability of public land using similar evidence:

Here, respondent failed to establish, by the required evidence, that the land sought to be registered has been classified as alienable or
disposable land of the public domain. The records of this case merely bear certifications from the DENR-CENRO, Region IV, Antipolo
City, stating that no public land application or land patent covering the subject lots is pending nor are the lots embraced by any
administrative title. Said CENRO certifications, however, do not even make any pronouncement as to the alienable character of the lands
in question for they merely recognize the absence of any pending land patent application, administrative title, or government project being
conducted thereon. But even granting that they expressly declare that the subject lands form part of the alienable and disposable lands
of the public domain, these certifications remain insufficient for purposes of granting respondent's application for registration. As constantly
held by this Court, it is not enough for the CENRO to certify that a land is alienable and disposable. The applicant for land registration
must prove that the DENR Secretary had approved the land classification and released the land of the public domain as alienable and
disposable, and that the land subject of the application for registration falls within the approved area per verification through survey by
the PENRO or CENRO. Unfortunately for respondent, the evidence submitted clearly falls short of the requirements for original registration
in order to show the alienable character of the lands subject herein.

Applying these standards to the instant case, we declare that the RTC did not have sufficient basis for its finding that the property in
question was alienable and disposable.

The Court also finds that the ruling of the CA on the evidentiary value of the private survey is untenable. The fact that the land has been
privately surveyed is not sufficient to prove its classification or alienable character. While the conduct of a survey and the submission of
the original tracing cloth plan are mandatory requirements for applications for original registration of land under P.D. 1529, they only serve
to establish the true identity of the land and to ensure that the property does not overlap with another one covered by a previous
registration.74 These documents do not, by themselves, prove a1ienability and disposability of the property. In fact, in several cases,75
the Court has declared that even a survey plan with a notation that the property is alienable cannot be considered as sufficient evidence
of alienability. Here, the survey plan and original tracing cloth plan submitted by respondent does not even bear that notation.
Consequently, it was grave error for the CA to consider the mere conduct of a private survey as proof of the classification and the
alienability of the land.
Respondent has failed to prove that the land subject of the application is part of the patrimonial property of the State that may be acquired
by prescription under Section 14(2) of P.D. 1529.

As previously noted, the CA also allowed the registration of the property under Section 14(2) of P.D. 1529 based on the following findings:
(1) the property is "private in nature" as shown by the fact that it is "covered by a private survey";76 (2) respondent had occupied the land
continuously for more than 30 years from the time of the filing of the application in 1996;77 and (3) the land is not covered by any public
land application based on the DENR-CENRO Certifications submitted by respondent.78

We do not agree. The Court fields no sufficient basis to allow the registration of the property under Section 14(2).

By express provision of the law, only private lands that have been acquired by prescription under existing laws may be the subject of
applications for registration under Section 14(2). The starting point of the Court's evaluation must, therefore, be whether the property
involved falls within the scope of the paragraph.

Under the Civil Code, all things within human commerce are generally susceptible of prescription.79 Properties of the public dominion,
or those owned by the State, are expressly excluded by law from this general rule,80 unless they are proven to be patrimonial in character.
As the Court explained in Republic of the Philippines v. Tan:

Only private property can be acquired by prescription. Property of public dominion is outside the commerce of man.1âwphi1 It cannot be
the object of prescription because prescription does not run against the State in its sovereign capac.ty. However, when property of public
dominion is no longer intended for public use or for public service, it becomes part of the patrimonial property of the State. When this
happens, the property is withdrawn from public dominion and becomes property of private ownership, albeit still owned by the State. The
property is now brought within the commerce of man and becomes susceptible to the concepts of legal possession and prescription.81
(Emphasis supplied)

To establish that the land subject of the application has been converted into patrimonial property of the State, an applicant must prove
the following:

l. The subject property has been classified as agricultural land.82

2. The property has been declared alienable and disposable.83

3. There is an express government manifestation that the property is already patrimonial, or is no longer retained for public service or the
development of national wealth.84

It must be emphasized that without the concurrence of these three conditions, the land remains part of public dominion and thus incapable
of acquisition by prescription.85

Here, the records show that respondent has failed to allege or prove that the subject land belongs to the patrimonial property of the
State.1âwphi1 As earlier discussed, the evidence she has presented does not even show that the property is alienable and disposable
agricultural land. She has also failed to cite any government act or declaration converting the land into patrimonial property of the State.

Contrary to the ruling of the CA, the DENR-CENRO Certifications submitted by respondent are not enough; they cannot substitute for the
three conditions required by law as proof that the land may be the subject of prescription under the Civil Code. For the same reason, the
mere conduct of a private survey of a property - even with the approval of the Bureau of Lands - does not convert the lot into private land
or patrimonial property of the State. Clearly, the appellate court erred when it relied on the survey to justify its conclusion that the land is
private in nature.

Considering the absence of sufficient evidence that the subject land is a patrimonial property of the State, we must consider it part of
public dominion and thus immune from acquisitive prescription.

As a final note, the Court must point out that proof of the classification, alienability and disposability of the subject property is of particular
significance in applications for the registration of land. Given the general rule that public lands may not be alienated,86 it is the burden of
applicants to prove that the land they seek to register falls within the classifications enumerated in Section 14 of P.D. 1529; in particular,
the specific paragraph they invoke as basis for registration.87 Absent that proot: no length of possession or occupation would vest any
right of ownership over the property,88 and registration under P.D. 1529 cannot be sanctioned by this Court.

WHEREFORE, the Petition is hereby GRANTED.

REPUBLIC VS. NOVAL

On September 8, 1999, the applicants sought the registration of their titles over the subdivided portions of a land in Barangay Casili,
Consolacion, Cebu, designated as Lot 4287 of Consolacion Cadastre. They alleged to have acquired their respective portions of this land
by "purchase, coupled with continuous, public, notorious, exclusive and peaceful possession in the concept of an owner for more than 30
years including [the possession] of their predecessors-in-interest." They also alleged that they were in actual possession of their
respective portions of the property.4

The Republic through the Office of the Solicitor General, filed its Opposition on the ground that the applicants failed to prove open,
continuous, exclusive, and notorious possession of the property since June 12, 1945.5 It also argued that the property sought to be
registered was part of the public domain.6 It alleged that the tax declarations and tax payment receipts attached to the application were
not competent to show bona fide acquisition or open and continuous possession of the land.7

The applicants' immediate predecessor-in-interest was Cecilia Alilin Quindao (Cecilia), who was already 73 years old when she testified
before the trial court. She said that she was familiar with Lot 4287 since she was 15 years old. Her grandmother, Flaviana Seno Alilin
(Flaviana), had already possessed and owned this property and enjoyed the fruits of 15 coconut trees already growing there. Her
grandmother's possession was "peaceful exclusive, adverse, public and in the concept of [an] owner."8

Cecilia's father, Miguel Alilin (Miguel), inherited the property when Flaviana died.9 Cecilia was then 20 years old.10 Miguel tilled and
cultivated the land and planted root crops, corn and other plants.11 Their family enjoyed the fruits of his cultivation of the land.12 When
he died, Cecilia inherited the property.13 She also tilled the land and declared it in her name for taxation.14 She even shared the produce
of the land with her tenant.15 Later, she sold the property to Joel Noval (Joel) and Elizabeth Messerli (Messerli).16 Messerli sold her
property to the Spouses Noval and Refi.17 Soon the property was partitioned as follows: Lot 1 to the Spouses Noval; Lot 2 to Gertrudes
Noval, who later donated hiS, share to delos Reyes; Lot 3 to Lao; Lot 4 to Refi; Lot 5 to Dale; and Lot 7 to Dale and Morales.18 All of
them later on took possession of their respective portions and declared them in their respective names.19

The Municipal Trial Court granted their application for registration of title. It declared the applicants to be the absolute owners and
possessors of their respective lots, having established conclusively that they are the exclusive owners and peaceful possessors of the
properties. The trial court ordered the issuance of decrees of registration upon finality of its judgment.20

The Republic appealed the Decision of the trial court,21 arguing that the applicants failed to show open, continuous, exclusive and
notorious possession of alienable and disposable lands for 30 years.22 It reiterated that tax declarations may not be used as bases for
the grant of the application.23 It added that there was no Department of Environment and Natural Resources report submitted to show
when the properties were declared alienable and disposable, for the purpose of computin2 the 30-year period of possession required by
law.24

The Court of Appeals, however, affirmed25 the Decision of the Municipal Trial Court.26

The Court of Appeals found that the required period of possession in land registration cases was satisfied. It noted that Cecilia was
already 73 years old when she testified in 2000 that the property had already been owned and possessed by Cecilia's grandmother since
Cecilia was 15 years old. It held that at 15 years of age, she was already competent to perceive that her grandmother's possession was
in the concept of an owner.27

The Court of Appeals also found that while the applicants did not submit a Department of Environment and Natural Resources report
showing that the property had been declared alienable and disposable, the Republic was not relieved of the duty to present evidence that
the land belongs to the public domain. It ruled that the burden is upon the State to prove that land is public domain when it has been
possessed and cultivated by an applicant and his or her predecessors-in-interest for a considerable number of years without action from
the State. The Court of Appeals added that the open, continuous, adverse, and public possession of land from time immemorial confers
an effective title to the possessor.28

The Court of Appeals likewise recognized that while tax declarations are not conclusive evidence of ownership, they may give weight to
a claim of ownership when coupled with open, adverse, and continuous possession.29

The Republic sought the reconsideration of the Court of Appeals Decision, but this was denied in a Resolution30 dated October 28,
2005.31

Hence, this Petition32 was filed.

Petitioner argues that respondents failed to show that they or their predecessor-in-interest have been in open, continuous, exclusive, and
notorious possession and occupation of the land for the period required by law.33 It also contends that the tax declarations presented by
respondents are not conclusive evidence of ownership and possession for at least 30 years.34 It likewise asserts that the property may
not be registered without a certification from the Department of Environment and Natural Resources that it has been declared alienable
and disposable.35 Failure to show such certification means that the land belongs to the State.36 It submits that the burden of proof is
upon respondents to show that Lot 4287 had already been declared alienable and disposable at the time of their application.37

Respondents, on the other hand, counter that Cecilia's testimony was sufficient to establish the nature of her possession and that of her
predecessors-in-interest.38 Thy submit that the property has been declared for tax purposes since 194539 and that while the Department
of Environment and Natural Resources did not issue a certification, it did approve their survey plan when the property was partitioned.40

For this Court's resolution is the sole issue of whether or not the Court of Appeals erred in affirming the trial court decision to allow the
Spouses Joel and Andrea Noval, Ellen N. delos Reyes, Dale Y. Noval, Winnie T. Refi, Zenaida Lao, and Daisy N. Morales to register
their respective portions of Lot 4287.

Any person seeking relief under Commonwealth Act No. 141, or the Public Land Act, admits that the property being applied for is public
land.
Under the Public Land Act, public lands may be disposed of through confirmation of imperfect or incomplete titles.41 Confirmation of title
may be done judicially or through the issuance of a free patent.42 The process for judicial confirmation of title is outlined in Section 48 of
the Public Land Act, as amended by Presidential Decree No. 1073:43
Section 48. The following described citizens of the Philippines, occupying lands of the public domain or claiming to own any s ch lands
or an interest therein. but whose titles have not been perfected or completed, may apply to the Court of First Instance of the province
where the land is located for confirmation of their claims and the issuance of a certificate of title therefor, under the Land Registration Act,
to wit:

....

(b) Those who by themselves or through their predecessors in interest have been in the open, continuous, exclusive, and notorious
possession and occupation of agricultural lands of the public domain, under a bona fide claim of acquisition or ownership, except as
against the government, since July twenty-sixth, eighteen hundred and ninety-four, except when prevented by war or force majeure.
These shall be conclusively presumed to have performed all the conditions essential to a Government grant and shall be entitled to a
certificate of title under the provisions of this chapter.
When a person applies for judicial confirmation of title, he or she already holds an incomplete or imperfect title over the property being
applied for, after having been in open, continuous, exclusive, and notorious possession and occupation from June 12, 1945 or earlier.
The date "June 12, 1945" is the reckoning date of the applicant's possession and occupation, and not the reckoning date of when the
property was classified as alienable and disposable.44 In Heirs of Malabanan v. Republic:45
[T]he choice of June 12, 1945 as the reckoning point of the requisite possession and occupation was the sole prerogative of Congress,
the determination of which should best be left to the wisdom of the lawmakers. Except that said date qualified the period of possession
and occupation, no other legislative intent appears to be associated with the fixing of the date of June 12, 1945. Accordingly, the Court
should interpret only the plain and literal meaning of the law as written by the legislators.

Moreover, an examination of Section 48 (b) of the Public Land Act indicates that Congress prescribed no requirement that the land subject
of the registration should have been classified as agricultural since June 12, 1945, or earlier. As such, the applicant's imperfect or
incomplete title is derived only from possession and occupation sine June 12, 1945, or earlier. This means that the character of the
property subject of the application as alienable and disposable agricultural land of the public domain determines its eligibility for land
registration, not the ownership or title over it.46
Thus, a property applied for judicial confirmation of title may be classified as alienable and disposable at any time. For the purposes of
judicial confirmation of title, only possession and occupation must be reckoned from June 12, 1945.

II

The Public Land Act is a special law that applies only to alienable agricultural lands of the public domain, and not to forests, mineral lands,
and national parks.47Heirs of Malabanan v. Republic48 categorized alienable and disposable lands into: "(a) patrimonial lands of the
State, or those classified as lands of private ownership under Article 425 of the Civil Code, without limitation; and (b) lands of the public
domain, or the public lands as provided by the Constitution, but with the limitation that the lands must only be agricultural."49 Thus, for
Section 48(b) of the Public Land Act to apply, the property first, must be agricultural land of the public domain, and second, must have
been declared as alienable and disposable.50

Parenthetically, not all lands and natural resources, by default, belong to the State.

The theory that all lands belong to the State was introduced in this jurisdiction. during the Spanish colonization. When Spain transferred
sovereignty of the Philippines to the United States in 1898 through the Treaty of Paris, the United States opted not to adopt this concept.
Instead, it created new presumptions with respect to land ownership. This was thoroughly explained in Carino v. Insular Government:51
It is true that Spain, in its earlier decrees, embodied the universal feudal theory that all lands were held from the Crown, ... It is true also
that, in legal theory, sovereignty is absolute, and that, as against foreign nations, the United States may assert, as Spain asserted,
absolute power. But it does not follow that, as against the inhabitants of the Philippines, the United States asserts that Spain had such
power. When, theory is left on one side, sovereignty is a question of strength, and may vary in degree. How far a new sovereign shall
insist upon the theoretical relation of the subjects to the head in the pas{, and how far it shall recognize actual facts, are matters for it to
decide.

The Province of Benguet was inhabited by a tribe that the Solicitor General, in his argument, characterized as a savage tribe that never
was brought under the civil or military government of the Spanish Crown. It seems probable, if not certain, that the Spanish officials would
not have granted to anyone in that province the registration to which formerly the plaintiff was entitled by the Spanish laws, and which
would have made his title beyond question good. Whatever may have been the technical position of Spain, it does not follow that, in the
view of the United States, he had lost all rights and was a mere trespasser when the present government seized his land. The argument
to that effect seems to amount to a denial of native titles throughout an important part of the island of Luzon, at least, for the want of
ceremonies which the Spaniards would not have permitted and had not the power to enforce.

The acquisition of the Philippines was not like the settlement of the white race in the United, States. Whatever consideration may have
been shown to the North American Indians, the dominant purpose of the whites in America was to occupy the land. It is obvious that,
however stated, the reason for our taking over the Philippines was different. No one, we suppose, would deny that, so far as consistent
with paramount necessities, our first object in the internal administration of the islands is to do justice to the natives, not to exploit their
country for private gain. By the Organic Act of July 1, 1902, c. 1369, § 12, 32 Stat. 691, all the property and rights acquired there by the
United States are to be administered "for the benefit of the inhabitants thereof." It is reasonable to suppose that the attitude thus assumed
by the United States with regard to what was unquestionably its own is also its attitude in deciding what it will claim for its own. The same
statute made a bill of rights, embodying the safeguards of the Constitution, and, like the Constitution, extends those safeguards to all. It
provides that "no law shall be enacted in said islands which shall deprive any person of life, liberty, or property without d ue process of
law, or deny to any person therein the equal protection of the laws." § 5. In the light of the declaration that we have quoted from § 12, it
is hard to believe that the United States was ready to declare in the next breath that "any person" did not embrace the inhabitants of
Benguet or that it meant by "property" only that which had become such by ceremonies of which presumably a large part of the inhabitants
never had heard, and that it proposed to treat as public land what they, by native custom and by long association - one of the profoundest
factors in human thought - regarded as their own.52 (Emphasis supplied)
The United States chose to limit its sovereign exercise to the fiduciary administration of the Philippines. Instead of exercising absolute
power with respect to property rights, it chose to adopt due process as embodied in the Bill of Rights. This due process clause is already
found in our present Constitution. Thus, Article III, Section 1 of the Constitution states:
Section 1. No person shall be deprived of life, liberty, or property without due process of law, nor shall any person be denied the equal
protection of the laws.
Most notably, however, Carino created a presumption against State ownership and recognized private property rights independent of
State grant. Thus:
[E]very presumption is and ought to be against the government in a case like the present. It might, perhaps, be proper and sufficient to
say that when, as far back testimony or memory goes, the land has been held by individuals under a claim of private ownership, it will be
presumed to have been held in the same way from before the Spanish conquest, and never to have been public land.53
Carino did not qualify that the existence of property rights independent of State grant and the presumptions on land registration apply
only to the indigenous cultural communities, These principles can be seen in the present land registration laws.

Under the Public Land Act, ownership is recognized if possession dates back since June 12, 1945 or earlier.54 The law refers to this as
"judicial legalization," which allows for agricultural public lands to be disposed of by the, State and acquired by Filipino citizens.55

Presidential Decree No. 1529, or the Property Registration Decree, has a similar provision, but also recognizes ownership through
prescription.56 Section 14(1) of the Property Registration Decree provides:
Section 14. Who may apply. - The following persons may file in the proper Court of First Instance an application for registration of title to
land, whether personally or through their duly authorized representatives:

(1)
Those who by themselves or through their predecessors-in-interest have been in open, continuous, exclusive and notorious possession
and occupation of alienable and disposable lands of the public domain under a bona fide claim of ownership since June 12, 1945, or
earlier.
Section 14(1) does not vest or create a title to public land.57 The procedure of registering one's title "simply recognizes and documents
ownership and provides for the consequences of issuing paper titles."58

These provisions are the latest versions of a catena of provisions on judicial confirmation of imperfect or incomplete titles.59 All these
laws recognize ownership acquired through possession and occupation in the concept of an owner.

That the law provides for confirmation of titles based on possession and occupation is an acknowledgment of the existence of property
rights independent of State grants. It is ail. acknowledgment $at registration is a means only to document ownership already acquired.

Be that as it may, applicants for judicial confirmation of title must still comply with the requisites stated in Section 48(b) of the Public Land
Act and Section 14(1) of the Property Registration Decree:
The applicant, by himself or through his predecessor-in--interest, has been in possession and occupation of the property subject of the
application;

The possession and occupation must be open, continuous, exclusive, and notorious;

The possession and occupation must be under a bona fide claim of acquisition of ownership;

The possession and occupation must have taken place since June 12, 1945, or earlier; and

The property subject of the application must be an agricultural land of the public domain.60
III

Petitioner argues that respondents were unable to prove that they and their predecessor-in-interest were able to prove their open and
continuous possession and occupation of the property for the period required by law. It describes respondents' and their predecessor-in-
interest's possession as mere casual cultivation, which is not the possession contemplated by land registration laws.

Both the Municipal Trial Court and the Court of Appeals established that respondents and their predecessor-in-interest were the exclusive
owners and possessors of the land. Both courts affirmed that respondents have met the required period of possession for land registration
cases.61 They acknowledged the credibility of the testimony of respondents' predecessor--in-interest, which established possession of
Lot 4287 in the concept of an owner since 1942 or earlier.62 This means that respondents and their predecessor-in-interest have already
been in occupation and possession of the land for more than 50 years at the time of their application for registration.

Only questions of law may be raised in a petition for review on certiorari.63 This Court has repeatedly said that findings of facts of the
lower courts deserve high respect since they are in the best position to pass judgment on the credibility of the witnesses and their
statements. This Court rarely questions facts as determined by the lower court, especially when they are affirmed by the Court of Appeals.
The findings of facts are often conclusive upon this Court, subject only to a few exceptions:
(1) When the conclusion is a finding grounded entirely on speculation, surmises or conjectures . . .; (2) When the inference made is
manifestly mistaken, absurd or impossible . . .; (3) Where there is a grave abuse of discretion . . .; (4) When the judgment is based on a
misapprehension of facts . . .; (5) When the findings of fact are conflicting . . .; (6) When the Court of Appeals, in making its findings, went
beyond the issues of the case and the same is contrary to the admissions of both appellant and appellee . . .; (7) The findings of the Court
of Appeals are contrary to those of the trial court . . .; (8) When the findings of fact are conclusions without citation of specific evidence
on which they are based; (9) When the facts set forth in the petition well as in the petitioners' main and reply briefs are not disputed by
the respondents; and (10) The finding of fact of the Court of Appeals is premised on the supposed absence of evidence and is contradicted
by the evidence on record . . .64
This case does not fall under any of the exceptions. Since the Court of Appeals affirmed the findings of the trial court, and there is no
showing that the conclusions made by both courts are either made with grave abuse of discretion or contrary to the evidence presented
and the law, this Court will not disturb these findings.

Respondents' predecessor-in-interest recalled her grandmother to have already cultivated fruit-bearing trees on Lot 4287 when she was
15 years old. Possession prior to that "can hardly be estimated . . . the period of time being so long that it is beyond the reach of
memory."65

Hence, respondents' and their predecessor-in-interest's possession is, with little doubt, more than 50 years at the time of respondents'
application for registration in 1999. This is more than enough to satisfy the period of possession required by law for acquisition of
ownership.

IV

The burden of proving that the property is an alienable and disposable agricultural land of the public domain falls on the applicant, not
the State.66 The Office of the Solicitor General, however, has the correlative burden to present effective evidence of the public character
of the land.67

In order to establish that an agricultural land of the public domain has become alienable and disposable, "an applicant must establish the
existence of a positive act of the government such as a presidential proclamation or an executive order; an administrative action;
investigation reports of Bureau of Lands investigators; and a legislative act or a statute."68 It is settled that the declaration of alienability
must be through executive fiat, as exercised by the Secretary of the Department of Environment and Natural Resources.69Republic v.
T.A.N. Properties70 provided further:
The applicant for land registration must prove that the [Department of Environment and Natural Resources] Secretary had approved the
land classification and released the land of the public domain as alienable and disposable, and that the land subject of the application for
registration falls within the approved area per verification through survey by the [Provincial Environment and Natural Resources Officer]
or [City Environment and Natural Resources Officer]. In addition, the applicant for land registration must present a copy of the original
classification approved by the [Department of Environment and Natural Resources] Secretary and certified as a true copy by the legal
custodian of the official records.71
Admittedly, respondents have failed to present any document from the Secretary of the Department of Environment and Natural
Resources certifying that the property is part of the alienable and disposable land of the public domain. On the other hand, the Court of
Appeals observed, as this Court has, that the Office of the Solicitor General has failed to "present any evidence, testimonial or
documentary evidence to support its opposition."72

When the State has no effective opposition, except for a pro forma opposition, to controvert an applicant's convincing evidence of
possession and occupation, presumptions are tilted to this applicant's favor.73 In Republic v. Barandiaran:74
"[W]here it appears that the evidence of ownership and possession are so significant and convincing, the government is not necessarily
relieved of its duty from presenting proofs to show that the parcel of land sought to be registered is part of the public domain to enable
[the courts] to evaluate the evidence of both sides." . . . [W]hen the records shows that a certain property, the registration of title to which
is applied for has been possessed and cultivated by the applicant and his predecessors in interest for a long number of years without the
government taking any action to dislodge the occupants from their holdings, and when the land has passed from one hand to another by
inheritance or by purchase, the government is duty bound to prove that the land which it avers to be of public domain is really of such
nature.75 (Citations omitted)
Indeed, the Public Land Act itself establishes a conclusive presumption in favor of the possessor that all conditions essential to a State
grant, including the conversion of a land in the public domain to a private property, have been performed, entitling him or her to a certificate
of title.76

Therefore, when an applicant is shown to have been in open, continuous, exclusive, and notorious possession of a land for the period
required by law, he or she has acquired an imperfect title that may be confirmed by the State. The State may not, for the simple reason
that an applicant failed to show documents which the State is in the best position to acquire, indiscriminately take an occupied property
and unjustly and self--servingly refuse to acknowledge legally recognized rights evidenced by possession, without violating due
process.77

The burden of evidence lies on the party who asserts an affirmative allegation.78 Therefore, if the State alleges that lands belong to it, it
is not excused from providing evidence to support this allegation.79 This specially applies when the land in question has no indication of
being incapable of registration80 and has been exclusively occupied by an applicant or his or her predecessor-in-interest without
opposition-not even from the State.

Hence, when a land has been in the possession of the applicants and their predecessor-in-interest since time immemorial and there is
no manifest indication that it is unregistrable, it is upon the State to demonstrate that the land is not alienable and disposable. "[A] mere
formal opposition on the part of the [Solicitor General] . . ., unsupported by satisfactory evidence, will not stop the courts from giving title
to the claimant."81

This Court's previous rulings imposing the burden of overcoming the presumption that a land is public should only be strictly applied when
a manifestly unregistrable land is in danger of fraudulent titling-not when it will promote unfairness and violation of due process rights.

Respondents' and their predecessor-in-interest's possession was never opposed, even at the time of application, by the government
agencies tasked to ensure that public lands remain public. There was neither indication nor mention that Lot 4287 was forest, timber land,
or belonging to a reservation.

The State also kept silent on respondents' and their predecessor-in interest's continuously paid taxes. The burden to prove the public
character of Lot 4287 becomes more pronounced when the State continuously accepts payment of real property taxes. This Court
acknowledges its previous rulings that payment of taxes is not conclusive evidence of ownership.82 However, it is good indicia of
possession in the concept of an owner, and when coupled with continuous possession, it constitutes strong evidence of title.

No person in the right mind would pay taxes on real property over which he or she does not claim any title.83 Its declaration not only
manifests a sincere desire to obtain title to a property; it may be considered as an announcement of an adverse claim against State
ownership.84 It would be unjust for the State to take properties which have been continuously and exclusively held since time immemorial
without showing any basis for the taking, especially when it has accepted tax payments without question.

However, despite these circumstances, petitioner failed to show any evidence that Lot 4287 remained public land. Instead, it conveniently
relied on the absence of a Department of Environment and Natural Resources certification.

Therefore, this Court is constrained to hold that respondents' evidence, coupled with the absence of contradictory evidence from
petitioner, substantially establishes that respondents have complied with the requisites of Section 48(b) of the Public Land Act and Section
14(1) of the Property Registration Decree. The Municipal Trial Court and the Court of Appeals did not err in approving the registration of
the property.

WHEREFORE, the Petition is DENIED.

DIAZ-ENRIQUEZ VS. DOL

THE FACTS

On 27 December 1974, Geronimo, Josefino, and Rodrigo, all surnamed Saclolo (the Saclolos) filed before the then Court of First Instance,
now Regional Trial Court, Naic, Cavite, a joint application for registration of title over three (3) parcels of land (subject lands), with a total
area of 3,752,142 square meters (375.2 hectares) and located at Sitio Sinalam, Bario Sapang, Ternate, Cavite.6 The Saclolos averred
that they had acquired title to the subject lands through purchase and that together with their predecessors-in-interest, they had been in
actual and exclusive possession, occupation, and cultivation of the subject lands since time immemorial.7

The government, thru the Director of Lands, Abdon Riego de Dios, and Angelina Samson filed oppositions to the application.8 The
Director of Lands argued that the subject lands are not alienable and disposable because: they are located within the Calumpang Point
Naval Reservation, segregated from the public domain by Proclamation No. 307, dated November 20, 1967; that by virtue of Republic
Act (R.A.) No. 6236, the right to judicial confirmation of imperfect title under Section 48 of the Public Land Law, with respect to lands
having an area of more than 144 hectares, has expired; that the Saclolos had not acquired title over the subject lands through any
recognized mode of acquisition of title; that the Saclolos and their predecessors-in-interest had not been in open, continuous, exclusive,
and notorious possession and occupation of the subject lands for at least 30 years immediately preceding the filing of the application;
and that PSU 68, 69, and 70, the plans which cover the subject lands, have not been verified by the Bureau of Lands as required by
Presidential Decree (P.D.) No. 239.9

On 27 December 1993, Trinidad Diaz-Enriquez (Enriquez) filed a motion for intervention alleging that the Saclolos had sold to her all their
interests and rights over the subject lands on 19 September 1976. The RTC allowed Enriquez's claim to be litigated.10

The RTC Ruling

In its Decision, dated 6 July 1995, the RTC ruled that the subject lands are alienable and disposable lands of the public domain because
Proclamation No. 307 itself stressed that the segregation of the Calumpang Point Naval Reservation was subject to private rights. It
opined that the pieces of evidence presented by the Saclolos proved that their rights over the subject lands, being private in nature and
character, were excluded from the reservation for military purposes. The fallo reads:

Wherefore, finding the evidence of applicants sufficient, their titles to the parcels of land applied for are hereby confirmed. The Land
Registration Authority is hereby Ordered to issue the corresponding decrees of registration and certificates of title in the names of the
applicants subject to the intervenor's rights upon finality of judgment.11

In its Order, dated 30 January 1996, the RTC modified its earlier decision by ordering the issuance of the decree of registration to
Enriquez.12

The CA Ruling
In its assailed decision, dated 26 May 2004, the CA declared that the subject lands are all within the Calumpang Point Naval Resevation,
as testified to by Eleuterio R. Paz, Chief of the Survey Division of the Bureau of Lands-Region 4; thus, the said lands could not be privately
titled. It held that even if Proclamation No. 307 qualifies the reservation as being subject to private rights, the Saclolos have not established
by adequate proof their open, continuous, exclusive, and notorious possession over the subject lands.

The appellate court observed that the informacion possessoria, upon which the Saclolos heavily rely to support their claim, did not at all
indicate the area covered by the claim. It added that the tax declarations, technical descriptions, sketch plans, tax receipts, deeds of sale,
and surveyor's certificates did not show the nature of the Saclolos' possession.

The CA stated that the trial court disregarded the fact that judicial confirmation of imperfect title under Section 48 of the Public Land Act
with respect to lands having an area of more than 144 hectares had lapsed pursuant to R.A. No. 6236, approved on 19 June 1971. It
further noted that the trial court's jurisdiction to entertain the application was not established since the plans had not been verified by the
Bureau of Lands as required by P.D. No. 239 and the alleged verifications in the plans were not authentic. The appellate court concluded
that the subject lands could not be registered because they lie within a naval reservation and most of them are forest and foreshore lands.
It disposed the case thus:

WHEREFORE, premises considered, the January 30, 1996 order of the trial court is REVERSED and SET ASIDE, and a new judgment
is entered DISMISSING the applications for registration of title to the subject three (3) lots in LRC Case No. TM-95 for lack of jurisdiction
and failure to prove acquisitive prescription.13

Aggrieved, the Saclolos and Enriquez moved for reconsideration, but the same was denied by the CA in its Resolution, dated 13 May
2005.

Hence, these consolidated petitions.

THE ISSUES

In G.R. No. 168070, the Saclolos raised the following issues:

WHETHER OR NOT THE RESPONDENT COURT OF APPEALS HAS DECIDED THE CASE ( CA- G.R. CV NO. 53838 (LRC CASE
NO. TM - 95 OF RTC, BRANCH XV, NAIC, CAVITE) IN A WAY NOT PROBABLY IN ACCORDANCE WITH LAW OR WITH THE
APPLICABLE DECISIONS OF THE SUPREME COURT.

WHETHER OR NOT THE RESPONDENT COURT OF APPEALS IN MAKING ITS FINDING, WENT BEYOND THE ISSUES RAISED
ON APPEAL AND THE SAME IS CONTRARY TO THE ADMISSIONS OF BOTH APPELLANTS AND APPELLEES.

WHETHER OR NOT THE RESPONDENT COURT OF APPEALS MANIFESTLY OVERLOOKED CERTAIN RELEVANT FACTS NOT
DISPUTED BY THE PARTIES AND WHICH, IF PROPERLY CONSIDERED, WOULD JUSTIFY A DIFFERENT CONCLUSION.

WHETHER OR NOT THE RESPONDENT COURT OF APPEALS HAS COMMITTED A GRAVE ABUSE OF DISCRETION WHEN IT
DECLARED THAT THE TRIAL COURT HAD NO JURISDICTION TO TRY THE CASE AND WHETHER OR NOT IN RENDERING THE
QUESTIONED DECISION DATED MAY 26, 2004, AND IN ISSUING THE QUESTIONED RESOLUTION, DATED MAY 13, 2005 THE
RESPONDENT COURT OF APPEALS COMMITTED A MISAPPREHENSION OF FACTS.

WHETHER OR NOT THE RESPONDENT COURT OF APPEALS ALSO COMMITTED A GRAVE ABUSE OF DISCRETION WHEN IT
DID NOT RESOLVE THE ISSUES RAISED BY PETITIONERS AS APPLICANTS-APPELLANTS IN CA-G.R. CV NO. 53838 OF THE
RESPONDENT COURT.14
On the other hand, in G.R. No. 168065, Enriquez submits the following assignment of errors:

The HONORABLE COURT OF APPEALS COMMITTED GRAVE ABUSE OF DISCRETION WHEN IT VIOLATED AND CONTRAVENED
SECTION 3, RULE 41 OF THE REVISED RULES ON CIVIL PROCEDURE.

THE HONORABLE COURT OF APPEALS COMMITTED GRAVE ABUSE OF DISCRETION IN FINDING THAT INTERVENOR HAS NO
REGISTRABLE TITLE.

THE HONORABLE COURT OF APPEALS CAPRICIOUSLY, ARBITRARILY AND WHIMSICALLY FOUND THAT THE REGIONAL TRIAL
COURT HAD NO JURISDICTION TO TRY THE CASE.15
In sum, the issues are: 1) Whether the appellate court may declare that the lands sought to be registered are not alienable and disposable
notwithstanding the failure of the Director of Lands to appeal from the decision of the trial court decreeing the issuance of certificates of
title; 2) Whether the appellate court may resolve issues which are not raised as errors on appeal; and 3) Whether the applicants for
registration of title have sufficiently proved that the subject lands are alienable and disposable.

In G.R. No. 168070, the Saclolos argue that the Director of Lands did not appeal from the RTC decision, thus, the facts pertaining to the
registration of titles are already final and settled; and that Proclamation No. 307 even strengthens their rights over the subject lands for
the same proclamation expressly recognizes the rights of private parties.

In G.R. No. 168065, Enriquez, citing Carrion v. CA,16 avers that the appellate court committed a reversible error when it modified the
decision of the trial court and granted to the Director of Lands, who did not appeal from such decision, affirmative reliefs other than those
granted to them by the trial court's judgment; that Proclamation No. 1582-A excluded the private occupants from the coverage of the
Calumpang Point Naval Reservation; that based on uncontroverted evidence, it has been established that the Saclolos' predecessors-in-
interest have declared the subject lands for taxation purposes as early as 1945; and that the Director of Lands should have raised the
plans' lack of verification during the trial of the case.

In his Comment,17 the Director of Lands, citing Baquiran v. CA, counters that issues, though not specifically raised in the pleadings in
the appellate court, may, in the interest of justice, be properly considered by the said court in deciding a case, if there are questions raised
in the trial court and are matters of record having some bearing on the issue submitted which the parties failed to raise or which the lower
court ignored; that Delfin Buhain, the alleged caretaker of the Saclolos and the husband of the Saclolos' alleged predecessor-in-interest
Pasencia Ruffy, testified that since he came to know of the land and up to the time it was sold to the Saclolos, his parents-in-law, his wife,
and brother-in-law Roman Bernardo Ruffy had possessed the same in the concept of a true and legal owner, though he could not
remember when the Saclolos bought it from his wife and brother-in-law; that the deed of sale between the Ruffys and Geronimo Saclolo
covers only 170 hectares, 156 of which are mountainous areas and only 14 hectares are planted to rice and corn; that the informacion
possessoria on which the Ruffys rely to prove that they had inherited the land from their parents does not even mention the area subject
thereof; that no effort was ever taken by the Saclolos to reconcile the glaringly disproportionate areas allegedly occupied by them and
their predecessors-in-interest, and the area being applied for, i.e., 325.1 hectares; that Marte Saclolo, son of Geronimo Saclolo and the
alleged administrator of the whole property, could only account for about 150 hectares devoted to rice, bamboo, mangoes, bananas and
other fruit-bearing trees while admitting that the rest of the area applied for are forest, foreshore, and mountain lands; and that the subject
lands form part of the Calumpang Point Naval Reservation, thus cannot be privately titled.

THE COURT'S RULING

The petitions are without merit.

The subject lands may still be declared public lands notwithstanding the Director of Lands' failure to appeal from the RTC decision.
In Laragan v. Court of Appeals,18 petitioners therein averred that the appellate court could not declare the parcel of land in question as
public land, because the decision of the Court of First Instance of Isabela ordering the registration of said parcel of land in their favor, had
already become final and executory for failure of the Director of Lands to appeal therefrom. The Court found such argument untenable,
viz:

xxx While it may be true that the Director of Lands did not appeal from the decision of the trial court, his failure to so appeal did not make
the decision of the trial court final and executory, in view of the appeal interposed by the other oppositors, Teodoro Leaño, Tomas Leaño,
Francisco Leaño, and Consolacion Leaño, who also seek the confirmation of their imperfect title over the land in question.

Neither did such failure of the Director of Lands to appeal foreclose the appellate court from declaring the land in question to be public
land, since the oppositors and the herein petitioners are both seeking the registration of their title pursuant to the provisions of Section
48 (b) of the Public Land Law where the presumption always is that the land pertains to the state, and the occupants and possessors
claim an interest in the same, by virtue of their imperfect title or continuous, open, exclusive and notorious possession and occupation
under a bona fide claim of ownership for the required number of years. Thus, in their application for registration, the petitioners alleged
that they "hereby apply to have the land hereinafter described brought under the operation of the Land Registration Act, and to have the
title thereto registered and confirmed." The petitioners are deemed to thereby admit that, until such confirmation, the land remains
public.19 (emphasis supplied and citations omitted)

In addition, an applicant is not necessarily entitled to have the land registered under the Torrens system simply because no one appears
to oppose his title and to oppose the registration of his land. He must show, even though there is no opposition to the satisfaction of the
court, that he is the absolute owner, in fee simple.20

Consequently, the appellate court may still determine whether the subject lands are indeed alienable and disposable lands of the public
domain, notwithstanding the Director of Lands' failure to appeal from the RTC decision.

The appellate court may reverse the decision of the trial court on the basis of grounds other than those raised as errors on appeal.
As a general rule, only matters assigned as errors in the appeal may be resolved. Section 8, Rule 51 of the Rules of Court provides:

SECTION 8. Questions that May Be Decided. — No error which does not affect the jurisdiction over the subject matter or the validity of
the judgment appealed from or the proceedings therein will be considered unless stated in the assignment of errors, or closely related to
or dependent on an assigned error and properly argued in the brief, save as the court may pass upon plain errors and clerical errors.

The exceptions to this rule have been enumerated in Catholic Bishop of Balanga v. Court of Appeals:21

[T]he appellate court is accorded a broad discretionary power to waive the lack of proper assignment of errors and to consider errors not
assigned. It is clothed with ample authority to review rulings even if they are not assigned as errors in the appeal. Inasmuch as the Court
of Appeals may consider grounds other than those touched upon in the decision of the trial court and uphold the same on the basis of
such other grounds, the Court of Appeals may, with no less authority, reverse the decision of the trial court on the basis of grounds other
than those raised as errors on appeal. We have applied this rule, as a matter of exception, in the following instances:

(1)
Grounds not assigned as errors but affecting jurisdiction over the subject matter;
(2)
Matters not assigned as errors on appeal but are evidently plain or clerical errors within contemplation of law;
(3)
Matters not assigned as errors on appeal but consideration of which is necessary in arriving at a just decision and complete resolution of
the case or to serve the interest of justice or to avoid dispensing piecemeal justice;
(4)
Matters not specifically assigned as errors on appeal but raised in the trial court and are matters of record having some bearing on the
issue submitted which the parties failed to raise or which the lower court ignored;
(5)
Matters not assigned as errors on appeal but closely related to an error assigned; and
(6)
Matters not assigned as errors on appeal but upon which the determination of a question properly assigned, is dependent.22 (citations
omitted)
In this case, there is no doubt that the application for registration of title hinges upon the determination of whether the subject lands are
alienable and disposable. Further, this is consistent with the appellate court's authority to review the totality of the controversy brought on
appeal.23

Applicants failed to prove that the subject lots are alienable and disposable.
The application of the Saclolos was filed on December 27, 1974. Accordingly, the law governing the application was Commonwealth Act
(C.A.) No. 141, as amended by R.A. No. 1942, particularly Section 48 (b) which provides that:

Those who by themselves or through their predecessors in interest have been in open, continuous, exclusive and notorious poss ession
and occupation of agricultural lands of the public domain, under a bona fide claim of acquisition of ownership, for at least thirty years
immediately preceding the filing of the application for confirmation of title except when prevented by war or force majeure. These shall be
conclusively presumed to have performed all the conditions essential to a Government grant and shall be entitled to a certificate of title
under the provisions of this chapter.

As can be gleaned therefrom, the necessary requirements for the grant of an application for land registration are the following:

The applicant must, by himself or through his predecessors-in-interest, have been in possession and occupation of the subject land;

The possession and occupation must be open, continuous, exclusive, and notorious;

The possession and occupation must be under a bona fide claim of ownership for at least thirty years immediately preceding the filing of
the application; and

The subject land must be an agricultural land of the public domain.24


Among these requirements, the question of whether the subject lands were declared alienable and disposable is of primordial importance
because it is determinative if the land can in fact be subject to acquisitive prescription and, thus, registrable under the Torrens system.
Without first determining the nature and character of the land, all the other requirements such as the length and nature of possession and
occupation over such land do not come into play. The required length of possession does not operate when the land is part of the public
domain.25

In Republic v. Heirs of Fabio,26 the Court similarly tackled the issue of whether certain parcels of land located within the Calumpang
Point Naval Reservation are alienable and disposable, to wit:

The three proclamations cited reserving the Calumpang Point Naval Reservation for the exclusive use of the military are the following:
(1) U.S. War Department Order No. 56 issued on 25 March 1904, (2) Proclamation No. 307 issued on 20 November 1967, and (3)
Proclamation No. 1582-A issued on 6 September 1976. Such proclamations state:

U.S. War Department General Order No. 56

U.S. War Department General Order No. 56


Washington, March 25, 1904.

For the knowledge and governance of all interested parties, the following is hereby announced:

The President of the United States, by the Order dated March 14, 1904, which provides that the reservations made by Executive Order
of April 11, 1902 (General Order No. 38, Army Headquarters, Office of the Adjutant General, April 17, 1902), at the entrance of Manila
Bay, Luzon, Philippine Islands, are arranged in such a way that will include only these lands as later described, whose lands were reserved
by the Order of March 14, 1904 for military purposes, by virtue of Article 12 of the Act of Congress approved on July 1, 1902, entitled "Act
providing for the Temporary Administration of Civil Affairs of the Government of the Philippine Islands and for Other Purposes" (32 Stat.
L., 691); namely:

1. In the northern side of the entrance to Manila Bay, in the province of Bataan, Luzon (Mariveles Reservation), all public lands within the
limits that are described as follows:

"Starting from the mouth of the Mariveles River in the eastern border and from here straight North to a distance of 5,280 feet; from this
point straight to the East to intercept a line, in a straight direction to the South from a stone monument marked U.S. (Station 4); from there
straight from the North until the aforementioned Station 4; from here straight to the East to a distance of 6,600 feet until a stone monument
marked U.S. (Station 5); from here straight South to a distance of 6,600 feet until a stone monument marked U.S. (Station 6); from here
straight to the East to a distance of 8,910 feet until a stone monument marked U.S. (Station 7); from here straight to the South to a
distance of 7,730 feet until a stone monument marked U.S. (Station 8), situated at the northwest corner of the second creek to the east
of Lasisi Point, 30 feet North of the high-tide mark; from there in the same direction until the high-tide mark; from here towards the East
following the shoreline up to the starting point."

2. In the southern side of the Manila Bay entrance, in the province of Cavite, Luzon (Calumpang Point Reservation), all public lands within
the limits that are described as follows:

"Starting from a stone monument marked U.S. (Station 1) situated in the cliff on the Eastern side of Asubig Point, 20 feet above the high-
tide mark and about 50 feet from the edge of the cliff and continuing from there to the South 28° 10' West, a distance of up to 22,000 feet
until a stone monument marked U.S. (Station 2); from here to North 54° 10' West at a distance of 5,146 feet until a stone monument
marked U.S. (Station 3); from here towards South 85° 35' 30 "West, at a distance of 2,455 feet until a stone monument marked U.S.
(Station 4), situated on the beach near the Northeast corner of Limbones Bay, about 50 feet from the high-tide mark and following in the
same direction until the high-tide mark; from here towards North and East following the shoreline until North 28° 10 ' East from the starting
point and from there encompassing more or less 5,200 acres. The markers are exact."

3. The islands of Corregidor, Pulo Caballo, La Monja, El Fraile, and Carabao, and all other islands and detached rocks lying between
Mariveles Reservation on the north side of the entrance to Manila Bay and Calumpang Point Reservation on the south side of said
entrance.

4. The jurisdiction of the military authorities in the case of reservations in the northern and southern beaches of the entrance to Manila
Bay and all the islands referred to in paragraph 3, are extended from the high-tide marker towards the sea until a distance of 1,000 yards.

By Order of the Secretary of War:


GEORGE L. GILLESPIE,
General Commander, Chief of Internal General Staff,
Official copy.
W.P. HALL, Internal Adjutant General. (Emphasis supplied)

Proclamation No. 307

... do hereby withdraw from sale or settlement and reserve for military purposes under the administration of the Chief of Staff, Armed
Forces of the Philippines, subject to private rights, if any there be, a certain parcel of land of the public domain situated in the municipality
of Ternate, province of Cavite, Island of Luzon, more particularly described as follows:

Proposed Naval Reservation

Calumpang Point

A parcel of land (the proposed Calumpang Point Naval Reservation), situated in the municipality of Ternate, province of Cavite. Bounded
on the NW., N. and E., by Manila Bay; on the SE. and S., by municipality of Ternate; and on the W., by Manila Bay. Beginning at a point
marked "1" on the attached Sketch Plan traced from Coastal Hydrography of Limbones Island.

thence N. 54 deg. 30' E., 750.00 m. to point 2;


thence N. 89 deg. 15' E., 1780.00 m. to point 3;
thence N. 15 deg. 10' E., 6860.00 m. to point 4;
thence N. 12 deg. 40' W., 930.00 m. to point 5;
thence S. 77 deg. 20' W., 2336.00 m. to point 6;
thence S. 49 deg. 30' W., 4450.00 m. to point 7;
thence S. 12 deg. 40' E., 2875.00 m. to point 8;
thence S. 30 deg. 30' E., 2075.00 m. to the point of beginning; containing an approximate area of twenty eight million nine hundred
seventy three thousand one hundred twelve (28,973,112) square meters. CHIEDS
NOTE: All data are approximate and subject to change based on future surveys."

Proclamation No. 1582-A

WHEREAS, Proclamation No. 307 dated November 20, 1967 and U.S. War Department Order No. 56 dated March 25, 1904 reserved
for military purposes, and withdrew from sale or settlement, a parcel of land of the public domain situated in the Municipality of Ternate,
Province of Cavite, more particularly described as follows: . . .

WHEREAS, the Philippine Navy and the Philippine Marines now need that portion of this area reserved under Proclamation No. 307,
particularly, Caylabne Cove, Caynipa Cove, Calumpang Cove and Sinalam Cove, for their use as official station, not only to guard and
protect the mouth of Manila Bay and the shorelines of the Province[s] of Cavite, Batangas and Bataan, but also to maintain peace and
order in the Corregidor area, which is now one of the leading tourist attractions in the country; . . .

. . . containing an approximate area of EIGHT MILLION EIGHTY NINE THOUSAND NINE HUNDRED NINETY (8,089,990) SQUARE
METERS, more or less.
The portion that remains after the segregation which are occupied shall be released to bona fide occupants pursuant to existing
laws/policies regarding the disposition of lands of the public domain and the unoccupied portions shall be considered as alienable or
disposable lands.

The proclamations established that as early as 1904 a certain parcel of land was placed under the exclusive use of the government for
military purposes by the then colonial American government. In 1904, the U.S. War Department segregated the area, including the Lot,
for military purposes through General Order No. 56. Subsequently, after the Philippines regained its independence in 1946, the American
government transferred all control and sovereignty to the Philippine government, including all the lands appropriated for a public purpose.
Twenty years later, two other presidential proclamations followed, both issued by former President Ferdinand E. Marcos, restating that
the same property is a naval reservation for the use of the Republic.27 (emphases in the original)

From the foregoing proclamations, four (4) things are clear: first, a parcel of land containing 28,973,112 square meters, located in Ternate,
Cavite, was withdrawn from sale or settlement and reserved for military purposes; second, by virtue of Proclamation No. 1582-A, the area
reserved for military purposes was limited to 8,089,990 square meters instead of the original 28,973,112 square meters; third, the
occupied portions, after segregating the 8,089,990 square meters, would be released to bona fide occupants; and fourth, the unoccupied
portions were declared alienable and disposable lands.

To reiterate, the Director of Lands insists that the subject lands are within the Calumpang Point Naval Reservation. This was bolstered
by the testimony of Eleutorio R. Paz, Chief of the Survey Division of the Bureau of Lands-Region 4.28 Thus, it was incumbent upon the
Saclolos and Enriquez to prove that the subject lands do not form part of the Calumpang Point Naval Reservation because "when a
property is officially declared a military reservation, it becomes inalienable and outside the commerce of man."29

Indeed, Proclamation No. 307 recognizes private rights over parcels of land included in the reservation. Further, Proclamation No. 1582-
A provides that the occupied portions which remained after segregating the 8,089,990 square meters shall be released to bona fide
occupants. Thus, a mere invocation of "private rights" does not automatically entitle an applicant to have the property registered in his
name. "Persons claiming the protection of private rights in order to exclude their lands from military reservations must show by clear and
convincing evidence that the pieces of property in question have been acquired by a legal method of acquiring public lands."30

In this case, however, none of the documents presented by the Saclolos and Enriquez prove that the subject lands are alienable and
disposable.

First, the Investigator's Report even contradicted the claim that the subject lands are alienable and disposable as it noted that these lands
are "within the extensive Calumpang Point Reservation however, the applicants assert their private rights to the subject area."31

Further, the informacion possessoria upon which the Saclolos heavily rely to support their claim neither states that the subject lands were
declared alienable and disposable nor indicates the area covered thereby. It merely describes it as "capacity of three cavans seed in
palay." What can only be determined from such certificate of possession is that a certain Bernabe Fabio had possessory title over a parcel
of land registered in 1895 but was subsequently lost and that the children of Fabio eventually sold such parcel of land to th e Spouses
Ruffy.32 This, however, does not prove that the subject lands were already legally acquired by the Saclolos and their predecessors-in-
interest at a time when such parcels of land were declared alienable and disposable by the government. Moreover, it is worthy to note
that P.D. No. 892 discontinued the system of registration under the Spanish Mortgage Law by categorically declaring all lands recorded
under the latter system, not yet covered by Torrens title, unregistered lands. P.D. No. 892 divests the Spanish titles of any legal force and
effect in establishing ownership over real property.33

Finally, in the Deed of Sale between the heirs of the Spouses Ruffy and Geronimo Saclolo, the parcel of land was described as containing
170 hectares (1,700,000 square meters).34 However, in the Saclolos' application for registration of title, the total area of the subject lands
is stated as 375.2 hectares. Further, Marte Saclolo, son of Geronimo, could only account for 150 hectares devoted to rice, bamboo,
mangoes, bananas and other fruit-bearing trees.35 Thus, the alienability and disposability of the subject lands and even the exact area
covered thereof lack factual bases.

In Heirs of Mario Malabanan v. Republic of the Philippines,36 the Court emphasized that lands of the public domain, unless declared
otherwise by virtue of a statute or law, are inalienable and can never be acquired by prescription. No amount of time of possession or
occupation can ripen into ownership over lands of the public domain. All lands of the public domain presumably belong to the State and
are inalienable. Lands that are not clearly under private ownership are also presumed to belong to the State and, therefore, may not be
alienated or disposed.

A positive act declaring land as alienable and disposable is required. In keeping with the presumption of State ownership, the Court has
time and again emphasized that there must be a positive act of the government, such as an official proclamation,37 declassifying
inalienable public land into disposable land for agricultural or other purposes.38 In fact, Section 8 of CA No. 141 limits alienable or
disposable lands only to those lands which have been officially delimited and classified.39

The burden of proof in overcoming the presumption of State ownership of the lands of the public domain is on the person applying for
registration (or claiming ownership), who must prove that the land subject of the application is alienable or disposable.40 To overcome
this presumption, incontrovertible evidence must be established that the land subject of the application (or claim) is alienable or
disposable.41 There must still be a positive act declaring land of the public domain as alienable and disposable. To prove that the land
subject of an application for registration is alienable, the applicant must establish the existence of a positive act of the government such
as a presidential proclamation or an executive order; an administrative action; investigation reports of Bureau of Lands investigators; and
a legislative act or a statute.42 The applicant may also secure a certification from the government that the land claimed to have been
possessed for the required number of years is alienable and disposable.43
In the case at bar, no such proclamation, executive order, administrative action, report, statute, or certification was presented to the Court.
The records are bereft of evidence showing that the subject lands were proclaimed by the government to be alienable and disposable.
Time and again, it has been held that matters of land classification or reclassification cannot be assumed. They call for proof.44

On a final note, it is worth emphasizing that as early as 1904, a certain parcel of land has already been reserved for military purposes. It
behooves the Court how the Saclolos remained oblivious to such fact despite a considerable lapse of time. Certainly, there would have
been several people who knew of such reservation considering that the same is not confidential information. The Saclolos and even
Enriquez failed to exercise such diligence as prudent men ordinarily would. As such, they only have themselves to blame for their
predicament. They should have taken full advantage of the opportunity to present during trial all pieces of evidence to prove that the
subject lands are alienable and disposable especially in the light of the fact that the government vehemently opposes the registration.
Thus, in view of the glaring lack of evidence as regards the alienability and disposability of the subject lands, the Court is constrained to
deny their registration of title.

WHEREFORE, the 26 May 2004 Decision and 13 May 2005 Resolution of the Court of Appeals in CA-G.R. CV No. 53838 are AFFIRMED
in toto.

NORTH GREENHILLS VS. MORALES

Factual Antecedents

Atty. Morales is a resident of North Greenhills Subdivision in San Juan City. His house is located alongside Club Filipino Avenue and
adjacent to McKinley Park, an open space/playground area owned and operated by NGA. He also has a personal access door, which he
built through a wall separating his house from the park. This access door, when unlocked, opens directly into the park.

On the other hand, NGA, an association composed of members of the subdivision, organized to promote and advance the best interests,
general welfare, prosperity, and safeguard the well-being of the owners, lessees and occupants of North Greenhills, is the undisputed
owner of the park. It has acquired ownership thereof through a donation made by the original owner, Ortigas &. Co. Ltd.

In June 2003, NGA started constructing a pavilion or kiosk occupying the side of the park adjacent to the residence of Atty. Morales. Part
of the design was a public restroom intended to serve the needs of park guests and members of NGA. Said restroom was constructed
alongside the concrete wall separating the house of Atty. Morales from the park.

Objecting to the construction of the restroom, Atty. Morales filed on July 23, 2003 a complaint before the HLURB, docketed as HLURB
Case No. NCRHOA-072303-309. On August 13, 2013, he amended his complaint and additionally sought the demolition of the pavilion
which was then being built.

In his Amended Complaint, Atty. Morales alleged that for a period spanning 33 years, he had an open, continuous, immediate, and
unhampered access to the subdivision park through his side door, which also served as an exit door in case of any eventuality; that
having such access to the park was one of the considerations why he purchased the lot; that the construction of the pavilion was illegal
because it violated his right to immediate access to the park, Presidential Decree No. 957 and the Deed of Donation of Ortigas & Co.
Ltd., which required the park to be maintained as an open area; and that the restroom constructed by NGA was a nuisance per se.

NGA, in its Answer with Compulsory Counterclaim, rejected the assertions of Atty. Morales. It contended that as the absolute owner of
the park, it had the absolute right to fence the property and impose reasonable conditions for the use thereof by both its members and
third parties; that the construction of the restroom was for the use and benefit of all NGA members, including Atty. Morales; and that Atty.
Morales' use of a side entrance to the park for 33 years could not have ripened into any right because easement of right of way could not
be acquired by prescription. NGA likewise sought the payment of P878,778.40 corresponding to the annual membership dues which Atty.
Morales had not been paying since 1980.

On April 13, 2003, the HLURB Arbiter conducted an ocular inspection of the park and noted that the construction started by NGA blocked
Atty. Morales' side access to the park.

On February 16, 2005, the HLURB Arbiter rendered a Decision,6 the decretal portion of which reads:

WHEREFORE, PREMISES CONSIDERED, judgment is hereby rendered ordering respondents of the removal of the pavilion and the
relocation of the common toilet in a place where it will not be a nuisance to any resident. Respondents are further directed to remove the
obstruction to the side door of the complainant. All other claims and counterclaims are hereby dismissed for lack of merit.

IT IS SO ORDERED.7

NGA appealed to the HLURB Board of Commissioners (HLURB Board). In its November 22, 2007 Decision,8 the HLURB Board modified
the ruling of the HLURB Arbiter, thus:

Further, the complaint against respondent Alviar should be dropped as no acts have been particularly attributed to him in his personal
capacity.
WHEREFORE, premises considered, the decision of the Regional Office is hereby MODIFIED. Accordingly, respondent NGA is ordered
to relocate the restroom constructed or being constructed in the McKinley Park away from the walls of any resident and where it will not
block complainant's side door access to the park.

SO ORDERED.9

NGA appealed to the Office of the President (OP).

On February 17, 2010, the OP rendered its decision, affirming in toto the ruling of the HLURB Board.

NGA moved for reconsideration, but its motion was denied by the OP in its August 8, 2013 Resolution.

Aggrieved, NGA filed a petition for review under Rule 43 of the Rules of Court before the CA, arguing that the OP erred in its findings.

Ruling of the CA

In its March 13, 2015 Decision,10 the CA affirmed the ruling of the OP. It found no error on the part of the OP in affirming the
characterization of the restrooms built as nuisance per accidens considering that the structure posed sanitary issues which could
adversely affect not only Atty. Morales, but also his entire household; that even if there existed a perimeter wall between the park and
Atty. Morales' home, the odor emanating from the restroom could easily find its way to the dining area, and the foul and noxious smell
would make it very difficult and annoying for the residents of the house to eat; and that the proximity of the restroom to Atty. Morales'
house placed the people residing therein at a greater risk of contracting diseases both from improperly disposed waste and human
excrements, as well as from flies, mosquitoes and other insects, should NGA fail to maintain the cleanliness of the structures.

The CA stated that NGA's fear of being exposed to outsiders and criminals because Atty. Morales' access was unfounded. It pointed out
that the door had been in existence for more than three decades and that if dangers truly existed, NGA should have taken immediate
action and blocked the side access years earlier. It then pointed out other ways to remedy the security concerns of NGA, such as placing
a wall strategically placed at the border of the park or additional guards to patrol the vicinity.

As to the counterclaim of NGA for association dues, the CA held that the claim was in the nature of a permissive counterclaim, which was
correctly dismissed by the OP.

NGA moved for reconsideration, but its motion was denied by the CA in its February 3, 2016 Resolution.

Hence, this petition.

GROUNDS:

I.

THE COURT OF APPEALS SERIOUSLY ERRED IN COMPLETELY DISREGARDING THE HLURB'S LACK OF JURISDICTION OVER
THE INSTANT CASE.

(1)

RESPONDENT MORALES FAILED TO ALLEGE IN HIS COMPLAINT (OR AMENDED COMPLAINT) THAT HE IS A MEMBER OF NGA
- A FATAL JURISDICTIONAL DEFECT FOR FAILURE TO PROPERLY LAY THE PREDICATE THAT WOULD HAVE ENABLED THE
HLURB TO ACQUIRE JURISDICTION OVER THE INSTANT ACTION.

(2)

IN THE CASE OF STA. CLARA HOMEOWNERS' ASSOCIATION V. GASTON (G.R. NO. 141961, JANUARY 23, 2002), THE
HONORABLE COURT RULED THAT WHERE THE BODY OF THE COMPLAINT FILED IN THE NOW HLURB FAILS TO MENTION
THAT THE COMPLAINANT IS A MEMBER OF THE ASSOCIATION HE IS SUING, SUCH COMPLAINT MUST BE DISMISSED FOR
LACK OF JURISDICTION.

(3)

PETITIONER NGA'S CLAIM FOR UNPAID ASSOCIATION DUES DOES NOT PRECLUDE IT FROM ASSAILING RESPONDENT'S
MEMBERSHIP IN THE NGA.

(4)

IN THE CASE OF GREGORIO C. JAVELOSA V. COURT OF APPEALS (G.R. NO. 124292, DECEMBER 10, 1996), THE HONORABLE
COURT RULED THAT "IT IS SETTLED THAT THE JURISDICTION OF COURTS OVER THE SUBJECT MATTER OF LITIGATION IS
DETERMINED BY THE ALLEGATIONS IN THE COMPLAINT. IT IS EQUALLY SETTLED THAT AN ERROR OF JURISDICTION CAN
BE RAISED AT ANY TIME AND EVEN FOR THE FIRST TIME ON APPEAL."

II.
THE COURT OF APPEALS SERIOUSLY ERRED AND IS MANIFESTLY MISTAKEN IN RULING THAT THE TOILET BUILT BY NGA
AT THE MCKINLEY PARK IS A NUISANCE PER ACCIDENS, ON THE BASIS OF MERE SPECULATION, SUPPOSITION AND PURE
CONJECTURE, CONSIDERING THE TOTAL LACK OF EVIDENCE ON RECORD TO PROVE SO.

(1)

RESPONDENT ATTY. MORALES DID NOT SET OUT TO PROVE THAT THE TOILET ADJACENT HIS HOUSE INJURED HIM OR
THAT FOUL ODOR EMANATED FROM IT BECAUSE HE MISTAKENLY ALLEGED THAT THE TOIILET WAS A NUISANCE PER SE.

(2)

BY FAILING TO ADDUCE EVIDENCE THAT THE TOILET, IN ANY WAY, ANNOYED RESPONDENT'S SENSES, OR THAT FOUL
ODOR EMANATED FROM IT, OR THAT IT POSED SANITARY ISSUES DETRIMENTAL TO HIS FAMILY'S HEALTH - THE SUBJECT
TOILET CANNOT BE LEGALLY CONSIDERED NUISANCE PER ACCIDENS.

(3)

INDEED, A CURSORY VIEW OF THE PERTINENT DISCUSSION IN THE ASSAILED DECISION REVEALS THAT THE COURT OF
APPEALS SADLY TOOK THE PATH OF SPECULATION, SUPPOSITION AND PURE CONJECTURE IN JUSTIFYING ITS DECISION.

III.

THE ASSAILED 13 MARCH 2015 DECISION IS PATENTLY ERRONEOUS AS IT IS BASED ON GRAVE MISAPPREHENSION OF
FACTS AND OF THE EVIDENCE - OR THE TOTAL LACK OF IT - ON RECORD.

(1)

INDEED, A PERUSAL OF THE RECORDS WOULD REVEAL THAT THERE WAS NO EVIDENCE WHATSOEVER ADDUCED BY THE
RESPONDENT DEMONSTRATING THAT THE SUBJECT TOILET HAS CAUSED PHYSICAL ANNOYANCE OR DISCOMFORT TO
HIM. NO TESTIMONY HAS EVER BEEN BROUGHT TO THE HLURB OR THE OFFICE OF THE PRESIDENT SHOWING THAT THE
TOILET EMITTED ANY FOUL SMELL, OR ODOR, OR AT THE VERY LEAST, ANNOYED RESPONDENT MORALES EVERY TIME HE
WOULD EAT IN HIS DINING AREA.

(2)

AS A MATTER OF FACT, IT IS WORTH TO NOTE THAT THE RESPONDENT DID NOT EVEN SUBMIT A POSITION PAPER BEFORE
THE HLURB TO ATTEST TO AND PROVE SUCH FACTUAL MATTERS.

(3)

IN THE VERY CASE CITED BY THE COURT OF APPEALS, SMART COMMUNICATIONS V. ALDECOA (G.R. NO. 166330,
SEPTEMBER 11, 2013), THE HONORABLE COURT STRUCK DOWN THE RULING OF THE LOWER COURT AND PRONOUNCED
THAT A DECISION THAT DECLARES A THING TO BE A NUISANCE PER ACCIDENS MUST BE SUPPORTED BY FACTUAL
EVIDENCE AND NOT BY MERE CONJECTURES OR SUPPOSITIONS.

IV.

THE COURT OF APPEALS SERIOUSLY ERRED IN UPHOLDING RESPONDENT ATTY. MORALES' UNBRIDLED ACCESS TO
MCKINLEY PARK, EFFECTIVELY CONSTITUTING AN EASEMENT OF RIGHT OF WAY WITHOUT ANY BASIS - AS AGAINST THE
CLEAR STATUTORY RIGHT OF PETITIONER NGA, AS THE OWNER OF MCKINLEY PARK TO FENCE AND PROTECT ITS
PROPERTY, GRANTED UNDER ARTICLES 429 AND 430 OF THE CIVIL CODE.

(1)

CONTRARY TO THE ASSAILED DECISION, IT IS NOT INCUMBENT UPON PETITIONER NGA TO PROVE THE LEGALITY OF ITS
ACT OF CONSTRUCTING THE SUBJECT TOILET ON ITS OWN PROPERTY. INDEED, THIS IS A BASIS STATUTORY RIGHT OF
NGA AS AN "OWNER".

(2)

RESPONDENT, ON THE OTHER HAND, BEING THE PROPONENT OF THE ACTION TO DECLARE THE TOILET A NUISANCE, IS
THE ONE SADDLED BY LAW WITH THE RESPONSIBILITY OF PROVING THAT THE STRUCTURE BUILT BY NGA IS A NUISANCE.
AS DISCUSSED, HOWEVER, RESPONDENT UTTERLY FAILED TO DISCHARGE SUCH BURDEN.

(3)
ARTICLE 430 OF THE CIVIL CODE GRANTS PETITIONER NGA OF ITS STATUTORY RIGHT TO FENCE OFF HIS PROPERTY. ART.
430 STATES THAT "EVERY OWNER MAY ENCLOSE OR FENCE HIS LAND OR TENEMENTS BY MEANS OF WALLS, DITCHES,
LIVE OR DEAD HEDGES, OR BY ANY OTHER MEANS WITHOUT DETRIMENT TO SERVITUDES CONSTITUTED THEREON."

(4)

MOREOVER, ARTICLE 429 OF THE CIVIL CODE LIKEWISE GRANTS PETITIONER NGA THE RIGHT TO EXCLUDE OTHERS FROM
ACCESS TO AND ENJOYMENT OF ITS PROPERTY.

V.

THE COURT OF APPEALS SERIOUSLY ERRED IN RULING THAT PETITIONER NGA'S COUNTERCLAIM TO COLLECT ON
RESPONDENT'S UNPAID ASSOCIATION DUES FOR THE PAST THIRTY-THREE (33) YEARS, IS NOT COMPULSORY BUT MERELY
PERMISSIVE.

(1)

AS A PERSON SUING NGA FOR THE EXERCISE OF HIS RIGHTS AS AN ALLEGED MEMBER THEREOF, NGA'S DEFENSE WILL,
AS A MATTER OF COURSE, INVOLVE THE CONTEST OF SUCH RIGHT. IN ORDER FOR NGA TO CONTEST RESPONDENT'S
RIGHT TO USE THE PARK AS A MEMBER OF NGA, THE LATTER HAS NO OTHER ALTERNATIVE BUT TO RAISE HIS NON-
PAYMENT OF MEMBERSHIP DUES IN ORDER TO ATTACK HIS RIGHT TO USE THE PARK, WHICH RIGHT INEXTRICABLY ARISES
OUT OF HIS STANDING AS AN ALLEGED MEMBER OF NGA.

(2)

AS A MATTER OF FACT, REPUBLIC ACT NO. 9904, OTHERWISE KNOWN AS THE "MAGNA CARTA FOR HOMEOWNERS AND
HOMEOWNERS' ASSOCIATIONS" MAKES IT A CONDITION SINE QUA NON THAT THE HOMEOWNER MUST PAY THE
ASSOCIATION FEES AND CHARGES BEFORE HE CAN ENJOY ITS FACILITIES.11

In its Resolution,12 dated May 30, 2016, the Court required respondent to file his Comment on the petition. To date, no Comment has
been filed. For said reason, the Court deemed, as it hereby deems, that respondent had waived his right to file one.

ISSUES

1.
WHETHER THE CA CORRECTLY RULED THAT THE HLURB HAD JURISDICTION OVER THE COMPLAINT FILED BY ATTY.
MORALES;
2.
WHETHER THE CA CORRECTLY RULED THAT THE RESTROOM BUILT BY NGA INSIDE THE MCKINLEY PARK IS A NUISANCE
PER ACCIDENS;
3.
WHETHER NGA HAS THE RIGHT TO BLOCK ATTY. MORALES' ACCESS TO THE PARK; AND
4.
WHETHER THE CA CORRECTLY RULED THAT THE COUNTERCLAIM OF NGA AGAINST ATTY. MORALES FOR UNPAID
ASSOCIATION DUES WAS A PERMISSIVE COUNTERCLAIM.
The Ruling of the Court

The Court partly grants the petition.

On Jurisdiction

Basic is the rule that jurisdiction over the subject matter of a case is conferred by law and determined by the allegations in the complaint
which comprise a concise statement of the ultimate facts constituting the plaintiff's cause of action. The nature of an action, as well as
which court or body has jurisdiction over it, is determined from the allegations contained in the complaint, irrespective of whether or not
the plaintiff is entitled to recover upon all or some of the claims asserted therein. Once vested by the allegations in the complaint,
jurisdiction remains vested irrespective of whether or not the plaintiff is entitled to recover upon all or some of the claims asserted
therein.13

Relative thereto is the rule that lack of jurisdiction over the subject matter may be raised at any stage of the proceedings.14 Jurisdiction
over the subject matter is conferred only by the Constitution or the law.15 It cannot be acquired through a waiver or enlarged by the
omission of the parties or conferred by the acquiescence of the court. Consequently, questions of jurisdiction may be cognizable even if
raised for the first time on appeal.16

NGA claims that the HLURB never had jurisdiction over the complaint filed by Atty. Morales considering that there was no allegation that
he was member of the association, entitling him to claim the use of the latter's facilities including the right of access to McKinley Park.
Citing Sta. Clara Homeowner's Association v. Gaston,17 NGA asserts that for HLURB to acquire jurisdiction over disputes among
members of an association, it is a requirement that the allegation of membership must be clear in the complaint, otherwise, no authority
to hear and decide the case is vested in the concerned agency. Membership in a homeowners' association is voluntary and cannot be
unilaterally forced by a provision in the association's articles of incorporation or by-laws, which the alleged member did not agree to be
bound to.18

In this case, it appears that Atty. Morales, by filing his complaint as a member whose rights have been allegedly violated, has satisfied
such requirement. His status as a member has not been questioned. It is worthy to note that NGA, in its counterclaim, demanded the
payment of association dues from Atty. Morales as he has been refusing to pay his dues for more than three decades. In sum, there is
no dispute that Atty. Morales is a member of NGA, albeit a delinquent member. In Tumpag v. Tumpag,19 the Court said:

Generally, the court should only look into the facts alleged in the complaint to determine whether a suit is within its jurisdiction. There may
be instances, however, when a rigid application of this rule may result in defeating substantial justice or in prejudice to a party's substantial
right. In Marcopper Mining Corp. v. Garcia, we allowed the RTC to consider, in addition to the complaint, other pleadings submitted by
the parties in deciding whether or not the complaint should be dismissed for lack of cause of action. In Guaranteed Homes, Inc. v. Heirs
of Valdez, et al., we held that the factual allegations in a complaint should be considered in tandem with the statements and inscriptions
on the documents attached to it as annexes or integral parts.20 [Citations omitted]

Considering that the requirement of membership is present, jurisdiction over the subject matter of the case was properly vested in the
HLURB.

On the finding that the restroom


was a nuisance per accidens

The CA in disposing the case, ruled that the restroom posed sanitary issues to Atty. Morales and is, therefore, a nuisance per accidens.
Such is a finding of fact, which is generally conclusive upon the Court, because it is not its function to analyze and weigh the evidence all
over again.

There are, however, well-recognized exceptions. These are (1) when the findings are grounded entirely on speculations, surmises or
conjectures; (2) when the inference made is manifestly mistaken, absurd or impossible; (3) when there is grave abuse of discretion; (4)
when the judgment is based on a misapprehension of facts; (5) when the findings of fact are conflicting; (6) when in making its findings
the Court of Appeals went beyond the issues of the case, or its findings are contrary to the admissions of both the appellant and the
appellee; (7) when the findings are contrary to that of the trial court; (8) when the findings are conclusions without citation of specific
evidence on which they are based; (9) when the facts set forth in the petition as well as in the petitioners main and reply briefs are not
disputed by the respondent; (10) when the findings of fact are premised on the supposed absence of evidence and contradicted by the
evidence on record; or (11) when the Court of Appeals manifestly overlooked certain relevant facts not disputed by the parties, which, if
properly considered, would justify a different conclusion.21

NGA avers that the case falls under the said exceptions considering that no proof was ever presented to prove that the restroom was a
nuisance per accidens. Absent such evidence, the CA's finding was only speculative, resulting in a grave misapprehension of facts.

The Court agrees.

A nuisance per accidens is one which depends upon certain conditions and circumstances, and its existence being a question of fact, it
cannot be abated without due hearing thereon in a tribunal authorized to decide whether such a thing does in law constitute a nuisance.22
Obviously, it requires a determination of such circumstances as to warrant the abatement of the nuisance. That can only be done with
reasonable notice to the person alleged to be maintaining or doing the same of the time and place of hearing before a tribunal authorized
to decide whether such a thing or act does in law constitute a nuisance per accidens.23

In other words, it requires a proper appreciation of evidence before a court or tribunal rules that the property being maintained is a
nuisance per accidens.

A reading of the CA's decision would easily reveal that its conclusions were merely speculative. It wrote:

The said toilet, to Our mind, poses sanitary issues which could adversely affect not only the Respondent but his entire household as well.
Even if there exists a perimeter wall between Respondent's house and the toilet, the odor emanating from the latter could easily find its
way to the dining area, and the foul and noxious smell would make it very difficult and annoying for the residents of the house to eat.
Moreover, the proximity of the toilet to Respondent's house places the people residing therein at greater risk of contracting diseases both
from improperly disposed waste and human excrements, as well as from flies, mosquitoes, and other insects, should petitioner NGA fail
to maintain the cleanliness in the said structure. Verily, the determining factor when the toilet is the cause of the complaint is not how
much it smells or stinks but where it is located as to produce actual physical discomfort and annoyance to a person of ordinary
sensibilities.24

By the use of the words "would, should, could," it can be discerned that the CA was not even sure that the restroom has caused such
annoyance to Atty. Morales or his family. Its declaration that the restroom is a nuisance per accidens had no basis in evidence. There is
nothing in the records which discloses that Atty. Morales had introduced any evidence, testimonial or documentary, to prove that the
restroom annoyed his senses, that foul odor emanated from it, or that it posed sanitary issues detrimental to his family's health. No
certification by the City Health Officer was even submitted to the HLURB to attest on such matters.

It was improper on the part of the CA to assume those negative effects because modern day restrooms, even those for the use of the
public, are clean, safe and emitting no odor as these are regularly maintained. For said reason, it was an error on the part of the CA to
rule that the restroom was a nuisance per accidens and to sustain the order that it should be relocated.
Clearly, its finding was based on speculations, and not evidence.

On the finding that Atty.


Morales had no access to
to McKinley Park

NGA claims that the CA erred in upholding Atty. Morales' unbridled access to the park, which effectively constituted an easement of right
of way without any basis as against the clear statutory right of NGA, as the owner of the park, to fence and protect its property on the
basis of Articles 429 and 430 of the Civil Code.

The Court agrees with NGA.

Under the Civil Code, NGA, as owner of the park, has the right to enclose or fence his land or tenements by means of walls, ditches, live
or dead hedges, or by any other means without detriment to servitudes constituted thereon. It also has a right to exclude others from
access to, and enjoyment of its property.

NGA's legal right to block the access door is beyond doubt. Courts have no business in securing the access of a person to another
property absent any clear right on the part of the latter.

The CA essentially violated the right of NGA. Atty. Morales never introduced any evidence that he had acquired any right by prescription
or by agreement or legal easement to access the park through his side door. Moreover, he never claimed that his side door was his only
access to the park. He has other means and, being adjacent to the park, going through other means is not cumbersome.

The conditions25 set forth under the Deed of Donation by Ortigas & Co. Ltd. to NGA could not be used by Atty. Morales in his favor.
Assuming that he has a right as a member to use the park, it does not mean that he can assert that his access to the park could only be
done through his side door. Atty. Morales knows very well that he can access the park through some other parts of the park.
Counterclaim for unpaid dues was a
permissive one and, therefore, the
affirmation of its dismissal was proper

A compulsory counterclaim is any claim for money or any relief, which a defending party may have against an opposing party, which at
the time of suit arises out of, or is necessarily connected with, the same transaction or occurrence that is the subject matter of the plaintiffs
complaint. It is compulsory in the sense that it is within the jurisdiction of the court, does not require for its adjudication the presence of
third parties over whom the court cannot acquire jurisdiction, and will be barred in the future if not set up in the answer to the complaint
in the same case. Any other counterclaim is permissive.26

The Court has held that the compelling test of compulsoriness characterizes a counterclaim as compulsory if there should exist a logical
relationship between the main claim and the counterclaim. The Court further ruled that there exists such a relationship when conducting
separate trials of the respective claims of the parties would entail substantial duplication of time and effort by the parties and the court;
when the multiple claims involve the same factual and legal issues; or when the claims are offshoots of the same basic controversy
between the parties.27

The criteria to determine whether the counterclaim is compulsory or permissive are as follows:

(a)
Are issues of fact and law raised by the claim and by the counterclaim largely the same?
(b)
Would res judicata bar a subsequent suit on defendants claim absent the compulsory rule?
(c)
Will substantially the same evidence support or refute plaintiffs claim as well as defendant's counterclaim?
(d)
Is there any logical relations between the claim and the counterclaim?
A positive answer to all four questions would indicate that the counterclaim is compulsory.28 Otherwise, the same is permissive.

Here, the main issues in the complaint are limited only to the propriety of barring Atty. Morales from accessing the park through the side
door and whether the restroom constructed by NGA is a nuisance per se. On the other hand, the counterclaim is simply concerned with
collecting from Atty. Morales his unpaid association dues for the past thirty (30) years. Suffice it to state that payment or non-payment of
association dues are distinct matters that do not relate to whether the main cause of Atty. Morales against NGA was proper. Whether
there was payment or otherwise is irrelevant to the main issues considering that the pleadings filed by the parties essentially reflected an
admission of membership of Atty. Morales in the association. The failure to raise the issue of unpaid association dues in this case or its
dismissal if properly raised will not be a bar to the filing of the appropriate separate action to collect it.

WHEREFORE, the petition is PARTLY GRANTED.

REPUBLIC VS. GO

Public land remains inalienable unless it is shown to have been reclassified and alienated to a private person.[1]
This resolves a Petition for Review assailing the Court of Appeals Decision dated January 21, 2011 and Resolution dated June 6, 2011
in CA-G.R. CV No. 93000, which affirmed the Decision of the Municipal Trial Court in Cities dated December 12, 2008 issuing the Decree
of Registration for Lot No. 4699-B of Subdivision Plan Csd-04-022290-D in favor of the Spouses Danilo and Amorlina Go.

On August 26, 2006, respondents Spouses Danilo and Amorlina Go (the Spouses Go) applied for the registration and confirmation of title
over Cadastral Lot No. 4699-B (Lot No. 4699-B), a parcel of land in Barangay Balagtas, Batangas City covering an area of 1,000 square
meters.[2]

The Spouses Go registered Lot No. 4699-B in their names for taxation purposes. They had paid the real property taxes, including the
arrears, from 1997 to 2006, as shown in Tax Declaration No. 026-04167.[3] They had also established a funeral parlor, San Sebastian
Funeral Homes, on the lot.[4] According to them, there were no other claimants over the property.[5]

The Spouses Go claimed to be in an open, continuous, exclusive, notorious, and actual possession of the property for seven (7) years
since they bought it.[6] They also tacked their possession through that of their predecessors-in-interest, as follows:

Sometime in 1945,[7] Anselmo de Torres (Anselmo) came to know that his parents, Sergia Almero and Andres de Torres (the Spouses
de Torres),[8] owned Lot No. 4699,[9] a bigger property where Lot No. 4699-B came from. According to Anselmo, the Spouses de Torres
paid the real property taxes during their lifetime and planted bananas, mangoes, calamansi, and rice on this lot.[10] His mother, Sergia
Almero (Sergia), allegedly inherited Lot No. 4699 from her parents, Celodonio and Eufemia Almero (the Spouses Almero).[11]

In the 1960s, Anselmo and his siblings inherited Lot No. 4699 from their parents upon their deaths.[12]

One of Anselmo's sisters, Cristina Almero de Torres Corlit (Cristina), then built a residential house on Lot No. 4699-B,[13] declaring this
parcel of land under her name for tax purposes, as evidenced by Tax Declaration No. 026-03492.[14] Meanwhile, Anselmo and his other
siblings built their homes on another portion of Lot No. 4699.[15] Anselmo, who was then 28 years old, started living in the eastern portion
from 1966.[16]

On January 26, 2000, the Spouses Go bought Lot No. 4699-B from the previous owners, siblings Anselmo, Bernardo Almero de Torres,
Leonila Almero de Torres Morada, and Cristina, as evidenced by a Deed of Absolute Sale.[17]

On August 26, 2006, the Spouses Go (respondents) applied for the registration and confirmation of title of Lot No. 4699-B.[18] They
attached the Report dated January 31, 2007 of Special Land Investigator I Ben Hur Hernandez (Hernandez) and the Certification dated
January 29, 2008 of Forester I Loida Maglinao (Maglinao) of the Batangas City Community Environment and Natural Resources Office
(CENRO) of the Calamba, Laguna, Batangas, Rizal, and Quezon (CALABARZON) Region of the Department of Environment and Natural
Resources (DENR).[19]

Hernandez's January 31, 2007 Report and Maglinao's January 29, 2008 Certification stated that the property was located in an alienable
and disposable zone[20] since March 26, 1928, under Project No. 13, Land Classification Map No. 718.[21] No patent or decree was
previously issued over the property.[22]

On November 3, 2006, the Republic of the Philippines (petitioner) opposed respondents' application for registration for the following
reasons: 1) Lot No. 4699-B was part of the public domain; 2) neither the Spouses Go nor their predecessors-in-interest had been in open,
continuous, exclusive, and notorious possession and occupation of the property since June 12, 1945 or even before then; 3) the tax
declaration and payment were not competent or sufficient proof of ownership, especially considering that these were relatively recent.[23]

Anselmo and his siblings had no proof of their inheritance. He claimed that the office having custody of the documentary proof of their
inheritance was burned[24] and they no longer had the original copy of the documents.[25]

In the Decision[26] dated December 12, 2008, the Municipal Trial Court in Cities confirmed the title of the lot in the name of the Spouses
Go. The dispositive portion read:
Considering that the applicants have duly established essential facts in support of the application, the Court hereby confirms title to Lot
4699-B, Cad 264 Batangas Cadastre covered in approved plan Csd-04-22290-D, containing an area of ONE THOUSAND (1,000)
SQUARE METERS situated at Barangay Balagtas, Batangas City in the name of Spouses Danilo Go and Amorlina A. Go, of legal age,
Filipino and residents of San Jose Subdivision, Barangay San Sebastian, Lipa City.

Once the Decision becomes final, let the corresponding Decree of Registration be issued.

SO ORDERED.[27]
Petitioner appealed directly to the Court of Appeals. In the Decision[28] dated January 21, 2011, the Court of Appeals denied the appeal:
WHEREFORE, premises considered, the appeal is DENIED. The assailed Decision, dated December 12, 2008, of the Municipal Trial
Court in Cities (MTCC), Branch 2, Pallocan West, Batangas City in Land Registration Case No. 2006-162, is AFFIRMED.

No pronouncement as to costs.

SO ORDERED.[29]
Petitioner filed its Motion for Reconsideration,[30] which was denied on June 6, 2011.[31]

Petitioner elevated[32] the case before this Court, arguing that Maglinao testified having investigated only 200 square meters of the 1,000-
square-meter land for registration.[33] She also admitted that her certification was based on the approved plan and not on the Land
Classification Map. She certified the lot only to determine "the point or monument of the entire or whole area" and not to id entify its
alienable character. Thus, petitioner argues that Maglinao's certification should not have been used to determine that the land was
alienable and disposable.[34]

Petitioner assails respondents' failure to submit a copy of the original classification map that bears the DENR Secretary's approval and
its legal custodian's certification as a true copy.[35] Petitioner argues that a CENRO Certification is insufficient to establish that a land
applied for registration is alienable.[36]

In the Resolution dated August 15, 2011, this Court required respondents to submit a certified true copy of any Presidential or DENR
Secretary's issuance stating Lot No. 4699-B as alienable and disposable.[37]

In their Compliance[38] dated September 25, 2011, the Spouses Go attached a certified photocopy of the CENRO Certification dated
January 29, 2008,[39] which this Court noted.[40] In the Resolution dated November 14, 2011, this Court informed the Spouses Go that
the CENRO Certification was not the submission required of them.[41]

On June 20, 2012, the Spouses Go's counsel, Atty. Jose Amor M. Amorado, was ordered "to show cause why he should not be
disciplinarily dealt with or held in contempt" for failure to comply with this Court's August 15, 2011 Resolution.[42] The Spouses Go
manifested that they had already complied with this Court's Resolution through their September 25, 2011 Compliance.[43] They re-
attached the CENRO Certification dated January 29, 2008.[44]

On September 24, 2012, this Court resolved[45] to require respondents to file their Comment. The Spouses Go failed to do so, which led
this Court to again require[46] their counsel to show cause for their failure to comply with the September 24, 2012 Resolution.

In their Compliance[47] dated August 15, 2013, the Spouses Go informed this Court that they would dispense with the filing of their
Comment.

For resolution before this Court is whether the Court of Appeals erred in issuing the Spouses Go a Decree of Registration over Lot No.
4699-B.

Any application for confirmation of title under Commonwealth Act No. 141[48] already concedes that the land is previously public.

For a person to perfect one's title to the land, he or she may apply with the proper court for the confirmation of the claim of ownership and
the issuance of a certificate of title over the property.[49] This process is also known as judicial confirmation of title.[50]

Section 48(b) of Commonwealth Act No. 141, as amended[51] by Presidential Decree No. 1073,[52] states who can apply for judicial
confirmation of title:
Section 48. The following described citizens of the Philippines, occupying lands of the public domain or claiming to own any such lands
or an interest therein, but whose titles have not been perfected or completed, may apply to the Court of First Instance [Regional Trial
Court] of the province where the land is located for confirmation of their claims and the issuance of a certificate of title therefor, under the
Land Registration Act, to wit:

....
(b) Those who by themselves or through their predecessors in interest have been in the open, continuous, exclusive, and notorious
possession and occupation of agricultural lands of the public domain, under a bona fide claim of acquisition or ownership, except as
against the government, since July twenty-sixth, eighteen hundred and ninety-four, except when prevented by war or force majeure.
Those shall be conclusively presumed to have performed all the conditions essential to a Government grant and shall be entitled to a
certificate of title under the provisions of this chapter. (Emphasis supplied)
Commonwealth Act No. 141 is a special law that applies to agricultural lands of the public domain, not to forests, mineral lands, and
national parks.[53] The requisite period of possession and occupation is different from that of land classification.

In an application for judicial confirmation of title, an applicant already holds an imperfect title to an agricultural land of the public domain
after having occupied it from June 12, 1945 or earlier.[54] Thus, for purposes of obtaining an imperfect title, the date it was classified is
immaterial.[55]

Classifying a land of the public domain as agricultural is essential only to establish the applicant's "eligibility for land registration, not the
ownership or title over it."[56] Heirs of Malabanan v. Republic of the Philippines[57] explained:
[T]he applicant's imperfect or incomplete title is derived only from possession and occupation since June 12, 1945, or earlier. This means
that the character of the property subject of the application as alienable and disposable agricultural land of the public domain determines
its eligibility for land registration, not the ownership or title over it.[58]
In Malabanan, the Court En Banc affirmed that June 12, 1945 is the "reckoning point of the requisite possession and occupation" and not
of the land classification as alienable and disposable:
[T]he choice of June 12, 1945 as the reckoning point of the requisite possession and occupation was the sole prerogative of Congress,
the determination of which should best be left to the wisdom of the lawmakers. Except that said date qualified the period of possession
and occupation, no other legislative intent appears to be associated with the fixing of the date of June 12, 1945. Accordingly, the Court
should interpret only the plain and literal meaning of the law as written by the legislators.
[A]n examination of Section 48 (b) of the Public Land Act indicates that Congress prescribed no requirement that the land subject of the
registration should have been classified as agricultural since June 12, 1945, or earlier.[59] (Emphasis supplied)
Thus, the land may be declared alienable and disposable at any time, not necessarily before June 12, 1945. The moment that the land
is declared alienable and disposable, an applicant may then initiate the proceedings for the judicial confirmation of title.

On the other hand, for the requisite duration of possession, an applicant must have had possession of the property under a bona fide
claim of ownership or acquisition, from June 12, 1945 or earlier. Such possession must have also been open, continuous, exclusive, and
notorious.[60]

Under Section 11(4)(a) of Commonwealth Act No, 141, the judicial confirmation of imperfect or incomplete titles, which the law describes
as "judicial legalization," allows for agricultural public lands to be disposed of by the State and acquired by Filipino citizens.[61]

Meanwhile, Section 14(1) of Presidential Decree No. 1529[62] provides for the procedure to register a title under the Torrens system:
Section 14. Who may apply. — The following persons may file in the proper Court of First Instance an application for registration of title
to land, whether personally or through their duly authorized representatives:

(1)
Those who by themselves or through their predecessors-in-interest have been in open, continuous, exclusive and notorious possession
and occupation of alienable and disposable lands of the public domain under a bona fide claim of ownership since June 12, 1945, or
earlier.
Section 14(1) of Presidential Decree No. 1529 does not vest or create a title to a public land that has already existed or has been vested
under Commonwealth Act No. 141.[63] The procedure of titling under Presidential Decree No. 1529 "simply recognizes and documents
ownership and provides for the consequences of issuing paper titles."[64]

Thus, under Section 48(b) of Commonwealth Act No. 141, as amended, and Section 14(1) of Presidential Decree No. 1529, Filipino
citizens applying for the judicial confirmation and registration of an imperfect title must prove several requisites. First, they must prove
that they, by themselves or through their predecessors-in-interest, have been in open, continuous, exclusive, and notorious possession
of the property. Second, it must be settled that the applicants' occupation is under a bona fide claim of acquisition or ownership since
June 12, 1945 or earlier, immediately before the application was filed. Third, it should be established that the land is an agricultural land
of public domain. Finally, it has to be shown that the land has been declared alienable and disposable.[65]

The Spouses Go's possession, by themselves or through their predecessors-in-interest, does not meet the statutory requirements.

The evidence the Spouses Go submitted to prove their required length of possession consist of Anselmo's testimony, Cristina's sole Tax
Declaration, and the Spouses Go's sole Tax Declaration. Other than these pieces of evidence, the Spouses Go could not support their
claim of possession in the concept of an owner, by themselves or through their predecessors-in-interest, from June 12, 1945 or earlier.

The records do not show that the Spouses Go's predecessors-in-interest fenced the original 3,994-square-meter Lot No. 4699, claiming
it as exclusively theirs or that they introduced improvements on it since June 12, 1945 or earlier. Cristina built a residential house on Lot
No. 4699-B[66] when her parents died in the 1960s,[67] while Anselmo started living in the eastern portion of Lot No. 4699 in 1966 when
he was 28 years old.[68] These events happened at least 15 years after 1945. Moreover, the siblings could not produce any documentary
proof of their alleged inheritance of this land from their parents.[69]

Apart from Cristina's single tax declaration and the Spouses Go's single tax declaration covering even Cristina's arrears from 1997 to
2000, nothing in the records shows that the Spouses Go's predecessors-in-interest religiously paid real property taxes. Payment of real
property taxes is a "good indicia of the possession in the concept of owner for no one in his [or her] right mind would be paying taxes for
a property that is not in his [or her] actual, or at the least constructive, possession."[70]

Anselmo only gave bare assertions that his parents paid the real property taxes during their lifetime.[71] Neither did the Spouses Go give
any proof of the alleged tax payments of the Spouses de Torres or of Anselmo's grandparents, the Spouses Almero.

Although not adequate to establish ownership, a tax declaration may be a basis to infer possession.[72] This Court has highlighted that
where tax declaration was presented, it must be the 1945 tax declaration because June 12, 1945 is material to the case.[73] The specific
date must be ascertained; otherwise, applicants fail to comply with the requirements of the law.[74] In Republic v. Manna Properties:[75]
It is unascertainable whether the 1945 tax declaration was issued on, before or after 12 June 1945. Tax declarations are issued any time
of the year. A tax declaration issued in 1945 may have been issued in December 1945. Unless the date and month of issuance in 1945
is stated, compliance with the reckoning date in [Commonwealth Act No.] 141 cannot be established.[76] (Emphasis in the original)
II

Even assuming that there is sufficient evidence to establish their claim of possession in the concept of an owner since June 12, 1945,
the Spouses Go nevertheless failed to prove the alienable and disposable character of the land.

The 1987 Constitution declares that the State owns all public lands.[77] Public lands are classified into agricultural, mineral, timber or
forest, and national parks. Of these four (4) types of public lands, only agricultural lands may be alienated. Article XII, Sections 2 and 3
of the Constitution provide:
Section 2. All lands of the public domain, waters, minerals, coal, petroleum, and other mineral oils, all forces of potential energy, fisheries,
forests or timber, wildlife, flora and fauna, and other natural resources are owned by the State. With the exception of agricultural lands,
all other natural resources shall not be alienated . . .
Section 3. Lands of the public domain are classified into agricultural, forest or timber, mineral lands, and national parks. Agricultural lands
of the public domain may be further classified by law according to the uses [to] which they may be devoted. Alienable lands of the public
domain shall be limited to agricultural lands . . . (Emphasis supplied)
Thus, an applicant has the burden of proving that the public land has been classified as alienable and disposable.[78] To do this, the
applicant must show a positive act from the government declassifying the land from the public domain[79] and converting it into an
alienable and disposable land.[80] "[T]he exclusive prerogative to classify public lands under existing laws is vested in the Executive
Department."[81] In Victoria v. Republic:[82]
To prove that the land subject of the application for registration is alienable, an applicant must establish the existence of a positive act of
the government such as a presidential proclamation or an executive order; an administrative action; investigation reports of Bureau of
Lands investigators; and a legislative act or statute. The applicant may secure a certification from the government that the lands applied
for are alienable and disposable, but the certification must show that the DENR Secretary had approved the land classification and
released the land of the public domain as alienable and disposable[.][83] (Emphasis supplied, citations omitted)
Section X(1)[84] of the DENR Administrative Order No. 1998-24 and Section IX(1)[85] of DENR Administrative Order No. 2000-11 affirm
that the DENR Secretary is the approving authority for "[l]and classification and release of lands of the public domain as alienable and
disposable." Section 4.6 of DENR Administrative Order No. 2007-20 defines land classification as follows:
Land classification is the process of demarcating, segregating, delimiting and establishing the best category, kind, and uses of public
lands. Article XII, Section 3 of the 1987 Constitution of the Philippines provides that lands of the public domain are to be classified into
agricultural, forest or timber, mineral lands, and national parks.
These provisions, read with Victoria v. Republic[86] establish the rule that before an inalienable land of the public domain becomes private
land, the DENR Secretary must first approve the land classification into an agricultural land and release it as alienable and disposable.[87]
The DENR Secretary's official acts "may be evidenced by an official publication thereof or by a copy attested by the officer having legal
custody of the record, or by his deputy."[88]

The CENRO or the Provincial Environment and Natural Resources Officer will then conduct a survey to verify that the land for original
registration falls within the DENR Secretary-approved alienable and disposable zone.[89]

The CENRO certification is issued only to verify the DENR Secretary issuance through a survey. "Thus, the CENRO Certification should
have been accompanied by an official publication of the DENR Secretary's issuance declaring the land alienable and disposable."[90] A
CENRO certification, by itself, is insufficient to prove the alienability and disposability of land sought to be registered.[91] In Republic v.
Lualhati:[92]
[I]t has been repeatedly ruled that certifications issued by the CENRO, or specialists of the DENR, as well as Survey Plans prepared by
the DENR containing annotations that the subject lots are alienable, do not constitute incontrovertible evidence to overcome the
presumption that the property sought to be registered belongs to the inalienable public domain. Rather, this Court stressed the importance
of proving alienability by presenting a copy of the original classification of the land approved by the DENR Secretary and certified as true
copy by the legal custodian of the official records.[93] (Emphasis supplied)
Here, in its Decision[94] dated December 12, 2008, the Court of Appeals concluded that the January 29, 2008 CENRO Certification,
which stated that Lot No. 4699-B was within alienable and disposable zone, was conclusive proof that this land applied for registration
was alienable. This Court disagrees.

To establish that a land is indeed alienable and disposable, applicants must submit the application for original registration with the CENRO
certification and a copy of the original classification approved by the DENR Secretary and certified as a true copy by the legal custodian
of the official records.[95]

Judicially entrenched[96] is the rule that it is the DENR Secretary who has the authority to approve land classification and release a land
of public domain as alienable and disposable. In Republic v. T.A.N. Properties:[97]
[I]t is not enough for the PENRO or CENRO to certify that a land is alienable and disposable. The applicant for land registration must
prove that the DENR Secretary had approved the land classification and released the land of the public domain as alienable and
disposable, and that the land subject of the application for registration falls within the approved area per verification through survey by
the PENRO or CENRO. In addition, the applicant for land registration must present a copy of the original classification approved by the
DENR Secretary and certified as a true copy by the legal custodian of the official records. These facts must be established to prove that
the land is alienable and disposable.[98]
Republic v. Hanover[99] ruled that a CENRO certification does not constitute incontrovertible proof that a piece of land is alienable and
disposable. This is because "the CENRO is not the official repository or legal custodian of the issuances of the DENR Secretary declaring
the alienability and disposability of public lands."[100] Republic v. Vda. De Joson explained:[101]
This doctrine unavoidably means that the mere certification issued by the CENRO or PENRO did not suffice to support the application
for registration, because the applicant must also submit a copy of the original classification of the land as alienable and disposable as
approved by the DENR Secretary and certified as a true copy by the legal custodian of the official records.[102]
III

The pieces of evidence the Spouses Go adduced fall short of the requirements of the law.

First, the Spouses Go failed to present a certified true copy of the original classification of the DENR Secretary. This Court has given
them enough chances to prove their claim. As a rule, this Court can only consider the evidence submitted before the trial court.[103]
Nevertheless, this Court gave respondents the opportunity to submit "a certified true copy of the Presidential or Department of
Environment and Natural Resources Secretary's issuance declaring the property alienable and disposable."[104] They failed to comply
despite being given a show-cause order.[105]

This Court also required them to file their Comment on petitioner's opposition to their original registration.[106] Instead of complying, they
asked that their Comment be dispensed with.[107]
Second, although the Spouses Go submitted a CENRO certification stating that the land was verified to be within alienable and disposable
zone under Project No. 13, Land Classification Map No. 718, Maglinao, the person who issued the CENRO Certification, testified
otherwise. She admitted in her testimony that, she certified the lot only to determine "the point or monument of the entire or whole area"
and not to identify its alienable character.[108]

The Spouses Go have the burden to show that the land for registration is alienable or disposable,[109] which they miserably failed to do
so. Without the original land classification approved by the DENR Secretary, the Spouses Go's application for registration must be
denied.[110] The land remains inalienable.

In sum, the Court of Appeals gravely erred in affirming the trial court's Decision that granted the Spouses Go's application for registration
of Lot No. 4699-B. The Spouses Go failed to adequately prove their claim of possession in the concept of an owner since June 12, 1945.
They likewise failed to establish that the land applied for registration is alienable and disposable. Thus, their occupation of this land, no
matter how long, cannot ripen into ownership and cannot be registered as a title.[111]

WHEREFORE, the Petition is GRANTED.

HI-LON VS. COA

FACTS:
Sometime in 1978, the government, through the then Ministry of Public Works and Highways (now DPWH),converted to a road right-of-
way (RROW) a 29,690 sq. m. portion of the 89,070 sq. m. parcel of land located in Mayapa, Calamba, Laguna, for the Manila South
Expressway Extension Project. The subject property was registered in the name of Commercial and Industrial Real Estate Corporation
(CIREC) under Transfer Certificate of Title (TCT) No. T-40999. Later on, Philippine Polymide Industrial Corporation (PPIC) acquired the
subject property, which led to the cancellation of TCT No. T-40999 and the issuance of TCT No. T-120988 under its name. PPIC then
mortgaged the subject property with the Development Bank of the Philippines (DBP), a government financing institution, which later
acquired the property in a foreclosure proceeding on September 6, 1985. TCT No. T-120988, under PPIC's name, was then cancelled,
and TCT No. T-151837 was issued in favor of DBP. Despite the use of the 29,690 sq. m. portion of the property as RROW, the government
neither annotated its claim or lien on the titles of CIREC, PPIC and DBP nor initiated expropriation proceedings, much less paid just
compensation to the registered owners.

ISSUE:

Whether or not HI-LON is entitled to just compensation for the 29,690 sq. m. portion of the subject property?

HELD:

NO. Having determined that HI-LON or its predecessor-in-interest TGPI does not own the RROW in question, as it has been the property
of the Republic of the Philippines since its acquisition by the DBP up to the present, the COA concluded that the proper valuation of the
claim for just compensation is irrelevant as HI-LON is not entitled thereto in the first place. Concededly, the 29,690 sq. m. portion of the
subject property is not just an ordinary asset, but is being used as a RROW for the Manila South Expressway Extension Project, a road
devoted for a public use since it was taken in1978. Under the Philippine Highway Act of 1953, "right-of-way" is defined as the land secured
and reserved to the public for highway purposes, whereas "highway" includes rights-of-way, bridges, ferries, drainage structures, signs,
guard rails, and protective structures in connection with highways. Article 420 of the New Civil Code considers as property of public
dominion those intended for public use, such as roads, canals, torrents, ports and bridges constructed by the state, banks, shores,
roadsteads, and others of similar character. Being of similar character as roads for public use, a road right-of-way (RROW) can be
considered as a property of public dominion, which is outside the commerce of man, and cannot be leased, donated, sold, or be the
object of a contract, except insofar as they may be the object of repairs or improvements and other incidental matters. However, this
RROW must be differentiated from the concept of easement of right of way under Article 649 of the same Code, which merely gives the
holder of the easement an incorporeal interest on the property but grants no title thereto, 20inasmuch as the owner of the servient estate
retains ownership of the portion on which the easement is established, and may use the same in such a manner as not to affect the
exercise of the easement.

BILAG VS. AY AY

Facts:

The instant case stemmed from a Complaint[5] dated August 12, 2004 for Quieting of Title with Prayer for Preliminary Injunction filed by
respondents Estela Ay-Ay, Andres Acop, Jr., Felicitas Ap-Ap, Sergio Ap-Ap, John Napoleon A. Ramirez, Jr., and Ma. Teresa A. Ramirez
(respondents) against petitioners Bernadette S. Bilag, Erlinda Bilag-Santillan, Dixon Bilag, Reynaldo B. Suello, Heirs of Lourdes S. Bilag,
Heirs of Leticia Bilag-Hanaoka, and Heirs of Nellie Bilag before the RTC Br. 61, docketed as Civil Case No. 5881-R. Essentially,
respondents alleged that Iloc Bilag, petitioners' predecessor-in-interest, sold to them separately various portions of a 159,496-square
meter parcel of land designated by the Bureau of Lands as Approved Plan No. 544367, Psu 189147 situated at Sitio Benin, Baguio City
(subject lands), and that they registered the corresponding Deeds of Sale[6] with the Register of Deeds of Baguio City. According to
respondents, Iloc Bilag not only acknowledged full payment and guaranteed that his heirs, successors-in-interest, and executors are to
be bound by such sales, but he also caused the subject lands to be removed from the Ancestral Land Claims. Respondents further
alleged that they have been in continuous possession of the said lands since 1976 when they were delivered to them and that they have
already introduced various improvements thereon. Despite the foregoing, petitioners refused to honor the foregoing sales by asserting
their adverse rights on the subject lands. Worse, they continued to harass respondents, and even threatened to demolish their
improvements and dispossess them thereof. Hence, they filed the instant complaint to quiet their respective titles over the subject lands
and remove the cloud cast upon their ownership as a result of petitioners' refusal to recognize the sales.[7]... on January 27, 1998,
respondents had already filed a complaint against them for injunction and damages, docketed as Civil Case No. 3934-R before the
Regional Trial Court of Baguio City, Branch 5 (RTC Br. 5), wherein they principally asserted their ownership over the subject lands.
However, RTC Br. 5 dismissed Civil Case No. 3934-R for lack of merit on the ground of respondents' failure to show convincing proof of
ownership over the same,[11] which Order of dismissal was then affirmed by the CA on appeal.[12] Eventually, the Court issued a
Resolution dated January 21, 2004[13] declaring the case closed and terminated for failure to file the intended petition subject of the
Motion for Extension to file the same. In view of the foregoing, petitioners contended that due to the final and executory ruling in Civil
Case No. 3934-R, the filing of Civil Case No. 5881-R seeking to establish the ownership thereof is already barred by res judicata.[14]

Issues:

whether or not the CA correctly set aside the dismissal of Civil Case No. 5881-R, and accordingly, remanded the case to the court a quo
for trial.

Ruling:

In a catena of cases,[27] and more importantly, in Presidential Decree No. (PD) 1271,[28] it was expressly declared that all orders and
decisions issued by the Court of First Instance of Baguio and Benguet in connection with the proceedings for the reopening of Civil
Reservation Case No. 1, GLRO Record 211, covering lands within the Baguio Townsite Reservation are null and void and without force
and effect. While PD 1271 provides for a means to validate ownership over lands forming part of the Baguio Townsite Reservation, it
requires, among others, that a Certificate of Title be issued on such lands on or before July 31, 1973.[29] In this case, records reveal that
the subject lands are unregistered and untitled, as petitioners' assertion to that effect was not seriously disputed by respondents. Clearly,
the award of lots 2 and 3 of the 159,496-square meter parcel of land designated by the Bureau of Lands as Approved Plan No. 544367,
Psu 189147 – which includes the subject lands – to Iloc Bilag by virtue of the reopening of Civil Reservation Case No. 1, GLRO Record
211, is covered by the blanket nullification provided under PD 1271, and consistently affirmed by the prevailing case law. In view of the
foregoing, it is only reasonable to conclude that the subject lands should be properly classified as lands of the public domain as well.

the Court ruled that the trial court therein correctly dismissed an action to quiet title on the ground of lack of jurisdiction for lack of authority
to determine who among the parties have better right over the disputed property, which is admittedly still part of public domain for being
within the Baguio Townsite Reservation, viz.:

The DENR Decision was affirmed by the Office of the President which held that lands within the Baguio Townsite Reservation belong to
the public domain and are no longer registrable under the Land Registration Act. The Office of the President ordered the disposition of
the disputed property in accordance with the applicable rules of procedure for the disposition of alienable public lands within the Baguio
Townsite Reservation, particularly Chapter X of Commonwealth Act No. 141 on Townsite Reservations and other applicable rules.

In sum, the decision rendered in civil case 1218 on October 28, 1968 is a patent nullity. The court below did not have power to determine
who (the Firmalos or the Tarucs) were entitled to an award of free patent title over that piece of property that yet belonged to the public
domain.

Under Articles 476 and 477 of the Civil Code, the two indispensable requisites in an action to quiet title are: (1) that the plaintiff has a
legal or equitable title to or interest in the real property subject of the action; and (2) that there is a cloud on his title by reason of any
instrument, record, deed, claim, encumbrance or proceeding, which must be shown to be in fact invalid or inoperative despite its prima
facie appearance of validity.

WHEREFORE, the petition is GRANTED.

CALDITO VS. OBADO

The Facts

This petition stemmed from a complaint5 for quieting of ownership over a parcel of land covering the 272.33 square meters eastern
portion of Lot No. 1633 situated at Barangay No. 5, San Vicente, Sarrat, Ilocos Norte, filed by Spouses Jenestor B. Caldito and Ma.
Filomena Tejada Caldito (Filomena) (petitioners) against Isagani V. Obado (Isagani) and Gereon V. Obado (respondents).

The record showed that as early as 1921, Lot No. 1633 was declared for taxation purposes in the name of Felipe Obado (Felipe). After
Felipe's death, Paterno Obado (Paterno), whom Felipe treated like his own son, subsequently occupied Lot No. 1633 and continued to
pay the realty taxes of the same.6

Sometime in 1995, Antonio Ballesteros (Antonio) executed an Affidavit of Ownership dated February 23, 1995 narrating his claim over
the subject parcel of land.· In his affidavit, Antonio claimed that Lot No. 1633 was co-owned by Felipe with his five siblings, namely:
Eladia, Estanislao, Maria, Severino and Tomasa, all surnamed Obado.7

On the next day following the execution of the said affidavit or on February 24, 1995, Antonio and Elena Ballesteros (Spouses Ballesteros)
sold the subject parcel of land to the petitioners for the sum of P70,000.000 evidenced by a Deed of Absolute Sale. Thereafter, the
petitioners declared the subject lot for taxation purposes and paid the realty taxes thereon.8
In 2002, the petitioners attempted to build a house on the subject parcel of land but the respondents prevented them from completing the
same. The respondents then filed a complaint before the barangay but no amicable settlement was reached between the parties.9 Hence,
on December 8, 2003, the petitioners instituted a complaint for quieting of ownership against the respondents before the RTC, as well as
an injunctive writ to prevent the respondents from interfering with the construction of their house.10

For their part, the respondents averred that the Spouses Ballesteros were not the owners and possessors of the subject parcel of land.
They maintained that Lot No. 1633 was inherited by their father, Paterno, from its original owner Felipe, and they have been paying the
real property taxes for the entire property. They asserted that the petitioners are buyers in bad faith since their family had been in
possession of the entire Lot No. 1633 since 1969 and had been in. open, peaceful and uninterrupted possession of the whole property
up to the present or for more than 30 years in the concept of an owner. 11

After trial, the court a quo rendered its judgment in favor of the petitioners. The trial court upheld the validity of the sale between the
petitioners and the Spouses Ballesteros and dismissed the respondents' claim of ownership over Lot No. 1633. The trial court held that
the petitioners presented convincing evidence of ownership over the subject parcel of land which consists of the following: (a) the Deed
of Absolute Sale executed between the petitioners and the Spouses Ballesteros; (b) the tax declarations all paid by the petitioners only;
and (c) the Affidavit of Ownership allegedly executed by Antonio. The trial court also found that the respondents have no successional
rights over the property of Felipe based on the governing law and on the order of intestate succession at that time and the established
facts. Thus, the RTC disposed as follows:

WHEREFORE, IN VIEW OF ALL THE FOREGOING PREMISES, the preponderance of evidence having substantially and sufficiently
tilted in favor of the [petitioners] herein and against the [respondents] herein named and their siblings, this Court hereby renders judgment
declaring the validity of the 272.33 square meters eastern portion of Lot No. 1633 in favor of the [petitioners] and, the [respondents] are
hereby ordered to do the following:

1. to respect, recognize and not to molest the lawful ownership and possession of the [petitioners] over the 272.33 square meters located
at the eastern portion of Lot No. 1633 of the Sarrat Cadastre;

2. to pay jointly and severally to the /petitioners} the total sum of:

2.a. PJJB,453.50- as and/or actual damages;

2.h. P400,000.00- as and/or moral damages;

2.c. PJ00,000.00- as and/or nominal damages;

2.d. P200,000.00- as and/or temperate damages; and

2.e. P300,000.00- as and/or exemplary damages or corrective.

With costs against the /respondents].

SO ORDERED.12 (Emphasis and italics in the original)

On appeal, the CA reversed and set aside the RTC decision upon finding that: (1) the petitioners failed to prove the title of their immediate
predecessors-in-interest, the Spouses Ballesteros; (2) the petitioners failed to support their claim that Felipe and his siblings, Eladia,
Estanislao, Maria, Severino and Tomasa, co-owned Lot No. 1633; (3) Antonio should have been called to the witness stand to testify on
the contents of his Affidavit of Ownership; (4) the Deed of Absolute Sale is not a sufficient and convincing evidence that the petitioners'
predecessors-in-interest have a title on the subject parcel of land. which they can transfer; (5) the petitioners are not innocent purchasers
for value since the subject lot is not registered and is in the possession of another person, other than the Spouses Ballesteros; (6) nothing
in the record could establish the relationship between Felipe and his supposed legal heirs; and (7) the respondents enjoy a legal
presumption of just title in their favor since they are in possession of the entire Lot No. 1633. The CA then ruled that:

For a party seeking to quiet their "ownership" of the portion in litigation, [the petitioners] have, for starters, miserably failed to prove the
title of their immediate predecessors-in-interest, the [Spouses' Ballesteros]. Except for the February 23, 1995 Affidavit of Ownership I
executed by [Antonio], there is, in fact, no evidence on record to support the claim that the subject parcel was, indeed, co-owned by
[Felipe] [and]:1 his siblings, Eladia, Estanislao, Maria, Severino and Tomasa, all surnamed Obado. To our mind, the fact that [Antonio]
was not even called to the witness stand to testify on the contents of his Affidavit of Ownership '~should have immediately impelled the
trial court to discount its probative value and, with it, the very foundation of [the respondents'] supposed cause of action.

xxxx

With even greater reason are we disposed towards the reversal of the trial court's holding that, pursuant to the provisions of the Spanish
Civil Code of 1889 on intestate succession, Eladia, Estanislao, Maria, s~verino and Tomasa, all surnamed Obado were the ones who
have rightfully inherited the subject parcel from their brother, [Felipe]. Except for the aforesaid February 23, 1995 Affidavit of Ownership
executed by [Antonio], [the respondents] correctly argue that there is nothing on record from which the relationship of said decedent and
his supposed legal heirs may be reasonably deduced. Even if said relationship were, moreover, assumed, the absence of evidence
showing that [Felipe] predeceased all of his supposed siblings impel us to regard, with considerable askance, the trial court's disposition
of the case by application of said rules on intestate succession. Litigations cannot be properly resolved by suppositions, deductions, or
presumptions, with no basis in evidence for the truth must have 1 to be determined by the hard rules on admissibility and proof. This is
particularly true. of the case at bench where the successional rights determined by the trial court are diametrically opposed to [Antonio's]
Affidavit of Ownership which dubiously claimed that the subject parcel was, in fact, co-owned by [Felipe] and his ostensible siblings and
had already been partitioned by and among them. 13

The petitioners moved for reconsideration14 but the same was denied. 15 Hence, this petition.

The Issue

WHETHER OR NOT THE PETITIONERS WERE ABLE TO PROVE OWNERSHIP OVER THE SUBJECT PARCEL OF LAND.

Ruling of the Court

The petition has no merit.

At the outset, it bears to emphasize that there is no dispute with respect to the fact that Felipe was the original owner of the entire parcel
of unregistered land known as Lot No. 1633 which he started declaring as his property for taxation purposes as early as 1921. When
Felipe died without issue, Lot No. 1633 was subsequently occupied by Paterno who then declared the same for taxation purposes and
paid the realty taxes thereon.

The petitioners' complaint styled as being for the "quieting of ownership" is in fact an action for quieting of title. The petitioners anchor
their cause of action upon the Deed of Sale and the Affidavit of Ownership executed by Antonio. On the other hand, the respondents
countered that: (1) they inherited from their father, Paterno, Lot No. 1633, of which the herein disputed subject parcel of land is part; and
(2) they have been in possession of the same for more than 30 years in the concept of an owner.

Essentially, the issues raised center on the core question of whether the petitioners were able to prove ownership over the subject parcel
of land.1âwphi1 In resolving this issue, the pertinent point of inquiry is whether the petitioners' predecessors-in-interest, the Spouses
Ballesteros, have lawful title over the subject parcel of land.

While the question raised is essentially one of fact, of which the Court normally abstains from, yet, considering the incongruent factual
conclusions of the courts below, the Court is constrained to go by the exception to the general rule and proceed to reassess the factual
circumstances of the case and make its own assessment of the evidence and documents on record. But even if the Court were to re-
evaluate the evidence presented, there is still no reason to depart from 'the CA' s ruling that Lot No. 1633 is owned by the respondents.

The Court concurs with the disquisition of the CA that the petitioners failed to: (1) prove the title of their immediate predecessors-in-
interest, the Spouses Ballesteros; and (2) present evidence supporting the claim that Lot No. 1633 was co-owned by Felipe and his
siblings, Eladia, Estanislao, Maria, Severino and Tomasa. Also, the Court finds that the RTC mistakenly relied upon the Affidavit of
Ownership, executed by Antonio, to conclude that the petitioners were possessors in good faith and with just title who acquired the subject
parcel of land through a valid deed of sale.

In this case, the petitioners' cause of action relates to an action to quiet title which has two indispensable requisites, namely: (1) the
plaintiff or complainant has a legal or an equitable title to or interest in the real property subject of the action; find (2) the deed, claim,
encumbrance or proceeding claimed to be casting cloud on his title must be shown to be in fact invalid or inoperative despite its primafacie
appearance of validity or legal efficacy. 16

From the foregoing provisions, it is clear that the petitioners' cause of action must necessarily fail mainly in view of the absence of the
first requisite since the petitioners were not able to prove equitable title or ownership over the subject parcel of land.

The petitioners' claim of legal title over the subject parcel of land by virtue of the Deed of Sale and Affidavit of Ownership issued by
Antonio cannot stand because they failed to prove the title of their immediate predecessors-in-interest, the Spouses Ballesteros. The
Court cannot give full credence to Antonio's Affidavit of Ownership for he simply made general and self-serving statements therein which
were favorable to him, and which were not supported with documentary evidence, with no specifics as to when their predecessors-in-
interest acquired the subject parcel of land, and when the Donations Propter Nuptias were made. Indeed, such is hardly the well-nigh
incontrovertible evidence required in cases of this nature. The petitioners must present proof of specific acts of ownership to substantiate
his claim and cannot just offer general statements which are mere conclusions of law than factual evidence of possession.17 Moreso,
Antonio was not even called to the witness stand to testify on the contents of his Affidavit of Ownership, thus, making the affidavit hearsay
evidence and its probative value questionable. Accordingly, this affidavit must be excluded from the judicial proceedings being
inadmissible hearsay evidence.

Furthermore, the said affidavit was executed by Antonio only a day before the subject parcel of land was sold to the petitioners.18 The
trial court should have considered this in evaluating the value of the said affidavit in relation to the ownership of the subject parcel of land.
The trial court's reliance on the Affidavit of Ownership executed by Antonio that the entire Lot No. 1633 was co- owned by Felipe and his
siblings, Eladia, Estanislao, Maria, Severino and Tomasa is misplaced, considering that nothing on record shows the relationship between
Felipe and his supposed legal heirs. It also indicates the fact that there is no evidence showing Felipe predeceasing all his supposed
siblings.19 Moreover, no other piece of evidence was ever presented to prove that Lot No. 1633 was ever subdivided. In fact, the
petitioners admitted that the subject lot has always been declared for taxation purposes in the name of Felipe and that the Spouses
Ballesteros or the siblings of Felipe have never declared the same for taxation purposes in their names.

While the petitioners submitted official receipts and tax declarations to prove payment of taxes, nowhere in the evidence was it shown
that Spouses Ballesteros dEclared the subject parcel of land in their name for taxation purposes or paid taxes due thereon. True, a tax
declaration by itself is not sufficient to prove ownership. Nonetheless, it may serve as sufficient basis for inferring possession.20 In fact,
what the petitioners presented as their pieces of evidence are receipts and tax declarations which they, as the new owners of the subject
parcel of land, have paid. Thus, the petitioners could not also rely on these tax declarations and receipts because those are of recent
vintage and do not reflect the fact that their predecessors-in-interest have been paying realty taxes for the subject parcel of land.

Be that as it may, the rights of the respondents as owners of Lot No. 1633 were never alienated from them despite the sale of the subject
parcel of land by the Spouses Ballesteros to the petitioners nor does the fact that the petitioners succeeded in paying the real property
taxes of the subject parcel of land. Besides, it seems that the petitioners knew of the fact that they did not have a title to the subject parcel
of land and could not, therefore, have validly registered the Same, because of the respondents' possession of the entire property.

The respondents also presented the following pieces of evidence: (1) old certified photocopies of declarations of real property and original
copy of tax receipts from year 1921 to 1944 in the name of Felipe, covering payments by the latter for Lot No. 1633 from which the subject
parcel of land was taken;21 (2) original copy of tax receipts from year 1961 to year 1989 in the name of the respondents' father Paterno,
covering payments by the latter for Lot No. 1633;22 (3) original copy of tax receipt dated July 21, 2004 in the name of Isagani, covering
payments by the latter for Lot No. 1633 ; 23 (4) original copy of the Certification issued by the Municipal Treasurer of Sarrat, Ilocos Norte
that Lot No. 1633 covered by Tax Declaration No. 03-001-00271 declared in the name of Felipe is not delinquent in the payment of realty
taxes. 24

Although tax declarations or realty tax payment of property are not conclusive evidence of ownership, as in the instant case, they are
good indicia of possession in the concept of owner, for no one in his right mind would be paying taxes for a property that is not in his
actual or constructive possession. 25 They constitute evidence of great weight in support of the claim of title of ownership by prescription
when considered with the actual possession of the property by the applicant.26

Indeed, the respondents' presentation of the tax declarations and tax receipts which all are of ancient era indicates possession in the
concept of an owner by the respondents and their predecessors-in-interests. The tax declarations in the name of Paterno take on great
significance because the respondents can tack their claim of ownership to that of their father. It is worthy to note that the respondents'
father Paterno to whom they inherited the entire Lot No. 1633 paid the taxes due under his name from 1961 to 1989; and subsequently,
the respondents paid the taxes due after the death of Paterno in 2003. 27 Granting without admitting that Felipe's possession of Lot No.
1633 cannot be tacked with the respondents' possession, the latter's possession can be tacked with that of Paterno. Thus, from 1961 to
the time of the filing of the quieting of title by the petitioners in 2003, the respondents have been in possession of the entire Lot No. 1633
in the concept of an owner for almost 42 years. This period of time is sufficient to vest extraordinary acquisitive prescription over the
property on the respondents. As such, it is immaterial now whether the respondents possessed the property in good faith or not.

Admittedly, the respondents built their house at the western portion of Lot No: 1633, and Isagani has declared that the eastern part was
their family's garden. Thus, it was fenced with bamboo and was planted with banana trees and different vegetables. Clearly, there is no
doubt that the respondents did not only pay the taxes due for the whole Lot No. 1633, in which the eastern portion is a part, but rather,
the respondents were able to prove that they have possession of the whole lot.

While the findings of the CA that the petitioners were a buyer in bad faith is in accord with the evidence on record, it must be pointed out,
however, that they overlooked the fact that Lot No. 1633 is an unregistered • piece of land. The Court had already ruled that the issue of
good faith or bad faith of a buyer is relevant only where the subject of the sale is a registered land but not where the property is an
unregistered land. One who purchases an unregistered land does so at his peril. His claim of having bought the land in good faith, i.e.
without notice that some other person has a right to, or interest in, the property, would not protect him if it turns out that the seller does
not actually own the property. 28 All the same, the application of this doctrine will not affect the outcome of this case.

Obviously, the petitioners cannot benefit from the deed of sale of the subject parcel of land, executed by the Spouses Ballesteros in their
favor, to support their claim of possession in good faith and with just title. The Court noted that in Filomena's testimony, she even admitted
that the respondents own the bigger portion of Lot No. 1633.29 Thus, it is clear that the petitioners chose to close their eyes to facts which
should have put a reasonable man on his guard. The petitioners failed to ascertain whether the Spouses Ballesteros were the lawful
owner of the subject parcel of land being sold. Far from being prudent, the petitioners placed full faith on the Affidavit of Ownership that
Antonio executed. Hence, when the subject parcel of land was bought by the petitioners, they merely stepped into the shoes of the
Spouses Ballesteros and acquired whatever rights and obligations appertain thereto.

It is also worthy to note of the respondents' reaction when the petitioners tried to construct a house in the subject parcel of land in 2002.
Upon learning that a house was being built on the eastern portion of Lot No. 1633, the respondents went to the barangay to file a
complaint.30 Clearly, this indicates the respondents' vigilance to protect their property. The Court also notes that in the respondent's
possession of the entire Lot No. 1633 for almost 42 years, there was no instance during this time that the petitioners or their predecessors-
in-interest, for that matter, questioned the respondents' right over Lot No. 1633.

From the foregoing disquisitions, it is clear that the petitioners were not able to prove equitable title or ownership over the subject parcel
of land. Except for their claim that they merely purchased the same from the Spouses Ballesteros, the petitioners presented no other
justification to disprove the ownership of the respop.dents. Since the Spouses Ballesteros had no right to sell the subject parcel of land,
the petitioners cannot be deemed to have been the lawful owners of the same.

WHEREFORE, the petition is DENIED.

REPUBLIC VS. ESTATE OF SANTOS

The Antecedents
On October 9, 2006, the Application for Land Registration3 of a parcel of land identified as Lot No. 10839-C (subject land) located at P.
Burgos St., Sta. Ana, Taguig City, with an area of 3,942 square meters and an assessed value of P82,400.00, was filed by respondent
Estate of Virginia Santos (respondent estate), through its administrator, Pacifico Santos (Pacifico). The subject land was a subdivision of
Lot No. 10839 described under survey Plan Csd-00-000352 (Subdivision Plan of Lot No. 10839, MCadm 590-D, Taguig Cadastral
Mapping).

Together with its application for registration, respondent estate submitted the following documents: (1) Letters of Administration4 showing
that Pacifico was appointed as the administrator of the estate of Virginia Santos (Virginia); (2) Oath of Office of Pacifico;5 (3) Subdivision
Plan6 of Lot No. 10839, MCadm 590-D, Taguig Cadastral Mapping (Csd-00-000352) with the annotation that the survey was inside L.C.
Map No. 2623 Proj. No. 27-B classified as alienable/disposable by the Bureau of Forest Development on January 03, 1968; (4) Technical
Description of Lot No. 10839-C, Csd-00-000352;7 (5) Certification in Lieu of Surveyor's/Geodetic Engineer's Certificate8issued by the
Land Survey Records Section, Department of Environment and Natural Resources (DENR), National Capital Region; (6) Tax Declaration
(T.D.) No. FL-013-01057;9 and (7) Extrajudicial Settlement of Estate by Sole Heir of the Late Alejandro Santos,10 dated March 27, 1975.

Respondent estate alleged that the late Virginia was the only child and heir of Alejandro Santos (Alejandro), who was the owner of the
subject land during his lifetime. It further asserted that on March 27, 1975, or after Alejandro's death, Virginia executed an Extrajudicial
Settlement of Estate by Sole Heir of the Late Alejandro Santos (Extrajudicial Settlement) and appropriated the subject land for herself.
Respondent estate further alleged that Virginia, by her and through her predecessor-in-interest, had been in open, continuous, exclusive,
and adverse possession of the property in the concept of owner for more than thirty (30) years.11

On October 9, 2006, the MeTC issued a notice of hearing setting the case for initial hearing on February 7, 2007.12

On April 30, 2007, petitioner Republic of the Philippines (Republic), through the Office of the Solicitor General (OSG), filed its Opposition13
to the Application, raising the following grounds: that neither the applicant nor the predecessors-in-interest of Virginia had been in open,
continuous, exclusive, and notorious possession and occupation of the subject land for a period of not less than thirty (30) years; that the
tax declarations and/or tax payment receipts attached to the application did not constitute competent and sufficient evidence of a bona
fide acquisition of the land applied for; that the claim of ownership in fee simple on the basis of a Spanish title or grant could no longer be
availed of by the applicant; and that the subject land was a portion of the public domain belonging to the Republic and not subject to
private appropriation.

On July 12, 2007, the Land Registration Authority (LRA) submitted its Report14 stating that the subject property, as plotted, did not appear
to overlap with any previously plotted decreed properties and that it was not in a position to verify whether or not the aforesaid land was
already covered by a land patent and previously approved isolated surveys.

Thereafter, trial ensued.

To support its allegation of possession and occupation, respondent estate presented Romualdo B. Flores (Romualdo) who testified that
Virginia owned the subject land; that he had been tilling the land since 1970; that his father, Sixto Cuevas Flores (Sixto), tilled the land
for Alejandro even before the Japanese occupation in 1941; and that he knew this for a fact as he was already nine (9) years old and
attained the age of reason at that time. Respondent estate also offered in evidence several tax declarations covering Lot No. 10839, the
earliest of which was T.D. No. 6532 issued on August 19, 1949.15

The MeTC Ruling

In its August 31, 2011 Decision16 the MeTC denied respondent estate's application for registration of the subject land. It opined that
respondent estate failed to present sufficient evidence to establish its claim of possession and ownership over the subject land. The
MeTC reasoned that mere casual cultivation of portions of the subject land did not constitute sufficient basis for a claim of ownership. It
did not give much weight either to the tax declarations offered in evidence as it stated that these documents were mere indication of claim
of ownership and not ownership itself. 17

The MeTC added that respondent estate failed to prove the alienable and disposable character of the subject land. It opined that the
certification at the dorsal portion of the survey plan was not the kind of evidence contemplated in an application for original registration of
title to land. The decretal portion of the decision, thus, reads: ChanRoblesVirtualawlibrary

WHEREFORE, all premises considered, the instant application for registration of land filed by the Estate of Virginia Santos represented
by Pacifico S. Santos, is hereby denied.

SO ORDERED.18

On September 16, 2011, respondent estate filed its Motion for Reconsideration (With Alternative Motion for New Trial).19 On February
24, 2012, the MeTC granted the motion and allowed respondent estate to present further evidence in support of its application. In granting
the motion, the MeTC explained that respondent committed mistake or excusable negligence which ordinary prudence could not have
guarded against xxx."20

Respondent estate presented, among others, Felino Flores (Felino), who, through his judicial affidavit,21 testified that he had been tilling
the subject land for Virginia and her estate since 1979; that before him, his father, Romualdo, tilled the land from 1969 until he took over
in 1979; that before his father, his grandfather, Sixto, tilled the land even before the Second World War; and that such claim was an
accepted fact in their family history.
On April 5, 2013, the MeTC issued the Order22 granting the subject application. In completely reversing itself, the trial court stated that
the tax declarations submitted by respondent estate and the certification appearing at the dorsal portion of the survey plan of Lot No.
10839, showing that the land was disposable and alienable, were already sufficient to establish respondent estate's claim over the
property as well as the alienable and disposable character of the subject land.

On the same day, the MeTC issued the Amended Order23 correcting the dispositive portion of the earlier order where the area o f the
subject property was omitted: ChanRoblesVirtualawlibrary

WHEREFORE, all premises considered, this Court hereby confirms the title of applicant ESTATE OF VIRGINA M. SANTOS, represented
herein by the duly appointed administrator, PACIFICO M. SANTOS, Filipino, of legal age, married to Priscilla Santos and a resident of
No. 93 P. Mariano Street, Ususan, Taguig City over the subject parcel of land designated as Lot 10839-C, as shown on subdivision plan
Csd-00-000352, being a portion of Lot 10839, MCadm-590-D, Taguig Cadastral Mapping, situated at Barangay Sta. Ana, Taguig City,
Metro Manila consisting of Three Thousand Nine Hundred Forty Two (3,942) Square Meters, more or less and hereby order the
registration thereof in its name.

After finality of this Decision and upon payment of the corresponding taxes due on the said lot, let an Order for the issuance of decree of
registration be issued.

SO ORDERED.24 [Emphasis and underscoring in the original]

Aggrieved, the Republic, through the OSG, elevated an appeal to the CA.25cralawred

The CA Ruling

In its assailed Decision, dated May 22, 2015, the CA dismissed the Republic's appeal and affirmed the Amended Order, dated August 5,
2013 of the MeTC. The appellate court noted that the cadastral survey in this case was the same cadastral survey in the case of Natividad
Sta. Ana Victoria vs. Republic26 (Sta. Ana Victoria), wherein the Court granted the application for registration of property. The CA
concluded that it could not take a view contrary to the ruling in the aforesaid case. It also concurred with the trial court that the DENR
certification at the dorsal portion of the subdivision plan of Lot No. 10839 was sufficient evidence to prove the character of Lot No. 10839-
C as alienable and disposable.

The appellate court further ratiocinated that the alleged discrepancies in the area of the property applied for could be explained by the
fact that the subject land was a subdivision of Lot No. 10839. It also found that respondent estate was able to prove its open, continuous,
exclusive, and notorious possession in the concept of owner. Relying again on Sta. Ana Victoria, the CA held that a tax declaration issued
in 1949 could be accepted as proof of open, continuous, exclusive, and notorious possession and occupation in the concept of an owner.
The dispositive portion of the said decision states: ChanRoblesVirtualawlibrary

WHEREFORE, the appeal is DISMISSED. The Amended Order dated April 5, 2013 of the Regional Trial Court (sic), Branch 74, Taguig
City in LRC Case No. 326, is AFFIRMED.

SO ORDERED.27

Hence, this petition, anchored on the following

GROUNDS

THE COURT OF APPEALS GRAVELY ERRED IN TAKING "JUDICIAL NOTICE" OF A "CADASTRAL SURVEY'' SUBMITTED IN A
DIFFERENT CASE ENTITLED "STA. ANA VICTORIA VS. REPUBLIC" TO PROVE, DURING THE APPEAL PROCEEDINGS, THE DATE
WHEN THE SUBJECT LAND WAS FIRST DECLARED ALIENABLE AND DISPOSABLE.

II

THE COURT OF APPEALS GRAVELY ERRED IN GRANTING THE SUBJECT APPLICATION FOR LAND REGISTRATION DESPITE
THE EXISTENCE OF DOUBT IN THE TOTAL AREA OF THE PARCEL OF LAND BEING APPLIED FOR REGISTRATION.

III

THE COURT OF APPEALS GRAVELY ERRED IN RELYING ON THE STA. ANA VICTORIA CASE AND IN UTTERLY DISREGARDING
THAT THERE IS ABSENCE OF EVIDENCE TO PROVE POSSESSION AND OCCUPATION BY RESPONDENT OR ITS
PREDECESSORS-IN-INTEREST SINCE JUNE 12, 1945, OR EARLIER.28

The Republic argues, first, that the CA gravely erred in its over-reliance on Sta. Ana Victoria. It posits that although the CA could take
judicial notice of Sta. Ana Victoria, it could not hastily rule that the subject land was also alienable and disposable based merely on the
allegation that the subject property and the property registered in the said case belonged to the same cadastral survey. Second, the
Republic asserts that respondent estate failed to establish its open, exclusive, continuous and notorious possession and occupation under
a bona fide claim of ownership over the subject land since June 12, 1945, or earlier. It contends that the tax declarations submitted by
respondent estate were considered not proofs of ownership. Moreover, the earliest tax declaration submitted by respondent estate was
for the year 1949, short of the required possession under the law. Lastly, the Republic insists that respondent estate's witnesses merely
gave shady statements on the supposed ownership of Virginia and Alejandro, without showing any manifestation of acts of dominion over
the property.

In its Comment,29 respondent estate countered that judicial decisions of this Court, including the findings of facts which were integral
parts thereof, formed part of the legal system which all other courts were bound to follow and be familiar with. It asserted that since the
subject land emanated from the same cadastral survey declared as alienable and disposable in Sta. Ana Victoria, the subject property
must likewise be declared as alienable and disposable. It further advanced that the contents of the certification at the dorsal portion of
the survey plan and the technical description of the property enjoyed the presumption of their accuracy.

With regard to possession and occupation, respondent estate averred that its witnesses testified on the identity of the property, the crops
planted thereon, and the three generations of tenancy agreement involving the subject land. It claimed that these testimonies were further
supplemented by the tax declarations it presented, which showed that Virginia and her predecessor-in-interest were in possession of the
subject land for more than fifty (50) years.

In its Reply,30 the Republic reiterated its position that respondent estate failed to adduce sufficient evidence of possession and occupation
on or before June 12, 1945; and that the appellate court erred in concluding that the subject land was declared alienable and disposable
based merely on the facts sustained in Sta. Ana Victoria.

The Court's Ruling

Essentially, the Court is asked to resolve the issue of whether the CA erred in granting respondent estate's application for registration
despite its failure to comply with the requirements for original registration of title to/and under Section 14 of P.D. No. 1529.

The petition is meritorious.

At the onset, the Court notes that there was some confusion as to what law on which the application for registration of the subject land
was based. As per examination of respondent estate's application, it would seem that the basis for their application was Section 14(2) of
P.D. No. 1529 considering its allegation of possession and occupation in the concept of owner for more than thirty (30) years. The MeTC,
and later the appellate court, however, granted the application under Section 14(1) of the same law making reference to June 12, 1945,
or prior thereto, as the earliest date of possession and occupation. Thus, the Court deems it proper to discuss respondent estate's
application for registration of title to the subject property vis-a-vis the provisions of Section 14(1) and (2) of P.D. No. 1529.

Respondent Estate Failed to Comply with the


Requirements under Section 14(1) of
P.D. No. 1529

In Republic of the Philippines vs. Cortez,31 the Court explained that applicants for original registration of title to land must first establish
compliance with the provisions of either Section 14(1) or Section 14(2) of P.D. No. 1529. Section 14(1) provides that:
ChanRoblesVirtualawlibrary

Sec. 14. Who may apply. The following persons may file in the proper Court of First Instance an application for registration of title to land,
whether personally or through their duly authorized representatives: ChanRoblesVirtualawlibrary

(1)
Those who by themselves or through their predecessors-in-interest have been in open, continuous, exclusive and notorious possession
and occupation of alienable and disposable lands of the public domain under a bona fide claim of ownership since June 12, 1945, or
earlier.
Under Section 14(1), applicants for registration of title must sufficiently establish the following: first, that the land or property forms part of
the disposable and alienable lands of the public domain; second, that the applicant and his predecessors-in-interest have been in open,
continuous, exclusive, and notorious possession and occupation of the same; and third, that it is under a bona fide claim of ownership
since June 12, 1945, or earlier.

The first requisite of Section 14(1) only entails that the property sought to be registered be alienable and disposable at the time of the
filing of the application for registration.32

In this case, to prove that the subject land formed part of the alienable and disposable lands of the public domain, respondent estate
relied on the annotation on the subdivision plan of Lot No. 10839 and on the certification issued by Rodelina M. De Villa, Forester II of
the Forest Management Services (FMS) of the DENR, which both stated that the subject land was verified to be "within the alienable and
disposable land under Project No. 27-B, Taguig Cadastral Mapping as per LC Map No. 2623."33

These pieces of evidence, however, would not suffice. The present rule is that to prove the alienability and disposability of the land sought
to be registered, an application for original registration must be accompanied by (1) a City Environment and Natural Resources Office
(CENRO) or Provincial Environment and Natural Resources Officer (PENRO) Certification; and (2) a copy of the original classification
approved by the DENR Secretary and certified as a true copy by the legal custodian of the official records.34 Clearly, the annotation on
the subdivision plan and the certification from the FMS fall short of these requirements.35
The judicial notice by the appellate court of the cadastral survey submitted in Sta. Ana Victoria will not cure respondent es tate's
shortcomings.

In Spouses Latip vs. Chua,36 it was ruled that a court cannot take judicial notice of any fact which, in part, was dependent on the existence
or non-existence of a fact of which the court has no constructive knowledge.37

In this case, in concluding that the subject land formed part of the alienable and disposable lands of the public domain, the CA, in effect,
assumed and took judicial notice that it was located within L.C. Map No. 2623. This is, however, erroneous considering that the CA had
no constructive knowledge as to the location of the subject land and the technical boundaries of L.C. Map No. 2623. Furthermore, the CA
erred in assuming the identity and location of the subject land because such matter was still under dispute. In fact, the Republic relentlessly
raised this issue even during the trial arguing that the identity of the land in question was doubtful. This position was further reiterated by
the Republic in its Reply when it argued that respondent estate failed to prove that the subject property was actually covered by the same
cadastral survey submitted in Sta. Ana Victoria.

Accordingly, the CA erred in taking judicial notice of the identity and location of subject land. Its declaration that the subject land was
alienable and disposable based merely on the declaration in Sta. Ana Victoria was erroneous.

Proof of Possession

Aside from the alienable and disposable character of the land sought to be registered, the applicant must also prove that he/she and/or
his/her predecessors-in-interest have been in open, continuous, exclusive, and notorious possession and occupation of the land under a
bona fide claim of ownership since June 12, 1945, or earlier. Possession is open when it is patent, visible, apparent, notorious, and not
clandestine. It is continuous when uninterrupted, unbroken and not intermittent or occasional. It is exclusive when the adverse possessor
can show exclusive dominion over the land and an appropriation of it to his own use and benefit. And it is notorious when it is so
conspicuous that it is generally known and talked of by the public or the people in the neighborhood.38 Respondent estate in this case
also failed to prove this requirement.

Respondent estate presented several tax declarations in the name of Virginia and Alejandro. The earliest of these tax declarations,
however, dates back to 1949 only, short of the requirement that possession and occupation under a bona fide claim of ownership should
be since June 12, 1945 or earlier.

Respondent also offered the testimonies of Romualdo and Felino to prove that Virginia's predecessor-in-interest had been in possession
and occupation under a bona fide claim of ownership since June 12, 1945. Romualdo testified as follows: ChanRoblesVirtualawlibrary

Atty. Valdez

Q.
At the time you started to farm the property, please describe the condition thereof?
A. It was being farmed and planted to rice, sir.
Q.
Who planted it with rice?
A.
My father, Sixto Cuevas Flores, sir.

Q.
Since when did your father start tilling the land?
A.
He started tilling the land even before the Japanese time in 1942?

Q.
How do you know?
A.
I have already reached the age of reason at the time being nine (9) years old in 1941, sir.39
It needs to be pointed out, however, that in Republic vs. Remman Enterprises, Inc.40 (Remman), the Court held that for purposes of land
registration under Section 14(1) of P.D. No. 1529, proof of specific acts of ownership must be presented to substantiate the claim of open,
continuous, exclusive, and notorious possession and occupation of the land subject of the application. "Applicants for land registration
cannot just offer general statements which are mere conclusions of law rather than factual evidence of possession. Actual possession
consists in the manifestation of acts of dominion over it of such nature as a party would actually exercise over his own property."41

In a plethora of cases, the Court has repeatedly held that unsubstantiated claims of cultivation of land do not suffice to prove open,
continuous, exclusive, and notorious possession and occupation of the public land applied for in the concept of an owner. In Remman,
the Court denied the application for original registration of title to land located in Taguig City as the testimony of the applicant's witness
lacked specifics as to the nature of the alleged cultivation. It was observed that: ChanRoblesVirtualawlibrary

Although Cerquena testified that the respondent and its predecessors-in-interest cultivated the subject properties, by planting different
crops thereon, his testimony is bereft of any specificity as to the nature of such cultivation as to warrant the conclusion that they have
been indeed in possession and occupation of the subject properties in the manner required by law. There was no showing as to the
number of crops that are planted in the subject properties or to the volume of the produce harvested from the crops supposedly planted
thereon.42 (Underscoring supplied)

In Aranda vs. Republic of the Philippines,43 the Court held that mere statements regarding cultivation of land would not establish
possession in the concept of an owner, stating that: ChanRoblesVirtualawlibrary

X x x And even assuming that Lucio actually planted rice and corn on the land, such statement is not sufficient to establish possession in
the concept of owner as contemplated by law. Mere casual cultivation of the land does not amount to exclusive and notorious possession
that would give rise to ownership. Specific acts of dominion must be clearly shown by the applicant.44 (Underscoring supplied)

In Republic vs. Candy Maker, Inc.,45 the Court did not give credit to the unsupported claim of the respondent-applicant's predecessor-
in-interest that he and his father cultivated the property applied for since 1937 by planting palay during the rainy season and vegetables
during the dry season. The Court emphasized the importance of showing specific acts of dominion by the applicant or his predecessors-
in-interest, to wit: ChanRoblesVirtualawlibrary

Fourth. When he testified on October 5, 2001, Antonio Cruz declared that he was "74 years old." He must have been born in 1927, and
was thus merely 10 years old in 1937. It is incredible that, at that age, he was already cultivating the property with his fa ther. Moreover,
no evidence was presented to prove how many cavans of palay were planted on the property, as well as the extent of such cultivation, in
order to support the claim of possession with a bona fide claim of ownership. (Underscoring supplied)

Similarly in this case, assuming the veracity of the claim that Alejandro and/or Virginia cultivated the subject land through Romualdo and
Sixto, the Court finds that the same could only be considered as a mere casual cultivation because his testimony was bereft of any
specificity to warrant the conclusion that Alejandro and/or Virginia had been indeed in possession and occupation of the subject land.
Romualdo's statements failed to show the nature of the cultivation and the volume of crops planted and harvested on the property.
Respondent estate, therefore, failed to satisfy the requisite exclusivity and notoriety of the possession and occupation of the property
because exclusive dominion and conspicuous possession over the subject land were not established.

Felino's testimony during the new trial of this case was likewise insufficient to prove the required possession and occupation since June
12, 1945 or earlier. Felino's pertinent testimony in his judicial affidavit was as follows: ChanRoblesVirtualawlibrary

Atty. Valdez
Q.
Since when did you start tilling the property?
A.
In 1979 at the age of 17.

Q.
Before you, who cultivated the property, if any?
A.
Romualdo Flores, my father then as tenant of the owner.

Q.
Since when did Romualdo cultivate or till the property?
A.
Since 1969.

Q.
As tenant, up to when did your father till the property?
A.
Up to 1979 when I took over.

Q.
In 1969 when Romualdo took over, who was cultivating or tilling the property, if any?
A.
Sixto Flores, his father and my grandfather.
Q.
Since when did Sixto start to cultivate the property?
A.
Before the Second World War. Q. How do you know when you were born only in 1962?
A.
It is an accepted fact in our family history. I heard my parents and grandparents talk about it very, very often. Everyone assumes it to be
true. Besides during the days of my grandfather Sixto, there was not much source of livelihood of the people but the farm. Many people
worked or derived their income from the farms.
Clearly, Felino failed to convincingly show that he had personal knowledge of the ownership or possession over Lot No. 10839-C on or
before June 12, 1945 having been born only in 1962. He also talked of how his father and grandfather cultivated the land based on their
family stories which were not substantiated. Hence, the above testimony of Felino does not deserve any credit for being hearsay.
From all the foregoing, the subject land cannot be registered in the name of Virginia and/or her estate under Section 14(1) o f P.D. No.
1529 for respondent estates failure to prove its alienable and disposable character, and its possession and occupation from June 12,
1945 or earlier.

Respondent Failed to Comply with the


Requirements under Section 14(2) of
P.D. No. 1529

The subject land cannot also be registered under Section 14(2) of P.D. No. 1529, which states: ChanRoblesVirtualawlibrary

(2) Those who have acquired ownership of private lands by prescription under the provision of existing laws.
In Heirs of Mario Malabanan vs. Republic46 (Malabanan), the Court explained that when Section 14(2) of P.D. No. 1529 stated that
persons "who have acquired ownership over private lands by prescription under the provisions of existing laws," it unmistakably referred
to the Civil Code as a valid basis for the registration of lands. The Civil Code is the only existing law that specifically allows the acquisition
of private lands by prescription, including patrimonial property belonging to the State.

Section 14(2) explicitly refers to the principles on prescription, as set forth in the Civil Code. In this regard, the Civil Code makes it clear
that patrimonial property of the State may be acquired by private persons through prescription. This is brought about by Article 1113,
which provides that all things which are within the commerce of man are susceptible to prescription, and that property of the State or any
of its subdivisions not patrimonial in character shall not be the object of prescription.47

This does not necessarily mean, however, that when a piece of land is declared alienable and disposable, it can already be acquired by
prescription. In Malabanan, this Court ruled that declaration of alienability and disposability was not enough — there must be an express
declaration that the public dominion property was no longer intended for public service or the development of the national wealth or that
the property had been converted into patrimonial, thus: ChanRoblesVirtualawlibrary

(2) In complying with Section 14(2) of the Property Registration Decree, consider that under the Civil Code, prescription is recognized as
a mode of acquiring ownership of patrimonial property. However, public domain lands become only patrimonial property not only with a
declaration that these are alienable or disposable. There must also be an express government manifestation that the property is already
patrimonial or no longer retained for public service or the development of national wealth, under Article 422 of the Civil Code. And only
when the property has become patrimonial can the prescriptive period for the acquisition of property of the public dominion begin to run.48
(Underscoring supplied)

In this case, and as already stated, respondent estate merely relied on the annotation on the subdivision plan of Lot No. 10839 and on
the certification issued by FMS-DENR which certified the subject land to be "within the alienable and disposable land under Project No.
27-B, Taguig Cadastral Mapping as per LC Map No. 2623." No certification or any competent evidence, however, was ever presented to
the effect that the subject land, or even the lands covered by L.C. Map No. 2623, were no longer intended for public service or for the
development of the national wealth pursuant to Article 422 of the Civil Code. The classification of the subject property as alienable and
disposable land of the public domain does not change its status as property of the public dominion under Article 420(2) of the Civil Code.
Thus, it is insusceptible to acquisition by prescription. Hence, respondent estate failed to prove that acquisitive prescription had begun to
run against the State, much less that it had acquired title to the subject property by virtue thereof.

In fine, respondent failed to satisfy all the requisites for registration of title to land under either Sections 14(1) or (2) of P.D. No. 1529.
Respondent's application for original registration of imperfect title over Lot No. 10839-C must be denied.

Without Prejudice

This denial, however, is without prejudice. As the FMS-DENR certified the subject land to be "within the alienable and disposable land
under Project No. 27-B, Taguig Cadastral Mapping as per LC Map No. 2623," the respondent must be given the opportunity to present
the required evidence. This is but fair and reasonable because a property within an alienable and disposable land must be deemed to be
of the same status and condition. As earlier stated, however, the respondent must prove that the subject property was actually covered
by the same cadastral survey and that they and their predecessors in interest were in possession and ownership since June 12, 1945 or
earlier.

WHEREFORE, the petition is GRANTED.

REPUBLIC VS. HEIRS OF ESTACIO AND OCOL

factual antecedents are as follows:

chanRoblesvirtualLawlibraryOn September 19, 2008,5 respondents, Heirs of Spouses Tomasa Estacio and Eulalio Ocol filed with the
RTC of Pasig City, Branch 266 an application for land registration under Presidential Decree No. 1529 (PD 1529) otherwise known as
the Property Registration Decree. The application covers three (3) parcels of land described as follows: a) Lot 2 under approved survey
plan Ccs-00-000258 with an area of 3,731 square meters; b) Lot 1672-A under approved subdivision plan Csd-00-001798 consisting of
1,583 square meters; c) a lot under approved survey plan Cvn-00-000194 consisting of 6,066 square meters.6 The total assessed value
of the parcels of land is P288,970.007chanrobleslaw

On October 6, 2008, the RTC issued a Notice of Initial Hearing, copy furnished the Land Registration Authority (LRA). The notice was
sent to the Official Gazette for publication and was served on all the adjoining owners. It was likewise posted conspicuously on each
parcel of land included in the application.8 During the initial hearing on January 13, 2010, respondents, by counsel, present ed the
jurisdictional requirements (Exhibits "A" to "I" and their sub-markings). There being no private oppositor, an Order of General Default was
issued except against the Republic of the Philippines.

At the ex-parte presentation of evidence on January 22, 2010, respondents Rosa Ocol, 72 years old, and Felipe Ocol, 70 years old,
testified that they are the children of the late Tomasa Estacio and Eulalio Ocol (Exhibits "U" and "V"). They inherited the subject lots from
their father and mother who died on February 1, 1949 and March 22, 1999, respectively. When Felipe Ocol was only about eight years
old and Rosa was still in grade school, their parents developed and cultivated the subject lots as rice fields. In the 1940's, there were only
a few houses around their house. At present, one of the lots is residential while the two remaining lots have become idle. Their parents
and grandparents had been in continuous, actual and physical possession of the lots without any interruption for more than sixty five (65)
years. Felipe and Rosa have been in possession of the land for more than fifty (50) years. There is n0 existing mortgage or encumbrance
over the said lots.9chanrobleslaw

Respondents presented witness Antonia Marcelo who was 85 years old at the time she testified. She is the neighbor of Tomasa Estacio
and Eulalio Ocol in Barangay Calzada where she has been residing for more than fifty (50) years. She testified that during her childhood
days, she used to play on the subject lots and had seen the spouses Ocol cultivate the lots by planting vegetables, rice and
trees.10chanrobleslaw

In support of their application, respondents presented documentary evidence which sought to establish the following:

The first lot which is Lot 2 of the conv. Subd. plan Ccs-00-000258 with an area of 3,731 square meters was declared for taxation purposes
in the names of Tomasa Estacio and Eulalio Ocol in the years 1966, 1974, 1979, 1985, 2000 and 2002 (Exhibits "T" to "T-7");

The second lot which is Lot 1672-A under approved subdivision plan Csd-00-001798 consisting of 1,583 square meters was declared for
taxation purposes in the names of Tomasa Estacio and Eulalio Ocol in the years 1942, 1949, 1966, 1974, 1979, 1985, 1994, 2000 and
2002 (Exhibits "R" to "R-10");

The third lot which is a lot under approved survey plan CVN-00-000194 consisting of 6,066 square meters, being a conversion of Lot
1889, MCadm, 590-D Taguig Cadastral Mapping, was declared for taxation purposes in the names of Tomasa Estacio and Eulalio Ocol
in the years 1949, 1974, 1979, 1985, 2000 and 2002 (Exhibits "S" to "S-6");

The subject lots used to have larger areas but certain portions were taken and designated as legal easements. On December 17, 2009,
the real property tax on the subject lots, declared in the names of Tomasa Estacio and Eulalio Ocol as owners, were paid (Exhibits "Q",
"Q-1" and "Q-2");

The subject lots were surveyed for Tomasa Vda. de Ocol as evidenced by the Geodetic Engineers' Certificates and Conversion
Subdivision Plans (Exhibits "J", "K", "L", "P", "P-1", and "P-2");

The subject lots are verified to be within alienable and disposable land under Project No. 27-B Taguig Cadastral Mapping as per LC Map
No. 2623 approved on January 3, 1968 as evidenced by Certifications dated January 28, 2010 issued by the Department of Environment
and Natural Resources-National Capital Region (Exhibits "J-3, "K-2" and "L-3").11
On February 11, 2010, respondents formally offered their documentary evidence. The RTC set the case for presentation of evidence of
the government on April 16, 2010. On the date of the hearing, there was no appearance from the government. Hence, the court, upon
motion of applicants, considered the case submitted for resolution.

On August 12, 2010, the RTC issued an Order granting the respondents' application for registration of title to the subject properties,
viz.:ChanRoblesVirtualawlibrary
WHEREFORE, judgment is hereby rendered thus: the title of the heirs of Tomasa Estacio and Eulalio Ocol, namely, Rosa Ocol; and
Felipe Ocol, to the three (3) parcels of land above-described is hereby CONFIRMED.

Upon the finality of the judgment, let the proper Decree of Registration and Certificates of Title be issued to the applicants pursuant to
Section 39 of P.D. 1529.

Let two (2) copies of this Order be furnished the Land registration Authority Administrator Benedicta B. Ulep thru Salvador L. Oriel, the
Chief of the Docket Division of said Office, East Avenue, Quezon City.

SO ORDERED.12chanroblesvirtuallawlibrary
The RTC found that respondents were able to prove that their predecessors-in-interest possessed the subject lots from 1966 until 2002
with respect to the first lot; from 1942 to 2002, with respect to the second lot; and from 1949 to 2002 with respect to the third lot, as shown
in the tax declarations. The court posited that even if the subject lots were declared as alienable and disposable public land only on
January 3, 1968, respondents had already "acquired title to the land according to P.D. 1529" by virtue of the continued possession of the
respondents and their predecessors-in-interest from January 3, 1968 to the present.13chanrobleslaw

A motion for reconsideration was filed by the petitioner raising the following grounds:

chanRoblesvirtualLawlibrary
(a)
Respondents did not comply with the requirements in acquiring ownership of the subject lots by prescription because the few tax
declarations of respondents failed to substantiate the requirement of open, continuous, notorious and exclusive possession of the subject
lots for the required period as stated in the case of Wee vs. Republic;14
(b)
The evidence is insufficient to establish the nature of possession because the testimony of witness Antonia Marcelo with regard to the
cultivation of the subject properties by spouses Ocol does not convincingly prove possession and enjoyment of the subject lots to the
exclusion of other people;
(c)
There was no declaration, either in the form of a law or a presidential proclamation, showing that the lots are no longer intended for public
use or for the development of national wealth, or that it has been converted to patrimonial property as stated in the case of Heirs of
Malabanan v. Republic.15

The Motion for Reconsideration was denied by the RTC on February 15, 2011.

The RTC opined that the case of Wee vs. Republic16 is not applicable in the instant case because the parcels of land involved in the said
case are "unirrigated ricefields". In the instant case, the first and third lots are ricefields while the second lot is a residential one as shown
in the tax declarations. The RTC averred that, even prior to the dates stated in the tax declarations specifically during the 1940s, spouses
Tomasa and Eulalio Ocol had started planting rice on the first and third lots as testified to by respondents. The testimony was corroborated
by witness Antonia Marcelo, who is 15 years older than the respondents, when she testified that she played on the subject lots and had
seen the spouses Ocol cultivate the same by planting vegetables, rice and trees in the 1930s. As to the second lot, the RTC gave
credence to the testimony of respondents that in the 1940s, respondents' house was already erected on the said lot. According to the
court, such is proof that the lot has been used for residential purposes even prior to 1942 which is the earliest date of the tax declaration
on the lot.

The RTC further held that the case of Heirs of Malabanan vs. Republic17 does not apply in the case at bar because the said case involved
a 71,324-square-meter lot, while the subject lots have a total area of 11,380 square meters only. The court pointed out that respondents
are not just entitled to a grant of their application under Section 14(1) of PD 1529 but also under Section 14(2) of the same law because
respondents had proven that their predecessors-in-interest were in possession of the subject lands earlier than 1945. Thus, there is no
need for an express government manifestation that the property is patrimonial, or that such is no longer intended for public service or for
the development of national wealth.

Aggrieved, petitioner filed an appeal before the CA. In a Decision dated February 20, 2013, the CA affirmed the Decision of the RTC. The
fallo of the Decision states:ChanRoblesVirtualawlibrary
WHEREFORE, the instant appeal is DISMISSED, and the Order dated August 12, 2010, of the Regional Trial Court of Pasig City, Branch
266, in L.R.C. Case No. N-11598 (LRA Record No. N-79393) is AFFIRMED IN TOTO.

SO ORDERED.18chanroblesvirtuallawlibrary
In affirming the RTC Order, the CA made the following ratiocinations:ChanRoblesVirtualawlibrary
In the case at bar, the applicants-appellees seek the confirmation of their ownership to the subject lands not based on prescription, but
based on their claim that "they have been in open, continuous, exclusive and notorious possession and occupation of alienable and
disposable lands of the public domain under a bonafide claim of ownership since June 12, 1945, or earlier". (Section 14[1], PD 1529).
The requirement of prior declaration that the property is patrimonial property of the State, therefore, does not apply. As explained in Heirs
of Malabanan, for application based on Section 14(1) of the Property Registration Decree, it is enough that the property is alienable and
disposable property of the State and the applicant has been in open, continuous, exclusive, and notorious possession and occupation of
the subject land under a bonafide claim of ownership from June 12, 1945 or earlier. Both of these requirements are present in this
case.19chanroblesvirtuallawlibrary
A motion for reconsideration was filed by the petitioner but the same was denied by the CA on July 26, 2013.

Hence, this petition, raising the following errors:ChanRoblesVirtualawlibrary


THE RECORD IS BEREFT OF PROOF THAT THE SUBJECT PROPERTIES HAD BEEN CLASSIFIED AS ALIENABLE AND
DISPOSABLE;

THE RECORD IS BEREFT OF PROOF THAT RESPONDENTS HAVE BEEN IN OPEN, CONTINUOUS, EXCLUSIVE AND NOTORIOUS
POSSESSION Of THE SUBJECT LOTS UNDER A BONA FIDE CLAIM OF OWNERSHIP SINCE JUNE 12, 1945, OR EARLIER;

ALTERNATIVELY, RESPONDENTS CANNOT INVOKE PRESCRIPTION UNDER SECTION 14(2) OF PRESIDENTIAL DECREE NO.
1529. THE SUBJECT LOTS HAVE NOT BEEN CONVERTED INTO PATRIMONIAL PROPERTY OF THE STATE.20
On the first ground, petitioner states that respondents failed to present a copy of the original certification, approved by the DENR Secretary
and certified as a true copy by the legal custodian, which would support respondents' claim that the subject lands are alienable and
disposable. The certification of Senior Forest Management Specialist Corazon D. Calamno and Chief of the Forest Utilization and Law
Enforcement Division of the DENR should not be treated as sufficient compliance with the requirements of the law because she was not
presented during trial to testify on the contents of the certification.

On the second ground, petitioner argues that there is insufficient evidence of acts of dominion on the part of respondents and their
predecessors-in-interest for the following reasons:

chanRoblesvirtualLawlibrary
(a)
Respondents did not explain how the properties were acquired. The only explanation as to the acquisition of Lot 1672-A was that it was
first acquired from a certain Gregorio, without even mentioning the date of acquisition as well as any document evidencing the same.21
(b)
It was unusual for respondents' parents to possess and occupy three (3) parcels of land that are not contiguous to one another;
(c)
Respondents were able to present a tax receipt only for the year 2009;
(d)
In terms of improvements, respondents did not go to the extent of specifying whether fences were erected on the lots. While they claim
that crops were planted, it did not appear that they exclusively and continuously enjoyed the possession of the lots;
(e)
While respondents consistently affirm the development of the lots as ricefields, they failed to consider the fact that the second lot, Lot
1672-A, is a residential land as stated on the tax declaration of the land.

On the third ground, petitioner avers that respondents cannot invoke prescription under Section 14(2) of P.D. 1529 because they failed
to present the necessary documents which would show that the subject properties are no longer intended for public service or no longer
used for the development of the national wealth. They did not present a declaration in the form of a law or a Presidential Proclamation.

In their Comment,22 respondents counter that the certifications issued by the DENR constitute substantial compliance with the legal
requirement, and that with their continuous possession of the subject lots for more than thirty (30) years, they had acquired ownership
over the subject lots through prescription under Section 14(2) of P.D. 1529.

In Reply,23 petitioner maintains that respondents failed to establish their compliance with the requisites for original registration either
under Section 14 (1) or Section 14 (2) of P.D. No. 1529. The certifications of Senior Forest Management specialist Corazon C. Calamno
and the Chief of the Forest Utilization and Law Enforcement Division of the DENR did not comply with the legal requirements for lack of
approval by the DENR Secretary and for lack of certification by its legal custodian. Respondents failed to establish that the State expressly
declared, either through a law or a presidential proclamation, that the parcels ofland are no longer retained for public service or the
development of national wealth, or that they had been converted into patrimonial properties. Without such, the subject lots remain part of
public dominion.

Petitioner further maintains that the tax declarations do not represent regular assertion of ownership because of the large gaps in the
years between declarations. Such sporadic assertion of alleged ownership does not prove open, continuous, exclusive and notorious
possession and occupation in the concept of an owner. And that, since the parcels of land are not contiguous, alleged possession and
occupation over one parcel of land cannot prove possession and occupation over the other parcels of land.24chanrobleslaw

The petition is meritorious.

Under the Regalian Doctrine, which is embodied in our Constitution, all lands of the public domain belong to the State, which is the source
of any asserted right to any ownership of land. All lands not appearing to be clearly within private ownership are presumed to belong to
the State. Accordingly, public lands not shown to have been reclassified or released as alienable agricultural land, or alienated to a private
person by the State, remain part of the inalienable public; domain. The burden of proof in overcoming the presumption of State ownership
of the lands of the public domain is on the person applying for registration, who must prove that the land subject of the application is
alienable or disposable. To overcome this presumption, incontrovertible evidence must be presented to establish that the land subject of
the application is alienable or disposable.25cralawredchanrobleslaw

Section 14 (1) of PD 1529, otherwise known as the Property Registration Decree provides:ChanRoblesVirtualawlibrary
SEC. 14. Who may apply. - The following persons may file in the proper Court of First Instance an application for registration of title to
land, whether personally or through their duly authorized representatives:

chanRoblesvirtualLawlibrary(1) Those who by themselves or through their predecessors-in--interest have been in open, continuous,
exclusive and notorious possession and occupation of alienable and disposable lands of the public domain under a bona fide claim of
ownership since June 12, 1945, or earlier.

(2) Those who have acquired ownership of private lands by prescription under the provision of existing laws.

xxx
In the Order of the RTC granting the registration of the subject lots, it was stated that respondents had "acquired title to the land according
to P.D. 1529" by virtue of the continued possession of the respondents and their predecessors-in-interest from January 3, 1968 to present.
On motion for reconsideration, however, the court added that respondents are not just entitled to a grant of their application under Section
14(2) of the P.D. 1529, but also under Section 14(1) of the same law because respondents had proven that their predecessors-in-interest
were in possession of the subject lots earlier than 1945. The CA explained, however, that the confirmation of the ownership to the subject
lots is not based on prescription, but on Section 14 (1), since it was established that the lots are alienable and disposable, and the
applicants are in continuous possession thereof since June 12, 1945 or earlier.

To distinguish between registration under Section 14(1) of P.D. No. 1529 from the one filed under Section 14(2) of P.D. No. 1529, this
Court held in the case of Heirs of Mario Malabanan v. Republic:26
Section 14(1) mandates registration on the basis of possession, while Section 14(2) entitles registration on the basis of prescription.
Registration under Section 14(1) is extended under the aegis of the Property Registration Decree and the Public Land Act while
registration under Section 14(2) is made available both by the Property Registration Decree and the Civil
Code.27chanroblesvirtuallawlibrary
Registration under Section 14(1) of P.D. No. 1529 is based on possession and occupation of the alienable and disposable land of the
public domain since June 12, 1945 or earlier, without regard to whether the land was susceptible to private ownership at that time. The
applicant needs only to show that the land had already been declared alienable and disposable at any time prior to the filing of the
application for registration.28chanrobleslaw

On the other hand, registration under Section 14(2) of P.D. No. 1529 is based on acquisitive prescription and must comply with the law
on prescription as provided by the Civil Code. In that regard, only the patrimonial property of the State may be acquired by prescription
pursuant to the Civil Code. For acquisitive prescription to set in, therefore, the land being possessed and occupied must already be
classified or declared as patrimonial property of the State. Otherwise, no length of possession would vest any right in the possessor ifthe
property has remained land of the public dominion.29chanrobleslaw

Moreover, Section 14(1) of P.D. No. 1529 refers to the judicial confirmation of imperfect or incomplete titles to public land acquired under
Section 48(b) of Commonwealth Act No. 141, or the Public Land Act, as amended by P.D. No. 1073.30 Under Section 14(1), respondents
need to prove that: (1) the land forms part of the alienable and disposable land of the public domain; and (2) they, by themselves or
through their predecessors-in-interest, have been in open, continuous, exclusive, and notorious possession and occupation of the subject
land under a bona fide claim of ownership from June 12, 1945 or earlier. These the respondents must prove by no less than clear, positive
and convincing evidence.31chanrobleslaw

In the case at bar, the first requirement was not satisfied. To prove that the subject property forms part of the alienable and disposable
lands of the public domain, the respondents presented three certifications - two are dated January 29, 2010 (Exhibits "J-3" and "K-2")
and one is dated January 28, 2010 (Exhibits "L-3") - issued by Senior Forest Management Specialist Corazon D. Calamno and Chief of
the Forest Utilization and Law Enforcement Division of the DENR-National Capital Region.32 The certification attests that the lots are
verified to be within alienable and disposable land under Project No. 27-B Taguig Cadastral Mapping as per LC Map No. 2623 approved
on January 3, 1968, thus:ChanRoblesVirtualawlibrary
This is to certify that the tract of land as shown and described at the reverse side hereof xxx as surveyed by Geodetic Engineer Jose S.
Agres, Jr. for Tomasa Vda de Ocol is verified to be within the Alienable and Disposable Land, under Project No. 27-B of Taguig City as
per LC Map 2623, approved on January 3, 1968.33chanroblesvirtuallawlibrary
However, the certifications presented by the respondents are insufficient to prove that the subject properties are alienable and disposable.
We reiterate the standing doctrine that land of the public domain, to be the subject of appropriation, must be declared alienable and
disposable either by the President or the Secretary of the DENR. Applicants must present a copy of the original classification approved
by the DENR Secretary and certified as true copy by the legal custodian of the records. In Republic of the Philippines v. T.A.N. Properties,
Inc.,34 this Court explicitly ruled:ChanRoblesVirtualawlibrary
Further, it is not enough for the PENRO or CENRO35 to certify that a land is alienable and disposable. The applicant for land registration
must prove that the DENR Secretary had approved the land classification and released the land of the public domain as alienable and
disposable, and that the land subject of the application for registration falls within the approved area per verification through survey by
the PENRO or CENRO. In addition, the applicant for land registration must present a copy of the original classification approved by the
DENR Secretary and certified as a true copy by the legal custodian of the official records. These facts must be established to prove that
the land is alienable and disposable. Respondent failed to do so because the certifications presented by respondent do not, by
themselves, prove that the land is alienable and disposable.36chanroblesvirtuallawlibrary
In Republic v. Bantigue Point Development Corporation,37 this Court deemed it appropriate to reiterate the ruling in T.A.N. Properties,
viz.:ChanRoblesVirtualawlibrary
The Regalian doctrine dictates that all lands of the public domain belong to the State. The applicant for land registration has the burden
of overcoming the presumption of State ownership by establishing through incontrovertible evidence that the land sought to be registered
is alienable or disposable based on a positive act of the government. We held in Republic v. T.A.N. Properties, Inc. that a CENRO
certification is insufficient to prove the alienable and disposable character of the land sought to be registered. The applicant must also
show sufficient proof that the DENR Secretary has approved the land classification and released the land in question as alienable and
disposable.

Thus, the present rule is that an application for original registration must be accompanied by (1) a CENRO or PENRO Certification; and
(2) a copy of the original classification approved by the DENR Secretary and certified as a true copy by the legal custodian of the official
records.

Here, respondent Corporation only presented a CENRO certification in support of its application. Clearly, this falls short of the
requirements for original registration.38chanroblesvirtuallawlibrary
Similarly, in Republic v. Cortez,39 this Court declared that:ChanRoblesVirtualawlibrary
xxx. To prove that the subject property forms part of the alienable and disposable lands of the public domain, Cortez adduced in evidence
a survey plan Csd-00-000633 (conversion-subdivision plan of Lot 2697, MCadm 594-D, Pateros Cadastral Mapping) prepared by
Geodetic Engineer Oscar B. Fernandez and dertified by the Lands Management Bureau of the DENR. The said survey plan contained
the following annotation:ChanRoblesVirtualawlibrary
This survey is inside L.C. Map No. 2623, Project No. 29, classified as cuienable & disposable by the Bureau of Forest Development on
Jan. 3, 1968.
However, Cortez' reliance on the foregoing annotation in the survey plan is amiss; it ciloes not constitute incontrovertible evidence to
overcome the presumption that the subject property remains part of the inalienable public domain. In Republic of the Philippines v. Tri-
Plus Corporation,40 the Court clarified that, the applicant must at the very least submit a certification froi:n the proper government agency
stating that the parcel of land subject of he application for registration is indeed alienable and disposable, viz.:ChanRoblesVirtualawlibrary
It must be stressed that incontrovertible evidence must be presented to establish that the land subject of the application is alie table or
disposable.
In the present case, the only evidence to prove the character of the subject lands as required by law is the notation appearinin the Advance
Plan stating in effect that the said properties are alienable and disposable. However, this is hardly the kind of proof required by law. To
prove that the land subject of an application for registration is alienable, an appficant must establish the existence of a positive act of the
government such as a presidential proclamation or an executive order, an administrative action, investigation reports of Bureau of Lands
investigators, andla legislative act or statute. The applicant may also secure certification from the Government that the lands applied: for
are alienable and disposable. In the case at bar, while the Advance Plan bearing the notation was certified by the Lands Management
Services of the DENR, the cert fication refers only to the technical correctness of the survey plotted in the said plan and has nothing to
do wh tsoever with the nature and character of the property surveyed. Respondents failed to submit a certification fromithe proper
government agency to prove that the lands subject for registration are indeed alienable and disposable.41chanroblesvirtuallawlibrary
Clearly, the aforestated doctrine unavoidably means that the mere certification issued by the DENR does not suffice to support the
application for registration, because the applicant must also submit a copy of the original classification of the land as alienable and
disposable as approved by the DENR Secretary and certified as a true copy by the legal custodian of the official records.42chanrobleslaw

Hence, in the instant case, the DENR certifications that were presented by the respondents in support of their application for registration
are not sufficient to prove that the subject properties are indeed classified by the DENR Secretary as alienable and disposable. It is still
imperative for the respondents to present a copy of the original classification approved by the DENR Secretary, which must be certified
by the legal custodian thereof as a true copy. Accordingly, the lower courts erred in granting the application for registration in spite o£ the
failure of the respondents to prove by well-nigh incontrovertible evidence that the subject properties are alienable and
disposable.43chanrobleslaw

Anent the second requirement, the tax declarations do not prove respondents' assertion. Although respondents claim that they possessed
the subject lots through their predecessors-in-interest since the 1930s, their tax declarations belie the same. The earliest tax declarations
presented for the first lot was issued only in 1966, while the earliest tax declaration for the third lot was issued in 1949.

If it is true that the parents of respondents had been in possession of the properties in the 1930s as testified to by witness Antonia Marcelo,
why was the first lot declared for taxation purposes for the first time only in 1966, and the third lot was declared only in 1949? While
belated declaration of a property for taxation purposes does not necessarily negate the fact of possession, tax declarations or realty tax
payments of property are, nevertheless, good indicia of possession in the concept of an owner, for no one in his right mind would be
paying taxes for a property that is not in his actual or, at least, constructive possession.44chanrobleslaw

That the subject properties were first declared for taxation purposes only in those mentioned years gives rise to the presumption that the
respondents claimed ownership or possession of the subject properties starting in the year 1966 only with respect to the first lot; and year
1949, with respect to the third lot.45 The voluntary declaration of a piece of property for taxation purposes not only manifests one's sincere
and honest desire to obtain title to the property, but also announces an adverse claim against the State and all other interested parties
with an intention to contribute needed revenues to the government. Such an act strengthens ones bona fide claim of acquisition of
ownership.46chanrobleslaw

Likewise, this Court notes that the tax declarations on the subject properties presented by the respondents were only for the years 1966,
1974, 1979, 1985, 2000 and 2002 with respect to the first lot (Lot 2 of the conv. Subd. plan Ccs-00-000258 with an area of 3,731 square
meters); for the years 1942, 1949, 1966, 1974, 1979, 1985, 1994, 2000 and 2002 with respect to the second lot (Lot 1672-A under
approved subdivision plan Csd-00-001798 consisting of 1,583 square meters); for the years 1949, 1974, 1979, 1985,2000 and 2002 with
respect to the third lot (a lot under approved survey plan CVN-00-000194 consisting of 6,066 square meters being a conversion of Lot
1889, MCadm, 590-D Taguig Cadastral Mapping).

Thus, there are only six tax declarations for the first lot, nine tax declarations for the second lot and five tax declarations for the third lot
within the alleged actual and physical possession of the lands without any interruption for more than sixty five (65) years. In Wee v.
Republic of the Philippines,47 this Court stated that:ChanRoblesVirtualawlibrary
It bears stressing that petitioner presented only five tax declarations (for the years 1957, 1961, 1967, 1980 and 1985) for a claimed
possession and occupation of more than 45 years (1945-1993). This type of intermittent and sporadic assertion of alleged ownership
does not prove open, continuous, exclusive and notorious possession and occupation. In any event, in the absence of other competent
evidence, tax declarations do not conclusively establish either possession or declarant's right to registration of
title.48chanroblesvirtuallawlibrary
Moreover, this Court emphasizes that respondents paid the taxes due on the parcels of land subject of the application only in 2009, a
year after the filing of the application. There is no showing of any tax payments before 2009. This Court held in the case of Tan, et al. vs.
Republic:49
Tax declarations per se do not qualify as competent evidence of actual possession for purposes of prescription. More so, if the payment
of the taxes due on the property is episodic, irregular and random such as in this case. Indeed, how can the petitioners claim of possession
for the entire prescriptive period be ascribed any ounce of credibility when taxes were paid only on eleven (11) occasions within the 40-
year period from 1961 to 2001?50chanroblesvirtuallawlibrary
From the foregoing, this Court doubts the respondents' claim that their predecessors-in-interest have been in continuous, exclusive, and
adverse possession and occupation thereof in the concept of owners from June 12, 1945, or earlier. The evidence presented by the
respondents does not prove title thru possession and occupation of public land under Section 14(1) of P.D. 1529.

Further, the RTC ruled that with the continuous possession of the subject lots for more than 30 years, respondents had acquired ownership
over the subject lots through prescription under Section 14(2) of P.D. 529. This view was adopted by the respondents in their Comment,51
to the petition.

An application for original registration of land of the public domain under Section 14(2) of Presidential Decree (PD) No. 1529 must show
not only that the land has previously been declared alienable and disposable, but also that the land has been declared patrimonial property
of the State at the onset of the 30-year or 10-year period of possession and occupation required under the law on acquisitive
prescription.52chanrobleslaw

It was elucidated in Heirs of Malabanan53 that possession and occupation of an alienable and disposable public land for the periods
provided under the Civil Code will not convert it to patrimonial or private property. There must be an express declaration that the property
is no longer intended for public service or the development of national wealth. In the absence thereof, the property remains to be alienable
and disposable and may not be acquired by prescription under Section 14(2) of P.D. No. 1529.

This Court, therefore, stresses that there must be an official declaration by the State that the public dominion property is no longer
intended for public use, public service, or for the development of national wealth before it can be acquired by prescription; that a mere
declaration by government officials that a land of the public domain is already alienable and disposable would not suffice for purposes of
registration under Section 14(2) of P.D. No. 1529. The period of acquisitive prescription would only begin to run from the time that the
State officially declares that the public dominion property is no longer intended for public use, public service, or for the development of
national wealth54.

In Republic v. Rizalvo, Jr.,55 this Court reiterated the ruling in Malabanan, viz.:ChanRoblesVirtualawlibrary
On this basis, respondent would have been eligible for application for registration because his claim of ownership and possession over
the subject property even exceeds thirty (30) years. However, it is jurisprudentially clear that the thirty (30)-year period of prescription for
purposes of acqmring ownership and registration of public land under Section 14 (2) of P.D. No. 1529 only begins from the moment the
State expressly declares that the public dominion property is no longer intended for public service or the development of the national
wealth or that the property has been converted into patrimonial. xxx
In this case, there is no evidence showing that the parcels of land in question were within an area expressly declared by law either to be
the patrimonial property of the State, or to be no longer intended for public service or the development of the national wealth.

Evidently, there being no compliance, with either the first or second paragraph of Section 14 of PD 1529, the Regalian presumption stands
and must be enforced in this case.

WHEREFORE, the petition is GRANTED.

MARAVILLA VS. TUPAS

FACTS:

The facts follow.

According to respondent, he, along with the other heirs of the late Asiclo S. Tupas, has maintained the occupation and possession of
certain portions of the property subject of this case. Thereafter, the late Zosimo Maravilla claimed ownership over 10,000 square meters
of said property by virtue of a Deed of Sale dated February 8, 1975, purportedly executed between him and the late Asiclo S. Tupas. The
property situated in Diniwid, Barangay Balabag, Malay, Aklan, is more particularly described as follows:

A parcel of land situated at Barangay Balabag, Malay, Aklan bounded on the North by Gil Aguirre, F. Flores; South by Antonio Tupas &
T. Sacapaño, East by Asicio (sic) Tupas, and West by Seashore L. Villanueva of approximately 1,000 hectares, assessed at P2,610.00
under Tax Declaration No. 1304, in the name of Maravilla, Ozosimo A. for the year of 1985.

Maravilla filed a case for quieting of title with recovery of possession and damages before Branch 9 of the RTC of Kalibo, Aklan, docketed
as Civil Case No. 4338. The dispositive portion of the Decision[4] reads:

WHEREFORE, decision is hereby rendered as follows:

1. Declaring the deed of sale (Exhs. A & 1) executed by Asiclo Tupas in favor of plaintiff Zosimo Maravilla over one-half (½) portion or
about 5,000 sq. m. of the conjugal property of the former as legal and valid;

2. Ordering that the portion sold be delineated from the shoreline with a length of at least 28 m. long from the southwestern direction
traversing in a straight line towards northeastern part between points 5-6 embracing an area of about 5,000 sq. m., depicted in Exh. G,
interpreted in relation to amended commissioner's report and sketch plan, dated August 25, 1992 (Exh. L) across Lots B and A; with the
northern portion of 5,000 sq. m. awarded to the defendants and the southern portion of 5,000 sq. m. to plaintiff; Defendants' cottages that
may be found in plaintiff's one- half portion shall be removed by the former at their expense within 30 days from the finality of this decision.
The existing muniments of the parties to the land in question like tax declarations, certificates of title, and other related documents are
ordered modified or corrected to conform to this decision;

3. Defendants are ordered jointly and severally, to refund plaintiff the amount of seven thousand pesos (P7,000.00), Philippine currency,
representing the consideration of the ½ portion of the land in question herein awarded to them; and

4. Plaintiff is ordered to pay defendants for attorney's fees and litigation expenses in the sum of ten thousand pesos (P10,000.00) and
the costs of the suit.

SO ORDERED.[5]
Maravilla filed an appeal with the CA questioning the RTC's decision that he is only entitled to ½ of the area sold even if the validity of the
deed of sale was upheld. The CA, in a Decision[6] dated August 28, 1996, ruled that:

WHEREFORE, the Decision of the court a quo is SET ASIDE and another judgment is issued declaring Zosimo Maravilla the owner of
10,000 sq. m. undivided share in the 36,382 sq. m. parcel of land of Asiclo S. Tupas and Francisca Aguirre and directing that this land
be partitioned, either extra-judicially or judicially, and that Maravilla's portion of the property be determined; and ordering the defendants
to turn over possession of the portion allocated to Maravilla.

Special Proceedings No. 39517 is DISMISSED.

No pronouncement as to costs.

SO ORDERED.[7]

On October 21, 1999, Maravilla filed another case for partition and damages before the RTC of Kalibo, Aklan, Branch 6, and on March
31, 2003, it disposed of the case as follows:[8]

WHEREFORE, judgment is hereby rendered containing that the one-hectare portion in the Sketch Plan [Annex B-1; Complaint] is the
rightful share of the plaintiff.

Defendants are ordered to restore possession thereof to the plaintiff, and to pay jointly and severally the latter the agreed monthly
reasonable compensation for the use and occupation thereof of P5,000.00 starting in 1990 until possession is fully restored to plaintiff.

Costs against the defendants.

SO ORDERED.[9]

Respondent appealed the decision with the CA, and in a Decision[10] dated April 13, 2007, the latter dismissed the appeal on the ground
of res judicata. The CA opined that the first case, the one for quieting of title and the second case for partition, both presented identity of
facts and evidence and that the truth of the matter is, part of the judgment of the first case ordered for partition of the subject parcel of
land to delimit the portion owned by herein petitioner.

On October 31, 2008, Maravilla filed a Motion for Execution[11] of the March 31, 2003 Decision of the RTC-Branch 6 of Kalibo, Aklan.

While the motion for execution was pending before the RTC-Branch 6 of Kalibo, Aklan, this Court, on October 8, 2008, declared Boracay
as government property in the consolidated cases of The Secretary of the Department of Environment and Natural Resources (DENR),
et al. v. Yap, et al. and Sacay, et al. v. the Secretary of the DENR, et al. (Boracay Decision)[12]

On February 2, 2009, a Resolution was issued by the RTC granting the motion for execution.

Respondent filed a motion for reconsideration, but the RTC denied the same in an Order dated April 7, 2009.

Thus, respondent filed a petition for certiorari with the CA assailing the Resolution and the Order issued by the RTC. Respondent raised
as an issue that the grant of the motion for execution is not in accordance with this Court's decision in The Secretary of the Department
of Environment and Natural Resources (DENR), et al. v. Yap, et al. and Sacay, et al. v. the Secretary of the DENR, et al., a supervening
event, and that the RTC erred in not declaring as null and void the deed of sale of unregistered land considering that Boracay has been
classified as an inalienable land. The CA granted the petition, thus:

Withal, the Petition is hereby GRANTED. The assailed Orders dated February 2, 2009 and April 7, 2009, respectively, issued by public
respondent are hereby declared NULL and VOID and SET ASIDE.

SO ORDERED.[13]

Maravilla's motion for reconsideration was denied in a Resolution dated March 17, 2010, hence, the present petition.[14]

Petitioners (the heirs of Maravilla) raise the following grounds:

In rendering the assailed Decision and Resolution, petitioners most humbly submit that the Court of Appeals gravely erred in making the
following legal conclusions that warrants the power of review and supervision by the Honorable Supreme Court:

I. The Court of Appeals so far departed from the accepted and usual course of judicial proceedings when it set aside the Orders of the
Regional Trial Court granting execution of the 31 March 2003 Decision of the Regional Trial Court in relation to the 28 August 1996
[Decision] of the Court of Appeals, both of which judgments have long become final and executory.

II. The Court of Appeals' finding that the Boracay Decision is a supervening event that prevents the trial court from implementing the writ
of execution is not in accord with the applicable decisions of this Honorable Supreme Court. The Court of Appeals erred in finding that:

a. the Boracay Decision had a direct effect on the issue litigated and settled with finality between the parties, and substantially changed
the rights and relations between the parties;
b. with the declaration of Boracay as state-owned, the claim of herein petitioners of rights to the Property is already without basis;

c. to allow execution of the judgment would be to give undue advantage to herein petitioners and would be a miscarriage of justice.[15]

They also bring up the following arguments:

I. Petitioners are entitled as a matter of right to the execution of the judgments that have long become final and executory.

II. The pronouncement of the Supreme Court in the Boracay Decision is not a supervening event:

A. The settled dispute between the parties as to who has the better right to the Property is distinct and separate from the issue of titling
sought in the Boracay Decision;

B. The Boracay Decision does not substantially change the rights and relations between the petitioners and respondent that were already
decided by the courts with finality;

C. Notwithstanding the Boracay Decision, it is still possible to execute the decision regarding the partition and restoration of the possession
of Property in favor of petitioners as against respondent;

III. The Boracay Decision does not render the execution sought by [the] petition as unjust or inequitable that precludes the execution of
the final and executory judgments.[16]

Petitioners insist that the CA's Decision dated August 28, 1996 in the original case for Quieting of Title with Recovery of Possession and
Damages entitled petitioners to the restoration of their possession of the property consisting of 10,000 sq. m. out of the 36,382 sq. m.
tract of land, after the validity of the sale to Maravilla by respondent's predecessor has been upheld by the court with finality. They further
claim that it is well entrenched in Our rules and jurisprudence that the prevailing party may move for the execution of a decision that has
become final and executory as a matter of right and the issuance of the writ of execution becomes a ministerial duty of the court.

The pronouncement in the Boracay Decision, according to petitioners, is not a supervening event. The Boracay Decision is simply a
recognition of the right of the State to classify the island and to pave the way for the eventual titling or formalization of ownership claims
of lands classified as alienable and disposable, and as to whether or not petitioners may secure title to the property is an issue that has
not yet ripened into a legal controversy between petitioners and the State. Petitioners argue that the settled dispute between the parties
as to who has the better right to the property is distinct and separate from the issue of titling sought in the Boracay Decision by the
claimants therein.

Furthermore, petitioners do not contest the legal status of the land; what they assert is the satisfaction of their right to enjoy whatever
imperfect rights that their predecessors had validly acquired from respondent's predecessor, as confirmed with finality by the courts.

The petition lacks merit.

The basic issue to be resolved is whether or not this Court's decision in The Secretary of the Department of Environment and Natural
Resources (DENR), et al. v. Yap, et al. and Sacay, et al. v. the Secretary of the DENR, et al. can be considered as supervening event
and if so, whether or not such supervening event can prevent the execution of a judgment that has already attained finality.

In the present case, petitioners' basis of their claim over the subject property is the Deed of Sale of Unregistered Land that the late Zosimo
Maravilla executed with the late Asiclo S. Tupas. This Deed of Sale has been acknowledged and adjudged by the RTC to be binding
between the parties, and in fact, has attained finality. This Court, however, in The Secretary of the Department of Environment and Natural
Resources (DENR), et al. v. Yap, et al. and Sacay, et al. v. the Secretary of the DENR, et al., ruled that the entire island of Boracay as
state-owned except for lands already covered by existing titles. To have a clearer view of the antecedents of the said case, the following
are thus quoted:

On April 14, 1976, the Department of Environment and Natural Resources (DENR) approved the National Reservation Survey of Boracay
Island, which identified several lots as being occupied or claimed by named persons.

On November 10, 1978, then President Ferdinand Marcos issued Proclamation No. 1801 declaring Boracay Island, among other islands,
caves and peninsulas in the Philippines, as tourist zones and marine reserves under the administration of the Philippine Tourism Authority
(PTA). President Marcos later approved the issuance of PTA Circular 3-82 dated September 3, 1982, to implement Proclamation No.
1801.

Claiming that Proclamation No. 1801 and PTA Circular No. 3-82 precluded them from filing an application for judicial confirmation of
imperfect title or survey of land for titling purposes, respondents-claimants Mayor Jose S. Yap, Jr., Libertad Talapian, Mila Y. Sumndad,
and Aniceto Yap filed a petition for declaratory relief with the RTC in Kalibo, Aklan.

In their petition, respondents-claimants alleged that Proclamation No. 1801 and PTA Circular No. 3-82 raised doubts on their right to
secure titles over their occupied lands. They declared that they themselves, or through their predecessors-in-interest, had been in open,
continuous, exclusive, and notorious possession and occupation in Boracay since June 12, 1945, or earlier since time immemorial. They
declared their lands for tax purposes and paid realty taxes on them.
Respondents-claimants posited that Proclamation No. 1801 and its implementing Circular did not place Boracay beyond the commerce
of man. Since the Island was classified as a tourist zone, it was susceptible of private ownership. Under Section 48 (b) of Commonwealth
Act (CA) No. 141, otherwise known as the Public Land Act, they had the right to have the lots registered in their names through judicial
confirmation of imperfect titles.

The Republic, through the Office of the Solicitor General (OSG), opposed the petition for declaratory relief. The OSG countered that
Boracay Island was an unclassified land of the public domain. It formed part of the mass of lands classified as "public forest", which was
not available for disposition pursuant to Section 3 (a) of Presidential Decree (PD) No. 705 or the Revised Forestry Code, as amended.

The OSG maintained that respondents-claimants' reliance on PD No. 1801 and PTA Circular No. 3-82 was misplaced. Their right to
judicial confirmation of title was governed by CA No. 141 and PD No. 705. Since Boracay Island had not been classified as alienable and
disposable, whatever possession they had cannot ripen into ownership.

During pre-trial, respondents-claimants and the OSG stipulated on the following facts: (1) respondents-claimants were presently in
possession of parcels of land in Boracay Island; (2) these parcels of land were planted with coconut trees and other natural growing trees;
(3) the coconut trees had heights of more or less twenty (20) meters and were planted more or less fifty (50) years ago; and (4)
respondents-claimants declared the land they were occupying for tax purposes.

The parties also agreed that the principal issue for resolution was purely legal: whether Proclamation No. 1801 posed any legal hindrance
or impediment to the titling of the lands in Boracay. They decided to forego with the trial and to submit the case for resolution upon
submission of their respective memoranda.

The RTC took judicial notice that certain parcels of land in Boracay Island, more particularly Lots 1 and 30, Plan PSU-5344, were covered
by Original Certificate of Title No. 19502 (RO 2222) in the name of the Heirs of Ciriaco S. Tirol. These lots were involved in Civil Case
Nos. 5222 and 5262 filed before the RTC of Kalibo, Aklan. The titles were issued on August 7, 1933.

RTC and CA Dispositions

On July 14, 1999, the RTC rendered a decision in favor of respondents-claimants, with a fallo reading:

WHEREFORE, in view of the foregoing, the Court declares that Proclamation No. 1801 and PTA Circular No. 3-82 pose no legal obstacle
to the petitioners and those similarly situated to acquire title to their lands in Boracay, in accordance with the applicable laws and in the
manner prescribed therein; and to have their lands surveyed and approved by respondent Regional Technical Director of Lands as the
approved survey does not in itself constitute a title to the land.

SO ORDERED.

The RTC upheld respondents-claimants' right to have their occupied lands titled in their name. It ruled that neither Proclamation No. 1801
nor PTA Circular No. 3-82 mentioned that lands in Boracay were inalienable or could not be the subject of disposition. The Circular itself
recognized private ownership of lands. The trial court cited Sections 87 and 53 of the Public Land Act as basis for acknowledging private
ownership of lands in Boracay and that only those forested areas in public lands were declared as part of the forest reserve.

The OSG moved for reconsideration, but its motion was denied. The Republic then appealed to the CA.

On December 9, 2004, the appellate court affirmed in toto the RTC decision, disposing as follows:

WHEREFORE, in view of the foregoing premises, judgment is hereby rendered by us DENYING the appeal filed in this case and
AFFIRMING the decision of the lower court.

The CA held that respondents-claimants could not be prejudiced by a declaration that the lands they occupied since time immemorial
were part of a forest reserve.

Again, the OSG sought reconsideration but it was similarly denied. Hence, the present petition under Rule 45.

G.R. No. 173775

On May 22, 2006, during the pendency of G.R. No. 167707, President Gloria Macapagal-Arroyo issued Proclamation No. 1064 classifying
Boracay Island into four hundred (400) hectares of reserved forest land (protection purposes) and six hundred twenty-eight and 96/100
(628.96) hectares of agricultural land (alienable and disposable). The Proclamation likewise provided for a fifteen-meter buffer zone on
each side of the centerline of roads and trails, reserved for right-of-way and which shall form part of the area reserved for forest land
protection purposes.

On August 10, 2006, petitioners-claimants Dr. Orlando Sacay, Wilfredo Gelito, and other landowners in Boracay filed with this Court an
original petition for prohibition, mandamus, and nullification of Proclamation No. 1064. They alleged that the Proclamation infringed on
their "prior vested rights" over portions of Boracay. They have been in continued possession of their respective lots in Boracay since time
immemorial. They have also invested billions of pesos in developing their lands and building internationally-renowned first class resorts
on their lots.
Petitioners-claimants contended that there is no need for a proclamation reclassifying Boracay into agricultural land. Being classified as
neither mineral nor timber land, the island is deemed agricultural pursuant to the Philippine Bill of 1902 and Act No. 926, known as the
first Public Land Act. Thus, their possession in the concept of owner for the required period entitled them to judicial confirmation of
imperfect title.

Opposing the petition, the OSG argued that petitioners-claimants do not have a vested right over their occupied portions in the island.
Boracay is an unclassified public forest land pursuant to Section 3 (a) of PD No. 705. Being public forest, the claimed portions of the
island are inalienable and cannot be the subject of judicial confirmation of imperfect title. It is only the executive department, not the
courts, which has authority to reclassify lands of the public domain into alienable and disposable lands. There is a need for a positive
government act in order to release the lots for disposition.

On November 21, 2006, this Court ordered the consolidation of the two petitions as they principally involve the same issues on the land
classification of Boracay Island.[17]

The consolidated petitions basically raise the issue of whether or not private individuals may acquire vested right of ownership over the
island, considering that they have been in open and continued possession for several years. With such factual antecedents, this Court
adjudicated that Boracay is classified as a public land, in particular, a forest land, thus:

Except for lands already covered by existing titles, Boracay was an unclassified land of the public domain prior to Proclamation No. 1064.
Such unclassified lands are considered public forest under PD No. 705. The DENR 109 and the National Mapping and Resource
Information Authority certify that Boracay Island is an unclassified land of the public domain.

PD No. 705 issued by President Marcos categorized all unclassified lands of the public domain as public forest. Section 3 (a) of PD No.
705 defines a public forest as "a mass of lands of the public domain which has not been the subject of the present system of classification
for the determination of which lands are needed for forest purpose and which are not". Applying PD No. 705, all unclassified lands,
including those in Boracay Island, are ipso facto considered public forests. PD No. 705, however, respects titles already existing prior to
its effectivity.

The Court notes that the classification of Boracay as a forest land under PD No. 705 may seem to be out of touch with the present realities
in the island. Boracay, no doubt, has been partly stripped of its forest cover to pave the way for commercial developments. As a premier
tourist destination for local and foreign tourists, Boracay appears more of a commercial island resort, rather than a forest land.

Nevertheless, that the occupants of Boracay have built multi-million peso beach resorts on the island; that the island has already been
stripped of its forest cover; or that the implementation of Proclamation No. 1064 will destroy the island's tourism industry, do not negate
its character as public forest.

Forests, in the context of both the Public Land Act and the Constitution classifying lands of the public domain into "agricultural, forest or
timber, mineral lands, and national parks", do not necessarily refer to large tracts of wooded land or expanses covered by dense growths
of trees and underbrushes. The discussion in Heirs of Amunategui v. Director of Forestry is particularly instructive:

A forested area classified as forest land of the public domain does not lose such classification simply because loggers or settlers may
have stripped it of its forest cover. Parcels of land classified as forest land may actually be covered with grass or planted to crops by
kaingin cultivators or other farmers. "Forest lands" do not have to be on mountains or in out of the way places. Swampy areas covered
by mangrove trees, nipa palms, and other trees growing in brackish or sea water may also be classified as forest land. The classification
is descriptive of its legal nature or status and does not have to be descriptive of what the land actually looks like. Unless and until the
land classified as "forest" is released in an official proclamation to that effect so that it may form part of the disposable agricultural lands
of the public domain, the rules on confirmation of imperfect title do not apply.

There is a big difference between "forest" as defined in a dictionary and "forest or timber land" as a classification of lands of the public
domain as appearing in our statutes. One is descriptive of what appears on the land while the other is a legal status, a classification for
legal purposes. At any rate, the Court is tasked to determine the legal status of Boracay Island, and not look into its physical layout.
Hence, even if its forest cover has been replaced by beach resorts, restaurants and other commercial establishments, it has not been
automatically converted from public forest to alienable agricultural land.

Private claimants cannot rely on Proclamation No. 1801 as basis for judicial confirmation of imperfect title. The proclamation did not
convert Boracay into an agricultural land. However, private claimants argue that Proclamation No. 1801 issued by then President Marcos
in 1978 entitles them to judicial confirmation of imperfect title. The Proclamation classified Boracay, among other islands, as a tourist
zone. Private claimants assert that, as a tourist spot, the island is susceptible of private ownership.

Proclamation No. 1801 or PTA Circular No. 3-82 did not convert the whole of Boracay into an agricultural land. There is nothing in the
law or the Circular which made Boracay Island an agricultural land. The reference in Circular No. 3-82 to "private lands" and "areas
declared as alienable and disposable" does not by itself classify the entire island as agricultural. Notably, Circular No. 3-82 makes
reference not only to private lands and areas but also to public forested lands. Rule VIII, Section 3 provides:

No trees in forested private lands may be cut without prior authority from the PTA. All forested areas in public lands are declared forest
reserves.
Clearly, the reference in the Circular to both private and public lands merely recognizes that the island can be classified by the Executive
department pursuant to its powers under CA No. 141. In fact, Section 5 of the Circular recognizes the then Bureau of Forest Development's
authority to declare areas in the island as alienable and disposable when it provides:

Subsistence farming, in areas declared as alienable and disposable by the Bureau of Forest Development.

Therefore, Proclamation No. 1801 cannot be deemed the positive act needed to classify Boracay Island as alienable and disposable land.
If President Marcos intended to classify the island as alienable and disposable or forest, or both, he would have identified the specific
limits of each, as President Arroyo did in Proclamation No. 1064. This was not done in Proclamation No. 1801.

The Whereas clauses of Proclamation No. 1801 also explain the rationale behind the declaration of Boracay Island, together with other
islands, coves and peninsulas in the Philippines, as a tourist zone and marine reserve to be administered by the PTA — to ensure the
concentrated efforts of the public and private sectors in the development of the areas' tourism potential with due regard for ecological
balance in the marine environment. Simply put, the proclamation is aimed at administering the islands for tourism and ecological purposes.
It does not address the areas' alienability.

More importantly, Proclamation No. 1801 covers not only Boracay Island, but sixty-four (64) other islands, coves, and peninsulas in the
Philippines, such as Fortune and Verde Islands in Batangas, Port Galera in Oriental Mindoro, Panglao and Balicasag Islands in Bohol,
Coron Island, Puerto Princesa and surrounding areas in Palawan, Camiguin Island in Cagayan de Oro, and Misamis Oriental, to name a
few. If the designation of Boracay Island as tourist zone makes it alienable and disposable by virtue of Proclamation No. 1801, all the
other areas mentioned would likewise be declared wide open for private disposition. That could not have been, and is clearly beyond, the
intent of the proclamation.

It was Proclamation No. 1064 of 2006 which positively declared part of Boracay as alienable and opened the same to private ownership.
Sections 6 and 7 of CA No. 141 provide that it is only the President, upon the recommendation of the proper department head, who has
the authority to classify the lands of the public domain into alienable or disposable, timber and mineral lands.

In issuing Proclamation No. 1064, President Gloria Macapagal-Arroyo merely exercised the authority granted to her to classify lands of
the public domain, presumably subject to existing vested rights. Classification of public lands is the exclusive prerogative of the Executive
Department, through the Office of the President. Courts have no authority to do so. Absent such classification, the land remains
unclassified until released and rendered open to disposition.

Proclamation No. 1064 classifies Boracay into 400 hectares of reserved forest land and 628.96 hectares of agricultural land. The
Proclamation likewise provides for a 15-meter buffer zone on each side of the center line of roads and trails, which are reserved for right
of way and which shall form part of the area reserved for forest land protection purposes.

Contrary to private claimants' argument, there was nothing invalid or irregular, much less unconstitutional, about the classification of
Boracay Island made by the President through Proclamation No. 1064. It was within her authority to make such classification, subject to
existing vested rights.[18]

Therefore, the island, being owned by the State, can only be declared or made subject of private ownership by the Government. And only
the Government can determine the manner in which the island should be disposed of or conveyed to private individuals, pursuant to the
Regalian Doctrine as this Court ruled in Secretary of the Department of Environment and Natural Resources v. Yap:[19]

The Regalian Doctrine dictates that all lands of the public domain belong to the State, that the State is the source of any asserted right to
ownership of land and charged with the conservation of such patrimony.[20] The doctrine has been consistently adopted under the 1935,
1973, and 1987 Constitutions.[21]

All lands not otherwise appearing to be clearly within private ownership are presumed to belong to the State.[22] Thus, all lands that have
not been acquired from the government, either by purchase or by grant, belong to the State as part of the inalienable public domain.[23]
Necessarily, it is up to the State to determine if lands of the public domain will be disposed of for private ownership. The government, as
the agent of the state, is possessed of the plenary power as the persona in law to determine who shall be the favored recipients of public
lands, as well as under what terms they may be granted such privilege, not excluding the placing of obstacles in the way of their exercise
of what otherwise would be ordinary acts of ownership.[24]

It was only in 2006 when certain parts of Boracay became agricultural land when then President Gloria Macapagal-Arroyo issued
Proclamation No. 1064, positively declaring parts of Boracay as alienable and opening the same to private ownership.

As such, the CA is then correct in ruling that with this Court's pronouncement that Boracay is state-owned, petitioners' claim of ownership
over the subject property is negated, thus:

With the latest pronouncement of the Supreme Court of Boracay as state-owned, private respondent's ownership over the property in
dispute is defeated. As discussed at length by the highest tribunal in the consolidated cases of The Secretary of DENR, et al. v. Yap, et
al. in G.R. No. 167707 and Sacay, et al. v. The Secretary of DENR, et al. in G.R. No. 173775, Boracay is an unclassified land of public
domain. Thus, where land is not alienable and disposable, possession of the land, no matter how long cannot confer ownership or
possessory right.

It follows then that Asicio (sic) S. Tupas was not in a position to sell that which he did not own in the first place. This is because at the
time the sale was entered into between private respondent and the late Asicio (sic) S. Tupas, the land in dispute was not alienable and
subject to disposition. Since private respondent derives title from whatever right his predecessor-in-interest had, which unfortunately
Asicio (sic) S. Tupas had none, his claim is no longer tenable. Private respondent cannot acquire a right greater than what his
predecessor-in-interest had. To allow the execution of judgment would be to give undue advantage to private respondent whose very
basis of claim is no longer tenable.[25]

The above reasoning of the CA has its basis on a simple logic that one cannot dispose of a thing he does not own. In this case, at the
time of the sale of the subject property, the late Asiclo S. Tupas had no right to sell a property that has not been declared alienable by
the State; hence, he cannot pass unto another any right or title to own or possess the land. Therefore, the "Sale of Unregistered Land"
entered into between the late Asiclo S. Tupas and the late Zosimo Maravilla on February 8, 1975, previously considered valid and
legitimate and became the basis used by the RTC to settle the dispute between the parties as to who has the better to right to the property,
has become null and void because the subject property of the contract is a forest land and cannot be alienated at the time the said deed
of sale was executed. Article 1347 of the Civil Code provides that only things, which are not outside the commerce of man, including
future things, may be the objects of the contracts and Article 1409 of the Civil Code also states that contracts whose objects are outside
the commerce of man are non-existent and void ab initio.

With the above disquisitions, this Court's decision in The Secretary of the Department of Environment and Natural Resources (DENR),
et al. v. Yap, et al. and Sacay, et al. v. the Secretary of the DENR, et al. is, therefore, considered as a supervening event that can stay
the execution of a judgment that has already attained finality. In Abrigo, et al. v. Flores, et al.[26] this Court ruled that:

Once a judgment becomes immutable and unalterable by virtue of its finality, its execution should follow as a matter of course. A
supervening event, to be sufficient to stay or stop the execution, must alter or modify the situation of the parties under the decision as to
render the execution inequitable, impossible, or unfair. The supervening event cannot rest on unproved or uncertain facts.

xxxx

We deem it highly relevant to point out that a supervening event is an exception to the execution as a matter of right of a final and
immutable judgment rule, only if it directly affects the matter already litigated and settled, or substantially changes the rights or relations
of the parties therein as to render the execution unjust, impossible or inequitable.[27] A supervening event consists of facts that transpire
after the judgment became final and executory, or of new circumstances that develop after the judgment attained finality, including matters
that the parties were not aware of prior to or during the trial because such matters were not yet in existence at that time.[28] In that event,
the interested party may properly seek the stay of execution or the quashal of the writ of execution,[29] or he may move the court to
modify or alter the judgment in order to harmonize it with justice and the supervening event.[30] The party who alleges a supervening
event to stay the execution should necessarily establish the facts by competent evidence; otherwise, it would become all too easy to
frustrate the conclusive effects of a final and immutable judgment.[31]

WHEREFORE, the Petition for Review on Certiorari under Rule 45 of the Rules of Court dated May 25, 2010 of petitioners heirs of Zosimo
Q. Maravilla is DENIED for lack of merit.

REPUBLIC VS. BAUTISTA JR

DOCTRINE:

The applicant for judicial confirmation of imperfect title must trace his possession of the subject land to June 12, 1945, or earlier. Any
length of possession that does not comply with the requirement cannot support the application, which must be then dismissed for failure
to comply with Commonwealth Act No. 141 (Public Land Act) and Presidential Decree No. 1529 (Property Registration Decree).

FACTS:

Apolonio, Sr. had been the sole and exclusive possessor and occupant from the time of acquisition until his death, with no party
questioning his possession and ownership, or staking any adverse claim against him thereon. He died in 1987, and was succeeded by
his children, namely: respondent Apolonio, Jr. and his siblings. Apolonio, Sr.'s children executed an extra-judicial settlement of their
father's estate, whereby Apolonio, Jr.'s brothers and sisters waived their rights in his favor. Thus, the property was declared for taxation
purposes in Apolonio, Jr.'s name.

On October 21, 1996, Apolonio Jr. commenced LRC Case No. N-12-10-96 in the MTC. The Government did not interpose any timely
objection to the testimony of Apolonia, Jr. It did not also object to the documentary evidence (i.e., the deeds of absolute sale and tax
declarations) offered by him. Hence, the MTC admitted all the evidence presented by Apolonia, Jr. In due course, the MTC granted
Apolonia, Jr.'s application, and declared him as the owner in fee simple of the land, and confirmed his ownership thereof. The Government
appealed the decision to the Court of Appeals (CA), which, on September 30, 2004, promulgated its assailed decision affirming the ruling
of the MTC. The CA pointed out that the Government did not present evidence against the claim of Apolonio Jr.; and that the Government
did not timely object to his testimony on the ground of its being hearsay.

ISSUE:

In response, Apolonio Jr. insists that he had duly established his lawful occupation of the land as owner in fee simple; that the Government
did not timely object to his testimony, and did not also controvert his evidence; that the property had been properly identified; and that the
lower courts had observed the legal safeguards and guidelines in granting his application for judicial confirmation of his ownership in fee
simple.
RULING:

The Government has correctly insisted that the requisite period of possession of the property should conform to that provided for in
Section 48(b) of the Public Land Act, as amended by Presidential Decree No. 1073, which has limited the right to apply for judicial
confirmation to citizens of the Philippines "who by themselves or through their predecessors in interest have been in open, continuous,
exclusive, and notorious possession and occupation of alienable and disposable lands of the public domain, under a bona fide claim of
acquisition of ownership, since June 12, 1945, or earlier, immediately preceding the filing of the application for confirmation of title except
when prevented by war or force majeure. x x x" The provision is reprised by Section 14(1) of Presidential Decree No. 1529 (Property
Registration Decree), adopting the length of possession and occupation of alienable and disposable lands of the public domain under a
bona fide claim of ownership since June 12, 1945, or earlier.

We note that in its amendment of the Public Land Act that took effect on January 25, 1977, Presidential Decree No. 1073 changed the
length of the requisite possession from "thirty (30) years immediately preceding the filing of the application" to possession "since June
12, 1945, or earlier."

Based on the records before us, Apolonia, Jr. presented only himself to establish the possession and ownership of his father, Apolonia,
Sr., who was his immediate predecessor-in-interest. He did not present as witnesses during the trial either of the transferors of Apolonia,
Sr. - that is, Mario Jardin or Cornelia Villanueva - to establish the requisite length of the possession of the predecessors-in-interest of the
applicant that would be tacked to his own. His personal incompetence to attest to the possession of the property within the time required
by law underscored the weakness of the evidence on possession, particularly as it has not been denied that the applicant had arrived in
the Philippines only on November 28, 1987. Considering that the possession and occupation of the property in question by Apolonia, Jr.
and his predecessors-in-interest were not shown in the records to have been "since June 12, 1945, or earlier," the application must be
rejected.

We should stress that only the title of those who had possessed and occupied alienable and disposable lands of the public domain within
the requisite period could be judicially confirmed. Indeed, alienable public land held by a possessor, either personally or through his
predecessors-in-interest, openly, continuously and exclusively during the prescribed statutory period is converted to private property by
the mere lapse or completion of the period.

That the Government did not timely object to the admission of the testimony of Apolonia, Jr., or of the other evidence presented by him
was of no consequence to the success of the application. If he had no personal knowledge of the facts establishing the possession of
property for the requisite period, no court can give any value to his assertion, particularly as it was conceded by him no less that he had
no personal or direct competence to know the truth of his assertion. It was one thing for the trial court to admit the evidence, but quite
another to give it any worth for purposes of judicial adjudication.

ALOLINO VS. FLORES

FACTS:

Alolino is the registered owner of two (2) contiguous parcels of land. Alolino initially constructed a bungalow-type house on the property.
In 1980, he added a second floor to the structure. He also extended his two-storey house up to the edge of his property. There are
terraces on both floors. There are also six (6) windows on the perimeter wall: three (3) on the ground floor and another three (3) on the
second floor.

In 1994, the respondent spouses Fortunato and Anastacia (Marie) Flores constructed their house/sari sari store on the vacant
municipal/barrio road immediately adjoining the rear perimeter wall of Alolino’s house. Since they were constructing on a municipal road,
the respondents could not secure a building permit. The structure is only about two (2) to three (3) inches away from the back of Alolino’s
house, covering five windows and the exit door. The respondents’ construction deprived Alolino of the light and ventilation he had
previously enjoyed and prevented his ingress and egress to the municipal road through the rear door of his house.

The respondents contend that the said barrio road which they built on was already converted to patrimonial property of the state and
hence they had validly built their store.

ISSUE:

Whether or not respondent’s contention is correct.

RULING:

NO! To convert a barrio road into patrimonial property, the law requires the LGU to enact an ordinance, approved by at least two-thirds
(2/3) of the Sanggunian members, permanently closing the road. In this case, the Sanggunian did not enact an ordinance but merely
passed a resolution. Properties of the local government that are devoted to public service are deemed public and are under the absolute
control of Congress. Hence, LGUs cannot control or regulate the use of these properties unless specifically authorized by Congress, as
is the case with Section 21 of the LGC. In exercising this authority, the LGU must comply with the conditions and observe the limitations
prescribed by Congress.

As a barrio road, the subject lot’s purpose is to serve the benefit of the collective citizenry. It is outside the commerce of man and as a
consequence:
(1) it is not alienable or disposable;
(2) it is not subject to registration under Presidential Decree No. 1529 and cannot be the subject of a Torrens title;
(3) it is not susceptible to prescription;
(4) it cannot be leased, sold, or otherwise be the object of a contract;
(5) it is not subject to attachment and execution; and
(6) it cannot be burdened by any voluntary easements.

As it remained a road, it is within the purview of Sec. 28 of Urban Development and Housing Act and hence can be demolished.

REPUBLIC VS. TAN

FACTS:

Tan applied for the original registration of title of Lot No. 4080, Cad. 545-D situated in Casili, Consolacion, Cebu. She alleged that she is
the absolute owner in fee simple of the said 7,807 square-meter parcel of residential land she purchased from a certain Julian Gonzaga
on September 17, 1992. Land registration court granted Tan’s application. The court confirmed her title over the subject lot and ordered
its registration. The Republic appealed the case to the CA, arguing that Tan failed to prove that she is a Filipino citizen who has been in
open, continuous, exclusive, and notorious possession and occupation of the subject lot, in the concept of an owner, since June 12, 1945,
or earlier, immediately preceding the filing of her application.

The CA noted that before land of the public domain can be acquired by prescription, it must have been declared alienable and disposable
agricultural land. The CA pointed to the certification issued by the Community Environment and Natural Resources Office (CENRO) as
evidence that the subject was classified as alienable and disposable on September 1, 1965, pursuant to Land Classification Project No.
28. The CA concluded that Tan had already acquired the subject lot by prescription.

ISSUE:

Whether or not the CENRO certification and tax declarations presented were insufficient to prove that the subject lot was no longer
intended for public use.

RULING:

YES! In Malabanan case, we already held en banc that a declaration that property of the public dominion is alienable and disposable
does not ipso facto convert it into patrimonial property. While a prior declaration that the property has become alienable and disposable
is sufficient in an application for judicial confirmation of title under Section 14(1) of the PRD, it does not suffice for the purpose of
prescription under the Civil Code. Before prescription can even begin to run against the State, the following conditions must concur to
convert the subject into patrimonial property:

1. The subject lot must have been classified as agricultural land in compliance with Sections 2 and 3 of Article XII of the Constitution;
2. The land must have been classified as alienable and disposable
3. There must be a declaration from a competent authority that the subject lot is no longer intended for public use, thereby converting it
to patrimonial property.

Only when these conditions are met can applicants begin their public and peaceful possession of the subject lot in the concept of an
owner.

In the present case, the third condition is absent. Even though it has been declared alienable and disposable, the property has not been
withdrawn from public use or public service. Without this, prescription cannot begin to run because the property has not yet been converted
into patrimonial property of the State. It remains outside the commerce of man and the respondent’s physical possession and occupation
thereof do not produce any legal effect. In the eyes of the law, the respondent has never acquired legal possession of the property and
her physical possession thereof, no matter how long, can never ripen into ownership.

NOVAI VS. REPUBLIC

FACTS:

A Transfer Certificate Title (TCT) issued in Navy Officers’ Village Association, Inc (NOVAI)’s name covers a land situated inside the former
Fort Andres Bonifacio Military Reservation in Taguig. This property was previously a part of a larger parcel of land which TCT’s under the
name of the Republic of the Philippines.

The then President Garcia issued a Proclamation No. 423 which reserves for military purposes certain parcels of the public domain
situated in Pasig, Taguig, Paranaque, Rizal and Pasay City. Thereafter, then President Macapagal issued Proclamation No. 461 which
excluded Fort McKinley a certain portion of land situated in the provinces abovementioned and declared them as AFP Officers’ Village
to be disposed of under the provisions of certain laws. However, this area was subsequently reserved for veterans’ rehabilitation, medicare
and training center sites.

The property was the subject of deed of sale between the Republic and NOVAI to which the TCT was registered in favour of the latter.
The Republic then sought to cancel NOVAI’s title on the ground that the property was still part of the military reservation thus inalienable
land of the public domain and cannot be the subject of sale. The RTC ruled that the property was alienable and disposable in character.
The Court of Appeals reversed RTC’s decision.

ISSUE:

Whether or not the property covered by TCT issued under the name of NOVAI is inalienable land of public domain and cannot be the
subject of sale.

RULING:

YES! The property remains a part of the public domain that could not have been validly disposed of in NOVAI’s favor. NOVAI failed to
discharge its burden of proving that the property was not intended for public or quasi-public use or purpose.

It is settled that the land falls under those which are reserved for public use in CA 141. In a limited sense, parcels of land classified as
reservations for public or quasi-public uses under Section 9 (d) of C.A. No. 141 are still non-alienable and non-disposable, even though
they are, by the general classification under Section 6, alienable and disposable lands of the public domain. By specific declaration under
Section 88, in relation with Section 8 and Section 83, these lands classified as reservations are non-alienable and non-disposable.

The classification and disposition of lands of the public domain are governed by Commonwealth Act (C.A.) No. 141 or the Public Land
Act, the country's primary law on the matter.

Under Section 6 of C.A. No. 141, the President of the Republic of the Philippines, upon the recommendation of the Secretary of Agriculture
and Natural Resources, may, from time to time, classify lands of the public domain into alienable or disposable, timber and mineral lands,
and transfer these lands from one class to another for purposes of their administration and disposition.

In a limited sense, parcels of land classified as reservations for public or quasi-public uses under Section 9 (d) of C.A. No. 141 are still
non-alienable and non-disposable, even though they are, by the general classification under Section 6, alienable and disposable lands
of the public domain. By specific declaration under Section 88, in relation with Section 8, these lands classified as reservations are non-
alienable and non-disposable.

As provided in Article 420 of Civil Code, “property of the public dominion as those which are intended for public use or, while not intended
for public use, belong to the State and are intended for some public service”. In this case, the property was classified as military reservation
thus, remained to be property of the public dominion until withdrawn from the public use for which they have been reserved, by act of
Congress or by proclamation of the President. Since there was no positive act from the government, the property had to retain its
inalienable and non-disposable character. It cannot therefore, be subject of sale otherwise, the sale is void for being contrary to law.

Applying to the case at bar, Proclamation No. 478 was issued after Proclamation No. 461. Hence, while Proclamation No. 461 withdrew
a certain area or parcel of land from the FBMR and made the covered area available for disposition in favor of the AFPOVAI, Proclamation
No. 478 subsequently withdrew the property from the total disposable portion and reserved it for the use of the VRMTC.

REPUBLIC VS. MIC

FACTS:

Petitioner Republic, represented by the Department of Environment and Natural Resources - Region IV (DENR) filed a petition before
the Court of Appeals for annulment of judgment, cancellation of title, and reversion against the respondents including the R egister of
Deeds for the Province of Batangas (Tanauan, Batangas), and the Regional Trial Court of Lipa City.

Petitioner, through the Office of the Solicitor General (OSG), alleges that respondents Marjens and Villanueva appear as registered
owners of a land identified as Lot 1 (LRC) Pcs-943, which is a portion of Lots 1 and 2, plan Psu-114430 LRC (G.L.R.O.) Record No. N-
3454, with an area of five thousand (5,000) square meters, covered by Transfer Certificate of Title (TCT) No. T-18592 issued on April 7,
1976 by the Office of the Register of Deeds of Tanauan, Batangas.

In Land Registration Case No. 52, G.L.R.O. Rec. No. 3454, the then Court of First Instance of Batangas rendered a Decision dated March
30, 1951 x x x granting the application for registration of several parcels of land in favor of the applicants therein, Hammon H. Buck, et
al. It was established that the lands described in Plans Psu-118922 and 114430 were originally owned by Rita Vda. de Ilustre since 1890.

The OSG among others alleges that the land in question cannot be the subject of disposition or registration, and the trial court did not
acquire jurisdiction over said property, much less to decree the same as private property. Therefore, the registration proceedings, the
judgment in the subject case, the OCT No. O-669 issued pursuant thereto, and all subsequent titles are null and void. The land covered
by TCT No. T-18592, not having been legally registered, remains and forms part of the public domain of the State.

Respondents claim that their titles, their predecessors' titles, and their mother title are issued in accordance with law, and that the property
was registered and brought under the Torrens system. Respondents contend that the subject property was already private property even
before the Spanish Crown ceded sovereignty over the Philippine Islands to the United States of America. They assert that the government
has lost its rights by laches and estoppel to question the validity of the OCT No. 0-669, the proceedings in LRC Case No. 52, G.L.R.O.
Record No. N-3454, and the corresponding decree (Decree 6610) issued after almost 50 years have lapsed.
The Court of Appeals dismissed the petition applying the case of Cariho v. Insular Government of the Philippine Islands, which recognized
private ownership of lands already possessed or held by individuals under claim of ownership as far back as testimony or memory goes
and therefore never to have been public land that Spain could bequeath to the United States of America.

ISSUES:

1. Whether or not the subject property covered by TCT No. T-18592 is a private property or part of the public domain.
2. Whether or not the government is barred by laches and estoppel.

RULING:

YES! It is a private property. The records did not categorically state that Rita Vda. de Ilustre had Spanish title over the subject property.
But by virtue of her long continued, open, public, adverse possession and cultivation of the property in the concept of owner as against
the whole world she is deemed to have acquired ownership over the subject property.

As for respondents, it is undisputed that the property covered by TCT No. T-18592 traces its title to the property originally owned by Rita
Vda. de Ilustre since 1890. From her it passed on to several hands until it was transferred to Hammon H. Buck, who successfully registered
it in his name on February 18, 1952. From 1890, respondents' predecessors in interest had been in peaceful, open, continuous, exclusive,
adverse, and notorious possession in the concept of an owner of the subject property including the portion covered by TCT No. T-18592.
Following the Cariño ruling, the subject property had been a private land and excluded from the public domain since 1890 prior to the
signing of the Treaty of Paris on December 10, 1898. Therefore, it is not part of the public domain that passed on from Spain to the United
States of America.

For the same reason, it is also not part of the unclassified public forest as petitioner claims. In Republic v. Court of Appeals and Cosalan,29
the Court held that "[d]espite the general rule that forest lands cannot be appropriated by private ownership, it has been previously held
that 'while the Government has the right to classify portions of public land, the primary right of a private individual who possessed and
cultivated the land in good faith much prior to such classification must be recognized and should not be prejudiced by after-events which
could not have been anticipated...Government in the first instance may, by reservation, decide for itself what portions of public land shall
be considered forestry land, unless private interests have intervened before such reservation is made'"

The map (LC Map No. 3013), which is the basis of petitioner's claim, is inexistent at the time Hammon H. Buck was issued an original
certificate of title. Therefore, the subject property had been a private property before it was classified. Thus, the Court agrees with the
Court of Appeals' findings and upholds the private character of the subject property.

2. YES! Laches has been defined as the "failure or neglect for an unreasonable and unexplained length of time to do that which, by
observance of due diligence, could or should have been done earlier. It is negligence or omission to assert a right within a reasonable
time, warranting the presumption that the party entitled to assert his right either has abandoned or declined to assert it."

The following elements must be present in order to constitute laches:

(a) conduct on the part of the defendant, or of one under whom he claims, giving rise to the situation complained of; (b) delay in asserting
complainant's rights after he had knowledge of defendant's acts and after he has had the opportunity to sue; (c) lack of knowledge or
notice by defendant that the complainant will assert the right on which he bases his suit; and (d) injury or prejudice to the defendant in
the event the relief is accorded to the complainant.

We find it unnecessary to discuss further this issue in view of our ruling that Decree No. 6610, OCT No. 0-669, and TCT No. T-18592
registered in the name of respondents were validly issued.

Вам также может понравиться